Applied Mechanics PDF

Download as pdf or txt
Download as pdf or txt
You are on page 1of 348
At a glance
Powered by AI
The key takeaways are that the book provides students of engineering and allied constructive arts with practical principles of mechanics essential to understanding their occupations. It aims to avoid just presenting rules and formulas and provides worked examples and exercises.

The book is about applied mechanics for beginners. It aims to provide students with an understanding of the principles of mechanics needed for their studies and work in engineering and related fields.

The book covers subjects like practical geometry, machine or building construction, and practical mathematics as prerequisites. It then discusses principles of mechanics like dynamics, kinematics and provides examples and exercises on typical problems encountered in engineering work.

HANDBOUND

AT THE

UNIVERSITY OF
TORONTO PRESS
Digitized by the Internet Archive
in 2007 with funding from
Microsoft Corporation

http://www.archive.org/details/appliedmechanicsOOduncuoft
^f

77ff

APPLIED MECHANICS FOR BEGINNERS


^^PLIED MECHANICS
FOR BEGINNERS

BY

J. DUNCAN, Wh.Ex., A.M.I.M.E.


HEAD OF THE DEPARTMENT OF AND MECHANICAL ENGINEERING AT THE
CIVIL
MUNICIPAL TECHNICAL INSTITUTE, WEST HAM

fConbcn
MACMILLAN AND CO., Limited
NEW YORK : THE MACMILLAN COMPANY
1902

All right* reserved


1*1, t
\

GLASGOW PRINTED AT THE UNIVERSITY PRESS


:

BY ROBERT MACLEHOSE AND CO.


PREFACE.
In the preparation of this little book, the object has been to
provide students of engineering and allied constructive arts
with a practical statement of the principlesMechanics
of
essential to an intelligent interest While
in their occupations.
the book will serve as a sufficient preparation for the elementary
examination in Applied Mechanics of the Board of Education,
its contents are not limited by the syllabus of this or any other

examining body.
Before the student can profitably take up the study of
Mechanics, he should be acquainted with the elementary por-
tions of Practical Geometry, Machine or Building Construction,
and Practical Mathematics. This preliminary knowledge has
therefore been taken for granted.
A constant endeavour has been made to avoid producing a
mere collection of rules and formulae, sufficient explanations
being given to ensure for a careful reader a systematic know-
ledge of the principles discussed. To preserve a constant
connection between theory and practice, numerous worked out
examples of problems which present themselves in everyday
work are scattered throughout the chapters. Additional exer-
cises, suitable for home or class work, will be found at the end
of each chapter ; those with a date are from examination papers
of the Board of Education, South Kensington.
Importance should be attached to the performance by the
student of typical experiments. Descriptions of suitable forms
of apparatus are given, and practical exercises to be worked
out with them are suggested. These experiments have been
arranged in the form of a Laboratory Course, the subjects of
which are brought together at the end of the book.
:

vni PREFACE

The opportunity is here taken to make several grateful


acknowledgments : to Prof. Rowden, of Glasgow, from whom I
received my first lessons in Mechanics, and to whom many of
the methods, used in developing the principles elucidated, are due -
y

to Messrs. A. Walker and R. Macmillan, Demonstrators in


Engineering in "West Ham Technical Institute, for kindly
reading the proofs ; and to Prof. R. A. Gregory and Mr. A. T.
Simmons for useful hints and guidance in preparing the book.

Thanks must also be given to the following firms for permis-


sion to reproduce certain illustrations from their copyright lists

Messrs. Tangyes, Ltd., for Figs. 258, 310, 311, 312, 316, 318 ;

Messrs. Buck & Hickman, for Fig. 9 ; Messrs. Charles


Churchill & Messrs. Hoffman Mfg. Co. Ltd., for
Co., for Fig. 6 ;

Figs. 193, 228 ; Ludw. Loewe & Co., for Figs. 2, 3, 4, 5,


Messrs.
10 ; and Messrs. The Empire Roller Bearing Co. Ltd., for
Fig. 194.
The Tables of Logarithms and Trigonometrical Ratios are
reprinted from Mr. F. Castle's Practical Mathematics for
Beginners (Macmillan).
It is almost too much to hope that a book containing so many
exercises will be found entirely free from numerical errors, and
I shall be very grateful if any such are pointed out to me.

J. DUNCAN.
West Ham,
July, 1902.
CONTENTS

CHAPTER I.
PAGE
Introductory, - - 1

CHAPTER II.

Matter, Force, Weight, - ...- - - 13

CHAPTER III.

Two and Three Forces acting at a Point, - - - 21

CHAPTER IV.

Triangle and Polygon of Forces. Simple Structures, - 32

CHAPTER V.

Moments ; Parallel Forces ; Couples, ..... 42

CHAPTER VI.

Centre of Gravity Forces not all applied at the same


;

Point Hanging Chains Arch,


; ; 54

CHAPTER VII.

Stress ; Strain ; Elasticity ; Ultimate Tensile Strength.


Strength of Shells ; Shear, 67
x CONTENTS.

CHAPTER VIII.
PAGE
Strength and Stiffness of Beams, 88

CHAPTER IX.

Riveted Joints ; Shafts; Springs, 100

CHAPTER X.

Work Mechanical Advantage and Velocity Ratio of


;

Machines Energy Power Efficiency Diagrams


; ; ; ;

of Work ; Resilience, - 122

CHAPTER XI.

Friction of Dry and Lubricated Surfaces, - - 136

CHAPTER XII.

Velocity ; Acceleration ; Inertia ; Kinetic Energy ; Rela-


tive Velocity; Change of Velocity, - - 15$

CHAPTER XIII.

Mechanism ; Transmission of Motion and Power, - 166

CHAPTER XIV.

Actual Mechanical Advantage and Efficiency ; Experi-


ments on Simple Machines, 195

CHAPTER XV.
Indicated and Brake Horse-power ; Absorption and
Transmission Dynamometers ; Flywheels Steadi- ;

ness of Machines ; Momentum ; Impact ; Force of


a Blow ; Centrifugal Pump, 211
;

CONTENTS. xi

CHAPTER XVI.
I'AOB

Centrifugal Force ; Examples of Want of Balance in


Rotating Bodies ; Bursting Effect in Flywheels
Governors, 234

CHAPTER XVII.
Hydraulics ; Water Pressure and Pressure Machines, - 247

CHAPTER XVIII.

Hydraulics ; Flow of Water ; Water Motors, - - 268

CHAPTER XIX.
Materials, - - - - -- - - - 291

Course of Laboratory Work, - - - - 302

Tables
Useful Constants, 10
Weights and Specific Gravities, - - - - 17
Young's Modulus of Elasticity, 74
Six Standard Cases of Beams, - 100
...
-

Ultimate Strength of Materials, - 300


Logarithms, 308
Antilogarithms, 310
Trigonometrical Ratios, 312

Answers, 313

Index, - - 318
APPLIED MECHANICS
FOR BEGINNERS.
CHAPTER I

INTRODUCTORY.
MEASURING INSTRUMENTS.
Straight Edge. The engineer's straight edge consists of a
long strip of metal with one edge bevelled, this edge being such
that a straight line drawn from two points situated near the
ends of the edge will lie v ;olly in the edge. If a straight edge
has to be originated, it is necessary to make three at the same
time, then by a continual process of comparing one with the
others and removing the faulty parts by scraping, the edges
of all three may be brought nearly true.
Surface Plate. The surface plate consists of a rigid plate of
cast iron, having three feet on
its under side, in order always to

distribute the supporting forces


in the same manner and so pre-
vent the plate warping. The
upper surface of the plate is
brought, as far as possible, all
to lie in one plane. This is done
by constructing three plates at Fig. l Surface plate,

a time. The upper surfaces of


the plates having been carefully planed, are compared one
with the others. A little oil and colouring matter rubbed
a.m.b. a <E
APPLIED MECHANICS FOR BEGINNERS.

on the surface show those spots which are in error, and


these are removed by scraping. The operation is finished when
any two of the three, on being brought together, show contact
over a large number of bearing spots evenly distributed.
These spots will then all lie in one plane. Any one of the
plates may now be used for the reproduction of other plane
surfaces.
External and Internal Gauges. Sir Joseph Whitworth,
by constructing bars having plane ends perpendicular to their
axes, subdivided the standard yard into inches, etc. Engineers'
steel rules, subdivided with considerable accuracy into inches,
tenths, etc., are one of the results
of his work, and can be used for
producing objects having required
dimensions. For standards of refer-
ence in the shops, cylindrical external
Fig. 2. Standard cylindrical
gauges. and internal gauges are used. These
are shown
and consist of in Fig. 2,
a collar with a hole, and a plug. Both hole and plug are
brought nearly to size by machining, then hardened, ground
and lapped down to size by hand. Gauges such as this are
turned out true to Tuljoo th i ncn -

Standard gauges are not suitable for the reproduction of


dimensions to a given degree of accuracy, as calipers have
to be used in transferring
the dimension from the
gauge to the work. The
calipers are adjusted as
nearly as the workman
can tell by touch to the
Fig. 3. Internal limit gauge. standard gauge and then
are applied to the work.
In much is left to the skill and discretion
this process too
of the workman, and no limits of accuracy can be stated
and worked to. To secure good results, limit gauges are
necessary. Two of these are shown in Figs. 3 and 4. The
internal limit gauge is for measuring inside cylindrical holes.
One end is made slightlylarger in diameter than the other,
the difference being determined by the limits of accuracy
MEASURING INSTRUMENTS.

required in the work under execution. The smaller end must


" go in " to the finished hole, and the larger end must " not
go in."Consequently, the finished
hole larger in diameter than
is

the small end of the gauge and


smaller in diameter than the large
end, and so the work is kept within
the desired limits of accuracy. For
example, a be 1 inch
hole to
approximately in diameter would
be bored, using an internal limit
gauge having diameters 1 "006" and
0*994" respectively. The total
variation in the diameter of the
finished hole cannot exceed 0*0012".
The external limit gauge is used
for turning cylindrical pieces down Fig. 4. External limit gauge.

to size, and
used in a similar
is

manner. As nothing is left to the discretion of the workman,


interchangeable parts can easily be produced by the use of
these gauges.
Standard screw-gauges are also useful for reference, and
help to produce accurate work. Two of these, one external
and one internal, are shown in Fig*. 5.

Fig. 5. Standard screw-gauges.

For the more accurate measurement of dimensions than can


be secured by the use of calipers, micrometers are used. In
Fig. 6 a micrometer is shown having its outer parts shown trans-
parent. The instrument consists of a very finely cut screw,
which may be rotated by turning the outer milled thimble.
This screw works in a split nut, fitted with an adjusting nut to
APPLIED MECHANICS FOR BEGINNERS.

take up wear, and terminates, as shown, between the jaws of


the instrument, in a cylindrical portion having its end brought
plane and square to the axis of the screw.
The screw is protected from dust and grit by
the outer thimble casing. The object to be
measured is placed in the jaw of the instrument,
and the thimble turned until the object is
gently nipped. The dimension is then read
from two scales, one engraved along the barrel
longitudinally, and the other circularly round
the edge of the thimble. In the instrument
shown, the screw has 40 threads to an inch, and
one inch on the longitudinal scale is divided into
tenths of an inch, each tenth
being subdivided into four
parts each part will therefore
;

be jtq" or 0*025" long. One


revolution of the thimble will
consequently advance the screw
one part on the longitudinal
scale, or a distance of 0*025"
The circular scale on the thimble
has 25 divisions round the
complete circumference, conse-
quently revolving the thimble
through one division will ad-
vance the screw
J^x 0-025" =0-001".
The instrument can therefore be
Fig. Micrometer. used for taking dimensions to
Y^y inch. To read the scales,
suppose, as in Fig. 6, that the longitudinal scale shows three
parts beyond 0*1". This will be 0\L + (3 x 0*025) = '175". The
circular scale is set at or 25, so that in this position nothing
need be allowed for it. The dimension, as set, is therefore 0*1 75".
If the circular scale had been beyond the 0*175" mark on the
longitudinal scale, by, say, 14 divisions, then we should have
added 0*014" to the above reading, giving 0*175 +0"014 =0*189"
as the required dimension.
1
MEASURING INSTRUMENTS.

Micrometers should be tested occasionally for zero error by


running the screw right home until the points touch, and
ascertaining if the or 25 mark on the circular scale comes opposite
to the line on the longitudinal scale. If this is not the case, the
instrument can be adjusted by the screw point on the opposite
jaw, or a zero error may be allowed for in subsequent readings.
Verniers. The veimier is a device for subdividing the parts
of a scale into divisions that
VMltlH
would be too fine to be read
1 1

fi' v.
' 1

by the eye. It consists of a / !


V
i . . ' 1 1
i 1
I

sliding piece fitted to a main


scale and having a suitable Fig. 7.Vernier, set at zero.
scale engraved on it. In the
case shown in Fig. 7 the vernier scale has 10 divisions of total
length equal to 9 divisions on the main scale. Each division on
the vernier is therefore f^tt shorter than a division on the main
scale, so that if set with the arrow opposite a division on the
main scale, the next two divisions
will be ^
tt
of a division apart, *''
'i'i'i
|
i i
| ' |
I
1
' ' '
|
'

the next pair of divisions ^jth8


Fig. 8. Vernier, set at 074.
and so on. To read the instru-
ment, note the division on the main scale to the left of the
vernier arrow, in the case shown in Fig. 8 this is 0*7 ; then
look along the vernier to find a division on it exactly opposite
a division on the main scale and note the vernier division,
in the example this is 4 so that the vernier arrow is T% ths of
;

a main scale division beyond the 0*7 mark the reading is ;

therefore 0*74

iiiilii"l"i<liiiilwi)i|iili
S lll
M'ijW ll l l llj
l l

'i
l!ilLl

Fig. 9. Vernier caliper, set at 0-200".


Tl
Calipers for use in the workshop are often fitted with verniers.
The one shown (Fig. 9) can read up to about If" by ixfojf h of an
8 APPLIED MECHANICS FOR BEGINNERS.

inch. The mainscale has inches divided into tenths, and each
tenth subdivided into five parts, each part being therefore
is

Jjj inch. The vernier has 20 divisions of total length equal to


19 divisions on the main scale, so that each vernier division is
th
2*0 shorter than a main scale division. The vernier therefore
reads to x ^ ^= ^
T n inch. As set in Fig. 9, the instrument
reads 0'200" on the main scale and on the vernier, the
dimension is therefore 0'200". Had the vernier been set say at
11, the reading would be 0*21 1". Readings of this instrument
should be corrected for zero error in the same manner as for
micrometers.

Other Devices. End measuring rods (Fig. 10) are very useful
for calipering holes or distances between parallel faces, when
the dimensions are large. The ends of these rods are made
spherical, so that they cannot be jammed when in position,
no matter how they are
turned. The rod is held
by the vulcanite handle
at its middle. Flexible
steel tapes may be
used
for taking the diameter
of large cylindrical pieces.
By passing these tapes
round such a cylinder
its circumference can be
obtained, from which
measurement the dia-
Fig. 10.-End measuring rods, with spherical
lneter of tne Cylinder may
ends. be calculated.

Mensuration.
Determination of Areas.
Some of the ordinary rules of
mensuration are given here for future reference.
Square, side s ; area = s2 .

Rectangle, adjacent sides a and b ; area = a x b.

Triangle, base perpendicular height h area = ^6x/<.


b, ;

Parallelogram, area = one side x perpendicular distance from


that side to the opposite one.
MEASURING INSTRUMENTS.

Any irregular figure bounded by straight lines ; to find its


area split it up into triangles, find the area of each triangle

separately, and take the sum of these areas


for the area of the figure. A
Trapezoid, such as A BCD (Fig. 11),
area = BC x average height.
The average height will be EF, drawn
from the centre of BC, and will be equal to
\{AB + CD).
E
TIC
Area of trapezoid = -^- (A B + CD). Fro. 11.

Suppose we have a figure consisting of a number of trapezoids


same breadth a, of which the area is required.
(Fig. 12), all of the
"We may proceed to find the area of each separately and to sum
for the total area.

A ^+_A.,)
Th s ,area =
(
.^) + (^+^ a+ ^+^ a + a)

This gives us the trapezoidal rule for such areas, viz. take
half the sum of the first and last ordinate, add to this the sum
of all the intermediate ordinates, and multiply the result by
the common distance between the ordinates.

Fio. 12. Fia. 13.

Simpson's Rule for finding the area of a plane figure bounded


by a curve and end ordinates perpendicular to the base is

founded on the assumption that the curve is parabolic. The


rule only is given here. For a curve such as A BCD (Fig. 13)
divide it by an ordinate h % so as to bisect the base DC,
8 APPLIED MECHANICS FOR BEGINNERS.

Let a = the distance between the ordinates and h v h 2 ,

A3 =the heights of the


ordinates.
Then,

area - {h x + 4h 2 + h3 ).

It is sometimes con-
Fig. 14. venient to take more
ordinates as is shown
for the area EFGH (Fig. 14). The number of ordinates must
always be odd and they must be equidistant.

In this case, area EKLR = -^(h


o
+ 4h + A
x 2 3 ),

area KMNL = -o (A + 4A 3 4 + h 5 ),

area MFGN= -o (A + 4A + h
5 6 r ),

and total area =-(h 1 + 4h 2 + 2h 3 + 4A4 + 2 h5 + 4A 6 + h 7 ).

This rule may be stated thus :


Add the first and the last
ordinates ; to this sum add four times the sum of the even
intermediate ordinates and twice the sum of the odd inter-
mediate ordinates ; multiply this total sum by one third the
common distance between the
ordinates.
These and other convenient
rules for finding the areas of
figures bounded by curves are
more used by naval architects
than by engineers. The ordin-
ary process used by engineers for
Fig. 15. finding the area of a figure such
as ABCD (Fig. 15) is to divide
DC into 10 equal parts and
measure the height of the
to
diagram at the centre of each part. Sum these heights and
divide by 10. The result gives the average height of the
MEASURING INSTRUMENTS.

diagram and if this be multiplied by DC the final result will


give the area.

Circle, diameter, d or radius, r ; circumference = 7rd=27rr.

7r denotes the ratio of the circumference of a circle to its

diameter and is represented by the number 3'1416. For many


engineering calculations the value 3j2 is correct enough.

Area of circle = ^=
4
7rr
2 = 0'7854 d 2 . .

Determination of Volumes. Cube, edge, s ; volume =s 3 .

Prism, having ends perpendicular to


its its axis volume
;

= area of one end x length of prism.


Sphere, radius, r volume = -firr3
; .

(Area of curved surface = 47rr2.)


Pyramid, volume = area of basex^ perpendicular height.
Cone, volume = area of base x \ perpendicular height.
(Area of curved surface = circumference of base x \ slant
height.)

Right-angled triangle ABC (Fig. 16); AC = AB + BC


2 2 2
.

Measurement of Angles. Angles may be measured in


degrees, or in radians.
A degree is the angle at the centre of a circle subtended by
an arc of g^" 1
of the circumference.

r c
Fio. 16. Fio. 17. The radian.

A
radian is the angle at the centre of a circle subtended by an
arc equal to the radius of the circle; in Fig. 17, is one ABC
radian.
In a complete circle there are 360 degrees and 27r radians,

therefore
2ir radians = 360 degrees,
or 7r radians = 180 degrees. '
70 APPLIED MECHANICS FOR BEGINNERS.

An angle expressed in radians can be transformed to degrees


180
by multiplying by or if expressed in degrees, can be
;

transformed to radians by multiplying by


180
An angle may
be expressed in radians by dividing the length
of the circular arc subtending it by the radius of the arc, both
being in the same units.

Trigonometrical Ratios.

The following definitions should be understood. Given an


angle POM (Fig. 18), take any distance OP and draw PM
perpendicular to 0M.
Then the following ratios of the sides will be independent of
the length OP and will depend only on the magnitude of the
given angle.
PM
-- is called the sine of the angle POM,
written sin POM.
0M is called the cosine of the angle
Fio. 18.
OP
POM, written cos POM.
PM
- is called the tangent of the angle POM, written tan POM.
Values of the sine, cosine, and tangent of angles up to 90 are
given in the mathematical tables at the end of the book.

USEFUL CONSTANTS.
1 inch = 2*54 centimetres.
1 metre = 39-37 inches.
5280 feet = 1 mile.
6 feet = 1 fathom.
1 chain = 66 feet.
80 chains = 1 mile.
1 knot = 6080 feet per hour.
MEASURING INSTRUMENTS. 11

1 square inch =6*45 square centimetres.


1 square metre =1550 square inches.
1 cubic inch =1639 cubic centimetres.
1 cubic metre =61,025 cubic inches = 1*308 cubic yard.
1 litre =1000 cubic centimetres = 1*761 7 pint.
1 gallon =0*1605 cubic foot = 4*541 litres.

1 bushel =1*2837 cubic feet.


One pound avoirdupois = 7000 grains = 453*6 grams.
One kilogram = 2*205 pounds.
One gallon of pure water at 62 F. weighs 10 lbs.

EXERCISES ON CHAP. I.

1. Convert 9 ft. 6 in. to metres.

2. Convert 2*94 metres to feet and inches.


3. Convert 3 miles 15 chains to kilometres.
4. Convert 53*7 millimetres to inches.
5. A rectangle has sides 4|" and 2 J". Calculate its area.

6. A
triangle, base 8 cms. ;
perpendicular height, 13*25 cms.
Calculate its area.

7. Draw
carefully to scale a triangle having sides respectively
4"> d og".
3i"> Measure its perpendicular height from your
aTfl

drawing and calculate the area of the triangle from this and the
length of the base.

8. What is (a) the circumference and (b) the area of a circle


whose diameter is 14 cms. ? Take 7r = -y-.
9. Calculate the volume of a ball 9" diam.

10. Draw full size a 5-sided figure ABCDE from the following
particulars. Take measurements from your drawing and calculate
its area.

Sides, AB=4", BC=3", CD =2", DE=U", EA = \".


Diagonals, BD = 3", AD=l%'.
11. A figure stands on a base AB 5" loug. Its heights at intervals
of 1", starting from A, are 2", 4", 2", 3", 1", 5". Straight lines
joining the tops of these ordinates bound the top of the figure.
Calculate, using the trapezoidal rule, the area of the figure.

12. Draw at random any figure bounded by a curve at the top,


12 APPLIED MECHANICS FOR BEGINNERS.

and find its area by applying (a) Simpson's rule, (b) the ordinary
engineering rule.
13. Describe any micrometer screw gauge with which you are
acquainted, suitable for measuring to the YoV^th of an inch.
Sketch and describe carefully the method of graduation and the
position of the gauge when set to measure '374 inch. (1899.)
CHAPTER II.

MATTER, FORCE, WEIGHT.


Definition of terms. Applied mechanics treats of those laws
and the effects of force upon matter which apply to works
of force
of human art. As science stands at present, it is impossible to
state exactly what force and matter really are, and we are com-
pelled to explainthem by reference to some of their properties.
Matter is anything which our senses tell us exists. Matter
exists in many different forms, and can often be changed from
one form to another, but man cannot create it, nor can he
annihilate it. Matter always occupies space, and a given piece
of matter, occupying a definite space, is called a body.
Force may exert push or pull on a body, or may set it in
motion, or bring it to rest. The most familiar conception we
have of force is obtained from the manner in which our muscles
must be exerted when we support a body.
All bodies are measured, as regards the quantity of matter,
or mass, they contain, by comparison with a standard body.
The standard for this country is the quantity of matter con-
tained in a lump of platinum preserved in the Exchequer Office.
This quantity of matter is called one pound. In countries
using the metric system the standard mass is the gram, and
is the quantity of matter contained in a cubic centimetre of

water at the temperature of 4 C.


When two given bodies, of different materials but having the
same volume, are found to contain differing quantities of
matter, that which contains the greater quantity is said to be
more dense than the other. The density of a material is stated
by the quantity of matter, or mass, of a cubic unit of it. Thus,
the density of water is about 62 "3, there being 62*3 lbs. mass in
one cubic foot of water wrought iron has a density of about 480.
;
14 APPLIED MECHANICS FOR BEGINNERS.

Specific density is the density of a substance when compared


with that of a standard, water being taken for this purpose.
Thus, the specific density of water being 1, the specific density
of wrought iron is 7 '7.
The most familiar force we have is weight, which all bodies
possess. Weight is due to gravitation, which is manifest in the
attraction which all bodies have for one another. Gravitational
effort is proportional to theproduct of the masses of the two
bodies and inversely proportional to the square of the distance
between them. It is very small for bodies of ordinary size, but
is perfectly evident when one, or both bodies, possesses a large

quantity of matter. Thus, for bodies near the surface of the


earth, the is seen by what we call the
gravitational effort
weight of the body. Weight means the tendency towards the
earth's centre possessed by all bodies.
The weight of a body may vary. Weight, as we have seen
above, is proportional to the product of the masses of the earth
and of the body. Assuming these to be constant, a given body
will always have the same weight at the same place on the
earth. Weight, however, is inversely proportional to the
square of the distance from the earth's centre to the centre of
the body, and therefore any change in this distance will produce
a change in the body's weight. Thus, the weight of a given
body is slightly less at the top of a mountain than at sea level.
The earth is not perfectly spherical, but is flattened towards
the poles. Consequently a body at sea level near the poles will
be nearer to the earth's centre than one at sea level near the
equator. Therefore a body near the poles has a greater weight
than it would possess near the equator.
The effect caused by the whirling
of the earth on its axis also makes
the apparent weight of a body near
the poles slightly greater than it
would possess near the equator.
Measurement of mass. Quan-
Fig. 19. Ordinary balance, tities of matter can be measured by
comparing their weights, using an
ordinary balance for this purpose. The balance beam will become
horizontal when equal vertical forces acting downwards are
MATTER, FORCE, WEIGHT. Ifi

applied to its ends A and B (Fig. 19). These forces are produced
by the weights of the bodies placed in the pans, and when the
weights, as shown by the beam, are equal, we have equal masses
in the pans. Using a standard lb. mass in the pan C, we can
obtain another lb. mass by this means in pan D, and this can be
done at any place without variation in the mass measured, as
equal masses have equal weights when both are at the same place.
Spring balances (Fig. 20), which measure force applied to
them by the extensions of a spring, show the actual weight of
bodies placed in their pans. These appliances,
therefore, will indicate different readings with the r/^\
same body placed in the pan at different places ^\
on the earth's surface. Thus, it can be shown that
a body, the weight of which is 32,088 lbs. at the
equator, will have a weight of 32,252 lbs. at the
poles. Engineers use as their unit of force,
in most cases, the weight of the standard lb.
mass.
This, as we have seen, is indefinite unless we also
state the place where the mass has to be weighed.
Thus, if we
say, the weight of the lb. mass at sea
level at Greenwich, we have a perfectly definite
force and one which is used by many people.
It will be observed from the above figures that
the alteration in the weight of a body by trans-
ference from the equator to the poles is too small
to affect engineering work, being about 0*5 per cent.
Fig. 20. Spring
It is, therefore, generally neglected in engineering balance.
calculations, although this is no reason why the
student should be ignorant of the fact that such alteration exists.
Specific gravity. Specific gravity is the weight of a given
volume of a substance when compared with the weight of an
equal volume of water. It is usual in engineering work to
measure specific gravities at a temperature of 60 F. The
specific gravity of water being 1, the specific gravity of wrought
iron would be 7*7, and of lead 11*4. It will be noticed that the
number giving the specific gravity of a body will be the same
as that giving its specific density. Specific gravity, however,
refers to weight, and specific density to quantity of matter.
16 APPLIED MECHANICS FOR BEGINNERS.

Some important relations. The quantity of matter in, or


mass of, a given body may be calculated thus :

Let F=its volume in cubic feet,


8 = its specific density.
Then Fx 62*3 would be its mass if it were water,
and Vx 62-3 x 8 will be its actual mass, in lbs.
The weight of a body may be calculated thus :

Let F=its volume in cubic feet,


p = its specific gravity.
Then Vx 62'3 = its weight if it were water,
and Vx 62*3 x = its
/o actual weight, in lbs.
The specific gravity of a body can be found roughly by first
weighing it and then measuring it and calculating its volume

from the dimensions.


Suppose F=its volume in cubic feet.
IF=its weight in lbs.
Then W=
Vx 62-3 x p,
W
Example. A piece of flat bar iron, 12" long, section 2"x" is

found to weigh 3 '38 lbs. Find its specific gravity.

Here V= ^
12x2x4*

3-38x1728
cubic feet.

. p-
12 x 2 x x 62-3
3-38x144 _
62-3


Practical Applications. An important part of the routine
work of the engineer is the calculation from its drawings of the
weights of various parts of a structure or machine. This he does
by first calculating the volume of the part either in cubic inches
or cubic feet and then multiplying this volume by the weight of
the material per cubic inch or per cubic foot. Or, he may proceed,
after having obtained the volume in cubic feet, to multiply this
by 62*5,* which gives the weight of the part if made of water,
62*5 lbs. being the weight of 1 cub. ft. of water. If this result
be now multiplied by the specific gravity of the material, the
*62"3 more nearly, but 62*5 is near enough for almost all engineering
purposes.
MATTER, FORCE, WEIGHT. 17

result will be the weight of the part. This procedure has to be


followed out, especially in finding the weights of castings.
In finding the weights of plates, it is advantageous, if much
work has to be done, to tabulate for reference the weights of
plates one inch thick, of the substances used per square foot
superficial area. If the actual area of the plate in square feet
be now calculated, this, multiplied by the tabular number and
the thickness of the plate in inches, gives the total weight.
For bar iron and rolled sections of different materials, the
weight of each shape and size of section per foot running length
is tabulated the actual length, in feet, of stuff used multiplied
;

by the tabular number will give its weight.


In estimating the weights of castings, it is customary to divide
up the drawing of the casting into numerous parts, so chosen
as to simplify the necessary mensuration work of finding the
volume. Fillets, small bosses, etc., are omitted in this cutting
up and allowed for afterwards. In structural work where the
parts are riveted together or secured by bolts or pins, the heads
of rivets, bolts and pins and nuts are allowed for separately.
The following table gives the weights and specific gravities of
some common substances :

Weights and Specific Gravities.

Weight of Weight of a sheet


Specific
Material. 1" thick,
Gravity.
One cub. foot. One cub. inch. 1 sq. foot area.

lbs. lb. lbs.


Wrought iron, 480 0-28 40 7-7
Steel, 490 0-28 41 7-8
Cast iron, 450 0-26 37* 7-2
Copper, - 550 032 46 8-8
Brass, - 525 30 44 8-6
Gun metal, - 540 0-31 45 8-7
Aluminium, 165 095 14 2 6
m
-

Zinc, 450 0'26 7 2


Tin, 465 27 39 7-4
Lead, 710 041 59 11-4
Fresh water, -
62 5 036 10
Sea water, 64 037 1 024

A.M.
IS APPLIED MECHANICS FOR BEGINNERS.

A few examples are appended to show some of the methods


adopted.

Example 1. A cast-iron pipe 3" bore, 6 ft. long, metal of body

h" thick, circular flange at each end 7V diam. x f " thick (Fig. 21).
Calculate its weight.

*>-.
&
-U
Fig. 21.

Remove the flanges and calculate their volume separately from


the volume of the body of the pipe.
Volume of body of pipe = cross sectional area x length
= tt(2 2 -H 2 )x72
= 396 cubic inches.
Volume of each flange = tt(3| 2 - 22 x f )

= 23*7 cubic inches.


Total volume of metal = 396 + 23*7 + 23 7
= 443*4 cubic inches.
Now, cast iron weighs 0*26 lb. per cubic inch ;

.'. weight of pipe = 443 -4 x 0*26


= 115-3 lbs.

f Example 2. A wrought-iron plate,


"^ V thick, originally square, has a
_^ piece cut out of the corner as shown
(Fig. 22). Calculate its weight.

(i) Area of original square


= 4x4=16 square feet.

(ii) Area of circle 1 ft. diam.


= 0*7854 square foot.

P 1G 97.
(iii) Area of square 1 ft. side
= 1 *0 square foot.
Difference between (iii) and (ii) = 0*2146 square foot.
; ;

MATTER, FORCE, WEIGHT. 19

This difference is made up of the four shaded pieces, and three of


these are cut out together with the circle 1 ft. diam.

Area of piece cut out = 0'7854 + (f x 0*2146)


= 0*9463 square foot
.'. Area of actual plate = 16 - 9463
= 15*054 square feet.
Now, wrought iron weighs 40 lbs. per superficial foot if the plate
is 1" thick
.'. weight of plate = 15 '054 x 40 x \
= 301 lbs.

Example 3. A copper float ball 14" diam. is made of metal jjf


thick. Calculate its weight.
In this case, as the metal is thin compared with the diameter of
the ball, we may volume
of metal near enough for practical
find the
purposes by multiplying the spherical area by the thickness of metal.
To be quite exact we should have to calculate the volume of metal
by taking the volume of a sphere 13g" diam. from the volume of a
sphere 14" diam.
.Spherical area = 47rr 2
= (4 x %?- x 7 x 7) square inches.
Approximate volume of metal = (4 x 22 x 7 x -jV) cubic inches,
and as copper weighs 0*32 lb. per cubic inch,
weight of ball = 4 x 22 x 7 x TV x 0*32
= 12-3 lbs.

EXERCISES ON CHAP. II.

1. Find the weight of a piece of flat bar iron 24" long, section
2" x V'.
2. Find the weight of a wrought-iron bar, . 3' *i

13-16" long, section 1 -5" x 4".

3. A
piece of angle iron, section as in
Fig. 23, is 30 ft. long. Calculate its weight,
neglecting rounded corners.
4. A circular brass plate, 2 ft. diam. , is J"
*
thick. Calculate its weight.
5. A
hollow cylinder of wrought iron is 4"
inside diameter, 4g" outside diameter and fig. 23.
10 feet long. Calculate its weight.
6. A solid pyramid of lead, square base 4" edge, 8" high. Find
its weight.
20 APPLIED MECHANICS FOR BEGINNERS.

7. A copper cone, 10" diam. of base, 12" high, metal 0*05" thick,
no bottom. Find its weight.

8. A hollow conical vessel, 6" inside diam. at top, 9" deep inside,
is full of water. Calculate the weight
of water.

9. A cast-iron bracket, metal all over


i" thick, has dimensions as shown in
Fig. 24. Find its weight.
10. A Lancashire boiler is 7 ft. diam.
and 30 ft. long. The two
internal
tubes are each 3 ft. diam. Plates of
Fig. 24. cylindrical outside portion g" thick.
End plates f" thick. Internal tubes
TV' thick. Neglect lapping at joints, rivets, etc., and find weight
of material if of wrought iron throughout.

11. Calculate what weight of sheet lead, O'l" thick, will be required
for lining a timber tank, the internal dimensions of which are 6 ft.
long, 4 ft. broad, 3 ft. deep.

12. A solid ball of cast iron is to have a weight of 90 lbs. Calcu-


late its diameter.
CHAPTER III.

TWO AND THREE FORCES ACTING AT A POINT.

Representation of a force. To describe completely a force


acting on a body we require to state the following particulars,
(a) its magnitude, (b) its point of application, (c) its direction,

(d) its sense, i.e. to state whether the force is pushing or pulling
at the point of application.
A straight line may be employed to represent a given force,
for it may be drawn of any length and so represent to a given
scale the magnitude of the force, the end of the line shows the
point of application, the direction of the line gives the direction,
and an arrow point on the line will indicate the sense of the
force.
Thus, a pull of 5 lbs. acting at a point in a body (Fig. 25)
at 45 to the horizontal, would be completely .

represented by the line OA and arrow point / h

/f
as shown.
We often speak, as above, of a force "acting
at a point." Of course this must not be (Z/\ &
understood literally, for no material is so / o""j
J hard that it would not be r
very penetrated
, .. . .. , ., Fig. 25. Graphical re-
. .
by even a very small fore* applied to it at presentation of a force,
a mathematical point. What is meant by
'

the statement, is that the force may be imagined to be con-


centrated at the point in question without thereby affecting
the condition of the body as a whole.
Forces acting in the same straight line. A body is said
to be in equilibrium if the forces acting on it balance one
22 APPLIED MECHANICS FOR BEGINNERS.

another. Thus, two equal opposite pulls P, P (Fig. 26) be


if

applied at a point in a body both in the same straight line,


they will evidently balance one another and the body will be in
equilibrium.

Fig. 26. Fig. 27.Tie bar under pull.

A
tie bar subjected to two equal opposite pulls of 2000 lbs.
each (Fig. 27) acting in the direction of its length will be in
equilibrium. If one only of these pulls were
reduced or increased even by very little the
bar would move. This bar could not possibly
be imagined pulled with a force of 2000 lbs.
at one end only and yet to remain at rest, any
more than a pull of 5 lbs. could be applied by
the hand to one end of a piece of string unless
the other end were pulled with a force of 5 lbs.
in the opposite direction.
In the same way, if a column or strut (Fig. 28)
be pushed at one end and remain in equilibrium,
there must be an equal opposite push acting
in the same straight line at the other end.
It is impossible for a single force to act
alone. To every foree there must be an equal
20001BS.
opposite force, or what" is exactly equivalent
to an equal opposite force. This- equal opposite
Fig. 28. Column force is often called a reaction.
under push.
If several forces in the same straight line
act at a point, the point will be in equilibrium if the sum of
the forces of one sense is equal to the sum of those of opposite
TWO AND THREE FORCES ACTING AT A POINT. 23

sense. sums are not equal, then a force is required to


If these
balance the point, and its magnitude will be equal to the differ-

ence of these sums and the force must have the same sense as
the smaller sum. In the given case (Fig. 29) these sums are
2 + 3 + 5 = 10 lbs., sense from A to B ;

^ 8 + 1=9 lbs., sense from B to A.


And a force of (10-9) = 1 lb. of a sense B to A will produce
equilibrium.
Two intersecting forces. If two forces are given acting at
a point, their lines of direction intersecting, a single force may

"Q e
Fig. 29. Fio. 30 Parallelogram of forces.
be found which would have the same effect, if applied alone, as
the two forces together have. This single force is called the
Resultant of the given forces, and may be found by the following
construction.
Let P and Q be two pulls applied to a nail at (Fig. 30) ;

their joint tendency will be to carry the nail upwards to the


right. Set off 0A, to some suitable scale, equal to P, and OB,
to the same scale, equal to Q. Complete the parallelogram
OA CB and draw its diagonal OC. Measure off OC to the same
scale of force and this will give the magnitude of It. If a pull
R be now applied to the nail along the line OC, it will have
the same effect as and P
Q together have. This
method is called the
Parallelogram of Forces ;

P and Q are called Com-


ponents of It.

Let us now remove the


"
forces Pand&and instead Fm ^_ E&nd R balance .

apply It alone to the nail.


We may balance R by applying another pull E, equal and
opposite to R and in the same straight line (Fig. 31), and
24 APPLIED MECHANICS FOR BEGINNERS.

if we do so, there will now be no tendency to disturb the


nail. But since R is exactly equivalent to P and Q together, we
may replace R again
by P and Q (Fig. 32),
thereby giving three
pulls on the nail which
will balance one an-
other without any ten-
dency to disturb the
Fig. 32. P, Q, and E balance. position of the nail
in the board. E is

generally called the Equilibrant, meaning the force required to


keep the other forces in equilibrium.

Experimental verification. The most satisfactory way for
the beginner to verify the truth of the above principle is for
him to make an experiment illustrating it.


Expt. Procure three wooden pulleys about 2" or 3" diameter,
having their edges grooved to receive string, and with holes
through their centres
so that they will run
freely on bradawls.
Pin a sheet of paper
to a vertical board and
mount two pulleys at
.4 and B by means
of bradawls (Fig. 33).
Tie two strings to a
small split key ring,
pass a bradawl through
the ring into the board
at 0, and lead the
strings over the pulleys
at A and B. Fasten
any bodies of known
.V^^^^iOosr^otic^ weigntg Wij ^ t0 theW
ends of the strings.
We have now two forces W x and W2 acting on the bradawl
at along the strings OA and OB. Mark the directions
TWO AND THREE FORCES ACTING AT A POINT. 25

of these strings carefully on the paper and remove them to


construct the parallelogram of forces Oabc. R, the resultant
of W
1
and W 2 acting at 0, will thus be found. Produce bO and
replace the strings. By means of another pulley and bradawl
at C, arrange a third string tied to the ring to lie along bO pro-
duced. Tie a body of weight W 3 to the end of this string, W3
being equal to R This will give a third force
in magnitude.
E= W 3
= R acting at 0. If what has been done is correct, the
three forces v W W
2 and , E
should now balance one another, and
if they do, we should be able to remove the bradawl at without
the ring changing its position. Try if this is so.
You will probably notice that after the bradawl is removed
from the ring can be made to take up positions some little
distance from 0. This is due to the stiffness of the strings and
the friction of the pulleys on the bradawls, and these causes
prevent the perfect success of the experiment when regarded as
a means of testing the truth of the parallelogram of forces.

Fig. 34. Students' apparatus for experiments on forces.

Fig. 34 is reproduced from a photograph of an apparatus


arranged for students' use. The pulleys used are of aluminium,
with pivot bearings, and may be clamped to any part of the
edge of the board. The apparatus may be used also for testing
several of the following principles concerning forces. The
student should not forget in using it, that scale pans possess
weight, and that such weights should be added to those put into
26 APPLIED MECHANICS FOR BEGINNERS.

them to get the total pull in the cords. Several experiments


should be made by each student, using different weights and
pulley positions each time.
Notice that before attempting to apply the parallelogram
of forces to find the resultant of two forces, both given forces
must be made to act either towards or from the point of appli-
cation. Thus, given P' pushing and Q pulling at (Fig. 35),

Fig. 35.

the tendency will be to carry downwards to the right. Sub-


stitute P=P\ pulling at 0, for P'and take OA to scale to
represent P, also OB to represent Q. The parallelogram OA CB
may now be drawn, giving R, the resultant of P' and Q.
Substitution of components for resultant. Since the re-
sultant produces exactly the same effect as its components, we
may use either resultant or components in working out a ques-
tion. It is often convenient to substitute for a given force its

components along two given lines, which are usually taken


perpendicular to one another. Thus, if we are given P acting
at 0, and it would be more convenient instead of P to have
forces in OA and OB (Fig. 36), then, by setting off OC=P and
completing the parallelogram
OBCA, two forces S=OB
and T= OA are found, which
if substituted for P, would
have the same effect on 0.

Example 1. Two forces of


4 lbs. and 9 lbs. pull a nail
in directions at right angles
to one another. Find their resultant.
Draw OA and OB to scale (Fig. 37) to represent the given forces.
Complete the parallelogram ; its diagonal 00 will give B.
;

TWO AND THREE FORCES ACTING AT A POINT. 27

H may also be found by calculation. Thus


OC*=OB*+BO*
= OB2 + OA*=92 + 4?;
.-. OC =\/81 + 16=\/97,
or E = 9'8 lbs.
In solving questions on forces by construction, take care to
use a large scale. By doing so you will secure much more
accurate results. Thus, in the above case a scale of \" to a lb.
would be suitable.

Example 2. A horse exerts a pull of 200 lbs. on a tram car at 30


to the direction of the rails as seen in plan. Find the force urging
the tram forward, and that
tending to pull it off the
rails.

Set off OA (Fig. 38), to scale


to represent 200 lbs. acting at
30 to OB, the direction of the Fig. 38.
rails. Draw OC perpendicular
to OB. Complete the parallelogram OB AC ; then OB, to scale,
gives the force P, tending to urge the tram along the rails, and OC
gives Q, the force tending to pull it off.
By calculation, since in the triaugle OAB, the angle AOB is 30
and A BO is 90, the sides have the following proportion :

BA :A0:0B=1:2:>J3,
or OB:AO:OB=l:2:>JS;
:. Q :200:P=1 2 >/3: :

/. Q = x 200= 100 lbs,


and P = 1 00 x *J% = 1732 lbs.
Example 3. A load of 14 lbs. is hung by a cord 10 ft. long from
an overhead beam, the arrangement being that of a pendulum. Find
what horizontal force applied to the load will keep it 2 ft. from the
vertical through the point of support.
the figure to scale, as shown at OA B (Fig. 39).
Draw Let P be
the required force ; then the three forces acting on the load,
P, W, and the pull of the cord, T, keep it balanced, so that T must
be equal and opposite to the resultant of and W. Set off to P AC
28 APPLIED MECHANICS FOR BEGINNERS.

a suitable scale of force to represent 14 lbs.,and complete the


parallelogram AGDE. T will be represented by AD, and P by
AE, which being measured
gives P= 2 -86 lbs.
Expt. Test your con-
struction by actually
arranging cord anda
weight. Put a spring
balance in the cord and
apply P by means of
another spring balance
(Fig. 39). P and T can
now be measured directly.
If readings of P are taken
when W is at different
distances from the verti-

Fig. 39. Forces acting on a pendulum.


cal, a curve may be
plotted on squared paper
which will show the values of P for all positions of W. The
results for intervals of \ ft. are given in the table, the curve
shown (Fig. 40) being plotted by using the values of AB for
abscissae or horizontal distances, and the corresponding values
of P for ordinates or vertical distances.

/ AB P

/ ft. lbs.
/ 0-5 07
1-0 1-41

/
f
lo
20
2-12
2-86

/
/
2o
3
3 5
3 6
4-4
5 2
/ 4-0 6-1

/
Fig. 40. Plotted curve of force required to
keep the pendulum out of the vertical.
TWO AND THREE FORCES ACTING AT A POINT. 29

Example 4. The forces in the parts of a simple roof truss may be


easily found by applying the parallelogram of forces.
Thus, letAB and AC be the rafters and BC the tie bar (Fig. 41).
If a load W is applied at the top, the pushes in BA and CA may be
found at once by consider-
ing these as the components
of W. Thus, aA and cA
are the pushes in BA and
CA respectively. Now, if

the bar BA is exerting a


push a A at A, it must be
exerting an equal contrary
push at B, and it may be
Fio. 41. Forces in the parts of a simple roof
assumed that the wall is truss.
pushing vertically upwards
to support the truss at B, with a force, say, P ; this is called
the reaction of the support. There are, therefore, three forces
in equilibrium at B, viz., the push from AB, P, and a force in
the tie bar. Take this last as being equal and opposite t.o the
resultant of the other two, and find the resultant by setting off
dB = aA and completing the parallelogram dBef. Bf will give
the force in the tie bar BC and eB will give the reaction of the
support. In the same way AC is pushing at C with a force
equal and opposite to cA hence by the same method find the
;

reaction Q. It will be found


that Ch = Bf, showing that
the tie bar is pulling equally
at both ends, as we know
must be the case, and there-
fore if Ch and Bf be not
equal we know that some-
thing is wrong with our
work.

Fig. 42 shows a suitable


apparatus for the student
to use in order to test the
42. Experimental model of simple
above constructions. It
roof truss.
consists of two wooden
bars, connected loosely at the top by means of a bolt. The
lower end of one bar is pivoted to a bracket secured to the base
30 APPLIED MECHANICS FOR BEGINNERS.

board, and the lower end of the other is mounted on a roller to


minimise friction. A cord, with a spring balance, serves for a
tie, and a compression spring balance, consisting of two brass
tubes sliding one in the other, with a spring inserted, forms
part of each inclined bar. The forces in each rafter, and in the
tie, can be read from these three balances.

EXERCISES ON CHAP. III.

1. Represent graphically a pull of 10 lbs. acting at a point, its


direction being N. E. Scale " to a lb.
2. Represent graphically two pulls acting at a point, one of 5 lbs.,
direction S.W.; one of 8 lbs., direction E. Find their resultant.
Scale |" to a lb.
3. Represent graphically a push of 7 lbs. acting at a point,
direction N. ; also a pull of 4 lbs. acting at the same point, direction
S.E. Find their resultant. Scale ^" to a lb.
4. A pull of 25 lbs. and a push of 54 lbs. act at a point along the
same straight line in opposition to one another. Represent them
graphically, and find their resultant.
5. Draw a horizontal line, and mark a point in it near its
centre. Pulls of 2 lbs., 5 lbs., and 9 lbs. act at in the right-hand
portion of the line, and pulls of 4 lbs. and 6 lbs. together with
pushes of 3 lbs. and 12 lbs. in the left-hand portion. Find the
equilibrant.
6. Two pulls of 6 lbs. and 10 lbs. act on a point (a) at 90, (b) at
120, (c) at 60. Find their resultant in each case both by construc-
tion and calculation.
7. A push of 20lbs. and a pull of 30 lbs. act at the same point,
their lines making 40 with each other. Find their resultant.
8. The resultant of two forces whose lines are perpendicular to
one another is 15 lbs. One isa force of 4 lbs. Find the other.
9. A force of 100 lbs., acting in a horizontal line has to be
balanced by two forces, one of 50 lbs. and the other of 120 lbs.
Show their lines of action.
10. Three cords are attached to a ring one cord carries a weight
;

of 10 lbs.and hangs vertical. The other cords are attached to an


overhead beam and are inclined one at 45 and one at 60 to the
vertical. Find the pull in each.
11. An overhead pulley has a chain passing over it from a winch,
and a load of 5 cwts. is being hoisted. The chain carrying the load
hangs vertical and the chain leading to the winch makes 30 with
the vertical. Suppose the force in each part of the chain to be
5 cwts., and find the resultant force on the pulley.
TWO AND THREE FORCES ACTING AT A POINT. 31

12. Prove by diagrams and by experiment that the force required


to balance two given equal and opposing forces becomes smaller as
the given forces approach, being finally nearly in the same straight
line.

13. A
barge is pulled along the centre of a canal 60 ft. wide by a
horse on the tow-path whose centre is 4 ft. from the bank. The
horse pulls the rope, which is 80 ft. long, with a force of 120 lbs.
Find, by construction, the force urging the barge along the canal
and the force urging it towards the bank.
14. A horse draws a load up an incline of 1 in 20. The traces
are inclined at 30 to the horizontal and the pull of the horse on
them is 180 lbs. Find by construction the backward pull on the
horse taken parallel to the incline and the downward pull on the
horse taken at 90 to the incline.
15. A
man pulls a nail by means of a string in a direction at 30
to the board. If he exerts a force of 20 lbs., calculate the force
tending to draw the nail and that tending to bend it.

CHAPTER IV.

TRIANGLE AND POLYGON OF FORCES. SIMPLE


STRUCTURES.

Triangle of forces. Let us now consider what conditions


must be fulfilled in order that three forces acting at the
same point all in the same
plane may balance one an-
other.
It has been seen already
that if three forces, such as
P, Q, and S, act at a point
(Fig. 43), one of them must
be equal and opposite to the
resultant of the other two.
Find, by the parallelogram
&
Fig. 43. P, Q and S balance. . . A
of forces, R, the resultant or
,, .

P and Q, then S must be equal and opposite to R.


This proportion will evidently be true :

Q:P:R=OB:OA : CO.
Now R = S and OA = BC ;

Q:P:S=OB:BC:CO,
.-.

that is, the three given forces are' proportional to the sides of
the triangle OBC.
The equilibrium of P, Q, and S, may therefore be tested by
seeing whether a triangle can be drawn with sides proportional
to these forces. Thus, in Fig. 44, Ob is parallel and proportional
to Q, be to P, and cO to S. If the lines so drawn give a closed
triangle, then the given forces will be in equilibrium. This
TRIANGLE AND POLYGON OF FORCES. 33

triangle Obc is called the triangle of forces for the given forces

P, Q, and S.
Notice in drawing the triangle of forces that the sides must be
drawn in the proper order to represent the
sense of the forces. Thus, Ob is drawn to
the right to indicate that Q acts to the
right, be upwards as P acts upwards, and
cO down to the left to indicate the sense
of S. So long as attention is paid to Q *

this the triangle of forces may be begun Fig. 44. Triangle of


forces for P, Q and S.
with a line parallel and proportional to
any one of the given forces. The student should test this fact
for himself by actual construction.
Resultant of several forces.- -By means of the parallelogram
of forces, the resultant may be found of any number of forces
acting at a point, all being

in one plane. Thus, given


P, Q, S, T, acting at (Fig.
45), in the plane of the
paper ; to find their re-
sultant R, first find the
resultant of any pair, such
as Pand S, by the parallelo-
gram. Call this v Then, R
take the other pair, Q and
T, and find R^ their re-
sultant. Finally, find the Fig. 45. Resultant of several forces by the
parallelogram of forces.
resultant R, of R x
and R 2
.

This will clearly be the resultant of the given forces.


If we apply E, equal and opposite to R, R would be balanced
at 0, and therefore, of course, the given forces would also
be balanced by K So that by this method we may also find
the equilibrant of a given number of forces.

Expt. Try one or two examples of this on your experimental


board. Use attached to a ring and led over
five or six cords
pulleys, provided with scale pans.
their ends being Have
different weights in the pans and fix the ring by means of a
bradawl at a given position on the board. Find the resultant
A.M. b. c
34 APPLIED MECHANICS FOR BEGINNERS.

of these forces, and by means of another cord and pulley apply


a force equal and opposite to R to the ring. Now remove the
bradawl and see if the ring remains in equilibrium in its original
position.

Polygon of forces. By means of an extension of the triangle


of forces, we may ascertain whether a given number of forces
acting at a point, all in the same plane, are in equilibrium.
Thus, given P, Q, S, T, acting at (Fig. 46), all being in the
plane of the paper. Draw AB (Fig. 47) parallel and propor-

Fig. 46. Fig. 47. Polygon of forces.


tional to P, and BC in the same way to represent (J. Then, by
the triangle of forces, CA will give a force r, which, if applied as
a push downwards to 0, will balance P and Q. Reverse it as
shown at / (Fig. 46), and / will be the resultant of P and Q.
Remove P and Q and let / be applied instead.
Now, taking / and S AC (Fig. 47) represents /, draw CD to
;

represent S, and, by the triangle of forces, DA will give a force r t

which if applied at (Fig. 46) will balance r' and S. It there-


fore follows that P, Q, and S, will be balanced by the application
of rx to 0, and if r x is reversed, as shown by r\, then r\ will be
the resultant of P, Q, and S. Now if T, the last of the given
forces, is found to be equal and opposite to r\, that is, if it is
represented by the line DA in Fig. 47, T and r\ will balance
one another, and if this is the case, the given forces P, Q, S, and
T must be in equilibrium.
TRIANGLE AND POLYGON OF FORCES. 35

The test of equilibrium therefore is Can a closed polygon be


:

drawn with its sides parallel and proportional to the given


forces respectively, taken as before in proper order? If so,
then the forces are in equilibrium.
This polygon is called the polygon of forces for the giv%n
forces. It may be used to find the resultant of a number of

Fio. 48.

forces acting at a point, in one plane, which are not in equi-


librium.
Thus, if on drawing the polygon of forces for the given forces
P, Q, S, T (Fig. 48), as at ABCDE (Fig. 49), it is found that the

Fio. 50.

Fio. 51. Fio. 51a. An experiment on the polygon


of forces.

polygon does not close, then the closing line EA will give a
force Eq, which, if applied as a pull at in the sense from
36 APPLIED MECHANICS FOR BEGINNERS.

E to A, will balance the given forces, and will therefore if


reversed, as at R, be their resultant.
Notice that it need not concern us if the forces are not all
pushes or all pulls, provided we take them in their proper
order. Figs. 50 and 51 show a case of this, and in Fig. 51a an
example is shown experimentally worked out.
The student should work out some similar cases for himself,
and verify the fact that it does not affect the result which force
is taken first in drawing the polygon, or the numerical order in

which they are taken, provided their senses are properly


considered.
Experimental models of simple structures. A model
derrick crane is shown in Fig. 52 having arrangements provided

for experimentally finding


the forces in its parts. It
consists of an upright post
firmly secured to a base
board, an inclined jib, with
a compression spring bal-
ance fitted to it, and a cord
to serve as a tie and having
an ordinary spring balance
to indicate the pull. The
jib can be fitted with a
pulley at the end for the
cord supporting the weight
to run over, or it may be
used without the pulley,
-Experimental model of a derrick
fche supportillg cord be i ng

then simply secured to its


upper end. The method of using the model is as follows.

Expt. Place a known weight in the scale pan and then
measure the height of the post from the junction of the jib to
the junction of the tie, the length of the jib and the length of
the tie. From these dimensions make an outline diagram of
the crane and show the This is
verfical line of the weight.
shown at ABC (Fig. 53). If W
simply hung from A, then by
is

the parallelogram of forces ADEF, AD= W


being first set off,
the pull T of the tie and the thrust Q of the jib will be found.
TRIANGLE AND POLYGON OF FORCES. 37

Or, the triangle of forces, as at abc (Fig. 54), may be used. If


the pulley is fitted to the jib end, and the supporting cord

Fin. 53. Solution of the derrick crane Fig. 54. Solution of the derrick crane
by the parallelogram of forces. by the triangle of forces.

passed over it and led to a point D on the post (Fig. 55), this

will give an additional force W


which will be nearly equal to
W, the stiffness of the cord and the friction of the pulley making
it slightly different. Taking W'= IP, the polygon of forces will
give T and Q. This is shown at abed (Fig. 56). In either case,

Fio. 55.
Derrick crane, the chain Fig. 56. Solution by the polygon
passing to D on the post. of forces.

after the force diagram is drawn and Q and T determined


thereby, the spring balances should be read and compared with
the scaled results. Generally they will read more than those
found by construction, because the weight of the parts has been
neglected and these will cause forces in the tie and jib when W
is removed from the pan. Take W away and read the balances
again, these readings, subtracted from the previous ones, should
give results agreeing very closely with the scaled ones.
A wall crane (Fig. 57) can easily be arranged, using the jib
from the derrick crane. A B is the jib, arranged horizontally,
3S APPLIED MECHANICS FOR BEGINNERS.

BC an inclined tie, fitted with a spring balance. The cord


suspending W
may either be secured to B or passed over a
pulley there and led to any point D between A and C. The
method of using this model and the construction for the results
are precisely similar to those for the derrick crane.
Sheer legs are used for moving very heavy weights, such
as for placing engine parts and the boilers into position on
board a vessel. A model sheer legs is shown in Fig. 58. It

Fig. 57 Experimental model of Fig. 58. Experimental model of a sheer


a wall crane. legs.

consists oftwo jibs hinged together at the top, and each fitted
with a compression spring balance. A
tie, with spring balance,

passes from the top of the legs to the rear of the base board.
The bottom ends of the legs bear in notches cut in the base
board, there being a number of these, so that the legs may be
spread out more or be brought closer together. The cord sup-
porting the weight may be attached to the top of the legs or
passed over a pulley there and brought down to some point on
the base board.
Examining the forces acting at the top of the legs, we see that
in this case these are not all in thesame plane. If, however,
the resultant push of the two legs combined is known, this
resultant would fall in the same plane as the other forces, and
the solution would then be exactly the same as for the derrick
TRIANGLE AND POLYGON OF FORCES.

crane. The method of carrying out an experiment is as


follows.

Expt. Put a known weight in the scale pan, measure all the
dimensions required in order to make an outline diagram of the
model, and show the line of W
on the diagram (Fig. 59). Draw
in addition to the outline plan and elevation of the model, a
view showing the true shape of the legs. Taking the directions
from the elevation, and setting off ab = W
and be = W'= W, first,
the polygon of forces may be drawn (Fig. 59), giving cd=R

Fio. 59. Solution of the forces in the sheer legs.

and da=T. Setting off efR midway between the legs as


shown in the view giving their true shape, draw the parallelo-
gram of forces ehfg, and the thrusts QQ of each leg will be
found.
This construction gives all the required forces, which, when
measured to scale, should be found to agree closely with those
shown by the spring balances, after allowance has been made
for the effects of the weights of the parts, in the same manner
t for the derrick crane.
40 APPLIED MECHANICS FOR BEGINNERS.

Inclined Plane. A model of a small carriage resting

Fig. 00. Model of an inclined plane.

on an inclined plane is shown in Fig. 60. The forces re-


quired to support it if the plane were removed are shown
experimentally by the pulls of the two
cords, one arranged parallel to the plane
and the other at 90 to it. These, with
the weight of the carriage, give three
forces acting on it and keeping it in
equilibrium. Measure the height and
length of the plane and make an outline
diagram as shown at ABC (Fig. 61).
Find these forces by construction, using
the triangle of forces abc. Experiments
should be made and force diagrams
Fig. 61. Equilibrium drawn when P is acting parallel to the
of a carriage on an
inclined plane. base and also when P is applied at any
angle to the plane.

EXERCISES ON CHAP. IV.


AB and AC are two scaffold poles in the same vertical plane,
1.
lashed together at their tops. AB is 20 feet and AC 15 feet long.
The distance BC between their feet is 15 feet. Find by construction
the push in each pole when a load of 1 ton is hung from the top.
:

TRIANGLE AND POLYGON OF FORCES. 41

2. The jib of a model derrick crane is 47" long, the tie 38", and
the post 31". Find by construction the push in the jib and the pull
in the tie when a load of 4'7 lbs. is simply hung from the end of
the jib.
3. A crane jib measures 19 ft., the tie 17 ft., and the post 9 ft.
A load of 50 cwts. is attached to a chain which passes over a single
pulley at the top of the jib, then along the tie. Find the push in
the jib and the pull in the tie by construction.
4. Answer Question 3 supposing the chain, after leaving the
pulley at the top of the jib, to pass along the jib.
5. A a hinge fixed to a vertical wall 6 ft. vertically over
is
another, B. A
triangular frame ABC, AC=8 ft., BC=\0 ft, is
attached to A and B, the arrangement forming a wall crane. A
load of \ ton is attached to a chain which passes over a pulley at C,
then along CA to a winding arrangement on the other side of the
wall. Find by construction the forces in AC
and BC, indicating
whether they are push or pull.
6. In Question 5, turn the frame upside down and answer the
same.
7. Draw to scale a frame A BCD from these particulars
45=4 ft., AD = 4it., BC=oft., DC=6ft.
Diagonal bar = 5 ft. BD
The frame attached to a vertical wall at A and D, A being
is
uppermost. Find by construction the forces in ail the bars, marking
push or pull, when a load of 2 tons is hung from C.
8. A
boiler weighing 25 tons is placed on board a ship by means
of a pair of sheer legs. The bottom pivots of the legs are 30 feet
apart and each leg is 50 feet long. The distance, measured
horizontally from the pivots to the centre of the ship hatch, is 25 feet,
and the back leg of the sheers is 100 feet long. Assume that the
load is simply hung from the top of the legs and find the push in
each leg and the pull in the back leg when the boiler is going
through the hatch.
9. A load W of 2000 lbs. is hung from a pin P, at which pieces
AP and BP meet like the tie and jib of a crane. The angles WPB
and WPA are 30 and 60 respectively. Show by a sketch how to
find the forces in AP and BP. Distinguish as to each piece being a
strut or a tie. (1897.)
10. Two pieces in a hinged structure meet at a pin, and a load is
applied at the pin. Show how we find the pushing or pulling forces
in the pieces. Describe an apparatus which enables your method to
be illustrated, (1898.)
11. A
carriage mounted on frictionless wheels rests on a plane
inclined at 25 to the horizontal. If the carriage weighs 10 lbs.,
find by construction the force required to keep it in equilibrium,
(a) when the cord is horizontal, {b) when parallel to the plane,
(c) when at an angle of 30 to the plane.

CHAPTER V.

MOMENTS. PARALLEL FORCES. COUPLES.


Moments.The moment of a force means the tendency of the
body on which it acts about a given axis. The
force to turn the
moment of a force is measured by the product of the magnitude
of the force and the length of a line drawn from the axis
perpendicular to the line of action of the force.
Suppose a rod AB (Fig. 62) to be suspended by means of
a bradawl pushed through a hole at Ainto the experimental
board, so that it hangs vertically. Attach a
cord to the rod at (7, and lead it over a pulley
at D, so that the portion CD is horizontal. If
a weight W
1
be attached to the end of the cord,
this will give a horizontal pull, P= W\, to the
rod at C. The effect will be to turn the rod in
the same direction as the hands of a clock. By
doubling or trebling the load W x
the tendency
,

Ok to rotate doubled or trebled. By


the rod is

increasing the distance AC, the turning ten-


Fig. 62. Moment
of a force. dency is increased in the same proportion, so
that the turning tendency, or moment of the
force P, is proportional jointly to the magnitude of P, and the
perpendicular distance AC from the axis at A to the line of
action of P.
Notice not merely the distance from the axis to the point
it is

o.f P that is taken. For if this were so it is easily


application of
seen that the calculated moment of P about A would remain the
same no matter in what direction P is applied, provided it
MOMENTS. PARALLEL FORCES. COUPLES. 43

always acts at C, whereas, actually, inclining the line of P's


action diminishes the turning tendency, until finally if W
l
be
hung from C so that P is vertical, there will be no
direct
tendency whatever to turn the rod. Hence in calculating
moments the perpendicular to the line of action of the force
must always be taken.
Now suppose a weight U\ = 5 lbs. hung and that
to the cord,
AC is 10" (Fig. 63). The moment of P measured by
will be
taking the product of 5 and 10 ; thus the moment of P about
vl = 5xl0 lb.-inch units. In giving the numerical value of a
moment, the units of force and distance employed must always

dCT
C .0

w,

Fig.
w.
63.Two equal
dm,
forces, giving Fig. 64.
n;
Two unequal forces, giving
equal opposing moments. equal opposing moments.

be stated. Thus, ton foot, cwt. inch, gram centimetre, are units of
moment. The moment of P will be clockwise. The rod AB may be
balanced against rotation if another weight 2
= 1 be applied W W
as shown (Fig. 63), so as to produce a force Q equal to P pulling
horizontally at C in the opposite sense to P. Thus, moment of
# = 5x10 = 50 and will tend to turn AB in the
lb.-inch units,
opposite direction to the hands of a clock, that is, anti-
clockwise.
It will be also, by trial, that the rod will be balanced
found
if the weight W
be altered, say diminished, provided at the
2
same time the distance from A at which Q is applied be altered,
in the present case increased, viz. AC to AC
(Fig. 64). It will be
found in all these cases that QxAC
must always amount to
50 lb.-inch units. That is, for AB to be in equilibrium :

Clockwise moment of P = anticlockwise moment of Q.


44 APPLIED MECHANICS FOR BEGINNERS.

If the direction of P be altered by raising the left-hand pulley


to D (Fig. 65), the moment of P is now P x AM, clockwise. The
rod will be balanced by W 2i provided that matters are so
arranged that the product QxAAT which measures the anti-
,

clockwise moment of Q, is equal to Px AM.


The result may be stated thus Two forces which act on a body
:

free to rotate, having- equal moments of opposite sign about the


axis of rotation, will balance the body.
The student should test the truth of this statement by work-
ing carefully several experiments similar to that described
above, using different forces and distances in each case.

Qw,

Pio. 65.Two inclined forces, having Fig. 66. Disc in equilibrium under the
equal opposing moments. action of several forces.


Principle of Moments.- Expt. By means of a screw at (7,
mount a circular wooden disc on the vertical experimental board
(Fig. 66). Apply any number of forces, such as P, Q, S, T by
means of cords attached to the disc at a, 6, c, d, led over pulleys
and having weights at their ends. Let the disc find its position
of equilibrium. Calculate the moment of each force about C by
multiplying the magnitude of the force by the length of the
perpendicular from C to its line of action, producing this last if
necessary. Arrange these moments in two columns, one for
clockwise, one for anticlockwise moments. Take the sum of
each column, and we should expect to find that these are equal,
for, if the disc is in equilibrium, the total clockwise turning
tendency must be equal to the total anticlockwise turning ten-
dency, in order that rotation may not take place. This principle
may be used to solve a great many problems.
:

MOMENTS. PARALLEL FORCES. COUPLES. 45

Example 1. A beam 12 feet long, supported at its ends, carries


a load of 2 tons, 4 ft. from one end. Find the reactions of its
supports. Neglect meanwhile the
weight of the beam itself.
Let ABbe the beam (Fig. 67) j
and P and Q the reactionsnns of its Pf

supports. Imagine the beam to Fig. 67.

be free to rotate about B, and


take moments about B of the forces acting on it. Find P thus :

Clockwise moment of P about B = Px 12 ton-foot units.


Anticlockwise moment of W
about B Wx 8 = 2 x 8 = 16 ton-foot
units.
Q has no moment about B, as its line of action passes through B, and
therefore the perpendicular from B to the line of action has no length.
Now the clockwise moment must equal the anticlockwise moment.
.-. Pxl2=Wx8,
12P=16, P= 1& = 1 33 tons.

In the same way, take moments about A in order to find Q ; then


Anticlockwise moment of Q = Q x 12 ton-foot units.
Clockwise moment of 7^=^x4 = 2x4 = 8 ton-foot units.
Anticlockwise moment = clockwise moment.
/. Qxl2=Wx4:,
12Q = 8, # = = 0-66 ton.

Example 2. A beam 20 feet


long, supported at its ends,
jw,-*to w,-Tw
carries loads at intervals as
is a
a, jpLUZZ t
t
'
paE l .1

shown (Fig. 68). Find the


ie
I 3'-* V I 6' 3' > ' ^
f

reactions of the supports,


neglecting meanwhile the
weight of the beam.
Fl - 6S
Taking moments about B. -

Clockwise moment P x 20 ton-foot units.


:

Anticlockwise moments
x W
x 17 = \ x 17 = 8-5 ton-foot units.
JF2 xl3 = ixl3= 325
z x W
7 = 1 x 7= 7-0
J^x 4=2 x 4= 8-0
Total anticlockwise moment = 26 '75 ton-foot units.

Clockwise moments = anticlockwise moments.


/. Px20 = 26'75, P= 1-3375 tons.
48 APPLIED MECHANICS FOR BEGINNERS.

Taking moments about A,


Anticlockwise moment = Q x20 ton-foot units.
Clockwise moments :

W l
x 3 = Jx 3= 15 ton-foot units.
W 2
x 7 = x 7 = 1-75
W s 13=1x13 = 130
x ,,

JT4 x16=2x16= 320


Total clockwise moment = 48 '25 ton-foot units.

Anticlockwise moments = clockwise moments.


.-. Qx 20 =48 -25,
# = 24125 tons.

Notice in these questions, that a system of forces exists in


each case the lines of which are parallel to one another, and
that by the above solutions, the sum of the downward forces

Fio. 69. Apparatus for determining the reactions of the supports of a beam.

equals the sum of the upward forces. Thus, in Example 1, the


downward force is 2 tons and the two upward reactions have a
sum of l + = 2 tons.
In Example 2, the downward forces give
|+}+l+23f=3*75 tons -

The upward reactions give


1-3375 + 2-4125 = 3-75 tons.
Or, in a given system of parallel forces the sum of the forces of
one sense must in every case he equal to the sum of the forces of the
;

MOMENTS. PARALLEL FORCES. COUPLES. 47

other sense, otherwise the body will be displaced as a whole in the


same sense as the greater sum. Thus, if in the beam examples
the sum of the reactions falls below the sum of the loads, the beam
will move downwards and vice versa. Fig. 69 shows an apparatus
for experimentally finding the reactions of a loaded beam.

Expt. Hang the same rod AB as before vertically in front of
the experimental board using this time a long piece of cord for
the suspension (Fig. 70). Attach cords at C
and D and apply horizontal forces P and Q
by means of pulleys and weights. To balance
the rod AB, a force acting to the right, of
magnitude, by the foregoing, equal to the
sum of P and Q, must now be applied.
Attach a weight S = W P+Qto another cord
and by means of a pulley, apply a horizontal
force E=P+Q to the rod at F. This will CW,
prevent bodily movement of the rod to the
left, but, very likely, the rod will not now
hang vertically. In this case shift the cord at
F, up or down, until such a position is found
that the rod hangs vertical under the com- Fiffl. 70. Equilibi-ant
of two parallel forces
bined actions of the horizontal forces P, Q and of the same sense.
E. E is now the equilibrant of P and Q.
The position of F may be found thus
Imagine the rod to be
:

free to turnabout C and take moments of the horizontal forces


(which alone tend to rotate the rod) about C.
Clockwise moment, Qx CD=Qx(a + b).
Anticlockwise moment, Ex CF=Exa.
P passes through C and has therefore no moment.
Clockwise moment = anticlockwise moment
.'. Q(a + b) = Exa, and E=P+Q,
;. Q(a + b) = (P+Q)a;
/. Qa + Qb = Pa + Qa,
or Qb=Pa,
P b

or P:Q=b:a.
The point F therefore divides the distance CD between the
forces P and Q in inverse proportion to the forces
48 APPLIED MECHANICS FOR BEGINNERS.

Since E
is the equilibrant of P and Q, if its sense be reversed,

it willbe the resultant of these forces.


Resultant of parallel forces. We have, therefore, the follow-
ing means of finding the resultant of two
parallel forces of the same sense: R = P+Q ;

the line of R divides the distance between P


and Q in inverse proportion to P and Q.
If the forces are of opposite sense, the
student should verify experimentally the

n following statements
(1) R is
ence between
greater force, R = Q- P.
:

equal in magnitude to the differ-


P and Q ; thus, if Q is the

- C_3

r R acts
'

r t (2) in the same sense as the greater


force.

Fig. 71. Equilibrant


(3) P:Q = b:a. (Fig. 71.)
of two
parallel forces of
opposite sense.
Expt. Do this by actually applying
forces P and Q to the suspended rod.
Calculate the magnitude of E from (1) and its point of applica-
tion F from (3). Apply i?and see if the rod balances vertically.
Notice in this figure, that P, E and Q are arranged in an exactly
similar manner to P, Q and EmSo that since P, ^and
Fig. 70.
Q balance in both cases, the same results apply to both. Thus :

Given forces of same sense. Given forces of opposite sense.

(1) R=E=P + Q. (1) Q=P + E=P + R;


:. R=Q-P.
(2) R acts in same sense as (2) R acts in same sense as larger
given forces. given force.
(3) P:Q = b:a. (3) P:Q = b:a.

In (3) reference must be made respectively to Figs. 70 and 71.


It isseen that in each case, a is the distance from P to R, and b
is the distance from Q to R.
It may now be
inferred from the above statements, that
the resultant of any number of parallel forces has a magnitude
equal to the algebraic sum of the forces, and its position may be
calculated by taking moments about any fixed point in the body.
MOMENTS. PARALLEL FORCES. COUPLES. 4<l

Couples. It has now been seen that the resultant of two


parallel forces of opposite sense may be found from
R=P-Q (i)
P:Q=b:a (2)

and that if R be reversed, giving E=R, then P, Q and E will


balance.
. Notice that if P and Q are nearly equal to one another, that R
will become very small [from (1)], and that b and a must be
nearly equal to one another and therefore both must be very
large. So that as P
and Q become more nearly equal to one
another, E will become smaller and smaller and will move
further away from P and Q. In the particular case of P
becoming equal to Q, E will become zero and
its distance from P and Q will be infinitely

great. In this case P and Q are called a


couple, and it follows from the above that no
single force can balance a couple.
The moment of a couple is measured by the
product of one of the forces and the perpen-
dicular distance between them, called the arm
of the couple.
i

Expt. Using the suspended rod, Fig. 72,
apply two equal horizontal forces P, P, at A
and 2?, thus causing an anticlockwise couple of
pitUi
moment PxAB to act on the rod. It will
Fio. 72.
now be found impossible to keep the rod
vertical by the application of any single force.
balance, apply Q, Q, at C and D, thereby giving a clockwise
To
couple to the rod. Select the values of Q and the arm CD so
that the moments of the clockwise couple and the anticlockwise
couple are equal, that is :

PxAB=QxCD.
The rod now remains balanced vertically.

Example. Suppose in the above experiment that P, P, are


forces of 6 lbs. each, ^4 =10" and CD = 15*. Find the forces Q, Q.
PxAB=QxCD,
6x10 = ^x15,
Q=4 1bi.
A.M.B. d"
50 APPLIED MECHANICS FOR BEGINNERS.

Fig. 72 shows all the forces horizontal, but this is not essen-
tial. Any two couples of equal moment and opposite turning
tendencies will balance the rod.
Expt. Test this statement by inclining the lines of the P
couple and also the lines of the Q couple, but at different angles
for the two couples, and make their moments equal again by
adjusting the weights hung on.
Couples have many interesting and useful properties, but
most of them must be reserved until the student is more
advanced. One thing in particular should be noticed a couple
applied to a body will not displace it as a whole from its given
position, but will only cause it to rotate. Conversely, a body
which is beginning to rotate must have a couple acting on it.
This principle is made use of in the example following.

Example. Given the total force transmitted from the


piston to the crosshead of an engine, and the lengths of the
crank and connecting rod, to find the pressure on the guides
and the turning moment on the crank, neglecting frictional
effects.

Make an outline diagram of crank, connecting rod, and piston


rod to scale (Fig. 73). Let P= force along piston rod, T= force
along connecting rod, and
Q= reaction of guide,
which would be perpen-
~"}~~
dicular to the piston rod
if there were no friction.
_ P, T. and 0, acting at
Fig. 73. Forces on guide, and along connecting ,, , , . , ,

rod. the crosshead A, balance


one another. Set off
Aa = P to any convenient scale of force. By drawing the
parallelogram of forces Aabc, we obtain T, represented by Ac,
and Q represented by A b. The pressure on the guide will be
equal and opposite to Q. The connecting rod exerts a force
equal and opposite to T at its crank pin end B. This force
acting on the crank pin has a moment about C, and, in conse-
quence, the crank rotates.
Considering the crank separately, the force T (Fig. 74), applied
at 2?, requires an equal opposite force, which in the actual crank
MOMENTS. PARALLEL FORCES. COUPLES. 51

can only be applied at G', viz., the reaction from the crank shaft

bearings. These two parallel equal forces T, T, of opposite


sense, form a couple, of moment T x Bd, gives the turning
moment on the crank. This couple is
balanced by the couple due to the ***}-*.
resistance of the machinery driven
by the engine, not shown on the
diagram.
The force P transmitted by the pis-
ton rod does not remain constant Fig. 74. Forces on crank.
during the stroke, but it is a very
useful exercise at this stage to work out completely an example
in which it is assumed to be uniform. The actual practical
problem can be dealt with later.
The method which may be adopted is to draw outline
diagrams of the crank and connecting rod when the crank is
at intervals of 30 (Fig. 75), and then to find, by the paral-
lelogram of forces, the forces Q and T for each position,
assuming a constant value for P, say 1000 lbs. These values of

120 ISO 180 210

Values of T. Turning Moment on Crank

Fig. 75. Turning moment diagrams for an engine.


Q and T set off as ordinates on a base line taken to represent
the stroke, and a curve drawn through their tops (Fig. 75) will
show the values of Q and T for all positions of the crosshead.
Work out also the value of T x Bd for each position of the
crank. The values of this set off as ordinates on a base taken
to represent the revolution, and divided into intervals of 30,
;

52 APPLIED MECHANICS FOR BEGINNERS.

will give the turning moment on the crank at any crank


position (Fig. 75).
Consider again the rank in this position (Fig. 76). We notice
that the whole tendency of T is partly to
exert a push on the crank along BC and
partly to rotate it. This can be easily seen
if we resolve T into two forces, one along

the crank and one perpendicular to it


thus, V gives the push along BC and S
Fig. 76. Force acting
along the crank. produces the turning moment. Wheu the
crank is in any position between 90 and
180, and also between 270 and 360, V is a pull, and in other
positions it is a push.
Experiment on forces in an engine. It is easy to show
these forces experimentally. Fig. 77 shows a flat piece of wood

Fig. 77. Experimental apparatus for showing the forces in the parts of
an engine.

BC attached by a screw to a vertical board at (7, so that it


can turn freely. Another screw at B serves for a crank pin.
A is a small ring to represent the crosshead, and another ring,
fitting very loosely over the crank pin screw B, serves for con-
necting rod brasses. A
cord attached to A and led over a pulley
gives P. A
cord hanging vertically from A gives Q. spring A
balance introduced into a cord connecting A and B gives T, and
cords led from the ring at B over pulleys give V and S respec-
tively. The weights used should be fairly heavy, say 10 lbs. at
P and the others to correspond with the crank position taken,
in order to minimise as far as possible the disturbing effect pro-
duced by the weight of the spring balance.
MOMENTS. PARALLEL FORCES. COUPLES. 53

EXERCISES ON CHAP. V.
1. AB is a uniform bar pivoted at C, its centre of length. is W
a load of 5 lbs. placed at D, CD being 15". If we have to restore
balance by means of a 3 lb. weight, where must it be placed ?

2.A bent lever A CB is pivoted at C arm A G is horizontal and


;

9" long arm BC is vertical and 39" long. A load of 300 lbs. is hung
;

from A. Find what horizontal fo: ze at B will produce equilibrium.


Neglect the weight of the lever.
3. The arms of a bent lever A CB are perpendicular to one
another, and the lever is pivoted at C. Arm AC
is 6" long, and BC
is 27" long and inclined 30 to the vertical. Find what horizontal
force atP B
will balance a force Q =250 lbs. applied at A
at 90 to
AG. Neglect the weight of the lever.
4. A
rod 5 ft. long has a weight of 2 lbs. at one end and 3 lbs.
at the other, also a weight of 5 lbs. at its centre. Find the point
about which it will balance. Neglect the weight of the rod.
5.Give a dimensioned sketch of a practicable arrangement of
levers whereby a weight of \ ton may be balanced by one of 10 lbs.
6. A beam 12 ft. long, supported at its ends, carries a load of
1 tons at a point 4 ft. from one end. Find the reactions of the
supports, neglecting the weight of the beam.
7. A
beam 20 ft. long, supported at its ends, has a load of 2 tons
at the centre of its span, another of 1 ton at 3 ft. from one end, and
another of 3 tons at 4 ft. from the other end. Neglect the weight
of the beam and find the reactions of the supports.
8. AB is a beam
16 ft. long. It is supported at the end A and
at from the end B. A load of 4 tons is placed 6 ft. from A and
C 4 ft.
another of 2 tons at the end B. Neglect the weight of the beam and
find the reactions of the supports.
9. A
handle used for turning a machine is 15" radius. man A
exerts a constant force of 30 lbs. (a) continually in a horizontal
direction, (b) continually in a direction tangential to the circle of
rotation of the handle. Draw diagrams to show in each case the
turning moment for all positions of the handle.
10. A man whose weight is 160 lbs. can lift, unaided, a load of
3 cwts. Suppose he uses a lever 4 ft. long, the fulcrum being 3" from
one end, find what weight he can raise (a) if his end of the lever is
moving down, (6) if his end of the lever is moving up, the fulcrum
and weight changing places with each other.
11. The diameter of the safety valve of a steam boiler is 3 inches.
The weight on the end of the lever is 55 lbs. and the distance from
,

the centre of the valve to the fulcrum is 4*5 inches. What must be
the length of the lever from the centre of the valve to the point of
suspension of the weight, in order that the valve will just lift when
the pressure of steam in the boiler is 80 lbs. per square inch ? Neglect
the weight of the lever and the valve. (1896.)
CHAPTER VI
CENTRE OF GRAVITY. FORCES NOT ALL APPLIED AT
THE SAME POINT. HANGING CHAINS. ARCH.
Centre of parallel forces. Let two forces P and Q act on the
rod AB (Fig. 77a), their directions being perpendicular to AB.
The resultant R, found as before, will
pass through C, and will divide AB in
the proportion

P: Q = BC:AC.
Suppose we incline the directions of
''
\ P and Q, as at P' and Q', without
Fig. 77a. Centre of parallel altering theirmagnitudes. R will
now act parallel to P' and Q\ its
magnitude will be unaltered, and it may be seen, if we draw a
line through C, perpendicular to P' and Q\ that this line BE
is also divided inversely proportional to P and Q, that is,

P' :Q' = EC:CD.

It therefore follows that R', the resultant of P' and Q, will


also pass through O, and it may be shown in the same way, that
no matter how P' and $ are inclined, provided their magnitudes
are unaltered and that they are kept parallel to one another,
that their resultant always acts through the same point C. This
point is called the centre of the parallel forces P and Q.
If there are a number of parallel forces it will be easily seen
that their resultant also always passes through the same point
whatever may be the inclination of the forces.
CENTRE OF GRAVITY. 55


Centre of gravity. Suppose now we have a sheet of thin
metal. Every particle of the metal is being pulled towards
the earth's centre, so that we have a large
number of forces acting on the body in lines
which are practically parallel to one another
(Fig. 78). The resultant of these forces is

what we call the weight of the plate, W say.


Now no matter how the plate may be turned
(which equivalent to inclining the forces on
is

the particles to their first direction) there will


be a point in the plate through which W Fin. 7& -Centre
gravity.
of

always acts, this point being the centre of the


parallel forces acting on the particles. Let G be this point, then
G is called the centre of weight, or centre of gravity of the plate.
In a great many bodies, we may see by inspection where the
centre of gravity is. Thus, a uniform rod will have its centre of
gravity at the middle of its length. A square, at the intersection
of its diagonals, so also a rectangle and a parallelogram. A circle
will have its centre of gravity at its geometrical centre. In the
case of a triangle, the centre of gravity will be found one-third
way up a line from the centre of the base to the opposite corner.
Pyramids and cones have their centres of gravity ^ way up a
line from the centre of the base to the apex. In prisms, with
ends perpendicular to their axes, the centre of gravity will lie

at the geometrical centre of the middle cross section.


When we are considering the equilibrium of a given body we may
regard its whole weight as concentrated at its centre of gravity.
Centre of gravity by experi-

ment. Suspend a thin plate of
any irregular outline and of any
material by a cord attached at
A (Fig. 79). The pull in the
cord will be P equal to W, the
weight of the plate. If the
plate be at rest, these forces,
being the only two acting on it,
must be in the same straight Fio. 79.A plate Fio. 80. The same
hung from A. plate hung from B.
line. It follows therefore that
G, the centre of gravity of the plate, must fall vertically under A.
:>G APPLIED MECHANICS FOR BEGINNERS.

Produce the line of P downwards on the plate as shown, then


G is in this line. Now hang the plate from another point in it
such as B (Fig. 80). G again must be vertically under B, so if

the line of P be drawn on the plate, the


again- intersection of
this line with that first drawn will give G.
Expt. Find by experiment the centres of gravity of pieces
of card board cut into the following shapes triangle square : ;

with a piece cut off one corner shape of a letter H shape of a ; ;

letter L shape of a letter T.


;

Centre of gravity by calculation. For plates of fairly


regular outline, the position of the

a
MR b
centre of gravity may be easily cal-
culated by using the principle of
moments. Thus, to find the centre
of gravity of the plate shown in
Fig. 81. Divide it up into rectangles
as shown. Then, the centre of
M -f
c j
y gravity of abcg is at m, and of gdef
1
.;.:.- ~^ :: "
at n.
---...
i Also the weights of the rectangles
will be proportional to their areas.
Fig. 81. Centre of gravity of an
angle section. So that weight of abcg is proportional
to 2^x^ = 1 J, and weight of gdef
proportional to 3 x \ = H and the weight of the whole plate to 2f.
Let x be the distance of the centre of gravity of the plate
from afj then taking moments about af

-;
'
*=ix ^=1 = 0-932".
Now take moments about fe, and let y be the distance of the
centre of gravity from fe.
2xy=(lix}) + (l*xlf)

*
y=t*A=H=o-932".
The centre of gravity is therefore a point 0*932" from each of
the sides a/and/e.
CENTRE OF GRAVITY. m
Centre of gravity by a graphical method. We may
proceed in another way. Thus, to find the centre of gravity of
the plate shown in Fig 82, divide
it into triangles by the line bd.

Find the centre of gravity of each


by construction, i.e. bisect bd at e,
join ea and ec and measure rd up
each of them from e. This con-
struction will give Ci and c 2 the ,

centres of gravity of abd and bed.


The centre of gravity of the whole
Fig. 82. Centre of gravity of an
plate must be in the line joining cx irregular plate.
and c 2 Now divide the plate again
.

by the line ac. Find as before c3 and c4 the centres of gravity ,

of abc and acd. Join c3 c4 The centre of gravity of the whole


, .

plate must be in the line c3 c4 G will therefore be the point


.

where c x c2 and c3c4 intersect.


State of equilibrium of a body. A body is said to be in
stable equilibrium if, on being slightly disturbed, it tends to

return to its original position ; unstable if it tends to go over


further, and neutral if it will remain at rest indifferently in any
position. We may easily test for a body's equilibrium.
Thus, suppose a cone to stand on its base on a horizontal
Its weight being W, then R, the reaction of the
surface (Fig. 83).

Fia 84.
Stable equilibrium of a cone.

surface, will be equal to W and in same straight line. Disturb


the cone slightly. R shifts along to A (Fig. 84), and R and
W now form a couple tending to bring the cone back to
its original position. This is therefore a case of stable
equilibrium.
58 APPLIED MECHANICS FOR BEGINNERS.

Now stand the cone on its apex (Fig. 85). If we disturb it


slightly, R and W
form a couple tending to upset it (Fig. 86).
The position is therefore unstable.

Fig. 85. Fig. 86.


Unstable equilibrium of a cone.

A ball resting on a horizontal table will be in neutral


equilibrium, because R and W
(Fig. 87) will always be in the
same straight line no matter how the ball
is disturbed. It will therefore remain at
rest in any position.
A cylinder lying on its side (Fig. 88)
will also be in neutral equilibrium, but if

we cut a slice off the top, the equilibrium


will be found to be stable.. For the centre
FlQ
-
brnTm tf?bali
qUil1
" of g ravit7 G '
( Fi & 89 > wiU be h *^
below G by cutting off the slice, and conse-
quently, if the cylinder be slightly disturbed, as in Fig. 90, R and
W will give a couple tending to bring it back again to its original
position.

Fig. 88. Neutral equilibrium Fig. 89. Fig. 90.


of a cylinder. Portion of a cylinder in stable equilibrium.

It may be seen, from some of the above examples, that


another property of the centre of gravity of a body is that, if
the body is free to be moved by its weight from its given
position, it will always do so in such a way that the centre of
gravity is lowered thereby.
CENTRE OF GRAVITY. 59

Any forces acting in a plane. We may now consider what


conditions must be fulfilled if a number of forces, all in the
same plane, but not necessarily passing through the same point,
act on a body and produce equilibrium.
There are three conditions to be satisfied :

(1) There nuist be no tendency to produce vertical movement of


the body, either up or down.
(2) There must be no tendency to produce horizontal movement,
either to right or left.

(3) There must be no tendency to rotate the body.


The easiest way of whether these conditions are
testing
satisfied is to take components of each force in horizontal and
vertical directions. Then, that there may be no vertical move-
ment, the sum of the upward components must be equal to the
sum of those acting downward for no horizontal movement,
;

the sum of the components acting towards the right must equal
the sum of those acting towards the left and for no rotation, ;

the sum of all the clockwise moments, about any point, produced
by the components must equal the sum of the anticlockwise
moments about the same point.
Graphical solution by the link polygon.The equilibrium
of a number of given forces all in one plane may be tested by
this method. Given any number
of forces such as P, Q, S, T
(Fig. 91), all in one plane. Take
any one of them, e.g. Q, and
balance it by applying any two
forces, p v p 2 at any point such
,

as B, on #'s line. Applying the


triangle of forces to p v B, and p 2 ,

the magnitudes of p x and p., can


be found, as at Oab. Now
imagine that BC is a rod, push-
ing at B with force p 2 and its ,

other end pushing at C, on the


line of S, with equal force p 2 Fig. 01. The link potygon.
.

Balance S and p 2 at C by a third


force, p 3 acting along a rod DC.
, The direction and magnitude
of this force may be found from the triangle of forces Obc. Let
60 APPLIED MECHANICS FOR BEGINNERS.

the end D D on T's line with force p3


of the rod push at .

Balance T and p3 at D by
a force p4 the line and magnitude of
,

which will be found from the triangle of forces Ocd. We


have now balanced all the given forces except P, and this would
be balanced also provided the push p 4 acting at A' on jP's line,
the push p x acting at A on the same line, and P, balance one
another.
For this to be possible the following conditions must be
satisfied.

(1) Aand A' must coincide, for the three forces p v p^ and P
must pass through the same point.
(2) The triangle of forces for them must close, as shown at
aOd.

we have a closed system of


If these conditions are fulfilled,
rods or links A BCD, and
a closed force polygon abed
also
(Fig. 91). Notice that the closed force polygon abed will
have the same shape no matter which force we select to
begin with, for its sides are parallel and proportional to the
given forces. On the other hand the link polygon, as it is
called, will have a shape depending on how we select the
directions of the first two links AB and BC. This will alter
the position of in the force polygon, but it will not alter the
essential conditions.
In practice we proceed thus. First draw the force polygon
abed for the given forces. For equilibrium this must close. Then
select any point and join it to each corner of the force polygon.
Draw the links between the given forces parallel to these
lines Oa, Ob, etc., on the force polygon. Notice that Oa, on the
force polygon, comes between da representing P, and ab repre-
senting Q this means that the link connecting P and Q must
;

be drawn parallel to Oa attention paid to this point will save


;

mistakes. If the link polygon closes also the given forces are in

The method is suitable for use in many cases where graphical


solution is desirable.


Tensions in a hanging cord. Exft. Arrange an experi-
mental link polygon by attaching a cord ACDEFB to the ends
A and B of a rod (Fig. 92); hang the rod up by two cords
CENTRE OF GRAVITY. 61

attached to fixed supports and H Zona vertical board, and


to A and B. If weights are now hung
from <7, D, E, and F, the cord will take
a definite shape. To find its tensions at
*
any part of its length, we may apply the
triangle of forces. Mark on a piece of
drawing paper, stretched on the board,
the directions of the cord and of the
applied weights, and remove the paper.
As h W W W
2 3y and TP4 are known,
, ,

the triangle of forces can be now drawn


for each of the points (7, Z>, E, and F.
Calling the tensions in the different
parts of the cord T T^ T T^
ly Zi and T6 ,

abO will be the triangle of forces for


Wv T2 , and Tx acting at C; bcO that
for Wz, T3, and T2 acting at D ; cdO
that for W 3,
T, Tz acting at E; dfO
and
92. Tensions in a
that for TT4 , T5 ,T4 acting at F. The
and Fid.
hanging cord.
tensions 7\, T2 T3 T, and T6 will there-
, , ,

fore be given by the lines Oa, Ob, Oc, Od, Of respectively,


measured to the scale of force.
Tensions at the ends of a stretched chain. This construc-
tion enables us to solve some important practical problems.
Thus, noticing that af in
the force diagram re- \^j,
presents the total load
applied to the cord, and
that Oa and Of represent
the tensions in the two
end portions of the cord
and are drawn in the
same direction as these
portions, we can easily
find the tensions at the
, - , . , . Fio. 93. Tensions at the ends of a hanging cord,
ends of a hanging chain
if we only know the inclination of the chain there, and the total

applied weight.
Thus, suppose a chain (Fig. 93) hanging from two supports
62 APPLIED MECHANICS FOR BEGINNERS.

A and B has its ends making angles of 30 and 45 respectively


with the horizontal, and that the total applied weight is 50 lbs.
Set off ab to scale to represent 50 lbs., and draw aO and bO at
angles of 30 and 45 respectively to the horizontal. Then aO,
to the scale of force, gives 7\ equal to 36^ lbs., the tension at the
30 end of chain and Ob to the same scale gives T2 equal to
44^ lbs., the tension at the 45 end. The form taken by the
hanging chain is not required for the solution, and is therefore
not shown in the figure.
Calculation of same problem from dip and span. AB
(Fig. 94) represents a fine wire, or a chain stretched between
two supports on the
same level ; to find
its tensions at the
middle f the S Pan
Fio. 94.-Hanging chain or wire.'
and at the supports,
we may proceed as
follows. Imagine
the wire cut at (7,
Fig. 95.-Relatfons between the dip,
the centre of itg

span, and instead


of the pull of the left-hand portion, apply a horizontal force IT of
the same amount this will keep the right-hand portion of the
;

chain or wire in equilibrium. "We apply a horizontal force at Cy


because the chain or wire will be horizontal there as the supports
are on the same level, and also because the wire or chain is only
capable of resisting pull, consequently its tension at any point
will be always in the same direction as the chain at that point.
Consider now the equilibrium of the right-hand portion of the
chain or wire. T, its tension at the end B (Fig. 95), may be
split into horizontal and vertical components T v and Th . If W
isthe whole weight of the chain or wire, ^ will be the weight W
of the portion under consideration and may be placed at its
centre of gravity (7, which we may assume (without serious

error the dip of the chain is small compared with the span) to
if

be midway between the verticals through C and B. Let be D


the dip of the chain and S its span, both in the same units of
length. Notice now that two horizontal forces and two vertical
forces only act on the balanced half chain, consequently the
CENTRE OF GRAVITY.

vertical forces must be equal and opposite and so also the


horizontal forces ;two opposite equal couples are therefore
formed by these forces, giving

Moment of clockwise couple =HxD


=\Wx S
Moment of anticlockwise couple

Having found 27, and knowing that Th is equal to 27, we may


now find T from its known components Th and Tv thus ,

and acts at an angle to the horizontal the tangent of which is

Tv

KAn actual
example. An experiment was tried on a chain
20' 2" long,weight 3 lbs., when hung from two supports, 19' 9"
apart, both on the same level.
Instead of fixing the ends of the
chain, each end was supported by
cords lead over pulleys shown at
A and C, (Fig 96) the pulls in
-x
these giving T
and Th By the
v .

experiment it was found that


Tv =1-5 lbs.
and Th = H
- 4'24 lbs.
The dip at the centre of the span
was measured and found to be Fio.96. The forces at the end of a
chain, determined experimentally.
1-73 feet.
Sf
By calculation, from H=^ W-
jgr= 3xl9'75 = 4 .
281bg '
8x 1-73
showing an error of less than one per cent when compared with
the observed value of H obtained in the experiment.
;

64 APPLIED MECHANICS FOR BEGINNERS.

The arch. Supposing we have several vertical loads sup-


ported by means of a cord pulled at its ends (Fig. 97), and that
we know the shape taken up by the cord. Imagine the whole
apparatus to become rigid for a moment and turn it upside down
we should then have a structure in equilibrium (Fig. 98) under

Fig. 97. Hanging cord, supporting loads.

Fig. 98. Hinged bars, supporting the same loads.

Fig. 99. An arch, supporting the same loads.

the applied loads W W W W W


lt 2, 3, 4, 6,
if we substituted bars hinged
at (7,E, F, G, and replaced the pulls Tx and T2 by equal
Z>,

pushes applied in the same way at A and B. The whole


arrangement would, however, be very unstable, but could be
made stable by replacing the hinged bars with blocks properly
fitted together, asshown in Fig. 99. These blocks would push
one another at their joints in just the same way as the bars did,
only now we have the structure called an arch. The curve
CENTRE OF GRAVITY. 65

formed by the hinged bars is called the line of resistance of the


arch. It is interesting and instructive to watch the changes
produced in the line of resistance of a model arch such as that
shown in Fig. 100. Here the joints, instead of being flat, are

Fia. 100. Experimental model of an arch.

made rounded so that the blocks may roll on one another,


thereby giving by their places of contact, points on the curve of
resistance. The places of contact of the blocks have been
marked in white chalk on the model and are easily seen in the
figure. In an actual arch, the shape of the line of resistance
alters,when loads are applied, in the same way as it does on the
model, only, of course, as the joints are flat, the shape of the
arch itself does not change unless rupture occurs. In practice,
an arch is considered safe when the line of resistance for any
possible load passes through any part of the middle third of
every joint.

EXERCISES ON CHAP. VI.

1. A uniform plank, 20 ft. long, weight 90 lbs., rests on supports


at its ends. A load of 500 lbs. rests 8 ft. from one end. Find the
reactions of the supports.
2. A
uniform beam 12 ft. long, supported at its ends, carries a
distributed load, including its own weight, of ton per foot run.
A concentrated load of 1 ton rests 5 ft. from one end, and another of
3 tons, 4 ft. from the other end. Calculate the reactions of the
supports.
3. A
uniform beam 16 ft. long weighs 300 lbs. It is supported
at one end and at a point 4 ft. from the other end. Calculate the
reactions of the supports.
4. A
cone, 3 ft. diameter of base, 4 ft. high, stands on its base on a
horizontal surface. Specific gravity of material = 3. What horizontal
force at the top will turn the cone over ?
A.M.B. B
)

66 APPLIED MECHANICS FOR BEGINNERS.

5. A
triangular plate, ABC, of wrought iron 1" thick, lies on a
horizontal surface. AB=3
ft., 80=3% ft., C A = 4 ft. Find what
vertical lifting force applied at A
will raise that corner of the plate.
6. A
wall 8 ft. high, 14" thick, is built of material weighing
130 lbs. per cubic foot. The normal wind pressure on the face of
the wall is 50 lbs. per square foot of vertical surface. Consider a
piece of the wall one foot long, and calculate the overthrowing
moment of the wind on it and also the resisting moment of the
weight of the wall. Will the wall stand or fall ?
7. Show in a diagram the couples acting on a hinged door, 7 ft.
high, 3 ft. wide, weight 90 lbs. There are two hinges, placed one
foot from top and bottom of the door.
8. The
jib of a crane is 40 feet long and weighs \ ton. The tie
is 30 long and the post is 25 ft. high.
ft. Make an outline diagram
to scale and calculate the pull on the tie produced by the weight of
the jib. Take the centre of gravity of the jib at 16 ft. from the
lower end.
9. A symmetrical roof weighs 16 tons. On one side of it there
are 7 tons of snow equally distributed. Find the pressures on the
supports.
A horizontal beam 10 feet long weighs \ ton and is pivoted
10.
4 from one end. Its centre of gravity lies in the longer part,
ft.
1 foot from the pivot. Find where a load of 500 lbs. must be placed
to keep the beam balanced.
11. A ladder 24 ft. long weighs 50 lbs. and has its centre of
gravity 8 ft. from one end. A bag of tools, weight 100 lbs. is slung
,

at the centre of length of the ladder. A lad and a man carry the
whole between them, the lad being at the lighter end of the ladder.
Find where the man must be if his share of the load is 90 lbs.
12. A chain of weight 20 lbs. is stretched between two points
on the same level, 40 feet apart. If the dip is 4 ft. calculate the
,

pulls at the middle and at each end of the chain.


13. The weight of a chain hanging from two points of support
is 220 lbs. its inclinations to the horizontal at the points of support
;

are 25 and 42 respectively what are the tensions in the chain at


;

the points of support ? (1900.


14. A
symmetrical pair of steps, hinged together at the top and
connected together by a string at the bottom, stands on a smooth
horizontal plane. If the length of each side be 3 feet 3 inches, and
the string be 3 feet in length, find the tension of the string when a
person of 140 lbs. in weight stands on the steps at a height of 2 feet
from the ground. How is the tension in the string affected as the
person ascends the steps ? (1896.)
CHAPTER VII.

STRESS. STRAIN. ELASTICITY. ULTIMATE TENSILE


STRENGTH. STRENGTH OF SHELLS. SHEAR.

Stress. What happens to a piece of material when forces are

AC
applied to it in the direction of its length ? This is a question
which now requires to be studied. Suppose AB (Fig. 101) is a

Q
bar the ends of which B
are subjected to equal
and opposite pulls P, P.
,
-

D
@~*T
Imagine the bar to be Fi- 101. Bar under putt,

cut at CD at 90 to its axis, and consider what must be done to


preserve the equilibrium of the left-hand portion. In order to
balance the force P it will be necessary to apply an equal and
opposite force at CD (Fig. 102) ; let Q be this force. As the cut

y
Fig.
d~Vp
102. Equilibrium of the
-qS- =-*p-
Fig. 103. Equilibrium of the
left-hand portion. right-hand portion.

is only imaginary, the portion to the right of CD in the actual

bar must have supplied this force Q, in order to keep the left-
hand portion balanced. In the same way, the left-hand portion
of the bar exerts a pull Q equal to P acting towards the left
y

(Fig. 103), on the right-hand portion of the bar.


The force Q in the actual bar will be distributed in some
manner over the section of the material, the sum of the pulls on
every part of the section being equal to Q (Fig. 104). For
sections taken near the ends of the bar, where the forces P, P
OS APPLIED MECHANICS FOR BEGINNERS.

are applied, the distribution cannot be definitely stated, but at


some little distance from the ends, and right along the bar, the
distribution is probably uniform, each square unit of the sections
bearing equal forces. This force per
unit area of the section is called stress.
The may
be easily calculated by
stress
dividing the total force on the section
Fig. 104. Distribution of
by the area of the section. Thus, if the
the stress. bar is pulled at its ends with forces of
10 tons each, and its sectional area is 2| square inches, the stress
will be 10 divided by 2^, or 4 tons per square inch. If the bar
is of uniform section, a stress equal to this will be found on any

cross section, except those very near the ends where the distri-
bution is unknown.

Ties and Struts. Those portions of a structure which are
intended to be under pull are called ties, and if intended to be
under push, are called struts. Ties are said to be under tensile
stress when pulled, and struts under compressive stress when
pushed.
If we consider the case of a tie slightly bent at first and then
pulled, we can easily see that the tendency is to straighten it

A B
(Fig. 105) ; in the same
^~
^-{ep^---^ ::
~^- way
~~~::: a P u U e d string be-
Fig. 105. -Tie bar, originally bent. comes straight. Also, if
the tie is straight to begin
with, there will be no tendency to bend it when pulls are
applied. It follows therefore that since no lateral stiffness is
required in ties to resist bending, that the shape of the cross
section is immaterial. It is very different, however, in struts.
If the strut is originally
> (gp^--^ ^-~^T*7qN - bent, the tendency will

Fio. 106,-Strut, originally bent.


be t0 bend {t stiU m0re
consequently (Fig. 106),
the section must be chosen so as to give the necessary lateral
stiffness to resist this bending action. If the strut is straight
to begin with, kept straight when the pushes are applied,
and is

then the stress on any section not too near its ends will be
uniformly distributed, and will be found as before by dividing
the total force on the section by the area of the section.
STRESS. STRAIN. ELASTICITY.

Both ties and struts should be made straight to begin with,


as we have seen that ties tend to become straight if originally
bent, and struts will bend more if not at
first straight. Horizontal or inclined ties
of great length compared with their cross
sectional dimensions will have a bending
tendency due to their own weight. Such
ties should have a suitable cross section to
give stiffness, or should be supported at
intervals by suspending rods. Struts are
of many different forms depending on their
lengths and the magnitude of the loads. A
few common forms are shown in Fig. 107.
Columns are vertical pieces designed for
the purpose of carrying weights and come
under the heading of struts as they are

awM^w/Mi^
1

Fig. 108. Flanged column, Fig. 109.Flanged column,


load applied centrally. load applied at edge of

subjected to push forces. Lateral stiffness


must be arranged for in columns as in
struts. Short blocks used as columns fail
by crushing. Very long columns fail by
bending and thus breaking. Columns are
often made stiffer by putting flanges on
the ends, the effect being that the column bends as shown by
the dotted lines in Fig. 108, instead of as a whole. But if the
70 APPLIED MECHANICS FOR BEGINNERS.

load is not centrally applied at the ends, the effect may be

worse than if there were no flanges. As an extreme case, think


of W
applied at the edge of the flange (Fig. 109). It is easy to
see that the bending tendency is much greater

^ijiifin tnan would be the case if the flanges were absent.


Change of length of a loaded bar. All

4 materials stretch

With
when pull forces are applied
and become shorter with push
sufficiently
forces.
delicate apparatus,
these changes of length can be meas-
ured in metals. With material such
as rubber, an ordinary scale is sufficient
to measure the differences. Fig. 110
shows a round rod of rubber about
2 inch diameter and 3 or 4 feet long
tied to a support at A and having a
hook for carrying weights at B. C and
D are two needles pushed through the
rubber these needles will move on a
;

i scale E
when loads are applied, and
from their readings the changes in
length of the portion CD can be ob-
tained.

Fio. 110. Ap-


A simple apparatus for measuring
paratus for the extensions of wires under various
the" extend lads consists of two wires hung side
sums of rub- by side from the same support. One
wire AB (Fig. Ill) carries a constant
load sufficient to keep and has a scale of
it taut,
inches divided into tenths fixed at C. The other
wire DE is the wire under test. The load on it
Fig. 111. Ap-
can be varied, and when this is done a vernier fixed paratus for
measuring
at F will move over the scale C, and will give the the exten-
changes in length of the portion DF. The arrange- sions of a
pulled wire.
ment for carrying the scales prevents any drooping
of the support at AD from being measured as an extension of
the wire under test. The support for the wires should, in order
to have a long portion of the wire available for testing, be fixed
at the ceiling, or as high up the wall as possible.
STRESS. STRAIN. ELASTICITY. 71


Extensions in ordinary test pieces. Test pieces of ordinary
bars or plates used for engineering work must be subjected to
great loads before a measurable extension is produced. This is
done in large testing machines and the extensions are measured
by means of instruments called Extensometers. In Prof. Ewing's
Extensometer, the extension of the piece is measured by the
movement of a fine wire over the scale of a microscope. It is
possible with the instrument to measure a change of length of
th
50.000 * ncn on a test pi ece 8" long. The following results were
obtained by its use, and are given in illustration of an impor-
tant fact.
Test bar of flat iron 1 -501" wide, 0'492" thick, 8" long between the
test points. The load was applied in steps of 1 000 lbs. u ntil a maxi-
mum of 10,000 lbs. was reached. The bar carried this load for
2 or 3 minutes, and then the load was taken off 1000 lbs. at a time.

An Experiment with Ewing's Extensometer.

Load increasing. Load diminishing.

Differences per Differences per


Scale reading. Scale reading.
1000 lbs. load. 1000 lbs. load.

3 00 3 00
0-18 016
1000 318 3 16
018 018
2000 3 36 3 34
018 019
3000 3 54 3 53
019 019
4000 3 73 3 72
21 0-20
5000 3 94 392
0-19 0-21
6000 413 413
019 019
7000 432 4-32
0-20 20
8000 4 52 4 52
020 20
9000 4-72 4 72
019 019
10,000 4-91 491
72 APPLIED MECHANICS FOR BEGINNERS.

The scale reading was such that one part on the scale corresponds
to an extension of -^ inch. Consequently the total stretch for
10,000 lbs. load was Lz inch.

Plotting on squared paper column 1 for ordinates and column


2 for abscissae, we see that all the points lie practically on a
LOAD
LBS.
10000

9000

8000

7000
7
6000
7
SOOO
7
4000

3000
7
2000
7
1000

300 320 3-40 360 380 400 420 4i0 460 480 500
Scale Readings
Fig. 112. Curve showing extensions and loads for a pulled bar.
straight line (Fig. 112). We therefore infer that in this piece
the extensions have been practically proportional to the loads.
The same law is found to hold more or less nearly in all
engineering materials and is known as Hooke's Law.

Strain. The term strain is used to signify the change of
length or other dimensions, or the change of form which occurs
in a material when loads are applied. Strain is measured in a
pulled or pushed bar by stating the change of length per unit of
original length of the bar. To obtain it in any particular case,
divide the total extension by the original length of the bar.
Thus, in the above experiment,
T91
=-
tensile strain 5 = 0*0004778.
Strain is not measured in any units, as it is simply the ratio
of two lengths.
Some important definitions. If we go on loading a test
piece, we presently reach a point where Hooke's Law breaks

STRESS. STRAIN. ELASTICITY. 73

down, and the extensions cease to be proportional to the load,


but increase at a more rapid rate. Up to this point, if the load
be removed, the piece will come back to its original length, but
if the loading is carried beyond this point the bar will be found

to be permanently extended when the load is removed.


Elasticity is that property of matter by virtue of which it
tends to return, or spring back, to its original shape and
dimensions when the applied forces are removed.
Elastic Limit is the name given to the stress at which Hooke's
Law breaks down, and at which the bar takes up a permanent
extension. The permanent extension is called Permanent Set,
and material loaded beyond the Elastic Limit is said to be
overstrained.
Modulus of elasticity. Notice that both strain and stress
are proportional to the load producing them up to the elastic
limit, and that therefore they are proportional to one another
for a given material. It follows that if the stress be divided by
the corresponding strain, a constant number for the same
s ie ss
material will be produced. The fraction . , is called
strain
modulus of elasticity. Young's Modulus of Elasticity, is that
which refers to a bar pushed or pulled it is usually written E.
;

For calculation of E we have the following relations :

Let JF=load applied,


A sectional area of bar in square inches,
Z = length of bar,
e = change of length
produced by W,
both lengths being in the same units. Then

A
strain =p
and str
^= WA ^ Le
E = strain -

W.L

The modulus of elasticity will be described as tons per square


inch or lbs. per square inch depending on how the stress is
74 APPLIED MECHANICS FOR BEGINNERS.

measured. The average values of Young's Modulus for some


common materials are given in the following table :

Young's Modulus of Elasticity.

Material. E. (tons per square inch).

Wrought
Steel,
Cast iron,
....
iron,

....
- 13,000
13,500
6,000
Rolled copper,
Brass,
Gun metal,
.... - 6,200
5,700
5,000
Phosphor bronze, 6,000
Aluminium bronze, - 6,500

Phenomena beyond the elastic limit. In ductile materials,


such as iron and further loading be applied after the
steel, if
elastic limit is passed, a point is reached where the material
draws out considerably with no, or very little, increase to the
load. This point is Further loading
called the yield point.
produces considerable extension throughout the material which
can easily be seen even in a short piece, and it will be also
observed, if the diameter of the piece is measured from time to
time, that contraction is going on all over the specimen.
Presently the load is reached at which the wire is about to
break. At the place where fracture is about to occur consider-
able contraction will be observed, until finally rupture occurs.
In materials like cast iron and brass, there is no yield point, and
the total extension of specimens of these materials before
breaking is much smaller than for iron or steel. In general,
the absence of a yield point and small extension show hard
material lacking ductility ; this is further shown by small local
contraction at the fracture. Ductile material, carrying a con-
stant load beyond the elastic limit, goes on extending, or
creeping, during a long period of time.
It is usual to estimate the ductility of a material from the
extension on a measured length of the test piece, and also from
the local contraction at fracture. Plastic material showing
considerable extension before rupture is more suitable for with-
STRESS. STRAIN. ELASTICITY. 75

standing shocks when worked into a structure than hard stuff


which breaks with little extension.
off short The ductility of a
material is often tested by bending a bar of it to a given radius,
and its ability to resist shocks is tested by repeated blows
applied by a weight falling on the middle of the bar, the bar
being supported at its ends and turned over after each blow.
In practice the elastic limit of the material should not be
approached when the loads are applied, but the information
gained during tensile testing after the elastic limit is reached
is valuable for determining the qualities of the material.
During testing, the loads should be applied at a fairly uniform
rate and without shocks the material should not be given a
;

rest at any time after loading has once started, as this is liable
to alter its qualities.
Ultimate strength.The breaking stress, or ultimate strength
is measured by dividing the breaking load by the
of a material
original sectional area of the piece. This is always done for
engineering purposes, although it is fictitious, as owing to the
contraction at the fracture, the actual area over which the
breaking load is distributed is smaller than the original area,
and therefore the stress on the section at the fracture is higher
than would be shown by the above calculation. It is convenient,
however, in practice, to measure the ultimate strength as stated ;

for we wish to know what load would break a given piece in


order, by making the actual load a certain fraction of the break-
ing load, to prevent that occurring. Thus, if we know that
the ultimate strength of mild steel is 30 tons per square inch,
then a load of 37*5 tons would be required to break a bar
2" by section. To prevent this, we arrange that the actual
load shall be, say, one fifth of this, or 7*5 tons.

Factor of safety. The factor of safety is the ratio of the
breaking load to the working load. Thus,

Factor of safetyJ
= r a I .
3.,

working load
The magnitude of the factor of safety to be used in any given
case depends on the nature of the loading. A low factor of
safetymay be employed where the load is steady, or is applied
and removed very gradually. High factors of safety are em-
ployed where the load is suddenly applied, or where the loads
76 APPLIED MECHANICS FOR BEGINNERS.

are pushes and pulls alternating. The factor of safety allows a


margin for shocks and for our ignorance of the possible loads a
structure may have to bear.

Fatigue of materials. It is well known that a load which
would not break a piece if gradually applied, will ultimately
fracture it if applied and taken off many times. A. still smaller
load will produce the same result if continually applied as
alternately push and pull. Material treated in this way is said
to get into a state of fatigue, although it is difficult to say
exactly what happens Bauschinger found that a piece of
to it.

Bessemer mild steel plate which had an ultimate strength of


28*6 tons per square inch with the load gradually applied, broke
when a stress of 15 *7 tons per square inch was continually
applied and removed, and that a stress of 8 '6 tons per square
inch applied alternately as push and pull effected the same
result. Another specimen of the same plate having a constant
stress of 14*3 tons per square inch broke when an additional I
stress of 9*5 tons per square inch was continually applied and
taken off. This last case approximates to the conditions in an
actual structure. Other materials showed similar results.

Factors of safety used in practice. In practice, live loads,
such as the weight of a locomotive and train on a bridge, are
usually doubled and added to the dead load which consists of
the weight of the bridge itself. The working stress allowed in
bridge work of steel having an ultimate tensile strength of 30
tons per square inch ranges from 4 to 7^ tons per square inch,
the lower value being used for parts which are alternately
pushed and pulled. These stresses correspond respectively to
factors of safety of 7^ and 4. Pieces of wrought iron or steel
subjected to shocks should have a factor of safety of from 10 to
12. For from 5 to 15 this
cast iron the factor of safety ranges ;

material not suitable for withstanding shocks and the latter


is

factor is used for such cases. Timber is liable to sag under


loads, consequently factors of safety of from 8 to 20 are used.
Effect of heating and cooling. The injurious effects of
may be got
overstraining, or of repeated applications of loads
rid of by This consists of heating to redness and
annealing.
then cooling slowly. Bars and plates are usually partly
annealed on coming from the makers. This is due to them
STRESS. STRAIN. ELASTICITY. 77

leaving the rolls hot and then cooling down fairly slowly. Cold
rolling and wire drawing produces hardness by setting up over-
strain in the material. This can be got rid of by subsequent
annealing. Crane chains are occasionally passed through the fire
so as to anneal them and restore their original qualities.
Copper is hardened by mechanical treatment such as wire-
drawing or bending it may be softened and its ductility
;

restored by being heated to redness and plunged into cold water.


Steel containing more than 0*2 per cent, of carbon is made very
hard by the same treatment.

Tensile tests on wires. Experiments on pulling wires of
various materials until they break are very instructive and are
easily performed by students. It has already been shown how
the extensions of a wire may be measured for varying loads
within the elastic limit. The same apparatus (Fig. Ill) may
be used, after the elastic limit is passed, if a long scale, divided
in inches and tenths, is clamped to a firm support and arranged
so that the testing weights may move past it as they descend
while the wire is extending. Use the" vernier until the elastic
and the scale after. The slight error produced
limit is reached
by any drooping of the support after the material begins to
draw out can be safely neglected as it will be only a very small
percentage of the total extension.

Expt. Arrange apparatus as described above and carry out
tensile tests on wires of copper, brass, iron and steel. In each
case, note throughout the test the extensions produced by
gradually increasing loads. Plot these on squared paper and
work out the results of the tests, using the same method as has
been employed in the following record of a test on a copper
wire.
Tensile Test on Copper Wire.
Length, 9' 8j".

Diameter, 0036".
The load was applied in 2-pound increments. Up to the
I elastic limit the extensionswere measured by a vernier on the
test wire moving over a scale on another wire hung from
the same support. The vernier read to O'Ol". After the elastic
limit the extensions were measured by a long boxwood scale,
clamped to a fixed support.
78 APPLIED MECHANICS FOR BEGINNERS.

Load, Scale reading, Extensions,


inches.
Remarks.
lbs. inches.

TThis load was the weight of


1 3 00 o-oo
\ the hook.
3 3-02 02
5 3-03 003
7 3 05 05
9 3-06 0-06
11 3-08 0-08
13 3-09 009 Elastic limit reached.
15 3 2 0*20
Scale changed to boxwood rule.
19 3 4 4
21 3 9 9
23 4-8 1*8
25 60 30
27 7 6 4 6
29 9 6 6 6
31 12 9
33 149 11-9
35 19-0 16-0
37 26 23-0
39 32 29 Wire broke.

Columns 1 and 3 plotted give the complete curve as shown


in Fig. 113. The curve up to 15 lbs. load has also been plotted
in Fig. 114 to an increased scale of extensions.

**
*nn
&R0

at


eiAsnc
umr.

1 S i9 is i l i J

i
XTUSI0K

Fio. 113. Plotted load-extension diagram for a copper wire.


STRESS. STRAIN. ELASTICITY. 70

Calculation of results. Area of wire=7r =0001018 sq. inch.


Load at elastic limit = 13 lbs.
13
Elastic limit = = 12,770 lbs. per sq. inch
0-001018
= 5*7 tons per sq. inch.
Breaking load = 39 lbs
39
= 38,310 lbs. per sq. inch
0001018
= 17*1 tons per sq. inch.

~M2 ThR Tm fit W


Fig. 114. Elastic portion of Fig. 113 to an increased scale of extensions.

For Young's modulus, 12 lbs. load gives extension -


09" on a
length of 9' 8".
12
Stress = = 10,800 lbs. per sq. inch.
Qj0()1Q18
009
Strain = .0000772.
1165
stress 10800
= 13,990,000 lbs. per sq. inch
strain 0*000772
= 6250 tons per sq. inch.
The total extension on 116'5" was 29", or
29
Extension = - x 100 = 24*9 per cent.
1 1 o'o ^^~
Autographic records. Wire testing machines can be con-
veniently arranged so as to automatically draw a diagram
similar to that in Fig. 113. The diagram reproduced in
80 APPLIED MECHANICS FOR BEGINNERS.

Fig. 115 was drawn by such a machine while a copper wire was
under tensile test. These diagrams are called autographic
*f*o records. In Fig. 116 copies
of autographic records for
some well known materials
have been traced on the same
sheet. The loads have been
tons

( rWirt.O-
W/ong
plotted as stresses in
per square inch of sectional
area the extensions are those
;

on 10" of the specimens. The


comparative qualities of the
i
'
t i* 6
\lrafioM
materials for which the curves
Fig. 115. Autographic record for a copper are given in this diagram can
be understood by inspection.
Stresses in a cylindrical shell. It is interesting to consider
the case of a thin cylindrical closed vessel, or shell, as it is
STRESS
Tons per sat inch

40

t>*
*** HCMS
.'" sm St,
1"
...w fTZMCTAL }_
^>-*zjoS*s< irc //roVx
f^ 1
'4

5
\fj\3
J Toppe* *>

7
5 Inches
Extensions on 10'

Pig. 116. Autographic records for some well-known metals.

called, subjected to fluid pressure internally. As we shall see in


Chap.XVII.,the fluid pressure everywhere on the internal surface
is perpendicular to the surface of the vessel. Referring to Fig. 117,
Let d= diameter of shell, inches ;

p = fluid pressure, pounds per square inch ;

t = thickness of the plate, in inches ;

P= total pressure on end of vessel, then


STRESS. STRAIN. ELASTICITY. si

The material of the cylindrical part of the vessel is therefore


subjected to pull forces equal to P, and will consequently be

Section at AB. [B Longitudinal Saturn.

Fig. 117. Cylindrical shell under internal fluid pressure.


under tensile stress. To find the amount of this stress, the
total pull P is divided over the area of the section AB. This
area is given approximately by
Sectional area = circumference of shell x thickness of plate
=irdxt.
_P
stress on section AB-
irdt
Trd*

Trdt

lbs. per square inch.

The on a longitudinal section of the shell must now be


stress
determined. The whole length of the shell need not be con-
sidered, but only the portion between two cross sections taken
one inch apart, for if this portion is taken
from the ends of the shell,
sufficiently far
the staying action of the ends becomes
negligible, and the stresses on all such
rings will be alike. The fluid pressure
on the ring is represented by the arrows
shown (Fig. 118), everywhere directed
perpendicular to the curved surface of
the ring. Take components of these, as
shown, parallel and perpendicular to the
diameter AB, and consider the portion Fio. n 8. Ring, i" broad,
of the ring above AB. The horizontal
components acting towards the right and left will balance one
another, and need not be further considered. The vertical
A.M.I!. F
82 APPLIED MECHANICS FOR BEGINNERS/

components will have a resultant R v Similarly, on the portion


below AB, a resultant force R2 will act, equal and opposite to
Rv These two forces put the material of the ring at A and B
under tensile stress, which must now be calculated.
First to obtain R. There will be no difference experienced in
the equilibrium of the ring if we imagine it to be filled up to
the level of AB with cement. The pressure on the surface of

Fig. 119. Fig. 120.

the cement will be perpendicular to AB (Fig. 119) and the


resultant force due to this will be
Q=px area of surface of AB
=pxdx 1.
R and Q now preserve the equilibrium of the ring, there-
fore R= Q
=pxd.
Imagine the material at A and B to be cut, and that forces
y, T, are supplied at the sections in order to balance R (Fig. 1 20),
then if the stress on the sections at A and B be called q,

T= q x area cut at A or B,
=qxtx 1.
Also R = 2T.
.'. pd=2xqxt ;

q = lbs. per square inch.

It has already been found that a circumferential seam has


a tensile stress equal to -^-, consequently the stress on a longi-
tudinal seam is just double that on a circumferential seam. For
STRESS. STRAIN. ELASTICITY. 83

this reason, boilers are usually made with the longitudinal


joints much stronger than the circumferential joints.

Example. A boiler shell is 6 feet in diameter, and the metal is $'


thick. If 100 pounds per square inch, calculate
the steam pressure is

the stress on circumferential and longitudinal sections.

Total pressure on end of shell = 100 x

Area of circumferential section = irdt;

= lOOxTrd
2 100 xd
.'. Stress on circumferential section
4rrdt 4x<
100x72
4x
= 3600 pounds per square inch.

Stress on longitudinal section = 7200 pounds per square inch.

Thin flat plates, such as are used for boilers, are not able to
withstand pressure without bulging. This is easily understood
when we consider that the plates can practically only withstand
tension, not bending, and it must be remembered that a
comparatively small force P
(Fig. 121) suffices to put two
large forces T, T, out of line
with one another when P is
applied transversely to their lines of action. Flat surfaces sub-
jected to fluid pressure consequently require to be staged. The
student is referred to books on boilers for descriptive sketches
showing how this is done in different types. A spherical shell
would be self-staying, but practical considerations prevent this
shape being used for boilers, although it is occasionally used for
other vessels where no heat is to be applied.
The results obtained above are only applicable to vessels the
walls of which are thin compared to their diameters. Thick
walled vessels, such as hydraulic pipes and cylinders, have to be
considered from a different point of view, as the stresses in such
cases are not uniformly distributed over the sections of the
material as has been assumed for thin shells. The theory
involved is too complicated for beginners, and is therefore not
given here.
84 APPLIED MECHANICS FOR BEGINNERS-

Shearing action. Another kind of stress to which material


is often subjected, is called shear stress. Shear stress acts
tangentially to the surface or section of
the material. A body subjected to shear
stress tends to break at any section by
the two portions of the body sliding past
0^'M%%nM%^
/
one another. In Fig. 122, a piece of
material is shown between the blades of
a shearing machine. When the pressure
Fig. 122.
Plate under is applied, the material will fracture
shear stress at AB.
by its two portions sliding past one
another at AB. The section AB will therefore be under
shear stress.
In punching machines (Fig. 123), the piece punched out
separates from the surrounding metal by shearing. Again, in

OSl Mevation

tyarv
Fio. 123. Shear stress produced Fia. 124. Knuckle joint.
in a punching machine.

the common knuckle joint (Fig. 124), when pulls or pushes are
applied to the rod, the joint may break by the pin shearing at
two sections, the left-hand rod then carries away the centre por-
tion of the pin with it.

In all these cases, the shear stress is calculated by dividing the


total force producing the shearing action by the area of the
section of the material over which the shear is distributed.
Thus, in the punching machine,
Let force pushing the punch down = P tons ;

diameter of hole punched = d inches ;

thickness of plate = t inches ; then


area under shear = circumference of hole x t

= ird x t square inches.


p
Shear stress = -,- tons per square inch.
=

STRESS. STRAIN. ELASTICITY. 85

In the knuckle joint, if d is the diameter of the pin in inches,


a ltd? Trd 2
Area under shear =

Shear stress =
P 2P
'ire?
2
Example. Suppose two bars connected by a knuckle joint to be
pulled with forces of 2 tons. Find the diameter of the pin if the
safe shear stress is 4 tons per square inch.
2P
Shear stress '
n-d2
2x2

2x2x7
d? =
4x22 22'
d= 565 inch.

Strain produced by shear. The kind of strain produced by


shearing stress is change of shape. This can be clearly shown
by means of a thick book. Lay the book on the table and
sketch a square on its end edges as shown in Fig. 125. Then

AA
im7ZWZZ%mPWZV&VZW7Z7?

W/WM//W////WW///', '////'/////////?'/////#'/'/'/

Fig. 125. Shear strain illustrated bjT a Fig. 126. -Measurement of shear
book. strain.

apply shear stress by holding the bottom cover firmly on the


table and pushing the top cover tangentially in the manner
indicated. Every page of the book shears past the adjacent
page, and the result is to change the shape of the square to that
of a parallelogram. The strain is measured by stating the
angle 6, in radians, turned through by the vertical sides of the
square. Thus, if A (Fig. 126) comes to A' and B to B', then the
86 APPLIED MECHANICS FOR BEGINNERS.

angle BCB', equal to the angle ADA', measured in radians, is

the measure of the strain.


In engineering materials, the shear strain is always very
small, so that the angle is very small. We may therefore
measure by putting it equal to _~ radians. This assumes
that the small distance BB! is a circular arc instead of being
straight. The modulus of elasticity for this sort of stress and
strain is called the rigidity modulus, usually written C. Follow-
ing the same definition as before
~ _ stress _ shear stress applied
-
"strain radians
The average value of C for wrought iron is about 5000 tons
pei- square inch, and about 5400 for steel. The rigidity modulus
is of importance when dealing with the stiffness of shafts and

other bodies subjected to twisting.

EXERCISES ON CHAP. VII.

1. A tie bar 4" broad, " thick, is under a tension of 7 tons. Cal-
culate the tensile stress.
2. Find the working load for the bar in Question 1 if the tensile
stress is not to exceed 5 tons per square inch.
3. A
bar of square section. " edge, is 60 ft. long and is found
to stretch 0'6" when a certain pull is applied. Find the strain.
Suppose the pull applied to have been 1562 lbs., and find Young's
modulus of elasticity.
4. Suppose the tensile stress is not to exceed 4 tons per square
inch, find the diameter of a round tie rod which has to resist a pull
of 16 cwts..
5. Taking Young's modulus for wrought iron to be 29,000,000 lbs.
per square inch, what decrease in length will take place when a
column containing 12 square inches in section and 20 ft. high carries
a load of 36 tons ?

6. What load, in pounds, must be hung to an iron wire 50 ft.

long and 0*1" diameter to make it stretch -- 3770" inch.


7. An iron tie bar is 50 ft. long, its section being rectangular
4" x ". Its stretch must not exceed $f ; calculate the maximum
load it can carry.
8. Taking the shearing strength of iron to be 20 tons per square
inch, calculate the force necessary to punch a %' hole in a " plate.
Find also the stress on the punch.
)

STRESS. STRAIN. ELASTICITY. 87

9. Suppose it were attempted to punch a |" hole in the plate


of Question 8, make a similar calculation.
10. A
copper wire, previously pulled beyond the elastic limit,
was tested again, after 15 hours' rest, under tension. Length of
wire, 12' 3", diameter of wire 0*036". The following results were
obtained :

Scale Extension for


Load,
reading differences of Remarks.
(lbs.).
(inches). 2 lbs. (inches).

3 03 Load marked zero in column 1was


2 3 04 o-oi actually a load of 4 lbs., hung
4 3 06 002 on to keep wire taut.
6 3 08 002
8 310 002
10 312 0-02
12 314 0-02\
Elastic limit reached.
14 3 65 051/

Plot columns (1) and (2) on squared paper. Calculate the value
of Young's modulus for this sample of copper wire.
11. What do you understand by the terms tensile, compressive,
and shearing strength respectively of any material ? Define
"modulus of elasticity." If a wrought iron bar of 1 square inch
sectional area just breaks under a tensile stress of 60,000 lbs., what
would be the area of the section of a tie-rod which would just
support a load of 20 tons ? ( 1896.
12. How would you behaviour of steel
find out for yourself the
wire loaded in tension till it breaks. What occurs in the material?
Use the words stress and strain in their exact senses. (1897.)
13. What do we mean by stress, strain, and modulus of elasticity?
A wire 10' long and & sq. inch in sectional area is hung vertically,
and a load of 450 lbs. is attached to its extremity, when the wire
stretches 0*015" in length. What are the stress and strain respec-
tively ? And also the modulus of elasticity ? (1899.)
14. An
iron wire is loaded with gradually increasing tensile loads
till breaks.
it We
want to know its modulus of elasticity, its
elastic limit stress and its breaking stress. What measurements
and calculations do we make ? (1900.)
CHAPTER VIII.

STRENGTH AND STIFFNESS OF BEAMS.


Bending of a beam. Beams are parts of a structure, usually-
supported horizontally, for the purpose of carrying loads
applied transversely to their
lengths. Suppose we have
a beam consisting of a num-
ber of planks of equal
Fig. 127. Bending of a loose plank beam.
lengthslaid one on the
other, and supported at the ends, Aload IF, applied at the
centre of the span, will cause all the planks to bend in a
similar fashion, and consequently, as the lengths of all the
planks will remain the same, they will overlap at the ends
as shown (Fig. 127). Strapping the planks firmly together will
prevent this occurring and the beam will now bend as a whole, the
ends of the planks remain-
ing in one plane (Fig. 128).
Assuming the middle plank
to remain the same length
Fig. 128. -Bending of a strapped plank as at first, it is clear that
beam.
the top plank, and all those
above the middle must have become shorter, and those below
the middle one, longer than at first. The planks above
the middle must therefore have been subjected to compressive
stress in the direction of their length and those below the
middle to tensile stress in the same direction. The further we
get away from the middle, above or below, the greater will be
the change of length of the planks and therefore the greater
will be the compressive and tensile stresses producing these
STRENGTH AND STIFFNESS OF BEAMS. 89

changes of length. An ordinary solid metal or timber beam


may be looked upon as being built up of a large number of
fibres cemented together, corresponding to the planks in our
model beam. These fibres will be subjected to stresses in the
same manner as the planks. Therefore, in a loaded solid beam,
such as that in Fig. 129, the upper fibres will be under com-
pressive stress and the lower ones under tensile stress. These
stresses are shown at the section AB.

J B

Fig. 129. Tensile and compressive Fio. 130. Shear at the section AB.
stresses on the section AB.

Further, the action of P, the pfessure of the support, acting


on the left-hand side of the section AB, and of Wand Q acting
on the right-hand side of the section, will be usually to give the
material a tendency to slide past at the section as in Fig. 130.
The material at the section will therefore be under shearing
stress as well as the stresses mentioned above.
Models showing the forces in the material.These
stresses may be very well understood by examining models
such as those illustrated. Fig. 131
shows a cantilever, that is, a beam
fixed at one end only and free at
the other end. The cantilever has
been cut at AB. In order to balance
the portion outside of it is AB
necessary to put a cord connection
at A and a small strut at B. The
pull and push forces thereby
supplied counteract the bending
tendency. In addition, an upward
Fio. 131.Model of a cantilever, cut
force S has to be supplied to balance to show the forces at AB.
the tendency to shear. In the
uncut cantilever these forces would be supplied by the resis-
tance of the material at the section. It will be noticed in
the cantilever that the tensile stress occurs above, and the
90 APPLIED MECHANICS FOR BEGINNERS.

compressive stress below, the middle. This is just the reverse


of what we have seen for a beam supported at both ends.
Fig. 132 shows This particular model consists
this latter case.
of an I section, supported at the ends and cut into two pieces.
A compression spring balance at the upper flange of the beam

Fig. 132. Model of a cut beam.


and two ordinary spring balances at the lower flange enable the
forces at the section, produced by bending,
to be measured.
Shearing is balanced by weights applied as indicated, the
;

upward force being supplied by means of a cord passing over a


pulley above. ;

If we again examine the model beam built up of planks, we


may notice that when it is bent and the ends of the planks
overlap, that they have done so by the planks sliding on one
another in the direction of their lengths. The straps obviated
this by binding the planks firmly together and so preventing
this sliding, or shearing action taking place. So also in a solid
beam there are shearing stresses distributed over horizontal
longitudinal sections.
The actual distribution and calculation of these stresses is
beyond the scope of this book, but be said to
sufficient will
enable the student to solve many simple practical problems.
Bending moment. The bending moment at any section of a
beam is measured by the moment about that section of the
applied forces tending to turn the portion of the beam on the

STRENGTH AND STIFFNESS OF BEAMS. 91

right or left hand of the section. For example, in the beam in


Fig. 133, carrying a load of 10 tons at the middle of a 12 ft. span,
the bending moment at the middle section will be found by
considering the forces acting on the right hand or left hand
side. If we neglect the weight of the beam itself, there is only
one upward force on either side, and its moment about the

E_t
Fig. 133. Fig. 134.

middle section is 5 x 6 = 30 ton feet. The bending moment at


the middle section is therefore 30 ton feet. To find the bending
moment at A (Fig. 134), 3 ft. from the left-hand support
consider the left-hand portion of the beam, the only force is

an upward one of 5 tons, and its moment about A is 5x3 = 15


ton feet. If we consider the right-hand portion, there are two
forces to be dealt with, an upward one of 5 tons and a down-
ward force of 10 tons. Their moments about A being contrary
to one another, the total moment about A will be (5x9)- (10 x 3)
= 15 ton feet, as before. The bending moment at A will there-
fore be 15 ton feet.
In any given case therefore, proceed thus first, from the given :

loads and dimensions, calculate the reactions of the supports ; then,


to find the bending moment at any section,
calculate the algebraic sum of the
moments about the section of all the
forces acting either on the right-hand or
left-hand portion of the beam.
Shearing force. To calculate the
total shearing force at any section such
as AB in Fig. 135, notice that if the \
beam were cut at A B we should have
to balance the left-hand portion by
applying, at AB, a force <S\ = \Y1 -P;
to balance the right-hand portion, a
force S., = Q~ W., must be applied at
AB. These forces 8 l and S 2 in the uncut beam are mutual
92 APPLIED MECHANICS FOR BEGINNERS.

shearing actions of the two portions of the beam at the section,


and therefore must be equal to one another. In any given case,
therefore, we calculate the total shearing force at any section toy
taking the algebraic sum of all the applied forces acting either on
the right hand or left-hand portion of the beam.
It is customary to describe bending moments which cause a
beam to bend convex in a downward direction as positive, and

G J
Posmve Bending Negative Bending

Fig. 136. Fig. 137.

convex in an upward direction as negative. These are shown


in Figs. 136and 137. Shear which causes stresses of the kind

7T0HS
A ^cl
I
HcoativeShem

Fig. 138. Fig. 139.

shown at two sections close together AB and CD (Fig. 138), we


will call positive, and if of the kind as shown at EF and OH
negative.

Example. A beam A B, 12 ft. span, supported at the ends, carries


loads of 2 tons and 7 tons as shown in Fig. 139. Calculate the
Bending Moment and Shearing Force at the centre of the span,
neglecting the weight of the beam.
To obtain the reactions of the supports, taking moments about B.
Pxl2 = (2x8) + (7x5),
P = *i = 4-25 tons.
Taking moments about A (as a check on working),
Qxl2=(2x4) + (7x7),
Q = 4| = 4 -75 tons.

P + Q = 4 25 + 4 "75 = 9 tons = sum of loads.


#
STRENGTH AND STIFFNESS OF BEAMS. 98

Imagine the beam cut at G and consider the left-hand portion.


Bending moment at G=(P x 6) - (2 x 2)
= 4-25x6-4
= 21-5 ton feet ; positive.

Or, by considering the right-hand portion,


Bending moment at G=(Q x 6) - (7 x 1)
=475x6-7
= 21 '5 ton feet ; positive.
These results agree.
For shearing force, considering left-hand portion,
Shearing force at C=P-2
= 4-25-2 = 2-25 tons.

Or, by considering the right-hand portion,


Shearing force at G=Q-1
= 4-75-7= -2-25 tons.

The effect of these shearing forces


sections close together at G under
is to put
stresses as indicated
two
II
Shear Posnwe

in Fig. 140. The shearing force is therefore positive. Fig. 140.

Beam, load distri-


buted. A
beam carry-
ing a distributed load Smur Posjnvc shiak namrt
A DlSTKIBUTtO LOAO /JJH UK fOOTHUH.
must now be studied.
Fig. 141 shows a beam
supported at its ends,
20 ft. span, with a uni-
formly distributed load
1 ton per foot length.

The reaction of each


support will be 10 tons.
To calculate the bending
moment at any section
A, consider, say, the
left-hand portion of the
beam. There are two
forces acting on it, 10
tons upwards from the
support, and W, equal Fig. 141. Bending moment and shearing force
diagrams for a beam carrying a distributed load,
to the weight of the
distributed load on this portion of the beam, acting downwards
94 APPLIED MECHANICS FOR BEGINNERS.

at its centre of gravity. Supposing A


is 4 ft. from the left-hand

support, then W is equal to 4 tons.


Notice also that the turning
tendencies about A, of the reaction of the support and of W, are
contrary.
Bending moment at A =(10 x 4) ( Wx 2)
= 40 -(4x2)
= 32 ton feet ;
positive.
The shearing force at A will be = 10 - W
= 10-4
=6 tons.

The shearing force puts two adjacent sections at A under


stresses as shown and is consequently positive.
It is interesting in this case to calculate the Bending Moments
and Shearing Force at several sections and then plot the results.
Doing this for sections at intervals of two feet (Fig. 141) we get,
Bending moment at =
2andl8 = (10x 2)- (2 x 1) = 18 ton feet
4andl6=(10x 4)- (4x2) = 32
6andl4 = (10x 6)- (6x3) = 42
8 and 12 = (10 x 8)- (8x4) = 48
10 = (10xl0)-(10x5) = 50
Shearing force at = 10 tons
2 = 10- 2 = 8 tons
4=10- 4 = 6 >

6 = 10- 6 = 4
8=10- 8 = 2
10 = 10-10=
12 = 10-12=- 2 V
14 = 10-14=- 4 J

16 = 10-16=- 6 5)

18 = 10-18=- 8
20=10-20= -10
These results plotted as shown in Fig. 141 give Bending
Moment and Shear Diagrams. It will be noticed that the curve
in the Bending Moment Diagram is parabolic, and in the Shear
Diagram, a straight line. Notice that the bending moments
increase towards the middle of the span, and that the shearing
force diminishes towards the middle, where it is zero.
STRENGTH AND STIFFNESS OF BEAMS. 95

A common practical section. The strength of a beam of


I section may be calculated on the assumption that the
flanges supply all the resis-
tance to bending and the
web all resistance to shear-
ing. This assumption leads
to results which differ only
very slightly from the more
correct ones for sections s-p
Ma Imcki
,,ASQl
used in practice. Consider
a section in which the area
~V*r v
ASy/*os
|of each flange is .4 square
inches and the distance from Fio. 142. Beam of I section.

centre to centre of the


flanges D inches (Fig. 142). Let /= stress allowed on the
flanges, in tons per square inch. Then fxA = greatest force
which may act on one flange.
These forces fA (pull), fA (push), give a couple, the moment
of which is fA x D inch tons,
aDd this is the resisting
moment of the section. The
Bending Moment any
at
section of the loaded beam
must not exceed this quan-
tity.

Ex ample 1. Suppose we
have a beam 12 ft. long,
supported at its ends, I sec-
tion, 8" in. deep, 4" broad, Fio. 143.
metal of flanges V' thick. Find
the maximum load at the centre, if the stress due to bending is not
to exceed 5 tons per square inch.

Let IF = load at centre in tons.

Reaction of each support ^-g-


W
=^
W x 6 ton feet,
Bending moment at centre of span

= 3Wxl2
= 36. IT ton inches.
96 APPLIED MECHANICS FOR BEGINNERS.

Resisting moment of section =fAD


= 5x(4xi)x7
= 75 ton inches.
Bending moment = resisting moment
36 JF=75,

W = f| = 2 -08 tons.
Example 2. The shear on any section of the beam in the last
Example will be 1'04 tons. Suppose 4 tons per square inch to be
the shear stress allowed, what thickness of web is required ?

Shear = shear stress x area of web section,


1-04 = 4 x 7 xt,
1 -04.

Webs are not made so thin as this in practice, because there is

buckling to be guarded against as well as shear. In the case of this


beam, the actual thickness would be probably "
or -jV'.

Some other practical sections. Rolled steel beams are


usually made of the section considered in Example 1, above,
the flanges being made equal as there
J shown. This form of section gives equal
stresses on each flange. Cast iron beams
have the tension flange of larger area
than the compression flange (Fig. 144),
since this material is very much stronger
under push than under pull, and the
effect is to reduce the magnitude of the
tensile stress. A common practice is to
1 1
make the tension flange of four times the
Fig. 144. Section of a sectional area of the compression flange,
cast iron beam. ....
which gives a
.,
tensile stress
.

approximately
. .

equal to one quarter of the compression stress. As the larger


flange has to be under tensile stress, this must be arranged for
in placing the beam Thus, a cantilever will have
or cantilever.
the larger flange uppermost; and in a beam supported at the
ends, this will be the lower flange.
STRENGTH AND STIFFNESS OF BEAMS. <>7

The resisting moment of other two sections may be stated.


Let /= maximum stress allowed, in tons per square inch.
Then for a rectangular section, b inches broad, d inches deep
(Fig. 145),

Resisting moment 7; ton inches.

For a circular section, r inches radius (Fig. 146),

Resisting moment =^-j ton inches.

Modulus of the section is the name given to that quantity by


which the safe stress must be multiplied in order to give the

v^sy

Fig. 145.
H
WWVxs

'WA*
Fig. 146. Fig. 147. The resistance to shear supplied
by a diagonal chain or prop.

resisting moment of the section. The letter Z is usually taken


to represent the modulus of a section . Thus, in the case of the

rectangular section above, 2T= ; and for the circular section,


3
71-r
Z=
Distribution of material in girders.Turning again to the
model cut cantilever, if we remove the cord and weight which
supply the resistance to shear, and substitute a diagonal cord
or prop as shown in Fig. 147, we find that the cantilever will be
in equilibrium at the cut part. This gives us information as
A.M. 15. G
98 APPLIED MECHANICS FOR BEGINNERS.

to the
functions of the parts in a girder (Fig. 148). The
horizontal flanges, or booms as they are called, supply the
required resistance to bending, the top one in a girder sup-
ported at its ends being under push and the bottom one under

IT U TT
Fig. 148. Bridge girder.

pull. The diagonal bars supply the required resistance to


shearing, being under push or pull depending on the direction
in which they are inclined.
As we havealready seen in the case of the beam supported at
its ends and carrying a distributed load, the bending moment is

greatest at the middle of the span and diminishes to zero at the


ends, while the shearing force is greatest at the ends and
diminishes to zero at the middle. In consequence of this,
the booms of a lattice girder should be thicker towards the middle
in order to supply a greater resistance to the bending moment,
and the diagonal parts stronger towards the ends in order to cope
with the larger shearing forces there. The same thing is also

mm
generally done with large plate girders, that

Fig. 149.Plate girder.

of plates riveted together with angle irons (Fig. 149).


is,

the flanges supply almost all the resistance to bending and the
girders built

In these,
up

web almost all the resistance to shearing. We


generally find
that the flanges at the middle consist of several plates riveted
together, these being gradually reduced in number as the ends
are approached. The webs are often made of thicker plates
near the ends and thinner ones near the middle. The large
thin plates of which the webs of these girders are constructed
are liable to bulge or buckle ; to counteract this tendency it is

usual to stiffen them at intervals by vertical angle or T sections


riveted to them.
STRENGTH AND STIFFNESS OF BEAMS. 99

Six standard cases of beams. The student will have no


difficulty in understanding and working out for himself the
Bending Moments and Shearing Forces for the first four of the six
standard cases of beams on pp. 100-101. The process is exactly
similar to that used in the example of the beam supported at
both ends and loaded with a uniformly distributed load. The
Bending Moment and Shear Diagrams should be drawn from the
calculations. The last two cases require a more difficult theory
for their explanation than can be discussed in this book. For
the sake of comparing the strengths and stiffnesses of the beams
with different methods of supporting and loading, it is assumed
in the following results that the materials, total loads, and
lengths are the same, and that the sections are uniform in
dimensions and rectangular.

Beams of similar sections. Beams made of the same
material and having similar sections, but with varying dimen-
and span, are found to have strengths
sions of breadth, depth
and approximately with the following laws,
stiffnesses agreeing
the loading and manner of support being similar in the com-
pared beams.
Strength is proportional to the breadth, to the square of the
depth and inversely proportional to the length.

Stiffness is proportional to the breadth, to the cube of the depth


and inversely proportional to the cube of the length.
Strength is measured by the load which the beam will carry,
stiffness by the reciprocal of the deflection of the beam under a
given load.
These rules, taken together with the comparative strengths
and stiffnesses given in the table (pp. 100-101), enable us to
solve many practical problems.

Example. Suppose it is found that a beam of cast iron 1" broad


x 1" deep x 36" between supports breaks with a load of 6 cwts. at its
centre. Calculate the breaking load at the centre of the span for a
beam of cast iron 1" broad x 3" deep x 48" span.
Expressing the above rules for strength in proportional form

w
" w
1
-Mi
r*%.


j
2
.

M22
jr

100 APPLIED MECHANICS FOR BEGINNERS.

is

lit
&C 5

tti


o o

go bo
=i

/
/

J
p A

>
CO
1

/
^U .,
\
IE

S - so
II II II s ^ c t,

II o _" -g
o
CO o
|
a o OB
S3 & 3 ,3 O +s

H !<M
1
^ ,9
%* g S
HCS HN

^
H05 _^3
II II
3 II II II
*
e II
^ II II a>
6 g
o
* 1 i ts 1
a

*T3 a _
c3 O

I 1
r^ 2
3 d
fell
n3.2 a>
a*
BJ r
STRENGTH AND STIFFNESS OF BEAMS. 101

>-.
t.

A
/
1 A,
1 *
/> t
r" =e .

2
/
_+ J
/
! i \/
w S # y hr
w O
-a
o P O 9 M S3 p
N o a, S3 - X 03 03IS ^
.2: 'c- S
31- - ^-e, &a *
II
S II
Hog ^ !l
I o J3 o ft * _r^2
c * ^
c
s
S ,-r fe
!; o ;
e3 b
c,2
l. g ?2
eg ^ p o -S ?

HI<N

ii ii

3 g * *
*! 2* 25
a s -*
ii

<n s
8 p- = o c
> p. S3
II II
g II II II
3 33 P. S 8
- eg ii -s
O O o
" s oo

S *
*j S P. a
II II

s c
to -o-
S3
= S3 -^

"ill
102 APPLIED MECHANICS FOR BEGINNERS.

Suffix 1 refers to the given case and suffix 2 to the one to be


worked out ; this gives

5
* ^36" *
~tt~~'

48

6 x 3 x 9 x 36
2x48
60f cwts.

Using a factor of safety of 15, about 4 cwts. would be a safe


load for this beam.

Commercial tests.
Cast iron is generally tested for commer-
cial purposes by supporting pieces at a known span and
ascertaining what load at the centre of the span will break
them. The test pieces are generally 36" span, 1" broad and 1"
deep, or, better, 1" broad and 2" deep. The breaking load at the
centre of the span for pieces having the first dimensions ranges
from 6 to 8 cwts. and for pieces having the second dimensions
from 25 to 35 cwts.
Timber is also generally tested by bending. Test pieces of
large section are desirable as the effects produced by local flaws
are thus minimised.

Let W= the breaking load at centre of span, in tons.


Z = span in inches.
b = breadth in inches!
of rect lar secfcion .

a = depth in inches J

Then c = 3- WL
f-^ is a quantity which is called the Modulus of

Transverse Rupture, which may be defined to be the result of the


calculation from this formula, using experimental data actually ;

it has no meaning physically, although sometimes an erroneous

one is applied to it. Worked out values of c from the results


of tests enable us to calculate the breaking load of similar
pieces of the Thus, suppose we take the value
same material.
of c for cast iron to be 16 tons, and it is required to calculate
'

STRENGTH AND STIFFNESS OF BEAMS. 103

what central load would break a cast iron beam 36" span 1" broad
and 2" deep then;

16 =
3 W.L
2 b.d*

3 TTx36
"2 X1x4
or
16x8
W--
3x36
= 1-18 tons.


Experiments on beams. Students should carry out for
themselves some experiments on the stiffness and strength of
beams. Metal beams are best tested to breaking in a large

Fio. 150. Apparatus for experiments on the stl ffness and strength of beams.

testing machine ; wooden ones having a section l"x 1" and 36"
span can easily be broken with apparatus similar to that
described below, and the same apparatus will do for experiments
on the deflection of both metal and timber beams. The
apparatus (Fig. 150) consists of a lathe bed fitted with two or
three cast iron brackets which can be clamped anywhere to it.
These brackets are arranged to receive either steel knife edge
supports resting in V slots cut on the tops of the brackets, or
104 APPLIED MECHANICS FOR BEGINNERS.

cast iron caps held down by studs. The knife edges are used for
beams simply supported. The caps are employed for screwing
down on to the beam so as to fix it, the knife edges being first
removed. A wrought iron stirrup, with a knife edge for resting
on the beam, carries a hook for applying a load anywhere to the
beam. Deflections may be measured in various ways. If timber
is being experimented on, a pointer, fixed to the stirrup and

moving over a scale clamped to a support, suffices, as the


deflection is usually considerable. Or a light lever may be
used, pivoted to a fixed support and attached by a fine wire at
its shorter end to the stirrup, the longer end moving over a

fixed scale as the beam deflects. This lever has arms having a
ratio of 1 to 10, so that the deflection is multiplied 10 times at
the fixed scale. Using a scale of inches divided into tenths, the
deflection with this apparatus may be easily read to ^^ inch.
For very fine measurements a micrometer microscope is used,
the readings being taken from the movement of the stirrup as
shown by a fine silk fibre mounted on it. This instrument is
shown in the illustration and need not be described here.
Pieces of tool steel of various breadths, depths and lengths
form useful examples for verifying the comparative stiffnesses
of beams. These may be used as beams supported at the ends,
or as cantilevers, or as beams fixed at both ends. All the
foregoing numbers given for comparative stiffnesses may be
verified by use of these samples. Itmust not be expected,
however, that they will be arrived at absolutely by experiment,
but the results, if the experiments are carefully done, should
agree closely with them. Deflection tests also form a very
convenient method of approximately determining Young's
modulus of elasticity for a given material, as, using simple
means, much larger pieces can be tested by bending than by
direct pull, the latter test requiring the use of a large
machine.

Let TF=load applied, in lbs.

L = length of span, in inches.


D = deflection produced by W in inches.
y

b = breadth ^ of beam of rectangular section,

d= depth / both in inches.


STRENGTH AND STIFFNESS OF BEAMS. 105

Then, if the test piece is used as a cantilever with the load


applied at the end,
E=4
D b
.

.
= .
-
tf
3
lbs. per square inch.

And, if the test piece is used as a beam simply supported at both


ends, with the load applied at the middle of the span,

jf-i.
WU
4* D.b.d?
The theory involved in these equations for E is too complicated
to be dealt with here.
Test bars of ductile material. Good qualities of wrought

3" 4 "
01

Fig. 151. Autographic record of a Bessemer steel bar under bending test.
and mild steel bend over double without breaking, and con-
sequently experiments on their bending strength always stop
short after the piece has bent .

through any convenient angle.


Fig. 151 shows a copy of an auto-
graphic diagram obtained while
a bar of Bessemer steel, 24" span,
2 735" deep, 0873" thick was
under test in a 50-ton machine.
The bar was tested on edge,
simply supported at the ends
and the load applied at the
centre of the span. Ordinates
in this diagram give these loads,
abscissae the deflections corre-
sponding to them. The test was
stopped when the deflection Deflections
reached 5^ inches. Fio. 152. Autographic record of a cast
iron bar under bending.
Cast iron test bars 36" span,
1" broad, 2" deep usually have a deflection of from 035" to 04"
106 APPLIED MECHANICS FOR BEGINNERS.

at the middle of the span at breaking. The diagram usually


resembles that shown in Fig. 152. It is interesting to note
the effect of notching a bar of cast iron. For this purpose two
cast iron bars, 1" x 1" section

* . 1
C ay .
2E
i
everywhere but at the centre,.
were prepared. One of these, A
(Fig. 153), had a sharp-bottomed

j b !;
V notch cut in it, the other one,
A A . B, was thinned down
X;
[
25 gradually
< 36' 1
over a length of 3 Vat the middle.
^fi^B$&Erii?g t^ sectional areas both bars
at the middle section were the
f

same. A broke with a load of 2 '6 cwts. at the centre of the


span, the total deflection being 0*23". broke with a load B
of 4 cwts. at the middle, the total deflection being 0'44". The
inference from this is, do not have a sudden change of section
in a beam unless it is required to break easily.
Timber test bars. Samples of various timbers, 36" span,
1" x 1" section should be tested to breaking by the student
himself. The deflection
corresponding to equal
increments of loading
should be measured and
diagrams should be
plotted from these re-
sults to show the
behaviour of the various
pieces. The results for
such small samples of
the same timbers will
be found to vary con-
siderably, owing to the
very different qualities
found even in pieces cut
"TZZtSiimSS. from the same plank.
Fig. 154.
ways
Curves
;
of beams loaded in various
drawn from a bent knitting needle.
mi
-

v.~i/i
Lne P ieces asnouia De u
selected, as far as possi-
ble, freefrom knots, shakes, and flaws of any kind, and should
be as straight in the grain as can be obtained.
STRENGTH AND STIFFNESS OF BEAMS. 107


Curve of a bent beam. The actual shape of the curve in
which a given beam will bend may be examined by the student
by using a long thin knitting needle. This needle, placed on a
sheet of paper secured to a drawing board and "loaded" by
means of drawing pins pushed into the board where the
supports and applied loads would come in the actual beam, will
enable the curve to be drawn. A few cases are shown in
Fig. 154. Of course it must be remembered that the deflections
in the actual beams will be much smaller in magnitude.

EXERCISES ON CHAP. VIII.

1. A timber beam, 10
ft. span, supported at its ends, carries a
load of 600 at its middle.
lbs. Calculate the Bending Moment and
Shearing Force at places (a) very near its ends, (6) very near its
middle.
2. Acast iron cantilever projects 6 ft. from a wall and carries a
load of 500 lbs. at its end. Calculate the Bending Moment and
Shearing Force (a) at the wall, (6) at its middle.
3. A beam 20 ft. span, supported at its ends, carries a load of
2 tons at its middle, also loads of 1 ton at places 5 feet from each
end. Neglect the weight of the beam and calculate the Bending
Moment at each load, and also the Shearing Force at a place 6 ft.
from one end.
4. A cantilever projects 8 ft. from a wall and carries a load of
400 lbs. uniformly distributed. Calculate the Bending Moment and
Shearing Force at intervals of 2 ft. throughout its length.
5. Abeam 12 ft. span, supported at its ends, carries a uniformly
distributed load of 2,400 lbs. Calculate the Bending Moment and
Shearing Force at the middle and at 3 ft. from each end.
6. A beam span, section 6" deep, 3" broad,
of I-section, 10 ft.
metal ^" thick, is ends and has to carry a load at
supported at its
the centre of the span. Find this load if the maximum stress due
to bending is not to exceed 5 tons per square inch.

7. A beam of I-section, 25 ft. long, supported at the ends, has a


totaldepth of 15", and its top and bottom flanges are each 6"x".
Allowing a stress of 4 tons per square inch, find the load which the
beam may carry at its middle. Neglect the weight of the beam.
8. A girder, span 60 ft. weighs 3 tons and is supported at its
,

ends. A load of 1 ton per foot length is distributed over 20 ft.


length of the beam from one end. Find the pressures on the
supports.
108 APPLIED MECHANICS FOR BEGINNERS.

9. Calculate the bending moment at the middle of the span of


the girder in Question 8, and find the sectional area of each flange
there, if the stress allowed is 4 tons per sq. inch, the depth of the
girder being 6 ft.
10. Acast iron cantilever has a mean depth at wall of 12". The
large flange is 10" x 1", and the small flange 3" x 1". The canti-
lever projects 10 ft. and carries a uniformly distributed load of J ton
per foot length. Find the stresses in the top and bottom flanges at
the support.
11. Acast iron cantilever 1" long, 1" broad, 1" deep breaks with
a load of 30 cwts. at its end. Calculate the safe load for a canti-
lever 4" broad, 1J" deep, 3" long, taking a factor of safety = 12.
12. A bar 2 ft. long, 2" x 2" square section is supported at its ends
and breaks when a load of 3 tons is placed at its middle. Calculate
the working load of a bar 5 ft. long, 6" deep, and 4" broad, taking
a factor of safety = 12. What distributed load would be safe ?
13. A wrought iron shaft has its supports 5 ft. apart and carries
a load of 2 tons at the middle. If the shaft is 4" diam., what is
the maximum stress due to bending? Neglect the weight of the
shaft.

14. A bar of mild steel, section 1" x 1", is supported at points


40" apart. A load of 10 lbs. being applied at the middle of the
span, the deflection is observed to be 0"0053". Calculate the value
ofE for the material.

15. A beam of wood, rectangular in section, is fixed at one end


and loaded at the other. What is occurring at various places in
any imaginary cross section? Sketch anything you have seen or
used which illustrates your ideas about bending. (1897.)
16. Uniform beams of timber of the same sizes are loaded and
supported as follows 1. Loaded at one end, and fixed at the other.
:

2. Fixed at one end, and uniformly loaded all over. 3. Supported


at the ends, and loaded in the middle. 4. Supported at the ends,
and loaded uniformly all over. 5. Fixed at the ends, and loaded in
the middle. 6. Fixed at the ends, and loaded uniformly all over.
What are their relative strengths ? What are their relative stiff-
nesses? Where is each most likely to break? (1898.)

17. What are the functions of the top and bottom booms and of
the diagonal pieces of a railway girder ? Why are the booms usually
larger in section towards the middle of the girder, and the diagonal
pieces usually larger towards the ends of the girder ? (1901.)
CHAPTER IX.

RIVETED JOINTS. SHAFTS. SPRINGS.

Riveted joints.
Plates are permanently connected by
riveted joints. In the simplest form of joint, the edges of the
plates overlap, and the rivets are closed up in a single row of
holes. This joint is called a single riveted lap joint ; if there are
two rows of rivets a double riveted lap joint. In butt joints
the plates are brought together, edge to edge, and cover plates
running along the seam are placed either on one or both sides.
Rivet holes are either punched or drilled. Punching injures
the material of the plate round the hole, and this must be
removed by rymering out the holes, which, in this case, are
punched smaller in diameter than the rivet hole is to be,
or else the plate must be annealed after punching. Punching
must be done with the plates separate, and for this reason
the holes will not come exactly opposite one another when
the plates are brought together unless a special machine is
used for spacing them. The holes produced by punching are
slightly conical, and the plates are so punched that when they
are put together the smaller ends of the holes are on the inside.
This produces a sounder job after the rivets are closed.
Drilling does not injure the plate, and is usually done with
the plates in position, so that the holes are bound to come fairly
opposite one another. The slight burr raised round the edges
of the holes by drilling must be removed by separating the
and slightly countersinking the holes.
plates after drilling
Unless the plates are thin and the rivets small, the rivets are
heated before being put into the holes. The head is then
formed by hand hammers and finished by a snap, or else
:

110 APPLIED MECHANICS FOR BEGINNERS.

machines worked by hydraulic or pneumatic pressure are


employed. The plates require to be first drawn tight together
by bolts, and the rivet contracts and binds the plates together
after the head is formed, as it cools down the rivets being ;

thereby put under tensile stress of an indefinite amount.


Example. Calculate the force required to punch a g" diameter
hole in a plate |f thick, taking 24 tons per square inch as the
ultimate shear stress. Calculate also the compressive stress on the
punch.
Area of metal under shear ird x t (p. 84).
Force required to punch the hole = ird x t x 24

=
y x-x-x24
= 33 tons.

33
Compressive stress on punch
area of punch
33
~7d?
4
33x4x7x8x8
22x7x7
= 55 tons per square inch.

Strength of riveted joints.


Considering the strength of a single
riveted lap joint under pull, we see
that it may fail in one of four ways
(a) By the material between the
edge of the rivet hole and the edge
of plate opening out (A, Fig. 155).
This would be due to the holes
being too near the edge of the
plate. In practice it is found to
be sufficient to make the distance
from the edge of the hole to the
edge of the plate equal to the
diameter of the rivet for rivets
Fig. 155. -Ways in which a riveted
joint may fail. f" and greater in diameter, and
slightly more for rivets less than
this. Thus, for A" rivets, the distance is about f".
RIVETED JOINTS. SHAFTS. .SPRINGS. Ill

(6) By the material of the plate crushing at B (Fig. 155).


This would be due to the rivets being too small in diameter, thus
providing too small a bearing surface. In practice, a rule such as

t being the thickness of the plate in inches, is used and is found


by experience to be sufficient.
(c) By one of the plates rupturing under tension along the
line CD (Fig. 155).

(d) By the rivets shearing at EF (Fig. 155).

The most economical joint would be obtained by so designing it


that the liabilities to rupture in these four ways are equal to
one another. There being no exact mathematical information
as to the strength against rupture by methods (a) and (6), it is
customary to determine first the diameter of the rivet for the
given plates by the rule given in (b), and then to decide upon
the overlap of the plates as shown in (a). Afterwards (c) and
(d) are calculated so as to make the joint equally strong against
failure by tearing along the row and by shearing of the rivets.

Fig. 156. Fio. 157. Butt joint, single cover strap.

Bending action on joints. It must be further noticed that


when the pulls P, P, (Fig. 156) are applied to the joint, they
produce a couple tending
tomake the joint assume
a form resembling that
shown in the illustration, T&Z^
so that P, P, will act in
the same straight Line.
Fio. 158. Butt joint, double cover strap.
Joints are sometimes
made as shown in Fig. 156 in practice in order to prevent
this bending tendency. Butt joints are liable to the same
action if there is only one cover strap (Fig. 157), but with a
strap on each side (Fig. 158), this is prevented.
.

112 APPLIED MECHANICS FOR BEGINNERS.

Application to single riveted lap joint. Consider a strip


of the joint equal in breadth to the pitch of the rivets, i.e. the
distance from centre to centre of the rivets, measured along the
row. This will be the breadth of
joint supported by one rivet and
so the conclusions arrived at will
be true for the whole joint.
In Fig. 159 let

p = pitch of rivets, inches ;

d diameter of rivets, inches ;

t = thickness of plate, inches ;

Fig. 159.
f = tensile stress permitted
t ;

f = shear stress permitted.


t

Area under tensile stress = (p d)t

Area under shear stress = A


4
For equal strength, the following equation must be true :

Mp-d)t=f
r-
t

d2
^ ,
Taking d^V2<t, or iT^A* tnen

>> ,v d2 .ird 2

f {p-d)=f \*VU
t t

=0-36./,.7r,

or (p-d)=\\Zl
ft ranges in practice from 35,000 to 67,000 pounds per square
inch, and /, from 43,000 to 53,000 pounds per square inch.
The values to be taken in any given case depend on the number
ofrows of rivets, on the material (whether iron or steel), and on
whether the holes have been punched or drilled. For iron

plates and iron rivets, with drilled holes, the ratio ^ may be
taken as A Q^
which would give for the single riveted lap joint
(jt?-d)0-94 = 1-131.
RIVETED JOINTS. SHAFTS. SPRINGS. 113

Example. Calculate the diameter and the pitch of the rivets for
plates " thick connected by a single riveted lap joint.
d=\2sft

t,4 nearly
Vi"ra"S -

94=1 131,

= 2 " nearly.
Percentage strength of joint. It will be observed that the
sectional area of the plate along the centre line of the row has
been diminished after the rivet holes have been punched or
drilled, and that therefore the strength of the joint is less than
that of the unhurt plate. Taking a width of plate equal to p ;

its sectional area will be p multiplied by t in the unhurt plate

and {p-d)t at the centre line of the row of rivets ; therefore

(p d) t _p d
strength of joint _
strength of unhurt plate pxt p '

In the above example, this will give


21-1 = 1-25 n 59nearly;
==0 "
'
.

-V 2l25
or the strength of the joint is about 59 per cent, of the strength
of the unhurt plate.
In joints such as that shown in Fig. 158, the rivets are under
double shear, that is, they would have to shear at two places if

the joint gave way by fracture of the rivets. Usually butt


joints with double cover straps are double riveted, having four
rows of rivets in all. In this case, for a length of joint equal to
the pitch of the rivets, there are four rivet sections under shear,
but as these will probably not all be effective, it is customary
to reckon 3'5 rivet sections only. In joints such as this, a per-
centage strength of 75 can be obtained.

Twisting moment on a shaft. Shafts are pieces used for
the transmission of motion and power from one place to another.
They are usually made round, and receive a moment tending to
rotate them at one place, which moment is transmitted, by
a.m.b. H
114 APPLIED MECHANICS FOR BEGINNERS.

stresses in the material of the shaft, to the desired place. Let


us consider a shaft A B (Fig. 160) one end of which, A, is fixed
in some way, and having an arm
BC, mounted at the other end.
If a force P is applied at the end
of the arm BC, two things will
evidently occur : the shaft will
tend to rotate, or twist also it ;

will tend to droop, showing that


there is a bending action. We
may get rid of the tendency to
Fig. ItiO. Shaft under torque. bend by prolonging the arm on
the other side of B (as shown
dotted in Fig. 160) and applying a force equal and opposite
to P at the end E. These forces P, P, now form a couple
acting on the shaft, which in consequence is called upon to
resist rotation only, and its material will be subjected to pure
twist. The moment of the couple is called the Twisting Moment,
or Torque. Thus
Torque = T=PxB.
Shearing stress produced by torque. Consider now any
cross section of the shaft, such as that at F, taken perpendicular
to its axis. There will be a tendency, when the torque is
applied, for the material on one side of the
section at F, by rotation, to slide past the
on the other side of the section.
material
Consequently such sections will be under shear
stress. The shear stresses will not be uniformly
distributed over the section, and can for round
shafts be shown to be in proportion to the
Fig. 161.
distance from the axis of the shaft at any part
of the section. Let r be the radius of the shaft section, then
we should expect to find at a l (Fig. 161), which is at a distance
|r from the axis, a stress just half that at a 2 which is at a
,

distance r from the axis. The stress at the axis of the shaft
would be zero.

Moment of resistance. The stress on all parts of the section
will give forces, each having a moment about the axis of the
shaft, and the sum of all these moments will give a resultant
RIVETED JOINTS. SHAFTS. SPRINGS. 115
1 ;
"
moment, called the Moment This
of Resistance of the material.
Moment Twisting Moment applied
of Resistance balances the
to the shaft and consequently will be equal to it. For a round
solid shaft, the Resisting Moment can be shown to be

far*
Resisting Moment =?=- lb. inches.

Where /= maximum shear stress allowed, lbs. per sq. inch,

r = radius of shaft section, in inches.

Let 7*= the torque, in lb. inches, applied, then

T= J^r for a round solid shaft.

Example. What torque can be applied to a shaft 4" diameter if


the maximum shear stress is not to exceed 10,000 lbs. per square

inch?

2
_ 10,000 x22x2x2x2
7x2
= 125,700 lb. inches
= 10,470 lb. feet.
If we measure the strength of a shaft by the torque which
may be safely applied to it, the above equation shows us, that
if the shaft is under pure twist, without bending action, its

strength will be independent of length and directly proportional


its

to the cube of its radius. Thus, a solid shaft 4" diameter could
withstand safely 8 times the torque which could be safely
applied to a shaft of the same material, but only 2" diameter.

Hollow shafts. Since the material of a shaft near its axis
is only under a small shearing stress, and the arm of which, in

taking moments, is also small, we could obtain a stronger shaft


of the same weight by removing some of this material and
putting it instead at the outer circumference of the shaft. This
is often done in large shafts for the sake of lightness, and

weight for weight, a hollow shaft toill be stronger than a solid


one.
Flanged shaft couplings. Two pieces of shafting in the
same straight line are often connected by having their ends
116 APPLIED MECHANICS FOR BEGINNERS.

flanged, or flanged couplings keyed on and the flanges secured


together by fitted bolts. These bolts will be under shearing
stress when the torque is applied.
Let d be the diameter of the bolts and fg the shearing stress
they may have, then
2
ird
Shearing force on one bolt =/, x --.

If there are N bolts, the total shearing force will be


P. *./.-*?
Let R = radius of the bolt circle, then the Resisting Moment of
the bolts to shearing will be P multiplied by R, or
ird 2
Moment of Resistance =JV.ft .
--- . R.

And this must be equal to the torque applied ; hence, if we


know the maximum torque to be applied to the shaft, the
diameter of the bolts required may be
fWf'^KJ easily calculated.
Stiffness of wires under torsion.
The elastic properties of a wire under
torsion can be examined by a self-con-
tained machine such as is shown in Fig.

162. This particular machine was not


specially designed, but made out of some
materials which happened to be at hand.
It consists of an upper bracket for hold-
ing the top end of the wire under test,
supported by three mild steel rods, which
are tied together near their lower ends by
another similar bracket. Two pointers
can be clamped to any part of the wire,
and move over circular scales divided in
degrees. The torque is applied by two
cords coiled round a drum 5" diameter,

Fig. 162. Apparatus for


clamped to the wire, the cords being led
experiments on the tor- over pulleys and having scale pans at
sion of wires.
their ends. If equal loads are placed in
the pans they will produce a couple, giving pure twist to the
RIVETED JOINTS. SHAFTS. SPRINGS. 117

wire. A
permanent weight hung to the end of the wire keeps
it taut. This machine can be used for verifying the following
elastic properties of wires under torsion.
The angle of twist is proportional to the torque applied, directly
proportional to the length of the wire, and for wires of the same
material but of different diameters, inversely proportional to the
fourth power of their diameters.
The following results were obtained in an experiment for
verifying the first two statements.

An Experiment on Twisting.
Steel wire, annealed, 0*065" diam.

Load in each pan Angle of twist Angle of twist


Torque
+ weight of pan ;
= W x5 lb. inches. on 4-5" length. on 20-8" length.
W lbs. Degrees. Degrees.

00 o-o
20 10 10. 6
0-80 40 5 23
105 5 25 7-5 29
1-55 775 9 410
2 05 10 25 11 53-5
2-80 140 15-5 73
3 00 150 16 79
3 50 17 5 19 92
3 60 180 20 94
3-85 19-25 21-5 101-0

Columns 2 and 3 when plotted, and also columns 2 and 4, give


very nearly a straight line, showing that the angle of twist is
proportional to the torque (Fig. 163).
Selecting two values from the plotted curves, when the torque
is 14 lb.-inches, the angle of twist on a length of 45" is 15-5

degrees, and the angle for a length of 20'8" is 73'5 degrees.


73*5
Ratio of angles of twist = ^^ = 4*7.
lo'o

Ratio of lengths of wire = =4*6.

An approximation to the law that the angle of twist is

proportional to the length of the wire.


118 APPLIED MECHANICS FOR BEGINNERS.

Angle of twist after passing the elastic limit. Test pieces


of wrought iron or mild subjected to torsion, after the
steel
elastic limit of the material for shearing stresses is passed, do
not twist through angles proportional to the torque. After
this, the angle is much larger proportionally, and a piece of
great length compared to its diameter will twist through many
complete turns before fracture occurs. For this reason, it is
convenient to use rather short pieces of the material for such
testing purposes.
/0/7QVS
LB. Inches
20r
>
f
18
V
16
1
u /
/
/r
14 /I /
12
/ J*
/l yt
/W
10

8 *

6 /* /
4 /
2 /
/
10 20 30 40 50 60 70 80 90 100 110
AMLEorTwJST
Fig. 163. Plotted diagram, showing torque and angles Fig. 164. Helical
of twist for a wire under torsion. spring.

Springs. Springs are pieces intended to take a relatively


large amount of strain, although any piece of material which
can be strained and shows a strong tendency to recover its
original shape freely may be called a spring. Thus, a bar of
iron or steel, pulled within its elastic limit, may be called a
spring. The forms taken by springs depend on the purpose for
which they are intended. For spring balances, used to measure
forces, helical springs are used (Fig. 164). In these the forces
are applied in the direction of the axis of the spring and the
material of the spring is under torsion. Helical springs are
also occasionally subjected to couples, as shown in Fig. 165 in ;

this case the material is under bending. A spring resembling


this is used in the Wayne indicator.
RIVETED JOINTS. SHAFTS. SPRINGS. 119

Spiral springs, as in Fig. 166, are used for watches and clocks,

Fig. 165. Helical spring under torsion. Fig. 166. Spiral spring.

and have their material subjected to bending. Volute springs are


used for railway carriage buffers one is shown in Fig. 167.
;

Carriage springs consist of a number of


flat strips, of gradually increasing length,

secured together and loaded (Fig. 168).


Springs consisting of small discs corru-
gated in a circular direction (Fig. 169),
are occasionally employed for measuring,
by their deflection, the pressure of a
gas or liquid acting on them. Rubber
Fig. 167. Volute spring.
cushions are often used between the
body of a carriage and the steel springs, These rubber springs
are under compression.

xzxzxz

Fig. 168. Carriage spring. Fig. 169. Corrugated disc spring.

In employing rubber springs under compression it may be


noticed that although rubber can change its shape very easily,

it can only change its bulk very slightly even under very great

forces. Consequently, if confined so that it cannot swell out


laterally when the forces are applied, it will act not as a spring
but as a relatively rigid body.
Springs are used for measuring forces, for storing energy,
and for minimising the effects of shocks. In all cases, the
loading is kept within the elastic limit, and consequently, as a
120 APPLIED MECHANICS FOR BEGINNERS.

rule, the spring is distorted, or changes its length, by an amount


proportional to the applied torque or forces.
Elastic extension of springs.
The student should experiment
with a helical spring by loading it
with gradually increasing weights,
the extensions being given by a
pointer attached to the spring and
moving over a scale (Fig. 1 70). It
will be found that, if several springs
of the same material are available
for testing, the following laws are
approximately true :

The extensions are proportional


to the load, to the cube of the
radius of the helix, and to the
number of complete turns in the
helix ; also inversely proportional
to the fourth power of the radius
of the wire of which the helix is
made.
Fig. 170. Apparatus for measuring
extensions of a spring.
In these experiments, the wire
is assumed to be round, and the

springs made so that the coils lie close together.


The proportional laws may be represented
by an equation in this way.
Let
IT=load applied, lbs. (Fig. 171).

R radius of helix to centre of wire, in


inches.
N= number of complete turns.
r= radius of wire, in inches.
X= extension produced by W, inches.
Then
X=c W&N
Pio. 171.
where c is a constant depending on the elastic
qualities of the material. Its value for steel is approximately
c = 0-00000033.
RIVETED JOINTS. SHAFTS. SPRINGS. 121

EXERCISES ON CHAP. IX.


1. A hole |" diameter has to be punched in a plate %" thick.
Take the ultimate shearing stress as 24 tons per square inch and
calculate the force required.
2. The plates of a boiler are TV' thick, and the circumferential
seams are to be lap joints double riveted. Find the diameter and
pitch of the rivets and the percentage strength of the joint, assuming
that the sectional area under shear has to be equal to the sectional
area under tension.
3. A boiler 7' 0" diameter has to work at a pressure of 120 lbs.
per square inch. Calculate the thickness of plate required if the
maximum tensile stress is not to exceed 4 tons per square inch (a)
assuming the strength of the joint to be 100 per cent., (6) assuming
the strength of the joint to be 75 per cent.
4. Twoportions of a tie rod are connected by a knuckle joint.
The pull in the rodis 6 tons. Calculate the diameter of pin required
if the shearing stress allowed is 4 tons per square inch.

5. Two shafts, one 2" diameter the other 2" diameter, are sub-
jected to equal twisting moments. Compare their maximum shearing

6. A shaft 2" diameter is subjected to a torque of 12,000 lb.


inches. Calculate the maximum shearing stress produced.
7. Calculate the diameter of shaft required to carry a torque of
5000 lb. feet, if the maximum shearing stress allowed is 10,000 lbs.
per square inch.
8. Calculate what twisting moment can be applied to a shaft 10"
diameter if the shearing stress is not to exceed 4 tons per square
inch. What force applied at 90 to a crank at an arm of 27" will
produce this torque ?
9. Compare the Bending Moment and the Torque which may be
applied to a round solid shaft if the safe tensile, compressive, and
shearing stresses are assumed equal.
10. A wire of Siemens' steel, 0*1 inch diameter, is to be twisted
till it breaks. Sketch the arrangement and show how the angle of
twist and the twisting moment are measured, how the results may
be plotted on squared paper, and the sort of results that may be
expected. In what way may a wire of twice this diameter be
expected to behave? (1901.)
11. Two portions of a shaft 2" diameter are connected by a flanged
coupling whose four bolts have their centres on a circle concentric
with the shaft centre and 5%" diameter. Allowing a safe shearing
stress of 10,000 lbs. per square inch, calculate (a) the twisting moment
the shaft may be subjected to (b) the diameter of bolts required.
;

12. A helical spring is made of round steel J" diameter, and has
50 coils li" diameter. Calculate its extension when pulls of 5 lbs.
are applied.
CHAPTER X.

WORK. MECHANICAL ADVANTAGE. VELOCITY RATIO


OF MACHINES. ENERGY. POWER. EFFICIENCY.
DIAGRAMS OF WORK. RESILIENCE.
Definitions of terms. Aforce is said to be doing work
when it acts through a distance, overcoming resistance. The
quantity of work done is proportional jointly to the magnitude
of the force and the distance through which it acts, the distance
being always measured along or parallel to the line of action of
the force. The unit of work used by engineers in this country
is the foot pound, and is that quantity of work which is done when

a force of one pound acts through a distance of one foot in its line
of action. The inch-ton and foot-ton are also sometimes used,
these being the work done when a force of one ton acts through
a distance of one inch or one foot respectively.
The work done by any force is calculated by taking the
product of the magnitude of the force and the distance through
which it acts.

Example 1. If a weight of 4 tons has to be raised from the


bottom of a shaft 100 fathoms deep, find the work done.
Work done = force x distance
= 4x(100x6)
= 2400 foot-tons.

Example 2. A load of 2 cwts. is dragged along a level floor


through a distance of 10 feet by means of a rope inclined at 30 to
the floor (Fig. 172). The pull P is found to be 80 lbs. Calculate
the work done.
WORK. ENERGY. POWER. 123

First Solution. Notice that P does not act through a distance


AB=10 feet, but through AC=5s/S feet (Fig. 172). Hence,
work done = 80 x 5 x 1 *73
= 692 foot-lbs.

Fig. 172. Fig. 173.

Second Solution. Or, we may solve the question in another way :

Take horizontal and vertical components of the pull in the rope


(Fig. 173) giving 4W3 lbs. for the horizontal and 40 lbs. for the
vertical component.
This vertical component, while the weight is being dragged along,
will always be acting in parallel vertical directions, and its point of
application will move neither up nor down. Consequently, as it does
not act through a distance measured along or parallel to its line of
direction, no work is done by it. The horizontal component acts
through a distance of 10 feet measured along its line of direction,
consequently work done will be
W3 x 10
= 692 foot-lbs. , as before.

The same quantity of work may be done either by a small


force acting through a large distance, or a large force acting
through a small distance. Thus, suppose we have to do
1,000 ft. -lbs. of work, we may use a force of 1 lb. acting through
1,000 feet, or 500 lbs. acting
through 2 feet, obtaining the
desired result in each case.
Machines. Machines are
arrangements receiving work
from some outside source of
supply, which work is modified
by the machine and delivered in
some form more suitable for the
purpose required. Fio. 174. Simple winch.
As an example of a machine,
we may take a simple winch (Fig. 174) used for raising loads,
This machine takes in work from the pushes and pulls of two
124 APPLIED MECHANICS FOR BEGINNERS.

men at the handles, turning the drum. The drum produces a


modified pull on the lifting rope by which work is done on the
ascending weight.
Usually in machines such as hoisting tackle, machine tools,

ff- ft
etc., the force delivering work to the machine
v i is smaller than the resistance which is overcome
'{ by the use of the machine. The Mechanical
Advantage of a machine is the ratio of these two
I
\ forces.
\ Thus, in the above winch, suppose that
^'.-

*'2r'
r each man exerts a constant force of 30 lbs.
applied always tangential to the path of his
hand (Fig. 175), the load raised being 300 lbs., then the
Mechanical Advantage will be ^ =
Qf)f)
5 -

The Velocity Ratio of a machine is measured by dividing the


distance through which the force applied to the machine acts,
by the distance through which the resistance is overcome in the
same time. Thus, in the above winch, if the men's hands move
through a circumferential distance of 6 feet while the weight is
being raised a height of one foot, then the velocity ratio is 6.
Energy and its transformation. If work is imparted to a
body so that it stores it up and is capable cf giving it out
again, the body is said to possess Energy.
Energy means capability of doing work. If we raise a one
pound weight from the table a height of one foot, we have done
one foot-pound of work on the weight, and this work it can give
out again if we allow it to descend. The raised weight possesses
energy to the amount of one foot-pound. The individual who
raised the weight has had to part with one foot-pound of his
store of internal energy, and if he goes on for some time raising
weights, say for 4 or 5 hours, he will find that there is need to
replenish his store of energy by absorbing some food and resting
a little. Food possesses energy which, transformed by the organs
of our body, enables us to do mechanical work. The energy of
food is liberated in our bodies in the form of heat by a process of
slow combustion. Coal also possesses heat energy which is utilised
by a much more rapid combustion in boiler furnaces or other-
wise. Water at an elevation possesses energy which can be
transformed into mechanical work while the water is descending,
WORK. ENERGY. POWER. 125

and the atmosphere has energy also when in motion as wind.


The principal source of all these stores of energy is the heat
of the sun, which (i), in past ages, caused the vegetation to
grow from which we at present derive our coal supply, and
(ii) now gives us our food supply by giving life to plants.

The heat of the sun also raises water by evaporation and so


gives it its store of energy, and it also sets the atmosphere in
motion as wind.

Conservation of energy. The principle of the Conservation
of Energy asserts that man is not able to create or destroy energy,
he can only transform it from one form into anotJier. This
principle the result of the observation and experiment of
is

many people, including those who have sought in vain for


perpetual motion. To the engineer, it is of extreme importance.
Thus, if we impart to any machine a certain quantity of energy,
and no energy is lost in the machine or used to coil up a spring
belonging to the machine or do any other form of work in the
machine, then the machine will deliver an exactly equal quantity
to that given to it. It cannot deliver more, for then it would
create energy nor can it deliver less, for then energy would be
;

destroyed in the machine.


Actually, it is impossible to construct a machine in which
there is no energy lost, whether by the rubbing of surfaces on
one another, by churning the atmosphere, or by the develop-
ment of sound and other causes. But we can assert about all
machines :

Energy supplied = Energy given out + energy lost in over-


coming resistances in the machine.
If the machine is running light, i.e. doing no useful work
against resistances, then we must supply energy sufficient to
make good that lost by resistances in the machine. If the
machine is doing useful work, then we must in addition supply
energy equivalent to this useful work.

Efficiency of machines. The efficiency of any machine is
measured by the ratio of energy actually given out as useful
work to the energy supplied. Or,
_,, usefulwork done
.

Efficiency
J
= r^-r-
energy supplied
;

126 APPLIED MECHANICS FOR BEGINNERS.

Example. A certain machine is found to give out 125 foot pounds


work when 180 foot pounds of energy are supplied to it.
of useful
Find the efficiency of the machine.

Energy lost in machine = 180- 125


= 55 foot pounds.
Efficiency = yli = 0'69.
The efficiency may be stated as a percentage by multiplying this by
100, giving
Efficiency=0-69xl00
= 69 per cent.

Forms of energy useful to the engineer. The principal


forms of energy that the engineer has to deal with are Potential :

Energy, such as the energy of a raised weight or a coiled spring


Kinetic Energy, which a body possesses when it is in motion,
and can give out as mechanical work while it is coming to
rest ;both these forms of energy are stated in foot pounds.
One foot pound of potential energy is exactly equivalent to one
foot pound of kinetic energy. For example, a raised weight
possesses potential energy, and if it is allowed to fall freely,
doing no work against any resistance, the potential energy
will be converted into an exactly equal quantity of kinetic
energy.
Heat is a form of energy which can be converted into
mechanical work. The amount of mechanical work equivalent
to a given quantity of heat is known with considerable accuracy
from the experiments of Dr. Joule and others. Thus 772, 774,
or 778, foot pounds of mechanical work transformed into heat
would, if all the heat passed into one pound of water, raise its
temperature one degree Fahrenheit. The numbers given are
those used by various authorities, and it is immaterial which of
them is taken for engineering purposes, as the total difference
between the first and the last is less than one per cent., and no
engineering problem involving heat quantities can be worked
out from practical data to a greater degree of accuracy. The
quantity of heat which would raise the temperature of one
pound of water through one degree Fahrenheit is the unit
of heat used by British engineers, and is called the British
Thermal Unit.

WORK. ENERGY. POWER. 127

Electrical energy is measured in kilowatts performed per hour.


The kilowatt an electrical power unit, and consequently gives
is

the rate of energy production, corresponding to the horse-power


(p. 128). A kilowatt is equal to 1000 watts, the watt being the
rate of working when an electric current of one ampere flows
from one point of a conductor to another, the potential difference
between which is one volt. The product of Amperes and volts
is expressed in watts. 746 watts are equivalent to the mechanical
horse-power. It will be understood that just as we have to state
the time during which a given horse -power has to be maintained
in order to produce a certain amount of work, so we must state,
as above, the time during which one kilowatt has to be main-
tained in order to produce a given amount of electrical energy.
The Board of Trade unit of electrical energy is that given above
as one kilowatt maintained for one hour.

Loss of useful energy. It should be clearly understood that
although energy in one form may be exactly equivalent to a
certain quantity of energy in another form, that we never suc-
ceed, in any transformation of energy, in obtaining an exactly
equal quantity in the new form. There are always losses

sometimes great losses which are inevitable. As a common
example, and one which gives a fair idea of the magnitudes of
some of these losses, take the following case of electrical power
production. Suppose 100 units of energy to be liberated from
some coal in the boiler furnace. About 75 of these will enter
the steam and the remaining 25 will be lost by the passage of
the smoke and heated gases up the chimney, or by radiation and
other causes. Of the 75 units of energy reaching the engine in
the steam, about 6 will be converted into mechanical energy and
the remaining 69 will be lost. The 6 units of mechanical energy
given to the dynamo will produce about 5 units of electrical
energy, 1 unit being lost. If these 5 units be reconverted into
mechanical energy by an electrical motor, about 4*5 units will
be produced. We utilise in this way about 4'5 per cent, of the
original energy and lose 95 5 per cent.
-

Variation of the actual mechanical advantage. From


what has been said it will be seen that the velocity ratio of a
machine, which depends solely on the arrangement and nature
of its parts, does not change provided the arrangement remains
;

128 APPLIED MECHANICS FOR BEGINNERS.

the same. On the other hand, the mechanical advantage, or the


ratio of the resistance overcome to the force delivering energy to
the machine, depends on the extent of the losses in the machine
these again are variable, depending on the load and on the
condition of the machine as regards lubrication and state of the
bearing surfaces.
Let P=the force delivering energy to a machine, and TT=the
resistance overcome, both in same units of force. Let D = dis-
tance through which P acts
while W
is being overcome through

a distance = d, Dand d being in the same units of distance.


Then, if there were no losses in the machine,
Energy supplied = useful work done,
PxD=Wxd,
D W
d~ P'
Now D
- is

the velocity ratio of the machine and W is the

mechanical advantage, so that in this hypothetical case, the


velocity ratio and mechanical advantage are equal numerically.
Actually, however, W will always be less than its value assumed
above, and consequently the actual mechanical advantage will
always be less than the velocity ratio for any machine. In
Chap. XIV. it will be seen how the actual mechanical advantage
of a machine can be obtained. The velocity ratio can be
calculated from a knowledge of the mechanism, or by direct
measurement, at the places where P and Ware applied, of the
distances through which they act.

Power. If we state not only the quantity of work done by a
force or forces, but also the time in which it is done, this will
give us the rate at which work is being performed. Power, or
activity, is the name given to the rate of performing work. The
unit of power used generally by engineers in this country is
produced when 33,000 foot-pounds of work are done in one
minute. This unit was denned by James Watt, who found that
the average horse could do about 22,000 foot-pounds of work in
a minute. Watt added 50 per cent, to this and took 33,000
foot-pounds per minute as the unit of power to be used in
measuring the performance of his steam engines.
WORK. ENERGY. POWER. 129

The horse-power developed any given case will be ascer-


in
tained by first work done per minute, in foot-
calculating the
pounds, and dividing the result by 33,000.
Thus, suppose we take the previous example (p. 122), in which
4 tons were raised from a depth of 100 fathoms. If the work
is performed in 40 seconds, the horse-power required may be
calculated thus :

Work done in 40 seconds = (4 x 2240) x (100 x 6) ft.-lbs.

Work done in 60 seconds = 8960 x 600 x


40
ft.-lbs.

= 8,064,000 ft. -lbs.


Horse-power = - -
8,064,000
3
= 244-4.
3W -

In calculating the horse-power required in any given case,


the efficiency of the machine employed must be considered, as
power will be required for overcoming losses in the machine.
Thus, suppose in the above case, that the efficiency of the
mechanism of the winding engine employed to raise the 4 tons
is 60%. This means that 60% of the energy given to the engine
in a given time, say one minute, is turned into useful work.
Consequently, the 8,064,000 ft.-lbs. useful work above done per
minute is only TQ ths of the energy that must be given to the
engine per minute.
8,064,000 x 10 ri ,i
i- i
to engine per mm. =
ti

Energy supplied i.
ft.-lbs., ?

and, Actual Horse-power


v required
M = '-

6x33,000
= 407.
Graphic representation of work. Since work is measured
by the product of two quantities, force and
distance, we may represent it by the area
of a diagram. Thus, supposing a uniform
force P to act through a distance D, the work
done will be P x D. If we set off I) to scale Fig. 176. --Work done
in a diagram (Fig. 176), and erect ordinates
of constant height equal to P to scale, we obtain a rectangle
of area equal to PD, which therefore represents the work done.
A.M. II. I
130 APPLIED MECHANICS FOR BEGINNERS.

If the work is done by a varying force, and the amount of it

isknown at various points in the distance acted through, we


may set up ordinates to represent its value at these points,
giving a diagram as shown in Fig. 177 bounded by a curve at
the top. The work done in this case will be equal to the
average value of P
multiplied by D. Now the average will be P
represented by the average height of the diagram to scale and ;

as the average height multiplied by D


gives the area of the
figure, it follows that in this case also, the area of the figure
represents the work done.

Pig. 177. Work done by a varying Pig. 178. Work done in extending
force. a bar.

Work done in pulling a bar.A useful example of diagrams


of work isto be found when the work done against the resist-
ance of a bar to stretching is shown. As we have seen already
(p. 72), the resistance of the bar, or the stretching load, is
directly proportional to the extension produced, giving, when
plotted, a straight line up to the elastic limit of the material.
The area under this line will represent the work done in stretching
the bar. whole diagram is given showing the gradual
If the
loads applied and the extensions produced up to breaking,
then the whole work done in breaking the bar will be given by
its area. Thus, the area EOM (Fig. 178) represents the work
done on the bar up to its elastic limit this work, stored in the
;

bar as energy, and enabling it to spring back to its original


length is called the resilience of the bar. The whole area OEBN
represents the total work done in breaking the bar. The energy
which can be stored in a piece of good wrought iron by stretch-
ing it up to its elastic limit, that is, its resilience, is about
0*08 inch-ton, and the energy which must be expended while
pulling it to rupture is about 55 inch-tons, the bar being one

square inch in sectional area and ten inches long.


=
WORK. ENERGY. POWER. 131

The resilience of a palled bar may be easily calculated if the


value of Eand the elastic limit are known. Thus,
let /= elastic limit in tons per sq. inch.
Then, taking a bar 1 square inch sectional area,
Resilience = average force x extension up to elastic limit.
From the part of the diagram OEM (Fig. 178), we see that
the average force will be one half the maximum and is therefore
equal to \f tons.
The extension may now be calculated from

E= _/xZ tons per square inch.

Taking L=l", so that one cubic inch of material is being


considered,* this will become

or, extension = e = % inches.


Hi

"4
Therefore, Resilience \f x
= ^ inch-tons per cubic inch of
2 -^ material.

Effect of suddenly applied loads. It may now easily be


seen what effect a suddenly applied load
"W
has on the material of a bar. Suppose a
bar AB (Fig. 179) to be suspended verti-
cally and that a load W
is just touching

the collar on the end B of the bar. If


the supports of W
are released and it is
allowed to rest on the collar, we get a
conception of a suddenly applied load.
Work will be done by gravity on If, the
force acting being equal to Jr. Set this
off as OK (Fig. 180) to scale, and let

distances along OM represent extensions


of the bar. So long as the bar continues
to extend, work
will be done by gravity Fig. 179.- -Load suddenly
applied to a bar.
on W. This will be represented by a
rectangular area, the force being uniform. But the bar can
132 APPLIED MECHANICS FOR BEGINNERS.

only resist the load by stresses gradually increasing from zero,


so that the work taken up by the bar will be represented by
the area of a triangular diagram. At N
the resistance of the bar will be equal to
W but at this point more work has been
7
,

U done on W
than the bar has taken up. At
this point the work done by gravity is
represented by the rectangle OKNP, while
Q that taken up by the bar is represented by
FlG 18 agram f
the t rian g le 0NP tne work represented
*
W^k '>

by the shaded area remains to be yet taken


up by the bar, which consequently goes on extending. When the
work taken up by the bar, represented by OQM, is equal to the
work done by gravity, represented by OKLM, the extension
will cease. This is easily seen to occur when OM=20P, or
when MQ = 2ML = '2W. The bar will now spring back and
vibrate vertically until the damping action of molecular friction
brings it The whole effect, as regards extending the bar
to rest.
and the produced in it, is just as though a load equal to
stress
twice Whad been gradually applied.
Horse-power transmitted by shafting.The horse-power
which can be safely transmitted by a given shaft depends on
the uniformity or otherwise of the torque applied. Supposing
this to be uniform, we proceed thus :

Let P=the turning effort in lbs.

R= radius in feet at which P is applied.


Then Px 27ri?=work done in one revolution in foot-pounds.
Let N= revolutions per minute,
Work per minute = P x ZirR x N foot-pounds,
and H P = - 33,000
-
'
'

assuming, as we stated above, the torque to be T=PR


uniform. In the case of factory line shafts it is often fairly so,
but in other cases it varies considerably.
Take, for example, the drank shaft of an engine having one
double-acting cylinder only. The effort of the steam on the
piston does not remain constant during the stroke, also the effect
of the connecting rod acting at different angles to the centre
WORK. ENERGY. POWER. 133

line while transmitting the varying force from the piston rod to
the crank, cause the torque to vary widely during the stroke.
The maximum torque will occur usually when the crank and
connecting rod are at right angles to one another. The torque
will be zero when the crank is on the dead centres, as the crank
and connecting rod will then be in the same straight line.
If the steam pressure and dimensions of the engine are given,
we can easily obtain the torque when the crank and connecting
rod are at 90 to one
another, by setting out ,' ~
T*3r^_ f

an outline diagram of / j \ ^^"^^n


---^lfs~--~~-^

i~"~
the engine to scale. "t a"
?
p
Let AB (Fig. 181) \ /
represent the crank,
Flo 181 . Maxinmm torque on the crank shaft,
.

and BC the connecting


rod. Let P total force along the piston rod, Q= thrust along
the connecting rod, S= pressure of guide. P, Q, and S will
balance one another, and Q may be found by an application of the
parallelogram of forces. The connecting rod pushes the crosshead
at one end and the crank pin at the other with equal forces Q,
consequently the torque is equal to Q x R. This value must be
taken in estimating the diameter of crank shaft required.

EXERCISES ON CHAP. X.

1. A load of 2 tons is raised from the bottom of a shaft 300 ft.


deep. How much work is done? Draw a diagram to represent
this work.

2.In Question (1) the load is raised by a wire rope weighing


3 per foot length. Calculate the work done in raising the rope
lbs.
alone, and draw a diagram of work done.

3. A loaded truck,
weight 12 tons, is pulled along a level track.
motion are 1 1 lbs. per ton weight, calculate the
If the resistances to
work done in pulling the truck a distance of one mile.
4. A bridge girder weighs 15 tons, and is to rest on supports 25 ft.
above the level of the ground. Calculate the work done in raising
the girder into position.
5. A man weighs 140 lbs. Calculate the total work he has to do
in carrying his bicycle, weight 30 lbs., upstairs to a room 20 ft.
above the street level.
134 APPLIED MECHANICS FOR BEGINNERS.

6. A man
exerts a constant force of 30 lbs. in turning a handle of
14" radius calculate the work done per revolution if {a) the force
;

is always exerted in a horizontal direction, (b) the force is always


exerted tangential to the circle described by the handle.
7. A tank measures 10 ft. long x 6 ft. wide x 3 ft. deep, and is at
a height of 200 ft. above the level of the pump used for filling it
with water. Calculate the work done in filling it, taking one cubic
foot of water to weigh 62*5 lbs.

8. What work done in raising a bucket, weight 2 lbs., con-


is
taining 25 lbs. of water from a well the surface level of which is
12 feet below ground level ?
9. A shaft, 10 ft. diam., 100 feet deep, is full of water. Cal-
culate the work done in emptying it.

10. The weight of a pile driver is 1250 lbs., and it is raised 6 ft.
above the pile head before delivering a blow. Calculate its potential
energy when raised.
11. One cubic foot of a gas contains 600 British thermal units.
To how much mechanical work is this equivalent ?
12. Find the mechanical work equivalent to the heat contained
in a pound of petroleum of heating value 20,000 British thermal
units.

13. A horse exerts a constant pull of 80 lbs. in dragging a cart


along a level road. If he walks at the rate of 3 miles an hour, what
horse-power is he developing ?
14. A
locomotive exerts a steady pull of 2500 lbs. in hauling a
train along a level track. If the speed is 4200 feet per minute,
calculate the horse-power.
15. Calculate the horse-power required to pump 5000 gallons of
water per minute from a well 40 feet deep to the surface of the
water if the efficiency of the machinery employed is 60 per cent.
16. 15,000,000 ft. -lbs. of energy are given to an engine per hour,
and the horse-power developed is 1J. What is the efficiency of the
engine ?

17. A
shaft running at 120 revolutions per minute is subjected to
a torque of 7000 ft. -lbs. Calculate the horse-power transmitted.
18. A
bar of mild steel, 10 feet long, has a sectional area of 3
square inches. Calculate the work done in stretching it when a load
of 12 tons is applied gradually. Take .#=30,000,000.
19. What work in foot-pounds is done in raising the materials
for building a brick wall 50 high, 12' long, and 2' 3" in thickness, if
one cubic foot of brickwork weighs 112 lbs. ? (1897.)

20. A man of 150 lbs. climbs a hill regularly 1200' vertically per
hour ; at another time he climbs a staircase at 2' per second in ;

each case find the useful horse-power in lifting himself. (1897.)


: )

WORK. ENERGY. POWER. 135

21. A chain weighing 10 lbs. per foot of its length is 240 ft. long

and hangs vertically, what work is done in winding up the chain on


to a drum? (1899.)

22. A body weighing 1610 lbs. was


rope,
lifted vertically by a
there being a damped spring balance to indicate the pulling force
F lb. of the rope. When the body had been lifted x feet from its
position of rest, the pulling force was automatically recorded as
follows

X 11 20 34 45 55 66 76

F 4010 3915 3763 3532 3366 3208 3100 3007

Find approximately the work done on the body when it has risen
70 feet. How much of this is stored as potential energy, and how
much as kinetic energy ? (1901.
;

CHAPTER XI.

FRICTION OF DRY AND LUBRICATED SURFACES.


Definitions.
When two bodies are pressed together, resistance
has to be overcome before they can be made to slide on one
another. This resistance is called the force of friction. The
force which friction gives to a body always acts contrary to the
direction of motion of the body and tends to arrest the motion,
or, if the body is at rest, to prevent

motion taking place. In many engineer-


ing problems, frictional forces have to be
considered with a view to their reduction
in others, friction is useful by preventing
slipping taking place.
Fig. 182. Frictional resist
ance to sliding. If two bodies A and B
(Fig. 182) are
pressed together so that the mutual pres-
sure perpendicular to the surfaces in contact is R, and if F is

the force of friction which has to be overcome before sliding

will take place, then


F
g is called the coefficient of friction of rest,

or the static coefficient of friction of the bodies. If the bodies


are sliding steadily on one another, and F' is the steady
F'
resistance to sliding, then -^ is called the coefficient of friction of

motion, or the kinetic coefficient of friction of the bodies.



Conditions influencing friction. The value of the coefficient
of friction for two given bodies depends on the nature of the
materials of which they are made, especially on their hardness
and ability to take on a smooth regular surface, and on the state of
the rubbing surfaces as regards cleanliness and lubrication. With
FRICTION OF DRY AND LUBRICATED SURFACES. 137

dry, clean surfaces, the force of friction is produced largely by


roughnesses on the surface of one body interlocking with
roughnesses on the surface of the other. The surfaces which
have to be dealt with in engineering work are usually of fair
shape and satisfactorily fitted to one another, but even these do
not bear on one another all over, but only in places, and when
sliding takes place, the projections on one body have to get
over, or, if the forces pressing them together are large enough,
to cut away, or abrade, the projections on the other. That body
which is the more easily replaced is generally made of softer
material, in order to confine the wear principally to it.
When clean, dry surfaces, well fitted to one another, are
brought together, a film of air may exist between them and
thus prevent the bodies from actual contact. This is very
noticeable when one surface plate is laid on another. If the
cast iron surfaces are perfectly clean, the upper plate seems to
float on the lower one. By pressure and working a little, the
air film may be eliminated. The plates then adhere strongly
together, or seize, partly on account of the vacuum between
them, but more, since the effect takes place even in a good
vacuum, on account of molecular forces of attraction being
brought into play. Seizing takes place more readily with bodies
of the same than with those of different materials. In practice
a film of lubricant is used to keep the rubbing bodies as far as
possible, from contact with one another, and the working load is
such that there is no danger of the film being squeezed out.
Laws of friction for dry surfaces. For bodies with rubbing-
surfaces dry, and perfectly clean or only slightly contaminated
by films of foreign matter, the following laws of friction have
been deduced from the results of experiments : The static
coefficient offriction is greater than the kinetic coefficient offriction ;
in other words, the resistance offered to sliding when the bodies are
at rest is greater than that after steady rubbing has been attained.
The force of friction is practically proportional to the perpendicu-
lar pressure between the surfaces in contact and is independent of
the extent of such surfaces and of the speed of rubbing, if moderate.
From this we infer that for two given bodies, the coefficient of
friction is practically constant for moderate pressures and speeds.
Considerable increase in the speed lowers the value of the
138 APPLIED MECHANICS FOR BEGINNERS.

coefficient of friction, and heating of the bodies produces the


same effect. It has alsobeen found that the coefficient of
friction is a little greater for light pressures on large areas than
for heavy pressures on small areas.

Experiments on friction. Students should verify experi-
mentally as many as possible of the above statements.

Expt. Set up a board AB (Fig. 183) as nearly horizontal as
possible, and arrange a slider C (which can be loaded to any

rt
r^
Fig. 183. Friction of a slider.

desired amount) with a cord, pulley, and scale pan, so that the
horizontal force P required to overcome frictional resistances to
sliding may be measured. Under these conditions, the per-
pendicular pressure between the surfaces in contact will be equal
to the weight of the slider and loads placed on it, say, and W
its actual distribution over the surfaces in contact need not con-
cern us at present. The F
will be equal to P,
force of friction
and this will very nearly equal theweight of the scale pan and
loads placed in it, provided the pulley used is finely mounted on
pivot bearings and oiled so as to run very freely.

The coefficient of friction will be


p
-^rr .
W
First, make a number of experiments on the static coefficient
of friction, using different loads on the slider. In each experi-
ment place loads carefully into the scale pan so as to avoid jerks
until the slider starts off. From the observations the static
coefficient will be found.
Using the same loads on the slider, perform the same pro-
cesses,only this time help the slider to start by jerking it.
Adjust the load in the scale pan until steady uniform motion,
as nearly as you can judge, has been obtained. From these
observations the kinetic coefficient will be found.
FRICTION OF DRY AND LUBRICATED SURFACES. 139

Some reaults are given in order to show the method of


recording.

An Experiment to Determine Coefficients of Friction.


Material of slider mahogany.
Material of board teak.
Rubbing surfaces slightly contaminated with dust and finger
marks.

Fig. 184. Apparatus for determination of the friction of a slider.


Area of sliding surface of slider =6" x 6" = 36 square inches.
Weight of slider = 0-701 lb.

Weight of scale pan and hook for applying P= 0*701 lb.

Grain of slider perpen-


Grain of slider parallel to direction
dicular to direction
of motion.
of motion.

w
lbs. Static Values. Kinetic Values. Kinetic Values.

Pi P* P
Pi lbs. P2 lbs. PglbS.
W W W
2-701 0-74 0-337 0-49 223 051 0-232
4-701 1-43 0-306 0-901 0191 0-901 0191
6-701 2 69 0-401 1-301 0194 1-301 0194
8 701 3 04 0-35 1-701 0195 165 019
10 701 3-54 0-335 2-201 0-206 2 09 0195
12-701 4-501 0355 2-501 0197 2-401 0189
14-701 4701 0-32 2901 0197 2-90 197
16-701 504 0-32 3 501 0-209 3-40 0-203
140 APPLIED MECHANICS FOR BEGINNERS.

Fig. 185 shows the plotted results for the experiments in which
the grain of the slider was parallel to the direction of motion.
The plotted points, especially those for the kinetic values, fall
approximately on the straight lines which have been drawn to
lie fairly among the plotted observations. The plotted points
would all lie exactly in a straight line had the friction been
proportional to the load and the experiment been perfectly per-
formed.
The coefficient of friction may also be found by another method.
Expt. Raise one end of the board XZ (Fig. 186), until the
block A, if started off, will slide down with steady speed. Con-
sider the forces acting on the block when sliding occurs. Its

yy -

s
,y
s *

i y s ^ HP
4* \ 10
* i
Fig. 185. Plotted results of an Fig. 186. Coefficient of friction
experiment on the friction determined by inclining the
of a slider. board.

weight W may be resolved into two forces, P parallel to the


board and perpendicular to it. R is
Q the reaction of the board
perpendicular to its surface and will be equal to Q. F is the
force of friction acting up the plane, and will be equal to P,
abed is the parallelogram of forces for W, P, and Q therefore
;

P:Q=cd:bc
= ab be. :

Now the triangles abc and XYZ are, from the construction,
similar to one another ; therefore
ab:bc = XY: YZ ;

.'. P:Q=XY: YZ,


or F:K = XY:YZ;
Coefficient of friction
F XY
= -^=
:. ^r?
K AZ
It therefore follows that, when the board is so adjusted that
1

FRICTION OF DRY AND LUBRICATED SURFACES. 141

the block will slide steadily down, the kinetic coefficient of


friction may be found by dividing the height of the plane by its

Repeat the experiment by raising one end of the board


until the block starts unaided. A calculation, similar to
that given above, will determine the static coefficient of
friction.

Limiting angle of resistance. Consider again the horizontal


board and slider. Let AB=R, and A C= F= the force of friction
when the block is about to move. Suppose a force Pl less than
F to be applied, as in Fig. 187, the
force of friction F = ALT will be equal
1 w
to P l5
and as this is less than F, the C
P
block will not slide. Under these C E A

conditions the table gives two forces


T" ""7

to the block, R and Fv Find the


resultant of these R1 by the parallelo- R
gram of forces. R 1
is the resultant /
reaction of the table on the block,
and acts at an angle BAG = (fi l , to the
perpendicular. As we increase Plt <j D G B
Fig. 187. Limiting angle of
will also increase, until a maximum resistance.
value <f>
will occur when P=F, and the
block will then slide. This angle DAB=<f>, is called the limiting
angle of resistance, or sometimes the angle of friction. Notice

Now
F
from the diagram that n=T5=
ItAn
the tangent of <f>.
-rz
K
is

the coefficient of friction, so that

Coefficient of friction = tan <f>.

Referring to the previous diagram (Fig. 186) of the slider on a


board with one end raised when the block is just beginning to
;

slide, <f>= angle acb = angle XZY, so that

Coefficient of friction = tan = tan XZY


<

XY as stated before.
YZ
The angle of resistance will be greater when the block is

just on the point of moving than after motion has occurred.


142 APPLIED MECHANICS FOR BEGINNERS.

Call <f>g
the angle in the first case and <f> k
the angle in the
second case, then
Static coefficient of friction = tan <f> s ,

Kinetic coefficient of friction = tan <f> k


.

If the coefficient of friction isknown, <j> may be obtained from


the table of natural tangents at the end of the book.

Expt. For showing any difference in the coefficient of friction
produced by changing the extent of the surfaces in contact, four
sliders cut from the same teak plank may

J be used. Each slider measures 6"x6".


One has the full surface, the others are
cut away (Fig. 188) on the under side so
J as to have rubbing surfaces respectively
Pie. 188. Sliders having of 27, 18 and 9 square inches. The rub-
r
bing surfaSes
aS f
b
king surfaces are thus in the proportion
1 075 0*5 025.: : :

The average results of some experiments are given, the sliders


having their grain parallel to the direction of motion and
sliding on a teak board.

t Effect of Extent of Surface on the Coefficient of


Friction.

Proportional Weight of Average


Slider. Area of Rubbing Slider, Coefficient of
Surface. lb. Friction.

A 10 0-88 0225
B 0-75 0-83 0-213

C 0-5 0-78 0-206

D 0-25 0-718 0-200

It will be observed from these figures that the coefficient of


friction is rather less for the smaller rubbing surfaces, that is,
it diminishes as the pressure per square inch rises. The law
that friction is independent of the extent of the surfaces in contact
is, however, shown to be approximately true.

The influence of speed on the coefficient of friction for unlubri-


cated surfaces may be tested by a machine such as that shown
FRICTION OF DRY AND LUBRICATED SURFACES. 143

in Fig. 189. In this machine a cast-iron pulley mounted on a


shaft can be driven at any desired speed. Blocks of different
materials have their under surfaces made concave to fit the rim

Fig. 189. Apparatus for experiments on the influence of speed on the


coefficient of frictiou.

of the wheel and rest on its top. These can be loaded by


weights placed in a scale pan underneath. The force of friction
is measured by the pull on the block of a horizontal cord, led

over a pulley and having a scale pan at its end. few results A
obtained with this machine are given.

Experiment on Influence of Speed on the Coefficient


of Friction.

Friction of brass block on cast-iron pulley at different speeds.


Surfaces dry and clean.
Diameter of pulley, 6".
Circumference of pulley, 1*571 feet.
N= revolutions per minute of pulley.

Rubbing speed = N x T571


feet per second,
ou
Weight of block, links and pan, 1 73 lbs.

Load in pan, 1 lb.


Constant load on block = 273 lbs.
Frictional resistances =J 1
lbs.
144 APPLIED MECHANICS FOR BEGINNERS.

Results of Experiment.

Coefficient of friction.
Revolutions per min. Rubbing speed, w p P
N. feet per second. lbs. lbs.
W'

1000 26 2 2 73 41 0-15
850 22 2 2 73 0-56 0-205
740 19 4 2 73 0-83 0-304
650 17 2-73 0-856, 0317
560 147 273 1-042 0-382
450 11-8 2 73 1-10 0-404
360 9 45 2 73 1-08? 0-396 ?

At 360 revolutions per minute there was a tendency to seize,


which made it difficult to obtain a reading of P. The increase
in the coefficient of friction as the speed falls is fairly regular.

Friction of a rope on a drum. When a rope is coiled round
a fixed drum, slipping will not occur until the pull on one end is
B considerably greater than that on the other
end. This is due to the friction between
the rope and the drum having to be over-
come. Suppose ABCD (Fig. 190) is a drum,
AC and BD being at 90 to one another,
and that a rope is coiled round 90 of arc
from A to B. We
might find that Pv when
W isdown, is equal to f W.
just sliding
If we give the rope other 90 of lap from
Fig. 190.
B to C, and apply P2 then, as the rope from
,

B to C is under the same conditions as between A and B, we


should expect to find

A = !A = fx |JF= 0-562 W.
Other 90 lap, to D }
should make
A = !A=fxfxfW' =0-422 JF,

and for one complete turn round the drum


^ = f^3=fxfxfxfW =0-315TF.
4
r

We see, then, that P is greatly reduced for the same load W


by coiling the rope further round the drum.
FRICTION OF DRY AND LUBRICATED SURFACES. 145

In an actual experiment a student obtained, using a silk cord


coiled on a pine drum, the following results :

An Experiment on Slipping.

Experimental Calculated
IF lbs. Plbs. Ratio Ratio
Angle of lap. P P
descending. ascending
W' w'

90 0-397 0-29 0-73 73


180 0-56 0-29 0-518 0-533
270 0-79 0-29 0-367 39
360 11 0-29 0-263 0-28

Taking the first ratio of -==0'73 for 90 lap,

. the ratio for 180 should be 0*73 x 0'73 = 0533 ;

for 270, 0-533 x 0'73 = 039 ;

for 360, 0-39 x 073 = 0*28.


The experimental results
agree as closely with the
calculated ones as we have
any right to expect when
we remember the assump-
tionwe have made with
regard to the condition of
the rope at different parte
of the circumference, and
also consider that the co-
efficient of friction will
certainly not be the same
at different parts of the
drum and cord. The effect
of want of perfect flexi-
bility of the cord also
tends to make the actual
from the cal-
results differ
culated ones.But the law
Fio. 191. Apparatus for experiments on the
simply stated as above friction of a cord coiled on a drum.

A.M.B.
146 APPLIED MECHANICS FOR BEGINNERS.

enables us to arrive at results, in questions concerning the


slipping of ropes on drums or belts on pulleys, which do not
very greatly from the actual ones. Suppose, for example
differ
that we had only performed the first experiment above,
W= 0*397 lb., P=0'29 lb. for 90 and we wished to predict
what W
would be for the same P with 360 lap.
For 90 lap, P = 0'73TF;
360 lap, P=(0'73)4 TF;
0-29 = 0-28 Pf,

Jf 103 lbs "


=o-l=
which does not differ greatly from the actual result 1*1 lbs.
For leather belts on iron pulleys the ratio of the pulls, when
the angle of lap is 180, is about 0*385 when the belt is just
slipping. It varies considerably with different cases, and the
number given is only of service to us when we want to calculate
the strength of belt required for a given
drive, and therefore require to calculate the
greatest pull in it. It should be noticed
that the diameter of the pulley or drum
does not affect the ratio of the pulls.
Friction of a rolling wheel. In rolling

o
friction,experiments show that the frietixmal
resistances are roughly proportional to the
load and inversely proportional to the radius
of the wheel or roller. In the case of a wheel
or roller rolling on a
plane surface, the
indented by the wheel, so that the
surface is
rim is in contact not at a line only as at A (Fig. 192), but over
a portion of the circumference BO. This introduces a certain
amount of sliding from B to O instead of pure rolling.
In roller and ball bearings the surfaces are usually lined with
hardened steel, and the rollers or balls are also of hard steel.
This reduces indentation as far as possible, and consequently
lowers the frictional resistances. In roller bearings, a cage is
necessary to keep the rollers in their proper relative positions,
and this is also often done in the case of ball bearings. An
example of a ball bearing for taking the thrust of a lathe
FRICTION OF DRY AND LUBRICATED SURFACES. 147

spindle is shown in Fig. 193, and a roller bearing for a truck


axle in Fig. 194.

Fig. 193. Ball bearing for a lathe spindle.


Experiment on rolling friction. In order to obtain some
idea of the resistance to rolling of a carriage along a level road of
different material, a small
three- wheeled carriage may
be used, the wheels being of
gun-metal and fixed to steel
axles carried on fine pivot
bearings so as to be as
frictionless as possible. The
roads consist of slabs of differ-
ent material, and are levelled
as accurately as possible. The FlG. 104. Roller bearing for a truck
axle.
horizontal force required to
maintain steady, slow motion is measured in the usual way by a
horizontal cord passing over a finely mounted pulley and having
a scale pan at its end.

Weight of carriage, 3"09 lbs.


Weight of scale pan, 0*04 lb.
W = weight of carriage -I- weight placed on it.

P= weight of scale pan + weights placed in it.


Diameter of wheels, If ".
Breadth of each of the pair on the same axle, ".
Breadth of the single one, ".
148 APPLIED MECHANICS FOR BEGINNERS.

The following are some of the results obtained :

An Experiment on Rolling Friction.

Resistance P lbs. to rolling on roads of


W lbs
Smooth cast iron. Teak. India-rubber.

3 09 03 0-047 047
10-09 0*125 016 0-20
17-09 0-20 0-25 0-36
24 09 0-28 0-36 051
31 09 0-36 0-47 0-70
38-09 0-44 061 0-95

The resistance to be overcome includes not only the rolling


friction, but also the frictional resistances of the pivot bearings
of the wheels and of the pulley used for the cord to run over.
The effect of the change of roads
is, however, clearly shown by
/ plotting the results (Fig. 195).
/
Notice that for the cast iron road,
/
/
the resistance is

portional to the load for


practically pro-
all loads,
/
<
/ /
/ and that for the teak and rubber
/ roads, especially the latter, the
f s
/
/S > & resistance is proportional to the
load up to a limit and then in-
/, xS creases much more rapidly. This
y.
/y
> p,
t

j i S 1 6
WHS
41
isdue to the wheels being forced
into the material of the road, and
thereby introducing a consider-

Fig. 195. Resistance to rolling on
able amount of slipping. We
roads of cast iron, teak, and rubber.
may deduce from these experi-
ments, that to minimise frictional resistances in wheels rolling on
roads, in roller bearings, or in ball bearings, that the materials
should be hard and not liable to be indented by the loads applied.
Fluid friction. The laws of friction for fluids differ con-
siderably from those stated for dry surfaces. For liquids such

FRICTION OF DRY AND LUBRICATED SURFACES. 149

as water and oils they have been shown experimentally to be as


follows :

The resistance is proportional to the extent of the surface wetted


by the liquid.
The resistance is independent of the material of which the
boundary is made and of its surface, provided it is not too rough.
The resistance is independent of the pressure to which the liquid
is subjected.
Rise of temperature of the liquid diminishes the resistance.
At sloio speeds the resistance is very small.
Below a certain critical speed, the resistance is proportional to
the speed; at speeds above this, the resistance is proportional to
some power, approximately the square, of the speed.
The critical speed depends on the liquid used and its tem-
perature. Below this speed the motion of the liquid is steady ;

above it, the liquid breaks up into eddies.


Liquids which can flow, or change their shapes more easily
than others, are said to be less viscous, or to possess less viscosity.
All liquids have one property in common they are unable to
resist shearing forces and yet remain at rest. Now friction is
always manifest as a tangential or shearing force, and it there-
fore follows that if a liquid is at rest there can be no frictional
resistances of any kind present.
Laws of friction for ordinary bearings.The laws of
friction for ordinary bearings are intermediate between those
for liquids and for dry surfaces. In bearings running in a bath
of oil, the laws have been shown to be approximately the same
as those of fluid friction, and in other bearings the resistances
experienced depend on the success which is achieved in getting
the oil into the bearing and in preserving the oil film. It is
not usual, in investigating the losses due to frictional resistances
of a number of bearings such as we find in any machine, to
attempt to ascertain how much is lost at each bearing, but to
find simply how much is lost at all the bearings collectively.
Almost always it is found that the frictional losses, when the
machine is loaded, are equal to those of the unloaded machine
together with a small fraction of the load. If we were to find
that the frictional losses in a machine were constant for all

loads, then we might deduce that the frictional resistances have


150 APPLIED MECHANICS FOR BEGINNERS.

been altogether due to fluid friction, this being independent of


the load.
Effect of friction in machines. It is useful, in dealing with
simple machines such as hoisting tackle, to deduce an equation
connecting the effect of friction in the machine with the actual
load applied.

Let P = force applied to work the machine ;

v = velocity ratio of machine ;

W= actual load raised ;

P and W being in the same units.

Suppose W to be raised one foot, then P will act through v


feet.
Energy supplied to machine =Pv ;

Useful energy obtained from machine = Wx 1 W.

Imagine the actual frictional resistances of the machine to be


removed, and an equivalent addition to the load IF to be made,
so that P is unaltered. Call this additional load F= effect of
friction, and measure it in the same units as P and W.
Energy lost in overcoming frictional. resistances Fx 1F.
By the principle of the conservation of energy, energy sup-
plied = useful energy obtained + energy lost.
.-. Pv=W+F,
or, F=Pv- W (1)

have been carried out on a given machine, a


If experiments
series of values of P and W
will have been obtained. From
(1), corresponding values of F
can be calculated. On plotting
the values of and F W
so found, it will generally be found that
the plotted points lie approximately on a straight line, showing
that the connection between and F W
can be represented by the
equation
F=aW+b, (2)

where a and b are constants for the machine.

Heating of bearings.
Almost the whole of the work done
in overcoming frictional resistances is transformed into heat.
The rubbing bodies therefore rise in temperature, until the loss
of heat by conduction, etc., balances the heat produced by the
FRICTION OF DRY AND LUBRICATED SURFACES. 151

continued rubbing. The temperature of the bodies then remains


constant.

Let i^=frictional resistance of a bearing, in lbs.,

V= distance in feet through which F is overcome in one


minute ;

then FV= work expended per minute, in foot-lbs.

.*. Horse-power absorbed in Overcoming friction = OQFV


nrtn
-

Taking 772 foot-lbs. as equivalent to one British thermal


unit (p. 126),

Heat generated per minute = rr B.T.U.


FV
<

oo,UU\J

Example. A makes 90 revolutions per minute.


shaft 6" diameter
The load on 4 tons, and the coefficient of friction 0*02.
it is

Calculate the H.P. absorbed in overcoming the friction of the


bearings and also the heat generated per minute.

TF = 4x 2240 = 8960 lbs.

Force of friction = 02 x 8960= 179*2 lbs.

F=7rrfx90

= 22 6
-=-x y^x 90=141 feet in one minute.

Work absorbed = 179-2 x 141


= 25,300 ft. -lbs. per minute.

25 300
Heat generated = -^^- = 32 -8 B.T.U. per minute.

EXERCISES ON CHAP. XL
1. found that a horizontal force of 8 lbs. can keep a load
It is
whose weight is 30 lbs. in steady motion along a horizontal surface.
What is the coefficient of friction?
2. A block
whose weight is 10 lbs. rests on a horizontal table ; it
is found that a pull of 4 lbs., applied at 30 to the table, just starts
it off. What is the static coefficient of friction ?

3. Answer Question 2 supposing the force applied had been a push.


4. The crank of an engine is 1 foot long and the connecting rod 5
feet. When the crank is at 90 to the centre line of the engine, the
push of the piston rod is 12,000 lbs. Taking the coefficient of friction
)

152 APPLIED MECHANICS FOR BEGINNERS.

as 0*06, find the frictional force opposing the sliding of the slipper
on the guide in this position.
5. The workingface of a slide valve of a steam engine measures
8^"x 15"; travel is 4".
its Steam pressure on the back of the valve,
120 lbs. per square inch. If the coefficient of friction is TV, calculate
the force required to move the valve and the horse-power absorbed
when the engine is running at 60 revolutions per minute.
6. An oak plank, 8 feet long, has a block of oak resting on it. If
the coefficient of friction is 45, how high must one end of the plank
be raised before slipping down will occur ?
7. In a belt driving a pulley, the ratio of the pulls in the two
parts of the belt is 0'4. A
difference in the pulls of 250 lbs. is
required for driving. Calculate the actual pulls.
8. A train whose speed is mile per minute has frictional resist-
ances amounting to 12 lbs. per ton weight of train. If this weight
is 150 tons, calculate the pulling force required and the horse-power
of the engine.
9. A shaft 4" diameter rotates 300 times per minute. If the load
on it is 1 tons, and the coefficient of friction 0"025, calculate the
H.P. absorbed in driving it and also the heat generated per minute.
10. What is friction ? What is meant by limiting friction, by
sliding friction, and by the coefficient of friction? A weight of
5 cwts. resting on a horizontal plane requires a horizontal force of
100 lbs. to move it against friction. What in that case is the value
of the coefficient of friction ? ( 1 896.

11.Define "force," "work," "foot-pound," and "horse-power."


A small metal planing machine, the table of which weighs 1 cwt.,
makes 6 backward and 6 forward strokes each of \\ feet in a minute,
and the coefficient of friction between the sliding surfaces is 0*07.
What is the work performed in foot-pounds per minute in moving
the table? (1896.)
12. How would you experimentally determine the nature of the
friction between clean smooth surfaces, say of oak, and what sort
of law would you expect to find ? (1897.)
13. An
express train going at 40 miles per hour weighs 150 tons ;

the average pull on it is 12 lbs. per ton, what is the horse-power


exerted? This power is only 40 per cent, of the total indicated
power of the engine find the indicated power.
; (1898.)
14.Describe any experiments you have made or seen for finding
the laws of solid friction. What are the laws so found ? Are they
quite true ? How do they differ from the laws of fluid friction ?
(1899.)
CHAPTER XII.

VELOCITY. ACCELERATION. INERTIA. KINETIC


ENERGY. RELATIVE VELOCITY. CHANGE OF
VELOCITY.


Velocity. The term Velocity has been used before when
considering machines. A commoner term with the same
meaning is speed. Both terms refer to the rate at which a body
is changing its position relative to other bodies. Velocity has
magnitude, direction, and sense, and, like force, may be
represented by a straight line.
Velocity is measured by stating the distance travelled by the
body in a given time. Thus, a velocity of 15 feet per second
means that in one second the body will travel a distance of 15
feet. When we say that the speed or velocity of a train is

55 miles an hour, we mean that if that velocity were kept up


constantly, the train would travel a distance of 55 miles in one
hour.
Velocities are seldom constant, but the velocity a body has at
any instant may be stated by considering what space the body
would travel in unit time if the velocity it has at the given
instant were kept constant. Thus, it is found that if a body
falls freely from a height, at the end of the first second of the
fall its velocity is This does not mean
32*2 feet per second.
that the body has fallen during the second a distance of
first

322 feet, or that it is going to travel that distance during the


next second. The meaning is that if the velocity possessed by
the body at the end of the first second were kept unaltered, it
would travel a distance of 32*2 feet during the next second.
Actually it does not remain unaltered, for at the end of the
154 APPLIED MECHANICS FOR BEGINNERS.

next second its velocity will be found to be 64*4 feet per second,
and at the end of the third second 96 6 feet per second and so
-

on, the speed continually increasing.


Uniform velocity will occur if the body travels over equal dis-
tances in equal intervals of time. Velocity which is not uniform
is said to be accelerating. If the speed is becoming greater the
acceleration is said to be positive, and if the speed is diminishing,
negative.

Parallelogram of velocities. Although it is impossible for
a rigid body to be moving in two directions at one and the
same time, yet it is often convenient to
think of a velocity as made up of two
component velocities. Thus, supposing
a point A to have component velocities
represented by AB=V 1
and AC= V 2,
its resultant velocity may be found by
Fig. 196. Parallelogram of ,, ,, ,
* , ... , . , .

velocities. the parallelogram or velocities, which is

similar to the parallelogram of forces.


On completing the parallelogram ABDC (Fig. 196), V AD=
will be the resultant velocity of the point A. Notice that
here, as with the parallelogram of forces, both velocities must
be either towards A or away from A before applying the
parallelogram.

Velocity-time diagrams. The velocity which a body has at
any instant can be very conveniently shown in a diagram.
Take the following case :

A train leaves Liverpool Street Station at 6.0 p.m., its speed


gradually increases and at 6.2 p.m. is 15 miles an hour, and
keeps uniform till 6.4 p.m., when brakes are applied, and at 6.4|
p.m. the train stops at Bethnal Green Station. After 1 minute
and in 1^ minutes its speed is 30
stop, the train starts again,
miles an hour and keeps uniform for 9 minutes. Brakes are
again applied, and it comes gradually to rest at Stratford Station
\ minute later.
Time being taken for abscissae and velocities for ordinates,
the diagram (Fig. 197) shows at a glance all that has occurred
to the train's speed.
The average speed of a body during a given journey can be
calculatedby dividing the total distance by the time taken to
VELOCITY. ACCELERATION. INERTIA. 155

perforin the journey. In the time taken should be included the


time lost in any stops. Thus, suppose a train takes 12 minutes

Fig. 197. Velocity-time diagram,

to perform a journey of 5 miles, its average velocity will be

12
= 0*4166 mile per minute, including stops.
Or, the average velocity may be found from its velocity-time
diagram by any of the well-known mensuration rules. In the
given diagram (Fig. 198), if the base be divided into 10 equal
parts and the sum of the velocities measured by the height of
the diagram at the centre of each part be taken, this sum
divided by 10 will give the average velocity.

Fig. 198. Fig. 199.


Acceleration. The case of a body gaining speed must now
be studied in more detail. Supposing it starts from rest at
time 0, and gains speed gradually throughout. Let its velocity
at the end of the first second be 4 ft. per second. Its velocity at
any instant during the first second will be shown by the height
of the diagram at that instant (Fig. 199). During the next
second, its velocity will gradually increase again, and at the end
of this second will be 8 feet per second, represented by 2B in
the figure. At the end of the third second its velocity will be
156 APPLIED MECHANICS FOR BEGINNERS.

12 feet per second and at the end of the fourth second 16 feet
per second, represented by 3C and 42) respectively. The gain
of velocity in any particular second, or positive acceleration, as
it is called, will be 4 feet per second. We state this by saying,
Acceleration = 4 feet per second, every second, or,
Acceleration = 4 (feet per second) (per second),

the part of the units in the first bracket referring to the gain of
velocity, and which that
in the second bracket to the time in
change of velocity took place.
Notice, in the diagram, that the total change in velocity in
4 seconds was 16 feet per second. So that we may find the
change per second in velocity by dividing the total change by
the time in which that change took place. Thus,
Acceleration =^ = 4 (feet per second) (per second).
We must be careful always to state not only the change in
velocity,but the time in which that change took place. Had the
body in the above case gone on moving for 10 seconds, its
velocity at the end would have been 40 feet per second.

Its acceleration = 40 (feet per second) (per 10 seconds),


or =16 (feet per second) (per 4 seconds),
or =4 (feet per second) (per second).

Referring to Fig. 199 again, if the body moves for 4 seconds,


its velocity at the end is 16 feet per second, its velocity at the
start is and the change was gradual and uniform. Its average
velocity will therefore be 8 feet per second. The distance
travelled may now be calculated from,

Distance = average velocity x time


= 8x4
= 32 feet.
This result can be represented by the area of the triangular
diagram 2)04 (Fig. 199), for its area will be the base 04
multiplied by half the height 2)4. Measuring the base in units
of time and the height in units of velocity, we get

Distance = area of 2)04 = 4 x ^ = 32 feet, as before.

Equations of motion. Taking an example with general


terms, let a body start from rest and gain every second a
VELOCITY. ACCELERATION. INERTIA. 157

velocity a feet per second. Let this go on for a time t seconds,


and call its velocity, at the end
of the time, v feet per second.
Let S be the distance travelled
in feet. The diagram shown
in Fig. 200 represents these
conditions.
The acceleration will be a
(feet per second) (per second),
Fig. 200.
and since a feet per second of
velocity are gained each second, in t seconds the gain will be
axt, so that
v = at (1)

The distance travelled, S, will be represented by the area of

the diagram, so that


S=b.t (2)

Or, substituting (a . t) for v in (2),

S=(%at)xt= :
, .(3)

Again, from (1) t = --

Substitute this in (3) giving

2a
v 2 = 2aS. (4)

Bodyfalling freely. We
have already seen that when a
body freely, its velocity increases by 32 '2 ft. per second
falls
every second. A
special symbol, g, is used for the acceleration
in this case, so that

<7 = 32'2 ft. per second per second.

It should be remembered that the acceleration of a falling


body is not always the same. magnitude depends on the
Its
latitude of the place. Applying the above equations to the case
158 APPLIED MECHANICS FOR BEGINNERS.

of a falling body, let h height fallen in time t seconds, and


v velocity at the end of this time, then

v=9* ()
h=%vt (6)
h=\9* (7)
v 2 =2gh (8)

In these equations the body is supposed to be falling freely,


that is, no atmospheric or other resistances oppose it.

Inertia. The importance of the study of acceleration from


the engineer's point of view lies in the fact that force must act
on a body to produce acceleration. no resultant force acts on
If
a body free to move when it is it will remain at rest, or,
at rest,
if the body is moving, it will continue to move with uniform
velocity in a straight line. This laziness, or inertia, as it is
called, of matter, must be overcome by a if any force or forces,
change in a body's velocity is to be made. Thus, if a machine
is at rest and is to be started, the pulls of the belt must

not only overcome the frictional resistances, but must also


overcome the resistances due to the inertia of the parts which
move when the machine is running. When the belt is thrown
on the loose pulley to stop the machine, the machine would go
on moving uniformly but for the frictional resistances applying
forces to the moving parts, thereby producing negative accelera-
tion, and so bringing the machine to rest.
We may estimate how much force is required to produce a
given acceleration in a body, by considering again the case of
a falling body. If the body has a mass of one pound, there will
be a force of one pound weight acting on it. This produces, in
all parts, practically, of the British Isles, an acceleration of 32*2
feet per second per second. If we could reduce the weight of
the body to h lb. without altering its mass, we should find that
the acceleration produced would be 16"1 feet per second per

second ; and if we could reduce the body's weight to --


32*2
lb. the

acceleration produced would be 1 foot per second per second.


In each of these cases the resistance due to the inertia of the

body will be an upward force of 1 lb., ^ lb., and lb. respec-


VELOCITY. ACCELERATION. INERTIA. 159

tively. Againif we could increase the mass of the body to

32'2 keeping its weight 1 lb., we should find that its


lbs., still

acceleration would be one foot per second per


Resistance due
second. In fact, the law is, the force required to tv Inertia -lie.

produce a given acceleration in a given body is


proportional to the product of the body's mass and
the required acceleration, and as we know that a
force of 1 lb. weight acting on a mass of 1 lb.
gives an acceleration of 32*2 feet per second per
Fla 201
second, we may calculate the force lbs. required P
to give an acceleration a feet per second per second to a mass
m lbs. from
n ma
P= 32-2
lbs -

If, instead of the pound weight as the unit of force, we were to

use a unit of force equal to -


>-'-
lb. weight, or - lb. weight, then
cj

this unit, acting on the one pound mass, would give an accelera-
tion of one foot per second per second. This unit of force is
called an absolute unit of force, and in our system of units
the
poundal. The absolute unit of force for the metric system is
the dyne, and is of such magnitude that it gives an acceleration
of one centimetre per second per second when it acts on a mass
of one grain. Using the poundal as the unit of force, the
equation stated above may be written
P=ma poundals.
Example. A body has a mass of 150 lbs. and we have to give it
an acceleration of 100 feet per sec. per sec. Find force required.

= ma 9 .,
lbs.

150x100
32-2
= 456 lbs.
Or in poundals,
P=456 x 32-2= 15,000 poundals.

Since the acceleration of a body, falling freely under the action


of its weight, is g feet per second per second, it follows that we

may express its weight, W, in poundals, from


W=mg poundals.
160 APPLIED MECHANICS FOR BEGINNERS.

Expt. To verify the law P= , arrange an apparatus as

shown in Fig. 202. This consists of two light pulleys attached


to a support as high as possible, and having a
WHtlllt,
light cord passing over them, with scale pans
at its ends.
1. Place equal masses in the pans. It will
be found that the pans remain at rest, and
that if motion is started by the hand, the
frictional resistances in the apparatus rapidly
bring it to rest.
2. Increase the mass in one of the pans,
^
mi
until its excess weight enables steadyJ
motion
Fio 202 Apparatus , , . , . ,
for verifying the to be maintained.
law p= . 3. An additional mass placed in the same
pan will now, by its weight, produce
acceleration in moving parts of the
the whole of the
apparatus. Place a known additional mass in this pan, and
elevate it a measured height. Allow it to descend, and note
the time of its descent, using a stop-watch for this purpose.
From these data, calculate the acceleration of the masses, and also
the force required to produce this acceleration, using the equa-

tion P= . The result should agree closely with the weight

of the additional mass used to produce acceleration in the


experiment if the law is true.
In one experiment the following results were obtained :

Mass of each scale pan = 0*72 lb.


Mass placed in A (Fig. 202) = 1 lb.
Mass placed in B to produce steady speed of descent = 1 "15 lbs.
Mass placed in B in acceleration experimental^ lbs.
Excess weight = force producing acceleration = 1*2 - 1*15
= 0-05 lb. weight.
In the acceleration experiment, B was allowed to descend
9 feet, and was found to do so in an average time of 6 "3 seconds.
s = $at 2 .

9=4a(6\3) 2 =a.x397
18
a=^r^r=0'45 feet per sec. per sec.
o9'7
VELOCITY. ACCELERATION. INERTIA. 161

Let ra=the whole mass set in motion, neglecting the masses of


the pulleys and cord.
m = 0-72 + 0'72 + 1 + + 0"2 = 364
1 lbs.

p= ma
9
364 x 0-45
= 0-0509 lb. weight,
322
which agrees closely with the actual P used in the experiment,
viz. 005 lb. weight.
Measurement of kinetic energy. When a body i i

falls freely from a given height, its potential -*-*

energy is gradually changed into kinetic energy


of an equal amount. Let W
be its weight in lbs.,
and h the height in feet from which it falls. Then
its potential energy = Wh foot-lbs. At the end
of its fall, its velocity will be

or v2 =2gh,

giving h = ~. Fig. 203.

We may therefore write,

Potential energy lost= JI7*

v2

= kinetic energy gained.

Consequently, the kinetic energy of a body is given by this


equation in terms of its weight and velocity. Notice that it
makes no difference in the kinetic energy possessed by a body
moving with a given velocity, whether it is moving vertically or
in any other direction, so that in general,

Kinetic energy = Wv foot-pounds,


2

or foot-tons if W is in tons.


Relative velocity. When we speak of a body being at rest,
what meaning do we attach to the statement ? Thus, a house
appears to be at rest, that is, it is not shifting its position on
A.M. 13. L
162 APPLIED MECHANICS FOR BEGINNERS.

the earth yet being attached to the earth, it possesses the


;

complicated motion of the earth at that place and is therefore


not actually at rest. In fact, no body is absolutely at rest.
When we speak of rest, we usually mean at rest relative to the
earth, that is, an observer standing on the earth perceives no
motion. If we say that a train has a velocity of 60 miles an
hour, we do not mean that this is the absolute velocity of the
train, but only its velocity relative to the earth. If two trains
are moving side by side with equal speeds, an observer in one
of them perceives no motion in the other and therefore says
that the relative velocity of the trains is zero. If the train
carrying the observer has a velocity of 30 miles an hour, and the
other, one of 35 miles an hour, he will see the other train
moving past him at a rate of 5 miles an hour, which velocity
he would call the relative velocities of the trains. If the second
trainwere going in the opposite direction at 35 miles an hour
the relative velocity would be 65 miles an hour. Relative
velocity of two bodies may be defined as the velocity which an
observer on one of them would perceive in the other. Thus, if a
stream of water moving at 8 feet per second reaches a water
wheel the buckets of which are moving at 6 feet per second, the
water will enter the buckets with a relative velocity of 2 feet
per second.
If two bodies A and B have velocities as shown at V1 and V2
in Fig. 204, their relative velocity can easily be obtained in the
following manner. Stop one of the bodies
by giving each of them a velocity equal
and opposite to the velocity of that body.
Then the resultant velocity of the other one
will be the relative velocities of the two
bodies ;thus, giving both A and B velocities
equal and opposite to V x
as shown in Fig.
204, the result will be to bring A to rest,
and B's velocity will now be VSi the
resultant of Vx
and V2 at B. This velocity
FlG
'city^andV
el '
Vr wiU be the relative velocity of A
and B.
As an example of relative velocity, take the case of a person
entering a compartment of a railway carriage when in motion.
VELOCITY. ACCELERATION. INERTIA 163

Suppose that the velocity of the carriage is 6 feet per second.


If the person desires to enter without being thrown against the
seats, he will contrive matters so that his velocity relative to the
carriage is along the line BA (Fig. 205), at 90 to the direction
of motion of the carriage. Suppose that this relative velocity is
to be 2 feet per second. Stop the carriage by giving both it and
the person at P velocities, towards the right, of 6 feet per second.
This is shown at P by the line PD = 6 feet per second. Now
VR = 2 by PE, has to be the relative
feet per second, represented
velocity of person and carriage, and hence must be the resultant
of his actual velocity along the platform and PD. Completing
the parallelogram PDEF, we get the velocity of the person along
the platform represented by PF= 6 32#
feet per second, and if he
runs along the platform in the direction PF
with this speed, he
will enter the compartment without shock along the line BA.


*>%-.
Fig. 205. Velocity of a person entering a Fio. 2U6. Velocity changed from
railway carriage. V\ along A B to V<t along BC.

Velocity changed in direction. Supposing a point is


moving along a line A B (Fig. 206), with a velocity Vlt and that
when it arrives at B, something is done to it, in consequence of

which it moves off along BC with a velocity V2 Let us .

examine what change must be effected in its velocity to produce


this result. First stop the point at B by giving it a velocity
equal and opposite to Vv This is shown by DB in the figure.
Then give it a velocity V2 in the direction BC, this will fulfil
the required conditions F 2 being represented by EB. To find
;

the resultant change in velocity, find the resultant of Vx and V2


by the parallelogram of velocities DBEF. Then FB= Pis the
resultant change in velocity. A moving point has been
considered instead of a body, in order to avoid drawing on the
164 APPLIED MECHANICS FOR BEGINNERS.

imagination in following out the process of reasoning, for we


have to think of velocities applied suddenly
at B, and a sudden change in^elocity
implies an infinitely great acceleration,
and therefore an infinitely large force to
be applied to the body at B. This, of
course, is but what actually
impossible,
occurs is a gradual change in velocity
causing the body to turn gently into the
C direction BC along a curve (Fig. 207).
F,
,n vel^UirC The line FB in Fig. 206 shows, however,
gradual. the total change in the velocity of the body.

EXERCISES ON CHAP. XII.

1. What distance will be travelled in 5 seconds by a train


running at 60 miles per hour ?
2. A train is observed to pass two points 480 ft. apart in
10 seconds. What is its speed in miles per hour ?

3. A ship is moving due north with a speed of 10 knots. A


person crosses the deck from port to starboard, a distance of
40 feet, in 10 seconds. What is his actual velocity relative to the
earth ? Give a diagram.
4. What is the average speed of a train which travels from
London to Edinburgh, a distance of 400 miles, in 8 hours ?

5. A ship steadily acquires a speed of 15 knots, from rest, in


5 minutes. What has been its acceleration in foot and second units?
6. A body falling freely passes two points, the vertical distance
between which is 120 feet, in two seconds. From what height
above the higher point was it dropped ?
7. A
train the mass of which is 300 tons is started from rest and
gains a speed of 30 miles an hour in 4 minutes. Calculate the force
required, additional to that utilised in overcoming frictional resist-
ances, to overcome the inertia of the train.

8. An engine, of stroke 8", makes 300 revolutions per minute.


What is the average speed of the piston in feet per minute ?
9. At 300 revolutions per minute, it is found that the piston of
an engine, when leaving the inner dead point, has an acceleration of
400 feet per second per second. The mass of piston, rod, and cross-
head is 150 lbs. Calculate the force required to overcome the inertia
of these parts.
,)

VELOCITY. ACCELERATION. INERTIA. 165

10. The moving parts of a steam hammer have a mass of 500 lbs.
and are raised a height of 3 feet above the work before each blow.
Wkftt is the kinetic energy of these parts when the hammer head
is just reaching the work, assuming no frictional losses, and that
steam is used for lifting the hammer only ?
11. What is the kinetic energy possessed by a hammer head, mass
2 lbs., moving with a velocity of 40 feet per second?

12. What kinetic energy has a ship of 15,000 tons mass when its
speed is 20 knots ?
13. What exactly does a man mean when he says " this train is
going at 30 miles an hour " ? Suppose you have a watch with a
seconds hand, and know that the telegraph posts are 200' apart, how
can you approximately find the speed of the train ? (1897.)
14. A body is moving towards the north at 40 feet per second.
In two seconds later, we find it moving towards the north at 50 feet
per second. What velocity has been added in these two seconds ?
(1898.)

15. A body
moving towards the north at 50 feet per second.
is
In two seconds afterwards we find that it is moving towards the
north-east at 60 feet per second. Find by drawing what is the
added velocity. State the magnitude and direction of the added
velocity. (1898.)

16. Suppose a body to have fallen h feet in t seconds from rest


according to the law A = 16*1 P. Find how far it falls between the
times t = 3 and 2 = 3'1 ; between 2 = 3 and = 3*01 ; between t = 3 and
t = 3*001 Find the average velocity in each of these intervals of
.

time. What do we mean by the actual velocity when t is 3 seconds?


(1898.)
17. A
bullet weighing 1 oz. leaves the muzzle of a rifle with a
velocity of 1350 feet per sec. What is the kinetic energy of the
bullet in foot-lbs. ? (1899.
CHAPTER XIII

MECHANISM. TRANSMISSION OF MOTION AND POWER.

Driving by belt. Motion may be transmitted from one


shaft to another in many different ways. If the shafts are
parallel to one another and
a considerable distance apart,
the most convenient way is
by the use of a belt, or rope,
lapping round pulleys fixed
Fig. 20S. Driving by open belt.
to the two shafts. Both
shafts will rotate in the same direction if the belt is open, as
in Fig. 208, and in opposite directions if the belt is crossed, as
in Fig. 209. A, the shaft
supplying motion, is called
the driver, B is the driven
shaft. The action of driving
is possible by reason of the
Fig. 209. Driving by crossed belt.
frictional resistance to slip-
ping of the belt on the pulleys, but there will always be a
certain amount of slipping, introducing some loss of motion.
Neglecting this, the velocity ratio of the shafts may be found
thus.
Let RA = radius of the pulley on A .

Rb = /;.

Then, if no slipping, the circumferences of both pulleys


there is

will move through the same distance in a given time, for each
will have the same speed as the belt. Suppose, then, that A
turns once; its circumference will travel a distance = 2irRA >
TRANSMISSION OF MOTION AND POWER. 167

The circumference of B will move through an equal distance


2ttRa
and consequently B will turn through
circumference of B
revolutions, or

Revolutions of B for one of A = -rr = W^-


Z7r/i fi tlB
If then, A rotates NA times in a minute, and B rotates NR
times also in a minute,
Nb_Ra
Na'RJ
or the revolutions of the shafts are inversely proportional to the
radii of the pulleys mounted on them.
The power transmitted in any given case can easily be cal-
culated if we know the tensions in the two parts of the driving
belt. This has been seen in Chap. XI. to be about 0*385 for a
leather belt on a cast iron pulley.
Let T = pull
x
in the tight part of the belt, lbs.
T2 = slack lbs.
r= distance travelled in one minute by a point on the
belt, in feet.
Then, considering the driver A (Fig. 210),
To is assisting the pulley to turn and Tx is

retarding it, so that the driver delivers a


net pull {Tx To) pounds, by means of the
belt, to the driven pulley. The work done ~*T.
in one minute will be

W
foot-pounds. Fio. 210. -Pulls trans-
mitted by the belt.
xr
W=(Ttm - m\
T2 )x i"..
T ft. -lbs. per minute, and
. . ,

(T - T.)V
Horse-power transmitted =

330OO
Let us take roughly T2 =0*4. Tv then
r -7 2 =0-6.7
i
T T
1

0*6 T
and Horse-power transmitted = ^-^. r.

Example. A belt running at 900 ft. per minute has a pull in its

tight part of 400 lbs. Calculate the horse-power transmitted.


0-6 x 400x900
"'^
" 33,000
=65.
-

168 APPLIED MECHANICS FOR BEGINNERS.

If instead of the speed of the belt, we are given the diameter,


D ft., and N, the number of revolutions per minute
of the pulley,
of the shaft on which it is mounted, then
V=ttDxN,
and Horse-power transmitted = ^ *
y .


Losses by slip. Slipping of the belt introduces a loss of
energy in overcoming frictional resistances between the belt
and pulley. The amount of slipping is variable, and depends
on the power transmitted and the tightness of the belt. Its
amount may be found experimentally by first calculating what
revolutions the driven shaft should make, using the expression
for the velocity ratio
Nb_Ra
Na~Rb
Then, actually count the revolutions of the two shafts for, say,
one minute, repeating two or three times and taking the
average. The actual velocity ratio will be found by dividing
the actual revolutions of A by those of B.
Slipping can then be expressed as a percentage. Thus, sup-
pose a pulley of 3 ft. diameter is driving one of 1 ft.diameter
and that the driver rotates 120 times in one minute.
3
The calculated velocity ratio = :r = 3, so that B should rotate

360 times in one minute. Suppose it is found on trial that B


rotates 340 times per minute Then .

Actual velocity ratio = '

= 283.
The lost revolutions of the driven shaft will be 20 in 360, or
20
k^x 100 = 5-5 per cent. We are therefore losing 5*5 per cent.
3o0
and also the same percentage
of the motion, of the power which
might have been transmitted if no slip.

Use of guide pulleys. The belt must always be delivered to
the pulley while moving in the same plane as the pulley is
rotating, but may leave the pulley in a different plane. There
is no difficulty in arranging this condition when the shafts are
TRANSMISSION OF MOTION AND POWER.

parallel they must


be simply placed opposite one another, but
in cases parallel, guide pulleys may be
where the shafts are not
required in order to direct the belt into the proper plane. A
case of this is shown in Fig. 211, where the belt leaving the top
of the driving pulley is directed into the plane of rotation of
the driven pulley by the guide pulley C. The lower side of the
belt would be similarly guided. Belt pulleys are generally
rounded on the face. The tendency is for the belt to climb to
the highest part of the rim and consequently this helps it to
stay on the pulley.

Fig. 211. Belt guided by pulleys at C. Fig. 212. Section of rim of a


rope pulley.

Ropes used for driving are generally of round white cotton.


The rims of the pulleys are grooved in this case (Fig. 212) to
receive the ropes. The effect of the V-shaped groove is to give
better adhesion and thereby minimise slipping.

Belt pulley arrangements. Belt driving can be conveniently
arranged for machines which have to be often stopped or started.
This may be managed as shown in
Fig. 213, by putting a pulley on F
the driven shaft, keyed to the V j\y Elevation \ By
shaft, and also one Z, arranged n
to run loose on the driven shaft, F
and L being close together. The
Fig. 213 -Airangement for starting
belt runs on a r J fixed at A
pulley
and stopping a machine.
on the driving shaft, this pulley
being as broad as F
and L together. The belt is guided on to
either F or L
by means of forks secured to a sliding bar C,
which may be moved by hand. These forks must engage with
the belt on that part which is advancing towards the pulleys

on B. When the belt is running on F, B will be driven, and


when on Z, B will stop, the pulley L then simply running on
B without driving it.
170 APPLIED MECHANICS FOR BEGINKERS.

An arrangement of crossed and open belts may also be used


for giving a reversing motion to a machine. Thus, in Fig. 214
B is the shaft from which the machine is driven, and it carries
two L x and X2 with a fast pulley between them.
loose pulleys, , F
T) is an open belt and E
a crossed one, both guided by forks on
a bar C. In the present position, the open belt D is on the fast
pulley F and the
machine will be driven in one direction. If
the bar C is moved downwards in the plan, D will run on L 2
and E
on L v the machine will therefore be at rest. Further
motion of C downwards will bring E
on to the fast pulley and F
the machine will consequently run in the reverse direction to
the former. The motion of C may be automatically effected, as
in an ordinary planing machine, by rods and levers worked
by the machine itself. The machine will then be self-acting.

e
A
Fig. 214.
Plan

Arrangement
machine.
for reversing a Fig.
Y
215. Speed cones.

Speed cones are belt pulleys having several steps on which the
belt may run (Fig. 215). Their object is to secure a velocity
ratio which may be varied to suit the particular work on which
the machine is engaged. The velocity ratio when the belt is on
any particular pair may be calculated
as before, from the radii of the steps.
Friction gearing. If the driving
shaft and the driven shaft are close
enough together, the rims of the
pulleys may touch one another, and
if enough friction
pressed together,
be
willproduced to enable the one to
Fio. 21ii. Friction wheels.
drive the other, provided the power
being transmitted is not too great. In Fig. 216, A is the
driver and B the driven pulley. It will be noticed that the
TRANSMISSION OF MOTION AND POWER. 171

shafts now rotate in opposite directions. There will always be


a certain amount of slipping with friction gears, and to mini-
mise this, one wheel may have its rim made of compressed paper
or leather, thereby giving a larger coefficient of friction than
can be obtained with metal surfaces. If we neglect slipping,
and as before, call
Radius of A = RM
Radius of B = RB ,

Revolutions of A = NA per minute,


Revolutions of B=NB per minute.
Then, since both circumferences will have the same speed if
there is no slip, we may obtain in the same manner as for belt

pulleys,
Na_Rb
n b r;
The pressure on the shaft bearings due to the forces pressing
the wheels together is objectionable, especially if the shaft is

running at high speed. This may be


got rid of by using two pulleys B and
C (Fig. 217), considerably larger than
the driver A, and with the bearings
mounted so that each has a limited Fig. 217.
horizontal travel. A
belt passed
round B and C will by its tensions cause the driving pulley
A to be nipped between B and C without thereby giving
pressures to the shaft bearings. B and C will each be driven
in the opposite direction to A, and
power may be taken from another
pulley mounted on the same shaft as
either B or C.
If the shafts are not parallel, but
have their axes inclined and meeting
at a point C (Fig. 218), then two cones,
one mounted on each, may be used
for driving. These must be pressed
Fig. 218. Friction cones.
together also. The radius of each
cone at any place will be proportional to the distance from 6',
and consequently the circumferences will also be proportional
R

172 APPLIED MECHANICS FOR BEGINNERS.

to the same distance. This being the case, when motion occurs,
the cones will roll on one another without slip at any of the
parts in contact, or if slip does occur, the amount will be
proportional anywhere to the distance from C.
Toothed wheels. To prevent slipping, the discs composing
friction wheels may have teeth cast or cut on the rims (Fig. 219),
the teeth on each disc
projecting beyond the
edge of the rim, and also
coming inside the edge.
The teeth on one wheel
engage with the teeth
on the other, and so
prevent any slip. The
edges of the original
have now dis-
discs
appeared, but we may
Fig. 219. Toothed wheels in gear.
imagine them still to be
there, and the wheels will rotate just as though these discs
were rolling on one another. This imaginary circle showing
the original discis called the pitch circle, and the distance

from centre to centre of two teeth measured along the pitch


circle is called the pitch of the teeth.

Let nA = number of teeth on A'b rim.


nB = B'a
Then, calling the pitch p, which must obviously be the same for
each wheel,
circumference of A = 2ttRa =p x nM
circumference of B=27rR B =p x n B ,

and from this,


RaJTIa
RB n B
., revolutions of A RA ,

revolutions of B~ B
'

or the revolutions per minute of the two wheels are inversely


proportional to their numbers of teeth.
TRANSMISSION OF MOTION AND POWER. 173


Use of idle wheels. Two toothed wheels in gear with one
another must rotate in opposite directions. If both are required
to rotate in the same direction, then another wheel, mounted on
an intermediate shaft, and gearing with both driver and driven
wheels, is required. This is shown in Fig. 220. A and B will
now rotate in the same direction, and since the speed of the
circumferences of all three pitch circles will be the same, it
follows that 2fA j{

also
alS
X-*>
NB -Rc
Multiplying the left-hand sides of these equations together and
also the right-hand sides, we get
NA x No Rr
Nc NB

Fig. 220. A drives B through the idle Fig. 221. C, D, and E are idle wheels.
wheel C.

The relative speeds of rotation of the driver and driven wheels is

therefore the same as if they


geared direct. The only object
of C is to change the direc-
tion of rotation, and as it does
not alter the velocity ratio it
is generally called an idle wheel. ELEVATION
In the same way we may show
that any number of idle wheels, mh ::z:
as in Fig. 221, may be inter-
nunnnn
posed without affecting the F
Ilkmnm mnnnni
velocity ratio.

Trains of wheels.Where PLAN.


Fig. 222. Train of wheels
a considerable velocity ratio is

required, trains of wheels arranged as in Fig. 222 may be


174 APPLIED MECHANICS FOR BEGINNERS.

employed. In this case, the velocity ratios of the various pairs


in gear will be respectively,

Rb Rd Rf .

Ra Rc Re
%b nj> n*
nA nc ne'

Supposing F to rotate once, the revolutions of E will be nF , and

I) will have the same number of revolutions. For one revolution

of D, C will have revolutions, and consequently for one of F,


Tic

will have x revolutions, and B will have the same


number. For one revolu-
tion of B, A will have
revolutions, and there-
nA
fore for one revolution of
F, A will have
nF
nD
nB
- x x revolutions.
n A n c nE
,
'

We see therefore, that


the velocity ratio of the
first and last wheels in the
train is found by taking
the product of the numbers
of teeth on all the drivers,
F, D
and B, and dividing
thisby the product of the
numbers of teeth on all
the driven wheels, E, G
and A.
Trains of wheels such
Fio. 223. Driving arrangements in a self-acting
lathe. as these are much used
in machinery, and the
engineering student will have no difficulty in observing many
examples of them for himself. Fig. 22o shows one which can
be examined on an ordinary self-acting lathe.

TRANSMISSION OF MOTION AND POWER, 175

Epicyclic trains of wheels contain usually one fixed wheel, such


as A (Fig. 224), secured to a bracket so that it may not rotate,
and several others in gear,
such as B and (7, mounted on
pins secured to an arm D which
can rotate about the axis of
A. Each wheel B and C may
rotate on its own axis at the
same time that it is carried
round A by the rotating arm.
The simplest way of finding
the revolutions of any wheel
PLAN.
of the set is to proceed thus :
Fig. 224. Epicyclic train of wheels.
Imagine the whole of the
wheels to be locked to the arm and the bracket carrying A to
be loosened from its supports. Looking at the plan (Fig. 224),
give the whole arrangement one clockwise revolution. Then we
have given revolutions to each wheel and the arm as shown :

A - c D

One revolution One revolution One revolution One revolution


clockwise clockwise clockwise clockwise

Now in the actual arrangement, A should not rotate, being a


fixed wheel,and as we have given it above a clockwise revolution,
we correct this by keeping the arm fixed and rotating the wheel
A once anticlockwise. This will give revolutions to B and C
thus :

A B C D

One revolution revolutions revolutions


anticlockwise
clockwise anticlockwise

We have now fulfilled in this manner the conditions of the


gear, and have given the aim D one revolution, clockwise,
176 APPLIED MECHANICS FOR BEGINNERS.

the fixed wheel A being where it was at first. The effect on


the other wheels will be given by the algebraic sum of the
corresponding columns of the tables, thus,

B will have ( 1 + ) revolutions, clockwise.

G will have f
\
1 -J revolutions, clockwise
nc l
if positive, anticlockwise if negative.

It will be noticed that if A and C have each the same number


of teeth,

si-i;
nc
and the revolutions of C will be
(1-1) = 0.
In this case Cdoes not rotate on its axis while the arm rotates.
A model epicyclic train such as shown above is easily arranged
and will be found very useful in explaining the action and what
has been discussed. Epicyclic gears have been used in various
machines, such as rope-spinning machines, and also for reducing
from a higher to a lower speed of rotation.
Shape of teeth.
If the velocity ratio of a pair of wheels in
gear is not to alter at any time, and this is essential to smooth
running, then the condition
,- J
v>- / which must be attended to
/ w*v' *n &i vm o the teeth their
\ f^L* [ proper shape, is that the

.'

" O -Jc
0----
common perpendicular to the
outline of two teeth, at any
\ ,'''
\ place where they are in contact,
must pass through the point
Fig. 225. The common perpendicular at where the pitch circles of the
P must pass through C.
r . ._. .
, ,._.*
two wheels touch (Fig. 225).
This condition is fulfilled if portions of cycloidal curves are
used for the teeth outlines, although other curves may be
used also. The student is referred to books on Machine
Design for the methods of drawing these curves and designing
the teeth.
=

TRANSMISSION OF MOTION AND POWER. 177

Power transmitted by toothed wheels. The resultant


tangential driving force between two toothed wheels may be
easily found if we know the power being trans-
mitted by one of the wheels and also its radius.
Thus, let .4 (Fig.*226) be a wheel the teeth
of which encounter a resultant resistance of
P lbs. from another wheel which it is driving.
Let R = radius of A in feet.
Then in one revolution of A, P is overcome Fig. 226.
through a distance = 2irR feet, and therefore
the work done = Px 2ttR ft. -lbs.

If the revolutions per minute = A7 then ,

PxZttRxN
Horse-power :

33,000 '

33,000 x Horse-power
or, '

2ttR*N
Having obtained P, the thickness of tooth required for
strengthmay be found by the ordinary proportional rules for
cantilevers as described in Chap. VIII.
Bevel wheels are evolved from coned friction wheels by giving
teeth to the cones in the same way as for ordinary toothed
wheels.
The screw consists of two portions, one A (Fig. 227) cylin-
drical, and free to rotate but not to slide axially, and the other B,

Fig. 227. Section through a nut, B, showing screw, A.

called the nut, free to slide axially but not to rotate. A helical
thread is cut on the outside of A and a corresponding one on
the inside of B, so that A may fit in B. If A is rotated, B will
slide axially. Combinations of the screw and nut are very
A.M.B. M
,

178 APPLIED iMECHANICS FOR BEGINNERS.

often used and take many forms. The threads also take
many different shapes. As regards the relative motions of
A and B. Let B have N
threads per inch, then the distance
from thread to thread, measured from corresponding places on

the threads, will be -y. This distance is called the pitch of the

thread, and may be written p inches. For one revolution of A,


B will slide a distance =p inches.

Example. The ordinary and nut


is an example of a screw.
bolt
Supposing a per inch, to be screwed down by a
%' bolt, 12 threads
spanner, the turning force being applied 7" from the axis of the
bolt, and equal to 20 lbs. what pull will be produced on the bolt,
,

neglecting friction ?
Let P = force on spanner, lbs.,
B= radius of P, inches,
Q = pull on bolt, lbs.
p = pitch of screw, inches.
Then, if there is no friction,
Work done by P in one revolution = Work done in overcoming Q
through a distance equal to the pitch.
Px2tB = Qx P ,

20 x 2 x 22 x 7 x 12
Q 7
10,560 lbs.

If 75 per cent, is lost in overcoming frietional resistances, then,


pull on the bolt will be
25
x 10,560 = 2640 lbs.
J
Worm-wheel gearing. A useful application of the screw is

to be found in the worm and worm wheel. A short screw of


comparatively large diameter is mounted on the driving shaft,
and this gears with specially shaped teeth on the rim of a wheel
mounted on a shaft, the axis of which is perpendicular to the
driving shaft. On the driving shaft rotating, the worm mounted
on it will drive the worm wheel, one tooth being advanced for
every revolution of the worm. The relative velocities of rotation
will therefore be simply equal to the number of teeth on the
worm wheel. The driving shaft must be prevented from axial
TRANSMISSION OF MOTION AND POWER. 179

movement under the thrust produced by driving the worm


wheel. In Fig. 228 the driving shaft is fitted with ball thrust
bearings so as to minimise as far as possible frictional losses

Fig. 228. Worm and worm wheel.

due to collar friction. The worm and worm wheel is often


employed for reducing from high to low speeds of rotation, and is
specially adapted for this on account of the large velocity ratio
easily obtainable.
Link mechanisms. Links are pieces used for transmitting
motion from one point to another, the motion being usually
modified. Thus, in the ordinary crank and connecting rod, the
reciprocating motion of the crosshead is transmitted by a con-

necting link the connecting rod to the crank pin, which has a
motion of rotation. In link work, the motion of each piece is
usually quite definite, thatis, it does not move at random in any

manner, but has a limited degree of freedom, being constrained


by the other pieces to which it is connected to move always over
the same path in the same manner. The principal problems we
require to solve in connection with link work mechanisms are :

(a) The complete path traversed by any given point of the


mechanism ;

(6) The velocity of any point at any instant ;

(c) The acceleration of any point at any instant.


The best way of answering (a) for any given mechanism is to
draw the arrangement to scale in several different positions of
180 APPLIED MECHANICS FOR BEGINNERS.

the motion, the point under consideration being marked on


each. A
curve showing the complete path can then be drawn
through these points. Such a curve is shown for a point on D
a connecting rod in Fig. 229.

Fig. 229. Path of a point D on the connecting rod.

Velocity of any part of a rotating body. If a body is


uniformly rotating, then the velocity of any point in it can
easily be found if we know the
velocity of a given point. Thus,
given any body rotating about
C in the plane of the paper
(Fig. 230), the velocity of a
given point A will be always
directed tangentially to the
circular path of A, and is shown
so in the given position, perpen-
dicular to the radius AC. Let
the velocity of A be Vv The
velocity of any other point in
the body, such as 2?, when the
Fig. 230. Velocities of two points, body is in this position, will be
A and B, in a rotating body.
V
perpendicular to the radius
2,
BC, that is, tangential to the path of B. In one revolution of
the body, A will move a distance equal to 27rx AC, and B will
move a distance 2tt x BC, so that
F
x
: V2 =2ttxAC:2ttxBC
= AC:BC
or the velocities of any two points are proportional to their radii.
The velocities of any point in a link may be easily found after
the following facts have been studied. Consider a point A
TRANSMISSION OF MOTION AND POWER. 181

moving with uniform velocity v in the circumference


(Fig. 231),
which is C. For an instant the point A
of a circle the centre of
may be considered to be moving with
uniform velocity v in the tangent to the
circle, AB, and we need
in imagining this,
not consider the magnitude of the radius
of the circle. In fact, A will be moving
in the direction AB if it is turning
about any centre whatever in the line
AC, or AC produced. Thus, D might
be taken for an instant as the centre of
rotation without thereby altering the
direction of the motion of A.
Crank and connecting-rod mechanism.
Let us now examine the connecting rod Fig. 231.
of a crank and connecting-rod mechanism.
In Fig. 232, AB is the connecting rod and BC the crank.
A always moves in the straight line AC and B always

Pig. 232. Velocity of any point in the connecting rod.


moves in the circumference of the circle LBR. Let the
velocity of B be uniform and equal to v. In the given position,
182 APPLIED MECHANICS FOR BEGINNERS.

B is actually turning about C, and the direction of its motion is

in the tangent BD This direction will not


to the circle at B.
be changed if we consider B to be moving about any point in
CB or CB produced thus, we might imagine it to be turning
;

about E for
an instant. A is actually moving in AC, but we
may imagine it to be turning about any centre in AF,
perpendicular to AC thus, if turning about
; for an instant, F
the motion of A will be along A C. Now B may turn about any
point in CB, and A may turn about any point in AF, so that if
we choose /, where CE and AF
intersect, both A and B may be
considered as rotating about the same point / for a very brief
interval of time. The velocity of A may now be found, for the
velocities of A and B will be directly proportional to their radii
AI and BL
Let V= velocity of A, then
v: V=BI:AI
vx AI
v=-
rr
sr -

Since two points, A and B in the rod, are turning for


an
instant about I, the whole rod about / for an
is also turning
instant, so that the velocity of any other point in it, such as G,
will be found by joining G to /. The velocity of G will be
directed at 90 to GI and can be found from
V6 :v = IG:IB
rr vx IG

/ is called the instantaneous centre of the rod for the given


position. In order to find the
velocity any point in the
of
rod, the mechanism should be
drawn to scale in the required
position, the instantaneous
centre found, and the radii
measured for substitution in
Fio. 233. Velocity curve for end A of the
connecting rod. the above equation. If this
is done for, say, twelve crank

positions differing by 30, values may be found for the velocity


of A, which, when plotted, will give the velocity of A for any
crank position. This is shown in Fig. 233.
TRANSMISSION OF MOTION AND POWER. 183

As another example, take two cranks CA and DB (Fig. 234),


connected by a link AB. Let the velocity of A be uniform and
equal to v it is required to find
;

V, the velocity of B in the given


I

position. The mechanism having


been drawn to scale, / will be at
the intersection of CA and DB,
and
v:V=AI:BI
V=
vxBI
AI
w hich may be found numerically
T

by measuring BI and A I and


Fig. 234. Double crank and con-
inserting the values in this equa- necting rod.
tion.
Use of models. In the crank and connecting-rod mechanism,
and, indeed, in any linkwork mechanism, a good deal can be
Fig. 235 shows a crank and
learned by studying simple models.
connecting-rod model in which the connecting rod may be

Fig. 235. Adjustable crank and connecting-rod model.

varied in length. The piston position for any crank angle can
be read on a scale against which a pointer attached to the model
piston slides. The disturbing effect of the connecting rod, and
the influence of its length may be conveniently studied with
such a model.

Acceleration of a point in a mechanism. Until the student
has acquired a more extensive knowledge of the subject, the
following approximation to the acceleration of a point in a link
may be used. Considering the velocity diagram shown in
Fig. 233 for the velocity of the crosshead at any crank
184 APPLIED MECHANICS FOR BEGINNERS.

position. The diagram is drawn on a base line showing equal


angles turned through by the crank, and as the crank is supposed
to be moving uniformly, we may think of this as being a base
line showing equal times.

Example. Suppose the crank is \ ft. radius, and that the


velocity of the crank pin is 3 ft. per second. Find the time in which
the crank rotates 30.

Circumference of crank pin circle = 2?r r .

= 2x
Tx2
22,
=y
.

feet.

,.
lime m
. , . ,
pin revolves once =
which crank r
, . . circumference
=

velocity
:-

22 1
=T X 3

= seconds.

22 30
And time in which it rotates 30 = ~y x ^
22
252

=0-0873 sec.

Each interval in the base line of the velocity diagram may


therefore be taken as 0-0873 second. Now in the interval from 0
to 30 the crosshead changes its velocity from to 184 feet per
second, so that its average acceleration during this interval will

be =21 'I feet per sec. per sec.

Repeating this process for all the intervals, we obtain a


showing the average acceleration of the cross-
series of values
head during each interval, and if these values are set up as
ordinates from the centres of the intervals, a curve may be
drawn showing approximately the acceleration of the crosshead
=

TRANSMISSION OF MOTION AND POWER. 185

throughout the crank revolution. The values are best set down
in a table thus :

Acceleration of Crosshead.

Velocity of Difference in Acceleration


Crank Angle. Crosshead, Velocity of difference . ,

feet per second. Crosshead. feet ' SeC 8CC -


0-0873 -

30 + 184 + 1-84 + 211


60 + 2-84 + 100 + 11-4
90 + 300 + 0-16 + 1-83
-0 70 - 8-02
120 + 2 30 -105 -1100
150 + 1-25 -1-25 - 14-3
180
210 -1-25
-1-25 -143
240 -230 -105 -1100
270 -300 -070 - 8-02
300 -2-84 + 016 + 183
330 -1-84 + 100 + 11-4
360
+ 1-84 + 211

This curve is shown in Fig. 236, positive accelerations being


shown above, and negative accelerations below, the base line.
The crosses show the points
plotted from the above table
and the dots are plotted from
exact data, both being given
in order to show how little

the actual acceleration differs


from that found approximately
above.
Infinite connecting - rod
mechanism. The disturbing
effects produced by the varying
inclination of the connecting
rod in a crank and connecting-
Fio. 236. Acceleration curve for end A
rod mechanism, can be got rid of the connecting rod.
of, if, for the connecting rod,

a slotted bar is substituted (Fig. 237), the bar being guided


to move in a straight line. The crank pin works in the
186 APPLIED MECHANICS FOR BEGINNERS.

slot, and, as the slot is perpendicular to the direction of motion


of the bar, horizontal movements of the crank pin are cancelled
by and its vertical ones alone are copied by the slotted
it

bar. A model of two of these bars, both worked from the


same crank pin, and furnished with scales for observing
positions of the bar corresponding to various crank positions,

Md

Fig. 9J7. Infinite connecting


rod relied to a donkey pump. Fig. 238. Model of infinite connecting rod.

is shown in Fig. 238. The mechanism is usually called the


infinite connecting- rod, motion of the sliding bar is
as the
the same as would be produced by a connecting rod of infinite
length.
Oscillating engine mechanism. In the mechanism of the
oscillating engine, the connecting rod dispensed with, and
is

the piston rod end connected direct to the crank pin, the
is

cylinder being mounted on trunnions, so that it may oscillate


and follow the motion of the crank pin. Fig. 239 shows a
model of the arrangement. Scales are attached for showing
crank angles, piston position and cylinder angles.
TRANSMISSION OF MOTION AND POWER. 187

Parallel motion mechanisms. Link work mechanisms for


producing straight line motion are commonly called parallel
motions. In the Scott-Russell parallel motion AP (Fig. 240) is
a rod, the end A being constrained to travel always in the
straight line AB C is the centre of AP and the parts CB, AC
;

and CP are equal. The link CB is connected to C and to a


fixed centre B.
Since CA, CB and CP are equal, a semicircle drawn with
centre C will pass through A, B and P. So that ABP, being

Fio. 240 -Scott-Russell parallel


motion.

Oscillating engine model. Fig. 241.

the angle in a semicircle, is a right angle. Now this is true


whatever be the position of the mechanism, so that P always
moves in the line BP perpendicular to AB, i.e. P has a straight
line motion.
Practically, it is not always convenient to make A slide in
guides to give it a straight line motion, and with little error to
the motion of P, A may
be connected by a link to a fixed centre
D (Fig. motion is in the arc of a circle of centre
241), so that J's
D. A'a travel is usually small, so that it does not appreciably
depart from the straight line AB. It will be found now that
Ps motion is practically straight over a considerable distance
above and below B.
188 APPLIED MECHANICS FOR BEGINNERS.

This mechanism is often used in steam engine indicators, the


piston rod being attached to a point between A and C by means
of a short link EF, and the
tracing pencil being attached
at P (Fig. 242). It is usual for
this purpose to alter the ratio
of the arms AC, CP, CB. The
position of P may be found,
after settling the lengths of
AD, A C, and CB, if this portion
be drawn to scale and the
instantaneous centre of AC
Fig. 242. Modified Scott-Russell parallel
motion applied to an indicator. found. This will be 7, the
point of intersection of DA
and BC produced. The AC
moving for an instant
link is

about /, so that a point in it which will be moving in a vertical


line will be at the end P of the horizontal radius IP. Thus P
is easily found. Or, CP may be found by calculation, thus, A
and B are usually at the same height, so that AB is horizontal ;

the triangles ABC, ICP are therefore similar, so that


AC:CB=CP:IC.
Now if the angle PAB is small (as it usually is), IC and A C
are very nearly equal to one another, so that

CBxCP=AC 2
.

From this equation, if CB and AC are settled first, CP may be


found, thus fixing the position of P.
The Thomson Indicator of Messrs. Schaffer and Budenberg is

fitted with this parallel motion, and the Crosby Indicator with
one slightly modified from this. The latter instrument is shown
on page 212, where its mechanism can be easily understood. The
Scott-Russell motion has also been used for guiding the top of the
piston rod in small engines known as grasshopper engines, from
their peculiar resemblance to grasshoppers when working.
Watt's parallel motion was used by Watt for guiding the top
of the piston rod in his beam engines. Its simplest form con-
sists of two equal links AB, CD (Fig. 243), swinging about
centres A and D, their ends B and C being connected by a link.
If we first put AB and CD horizontal so that BC is vertical, and
TRANSMISSION OF MOTION AND POWER. 189

then move the links up or down, we see that B deviates to the


one side of the vertical and C deviates an equal amount to the
other side of the vertical.
so that P, the centre of /
BC, will remain in the '

vertical. Therefore, for -DD


some distance on either
side of the mean position
of themechanism, P
will remain in a straight A ^
line, and this point may-
be attached to the top
Fig. 243. Watt's parallel motion simplest
of the piston rod so as to form.
guide it in a straight line.
If A B and CD are unequal (Fig. 244), the deviations of B and
C will be nearly inversely proportional to A B and CD of ;

course it is evident that the deviation of B is made greater by

Fig. 244. Fig. 245. Method of finding P, using


the instantaneous centre /.

decreasing the radius A B. If, therefore, P is to move in a


straight line, BP and PC must be inversely proportional to AB
and CD, that is,
BP:PC=CD.AB.
P may also be found by drawing the mechanism to scale and
finding the instantaneous centre of the link BC when some little

distance from its mean position. In Fig. 245, / is the point


where AB intersects DC, and P will be where a horizontal line
from / cuts BC.
.

190 APPLIED MECHANICS FOR BEGINNERS.

This parallel motion is often extended so as to give a second


point moving in a straight line. Thus, taking the simplest form
of the mechanism (Fig. 246), if A B is
equal to CD, and BP equal to PC, P
will move in a straight vertical line.
If DC be extended to E, making CE
equal to CD, and other two bars, EF
equal to BC, and FB equal to EC, be
added, then F will also move in a
straight vertical line.
Fio. 245.Watt's parallel
motion ordinary form.
;
For EFBC will always be a par-
allelogram in any position of the
mechanism, so that EF is always parallel to PC ; and since
ED = -LCD
and EF=2CP,
F, P and D will always be in one straight line, and FD will
always be double of PD, so that if Pis moving in a vertical line,
F will be also moving in a vertical line at twice P's distance
from D, i.e. the vertical through A.

Fig. 247. Model of Watt's parallel motion applied to a beam engine.

This is the arrangement which is commonly adopted for


beam engines, the piston rod being connected to F and the air
pump rod to P, or in compound beam engines, the high-pressure
piston rod to P and the low-pressure rod to F. A model of this
is shown in Fig. 247
TRANSMISSION OF MOTION AND POWER. 191

Peaucellier straight line motion. There are many other


straight line motions, mostly approximate only, like the above
instances. The Peaucellier links
give an absolutely straight line,
but owing to number of joints it
cannot be used for practical pur-
poses. It consists of a jointed
parallelogram CDEF (Fig. 248),
controlled by and
links EA, DB
Fig. 248. The Peaucellier links.
FB, to fixed and B,
centres A
EA being equal to AB. C moves in a straight line perpendicular
to BA. In Fig. 249 a model of this mechanism is shown.

Pig. 249.Model of Peaucellier straight line motion.

The Eccentric. Valves are generally driven by means of an


eccentric (Fig. 250), which consists of a circular disc of centre
B, having a hole bored
through it at centre C
to receive the shaft. A
strap J, is passed round
} To VALVE
the disc, a working fit,

so that the disc may


rotate inside of the strap.

Fia. 250. Eccentric, used for driving a slide valve.


A rod connected to the
strap drives the valve.
It will be 3een, by inspection of the mechanism, that it is
simply equivalent to a crank of radius BC, the disc forming the
192 APPLIED MECHANICS FOR BEGINNERS.

crank pin being so large in diameter that crank cheeks are


dispensed with, and the shaft hole can be bored in the crank pin.
The motion will be identical to that of the crank and connecting
rod the travel given to the valve will be 2BC.
;

Cams are also used for the purpose of obtaining a reciprocating


motion from a rotary one. In Fig. 251, a shaped cam BC is
mounted on a rotating shaft A. The rim of the cam has pro-
jections of any desired form, and, in the example shown, a lever

Fig. 251. Arrangement for operating a valve by means of a cam BC,


pivoted at E
carries a small roller, B, at its end bearing on the
edge of the cam. The lever will be motionless so long as the
roller bears on a circular portion of the edge of the cam, but
when the projecting parts of the cam reach the roller, the lever
will be operated. Evidently any desired motion may be given to
the lever by suitably forming the edge of the cam. Cams are
much used for operating the valves of gas and oil engines and
can be seen in such cases in their best forms.

EXERCISES ON CHAP. XIII.


1. An
engine driving a line of shafting by belts, has a belt pulley
24" diameter, that on the line shafting being 26" diameter. The
engine runs at 230 revolutions per minute. A counter-shaft is
driven from the line-shaft, the belt running on a pulley 3 feet
diameter on the line-shaft and on one 2 feet diameter on the counter-
shaft. What are the speeds of revolution of the line-shaft and of
the counter-shaft, (a) supposing no slip of belt, (b) supposing 2 per
cent, slip at each belt ?
2. A belt, running at a speed of 2000 feet per minute, has a
difference of 240 lbs. between the pulls on the tight and slack sides.
What horse-power is being transmitted ?
TRANSMISSION OF MOTION AND POWER. 193

3. A
line-shaft transmits 4 H.P. to a countershaft through a
belt running on a pulley 12" diameter on the line-shaft and one of
the same diameter on the counter-shaft. The speed of the line-shaft
is 150 revolutions per minute, and it is found that there is a great
deal of slip. On the pulleys being replaced by others 24" diameter
each, the slip is much reduced. Explain this, giving actual figures.
4. Give sketches and description of the shafts, pulleys, etc. , used
in distributing to the machines the power developed by the engine
in any engineer's workshop you are acquainted with.
5. Two toothed wheels, mounted on parallel shafts, are to be in
gear with one another. Their speeds of rotation are to be in the
ratio of 2 1. If the distance between the axes of the shafts is 12",
:

and the pitch of the teeth is to be as nearly 1" as possible, find the
numbers of teeth on each wheel.
6. Give sketches and description of the train of wheels connecting
the hour axle with the minute axle in a clock. Give suitable
numbers to the teeth.
7. The counter-shaft driving a turning lathe runs at a speed of
180 revolutions per minute. The largest step on the speed cone is
10" diameter and the smallest 4" diameter. Each pair of wheels in
the back gear have numbers of teeth 15 and 45 respectively. If the
belt is running on the smallest step of the countershaft cone, and
the back gear is "in," what will be the surface speed, in feet per
minute, of a piece of work 7" diameter ?
8. An epicyclic train consists of four wheels. A is fixed and
has 40 teeth ; B gears with A and has 20 teeth C gears with B
;

and has 30 teeth ; D


gears with C and has 15 teeth. B, C and D
are carried on an arm revolving on the axis of A. Find the revolu-
tions of each wheel if the arm is rotated once clockwise.
9. In Question 7, supposing the pitch of the leading screw to be
inch, and that it takes its motion from a wheel of 20 teeth on the
lathe spindle, give suitable numbers for the wheels driving the
leading screw for a feed of xV'*
10. The crank of an engine is 1 foot long, and the connecting rod
3^ feet. Revolutions per minute, 100. Find (a) velocity of crank
pin centre (supposed uniform), (b) average velocity of piston,
(c) actual velocity of piston at 0, 30, 60, 90, 120, 150, and 180
from inner dead point. Plot these velocities on a time base line.
11. Draw an approximate acceleration diagram for the piston in
Question 10.

In the parallel motion shown in Fig. 242, AD=\$', AC=\\",


12.
BC= 1". A and B are on the same horizontal line. Find, both by
calculation and construction, the length of CP, and prove your
work by drawing the motion in several positions so as to show the
path of P.
13. How would you determine the " pitch circles " and the proper
"pitch of the teeth" for a pair of spur wheels? What would be
A.M.B. N
)

194 APPLIED MECHANICS FOR BEGINNERS.

the diameter of the pitch circle of a spur wheel having 80 teeth of


| inch pitch? Three spur wheels A, B, Care on parallel axes, and
are in gear. A has 10 teeth, B has 35 teeth, and G has 55 teeth.
How many revolutions upon its axis will be made by A for every
4 revolutions of ? Why is B
called an idle wheel, and what is its
use? (1896.)
14. Describe the construction of the ratchet-brace employed in
drilling holes by hand. Explain, with the aid of a sketch, how the
drill is revolved always in the same direction, and also how the feed
is put on, whilst the handle is worked alternately backwards and
forwards. (1896.)
15. Sketch and describe an arrangement of belts and pulleys
whereby a reversing motion may be obtained from a single wide
pulley running on the main shaft at a constant speed in one direction
only. (1898.)
16. A rope transmits 20 horse-power to a rope pulley of 8 feet
diameter ; draw a section of the rope in its groove. If the pulley
makes 100 revolutions per minute, what is the speed of the rope
in feet per minute ? What is the difference of the tensile forces in
the rope on the two sides of the pulley ? As it is the difference
between the tensile forces in a belt or rope that is important for
power, why is it necessary to have any pull on the slack side ? (1898.)
17. What are cone or speed pulleys ? Describe the use of such
pulleys in any machine with which you are acquainted.
The spindle of a wood-turning lathe can, by moving the belt on
its cone pulleys, be driven at the rate of 400 revolutions per min.
when at its greatest and at 100 revolutions per min. when running
at its lowest speed. If the revolutions of the driving shaft are kept
constant throughout, and the largest diameter of the speed cones is
20", what must be the diameters of the smallest steps on the pulleys,
the speed pulleys on the two shafts being of the same size ? Sketch
the pulleys in position. (1899.)
18. Describe and sketch the arrangement of the mechanism by
which the saddle of a lathe is toaversed by hand along the bed. If
the slide rest of a screw-cutting lathe when in gear with the leading
screw moves along the bed for a distance of 14", while the leading
screw makes 56 revolutions, what must be the pitch of the thread
on the leading screw ? (1899.
19. Sketch and describe the arrangement of mechanism by which
the tool of a planing machine is traversed across the slide of the
machine at each stroke of the table. (1900.)
CHAPTER XIV.

ACTUAL MECHANICAL ADVANTAGE AND EFFICIENCY.


EXPERIMENTS ON SIMPLE MACHINES.
Simple pulley block. The simplest means we have for raising
loads consists of a pulley suspended from an overhead beam,
with a rope passing over it (Fig. 252). A
load secured to one end of the rope may be
raised by pulling on the other end. If there
were no losses by friction, stiffness of rope,
etc., the greatest load a man could support

would be equal to his own weight, and to do


this he would have to lift himself off the floor
by pulling on the rope. Frictional losses
always prevent so great a load from being
raised. By attaching two scale pans to the
rope ends, A and B, and placing loads in them,
the force P required to steadily raise a load
W may be found. If equal loads are placed in I

the pans, no movement will result, but


one if
ii

load is increased until, by slightly pulling the


rope so as to start motion it is found to
be sufficient to maintain steadily the move-
ment so produced, the weights in the pans
will give the value of P required for this Fio. -Simple
pulley block.
load W. In this arrangement the velocity
ratio is clearly 1 the mechanical advantage for any load
;

W W
is equal to p, which will always be less than 1, as IT is

always less than P. The imaginary load F, which, if placed in


196 APPLIED MECHANICS FOR BEGINNERS.

the same pan as would be equivalent to the fractional


IF,

resistances of themachine (p. 150), will be equal to P W,


for if there were no losses, a force P would raise a load P.
The efficiency of the machine (p. 125) will be found by
considering W
to be raised one foot P will then descend
;

one foot.
Useful work done on W= W x 1.

Energy supplied to effect this result = Px 1.


:. Efficiency
W x 100
= -p per cent.


Plan of procedure. In experimenting with any machine,
about 10 experiments should be made with loads increasing by
equal steps up to the maximum the machine can safely carry.
If scale pans or hooks are used for attaching P and IF, the
weights of these should be included in the recorded values of
P and W. In recording the results, a sketch showing the
mechanism clearly should first be inserted and a description of
the machine. The calculation for the velocity ratio of the
machine should then be given and its result stated. If suitable,
the velocity ratio should also be determined by direct measure-
ment at the places where W and P are applied. Weigh the
scale pans or hooks and state their weights separately. Oil the
parts of the machine requiring lubrication, make sure that
everything is running nicely and then make the experiments.
Record the results in the form of a table. Curves should then
be plotted on squared paper showing the relations of P to W, of
F to IT, and of the efficiency of the machine to W. From the
first two curves equations showing the connection of P and W
and of F
and Wmay be found. The third curve will show the
value of the efficiency of the machine for any load as a rule it ;

will be observed that the efficiency rises rapidly when the loads
are small, and tends to become constant as the maximum load is

approached.
As an example of the method, the following experiment is

worked out in full.


MECHANICAL ADVANTAGE AND EFFICIENCY. 197

Experiment on Pulley Blocks.

Pulleys usedA single pulley at A (Fig. 253) and another at


:

C, both hung from an overhead beam a single movable pulley


;

at B, from which W
is suspended.

These pulleys were of galvanised iron, small size, such as are


used for ordinary household purposes. thick cord attached A
to the pulley A at D passes down through
pulley B up over pulley A, then over
y

pulley C and a scale pan is attached to


its end for applying P. Another scale
pan attached to pulley B serves for
applying W. The pulley C would not
be usually employed in practice, its
present use is to keep the scale pan for
applying P clear of the other scale pan
and cords.
Velocity Ratio.If both cords and E F
were raised one foot each, would also W
be raised one foot. In the actual machine,
the cord F does not move until after it
passes B, so that if E alone is raised one
Fig. 253. Arrangement
foot, W ascend half a foot.
will The experiment.
of pulleys used in the

pulleys at A and C merely change the


direction of the rope E, so that raising E
one foot causes P to
descend one foot. The velocity ratio is therefore equal to 2.
This was confirmed by actual measurement of the distance
moved by when P W
was raised one foot.
We may also obtain the velocity ratio by considering the
machine as being free from frictional resistances. In this case,
as Wis suspended by two cords E
and F, the pull in each will
be W consequently P will be equal to \
; and the mechanicalW
advantage, neglecting frictional losses, would be equal to 2. In
this case (p. 128) the values of the mechanical advantage and
velocity ratio are equal, consequently the velocity ratio is 2.

Two experiments were made


sets of I., with the machine
;

unlubricated, after a period of some months' disuse II., after ;

oiling.
198 APPLIED MECHANICS FOR BEGINNERS.

Weight of scale pan for IF=0-68 lb.


P=0-68 1b.

I. Before oiling.

V*a 41 a B
c
Rais - o
Weigl
Applied,
Weigl
C ON >, o
^S II
fa a
I *
Pan, lbs. Pan, ii
W P'
Hi
Mechanical

Plbs.
Load Advantage,

IT

including
of
Force
including
of
f g 11X
Actual

uia.
s

0-68 114 2-28 1-6 0-6 29-8

1-68 1-88 3-76 2-08 0-89 44-7

2-68 2-68 5-36 268 10 50-0

4-68 4-42 8-84 416 106 53-0

6-68 6 02 12 04 5-36 Ml 55-5

8-68 7-42 14-84 6-76 117 58-5

10-68 912 18-24 7 56 117 58'5

12-68 10-72 21-44 8-76 M8 59


14-68 1222 24 44 9-76 1-2 600
16-68 13-72 27 44 10-76 1-21 60-7

II. After oiling.

0-68 092 1-84 116 0-74 37


1-68 1-52 3-04 1-36 113 56-3

2-68 2-22 4-44 1-76 1-21 60-4


4-68 3 42 6-84 216 1-37 68-5
6-68 4-72 9-44 2-76 1-42 70-9

8-68 5 92 11-84 316 1-47 73-3

10-68 7*22 14-44 3 76 1-48 73-9


12-68 8-52 17 04 4-315 1-49 74 4
14-68 9-72 19-44 4-76 1-51 75-5
16-68 11-2 22-4 5-72 1-49 74-5
MECHANICAL ADVANTAGE AND EFFICIENCY. 199

These results show an increased efficiency, due to oiling, of


about 15 per cent, with the highest loads.
PLUS*
A
a /
'

to

Ss
f A

r^
8 /

4
'/'

S
2 *

4 6 S JO 12 H a onus.
PULL* LOAD
Fig. 254. Plotted values of P and W.

On plotting the values of P and W (Fig. 254), also of F and


W (Fig. 255), the points are found to lie nearly on a straight
line. This line is found by
.'

stretching a thread on the /


paper and shifting it about /
/
until the plotted points are /
/
well divided on either side
1

of it. A mark is then made >


under each end of the thread > #'*
and the line drawn through it. / f
Equations for the ma-
/
/
*
chine. In all cases where a *>
/
straight line is given when A
i
the results are plotted, the
>
relation between the results
can be expressed by an equa- > 2 1 1 i

nicrton.
tion like
Fio. 255. Plotted values of F and IF.
P=aW+b, (1)

where P and W are the observed quantities and a and b are


constants.
If the constants are found and inserted in the equation, then
200 APPLIED MECHANICS FOR BEGINNERS.

this equation may be used for determining the value of P for


any value of W. These constants are found from the straight
line on the diagram thus Taking the diagram showing P and
:

W before oiling, we see from it that


P=22 lbs. when W=2 lbs., and
P=133 lbs. when W=16 lbs.
Fill in these values in giving two simultaneous
equation (1),
equations from which a and 6 will be found. Thus :

= (ax 2) + 6
2-2 (2)
13'3 = (axl6) + 6 (3)
Solving these, we find a 0'79,
6=0-62;
so that P= 0-79 TT+ 0-62 (4)
gives the required equation showing the connection between
P and W, with the machine unlubricated.
Taking now the diagram for P and W after oiling, we see
that P= 1-7 lbs. when W=2 lbs, and
P= 10-7 lbs. when W = 16 lbs.
These values inserted in (I) give l'7 = (ax 2) + 6 (5)
lC-7 = (axl6) + 6 (6)
Solving these gives a =0*64,
6 = 0-41 ;

so that P=0-64TT+0'41 (7)


gives the relation of P and W after oiling the machine.
Taking now the diagrams showing the relation of F and IP,

we see that

Before Oiling. After Oiling.

F= 1-3 lbs. when W= 0. F= 0-95 lb. when W= 0.


F= 10-62 lbs. when JF=161bs. F=5'24 lbs. when PT=16 lbs.
F=a'W+V, (1) F=a'W+V, .. ..(1)

giving 1-3 =0 + 6', (2) giving 0-95 = + 6', (2)

10-62 = (a'xl6) + 6', (3) 5-24 = (a'xl6) + 6', (3)

and from (2) and (3) and from (2) and (3)

a' = 0-58, a' = 0-27,


6'=13; 6' = 0-95;
/. F=0'58W+13. (4) .-. F= 0-27 JF+0-95.(4)
f

MECHANICAL ADVANTAGE AND EFFICIENCY. 201

Tabulating these equations :

Before Oiling. After Oiling.

/>=079H'+0'62. P=0-64W+0-41.
F=0 58W+l-3. ^=0-27^+0-95.

The unit used for P, F and W is the pound weight. These


equations give all the required information about the machine
under the two given conditions.
It should be noted that as the efficiency curve (Fig. 256) is
not a straight line, an equation like those given above cannot be

so
_
AFT& 0JUK___

iW
StfWOii

so
/
40
1
.V
f
to

K>

2468/0/2/4/6 EFFICIENCY* LOAD


emus.

Pio. 256. Curves of efficiency and load.

applied. Its equation is not so simple, and may be neglected


meanwhile by the student.
The above should be taken as showing the method which
must be employed in experimenting upon and working out the
results for all the simple machines described here. It may be
observed, in the example taken, that the pan and weights placed
in it are not the only loads raised. The bottom pulley B is also
raised, and part of the rope changes its height also. These loads
202 APPLIED MECHANICS FOR BEGINNERS.

need not be taken into account, as what we require to know


practicallyis not what parts of the machine itself can be raised

by a given force, but what useful load may be raised. Special


cases, however, may sometimes arise in which the weights of
parts belonging to the machine must be considered.
Another arrangement of pulley blocks. Keeping the
arrangement of ropes the same as in Fig. 253, a higher velocity
ratio may be obtained by increasing the number
of sheaves, or wheels, in each block. In Fig.
257 there are three sheaves in the upper block
'"> and two in the lower. W
is now supported by

5 ropes between the upper and lower block,


consequently the velocity ratio is 5. Usually
the sheaves belonging to each block are mounted
side by side on the same spindle.
Let W
lbs. = load actually raised.
P lbs. = pulling force.
Then, if there were no frictional losses, and if
the lower block and the rope had no weight, the
useful load raised would be
W^bxP.
W Therefore losses in machine = 5P
before. The mechanical advantage
W=F,
will
as
be
W
-p, and the efficiency
W
-^x 100 per cent. It will

be found, on experiments being made on a set


Fig. 257. Pulley
of actual blocks like this, that the efficiency is
blocks.
not so great as with those first given, even when
well oiled. The stiffness of the rope passing so many times
around the sheaves accounts for a large loss.
Weston's blocks are much used for hoisting loads. The top
block (Fig. 258) contains two sheaves of different diameters, cast
together so as to form one piece, the rims of each being formed
so as to receive the links of the hoisting chain and to prevent it
slipping. The bottom block contains one sheave only. The
chain is endless and passes first round the larger top sheave,
then round the bottom sheave, then round the top smaller sheave,
the two ends being connected and allowed to hang loose. In
raising a load, the loose chain passing from the larger top
MECHANICAL ADVANTAGE AND EFFICIENCY. 203

sheave is pulled ; in lowering, the other is


pulled.
The arrangement may be studied either
by use of the principle of moments, or by
calculating the velocity ratio direct.
1st, Let i? = rad. of large sheave = CF
r= small =CB.
Since W is sustained by the two chains
A and B (Fig. 259) the pull in each (friction
being neglected) will be ^W. If now we
consider DF
as a lever with fulcrum at C,
we see there are three forces acting on it,
at 2), E
and F, tending to turn it about C.
For balance,
clockwisemoments = anticlockwise moments,
(PxCF) + ($WxCE)=$WxDC,
or, (Px R) + (%Wxr) = lWx R
PxR=$WxR)-($Wxr)
=\w{R-r)

w 2ft

which gives the mechani-


cal advantage with no
friction. We see from
this equation that the
smaller the difference be-
tween R and r the greater
will be this mechanical
advantage, and conse-
quently the velocity ratio
which is equal to this
numerically.
2nd. The same result
may be obtained by con-
sidering the velocity ratio.
Thus, let the uppersheaves
turn round once clock-
Fig.259. Diagram of
Weston's blocks. wise to do this P must
;
Fig. 258. Weston's
blocks

204 APPLIED MECHANICS FOR BEGINNERS.

travel through a distance equal to the circumference of the


larger sheave, i.e. 2ttR. A will be raised a distance equal to
the circumference of the larger sheave 2ttR, and B will be
lowered a distance %-rrr. So that if B were kept fixed, A would
be raised a distance {2ttR %irr\ and W
would be raised one
half of this (ttR - ttt).

So that, distance travelled by P : distance travelled by


W=2TrR:(7rR-irr);

J ratio = >> - x
velocity
tt(R r)
_ 2R
~ R-r"
which is the same result as that found by the previous method.
Eunning down of the load. Notice the question of the
balance of the lever DF above, when both chains are hanging
loose.
Clockwise moments =\ W xr.
Anticlockwise moments = \ Wx R.
There is therefore an anticlockwise equal to \ W{R r)
moment
tending to turn the sheaves about C and so cause the load to W
run down. To prevent this, we have to depend, in this tackle,
on the friction of the spindles of the sheaves in the upper and
lower blocks, which must supply a clockwise moment equal to
\ W(R r). If, therefore, we want to raise a load easily by this

tackle, we must grease the bearings of the spindles, but not too
freely, as we may thereby reduce the friction so much as to
cause the load to run down, that is, the blocks cease to be self-
sustaining.
It can be shown that in any machine in which the removal of
the pulling force does nothing to alter the magnitude of the
frictional resistances, the suspended load will not run down if
the efficiency of the machine is less than 50 per cent. If,

however, the removal of the force causes alterations in the


frictional resistances, no general statement true for all machines
can be made, each case must be investigated separately.
Running down of the load when P is removed, may be
prevented by fitting a pawl and ratchet wheel to the machine
(Fig. 260). When the pawl engages the teeth, motion can only
take place in the direction necessary to raise the load. When
MECHANICAL ADVANTAGE AND EFFICIENCY. 205

lowering the load, the pawl is raised by hand, thereby


permitting the required motion to be effected.
The wheel and differential axle (Fig. 261) consists of a drum
BC, divided into two portions having different diameters, and

Front Elevation *0 Elevation


Fig. 260. Pawl and ratchet Fig. 261. Wheel and differential
wheel. axle.

mounted on the same spindle as a wheel A. A rope is attached


to B, and, after a few turns round B, is led down through a pulley
D, then up and turned round C in the opposite direction to the
winding on B, its end being made fast to C. A cord made fast
to A, passes two or three times round A and carries a load P for
working the machine. The load to be raised, W, is carried by
the pulley D.
Suppose the drum to rotate once. P will be lowered a
distance equal to the circumference of the circle in which the
cord sustainingP is wrapped. A point
on the cord E will be lowered, and one
on the cord F
raised, the amounts in
each case being equal to the circum
ference at the drum of the cord circle.
It will be noticed that for all three
cords, the circumference must be meas-
ured, not of the drum, but of the circle
Fig. 262.
at the cord centre ; for the inside of
the cord, in contact with the drum (Fig. 262), will have a shorter
206 APPLIED MECHANICS FOR BEGINNERS.

circumference than that of the outside part of the cord, and the
actual distance moved by the cord will be the mean of these two,
that is, the circumference of the cord centre.
To find the velocity ratio, let RA RB , , Re be the radii of A, B
and C, in each case measured to the centre of the cord. Let the
drum rotate once, then
Distance moved down by P=2ttRa
down by E=2irR B
up by F=2ttR c .

Consequently, as in the Weston's Blocks, W will be raised a


distance equal to h{2irRc - 2ttRb )\

Velocity ratio =
:.
J ^^^7)
2RA
Re Rb
The actual mechanical advantage will be found by experiment.
A From the results the effect of friction
and the efficiency will be calculated
as already explained.
Helical blocks are often used on
account of their self-sustaining quali-
ties. The upper block consists of a
chain wheel A (Fig. 263) carrying an
endless chain for working the blocks ;

a worm B is fixed to the same spindle


as A, and is driven by A this worm
;

drives a worm wheel C and also a chain


wheel Z), which is keyed on the same
spindle as C. A
chain is secured to
the upper block at E, passes down
through F, then up over I), its end
hangmg loose at G. The rim of D is
shaped to take the links of the chain
without any chance of slipping occur-
ring, and the chain is guided on and
. Diagram of helical off D. On turning A by pulling the
blocks.
endless chain, the wheel D either raises
or lowers the chain H
and so raises or lowers the load W.
MECHANICAL ADVANTAGE AND EFFICIENCY. 207

To calculate the velocity ratio : Let W ascend a distance


equal to the length, of the number of links D can take round its
complete circumference. Call this length h. Then, as the chain
is fixed at E, the wheel D
must rotate twice. Now, as one
revolution of the worm passes on one tooth of the worm wheel,
therefore for two revolutions of D to occur, the revolutions of the
worm must be equal to twice the number of teeth on the worm
wheel. Call the number of teeth N. Then the worm, and con-
sequently A, rotate 2iV times in raising W
through a height h.
Let the length of the number of chain links A can take round
its complete circumference be called L then, if A rotates once,
;

P will be lowered a distance Z, and for 2N revolutions of A, a


distance 1NL. The velocity ratio is therefore equal to 2JVL
h '

The crab is often used along with arrangements of pulley


blocks for raising weights. The rope from the pulleys is wound
round a barrel, having a
toothed wheel secured to
its spindle. A pinion, se-

cured to another spindle


parallel to the barrel, gears
with the toothed wheel, so
that if this second spindle

is turned by means of
handles, a considerable
velocity ratio is obtained.
This arrangement is said
to be single geared. In
double geared crabs, an
additional toothed wheel
and pinion are introduced
on another spindle, so that
a much greater velocity
obtained. The Fig. 264. Crab, arranged for performing
ratio is
experiments.
double geared crab is
usually so arranged that it can be rapidly converted to single
geared light loads can then be raised more quickly than by
;

use of the double gear. A band brake is fitted to the drum


spindle for use while lowering the load, and a pawl and ratchet
208 APPLIED MECHANICS FOR BEGINNERS.

wheel on the same spindle prevent the load running down if the
handles are released.
Fig. 264 shows such a double geared crab arranged so that
experiments may be performed. The handles have been taken
off and a wheel, having a rim
grooved to receive a cord, substi-
tuted. The pull required to work
the machine is supplied by weights
D = C placed in a scale pan at the end of
i 1 this cord, the cord being led over
I 1

a pulley secured overhead, so as


to obtain a considerable vertical
llll *
a _-z:
SHI
travel for the scale pan. Fig. 265
lift

I shows a diagram of the mechanism


from which the velocity ratio may
be calculated.
Let d x = the diameter to the
centre of the rope on the barrel A ;

Fig. 265. Diagram of the mech d2 = the diameter to the centre of


anism of the crab.
the cord on the wheel F.
Let B, C, D, E represent the numbers of teeth on the wheels
as shown.

Then, if A revolves once, F will revolve ^-L


x Jb
tmies an ^> W
will be raised a distance equal to irdi. In one revolution of F,
P will be lowered a distance equal to ird* ;therefore, for one
revolution of the barrel A, P will be lowered a distance
BxD
CxF x 7rd Consequently the velocity ratio will be
- 2.

BxD XTrd>
F= CxF
irdx

= BxJ)xd2
~CxEx d{
Screw jacks are used for heavy loads requiring a small lift

only. A hollow case A (Fig. 266) has a hole at its top screwed
to receive a strong square threaded screw B. The load is

applied at the top of this screw, on C, which is a piece free to


rotate on the top of B. B is turned by a tommy-bar inserted
MECHANICAL ADVANTAGE AND EFFICIENCY. 209

into holes in the screw head as shown, and as C is free to rotate


on B, the load is not turned by the rotation of the screw.
To obtain the velocity ratio :

Let R = radius at which P is applied, w


inches.
p= pitch of screw, inches.

Then, in one revolution of the


screw, P
moves a distance tangen-
tially equal to 2ttR, and moves a W
distance equal to p. Therefore the
velocity ratio is
2ttR
P
By substituting a wheel with a
grooved rim for the tommy bar, and
attaching a cord to it, led over a
pulley, a load may be hung on equiva-
lent to P, and experiments may be
Screw jack.
made to find the mechanical advantage,
effect of friction and efficiency, in the same manner as for the
other machines.

EXERCISES ON CHAP. XIV.


1. In a Weston's Block the large pulley is 6" diameter and the
small one 5" diameter, measured in each case to the chain centre.
Neglect friction and find what pull will be required to raise ton.
2. A single geared crab has handles 15" long and barrel 6"
diameter to the rope centre. The pinion has 20 teeth and the spur
wheel 60 teeth. Give an outline sketch and calculate the velocity
ratio.
A screw, 1" pitch, is used to raise a load of 5 tons. The screw
3.
isturned by means of a toothed wheel 15" effective radius. Calculate
the pressure required on the teeth tangential to the pitch circle,
supposing 50 per cent, to be lost in friction.
4. In a wheel and differential axle, the wheel is 24" diameter and
the drum has diameters of 7" and 6" respectively. Calculate the
velocity ratio.
5. A
worm and wormwheel are used for applying the twist to a
piece of material under torsion test. The worm is turned by a
handwheel, and the test piece is connected to the shaft on which the
wormwheel is mounted. If the wormwheel has 90 teeth, how many
A.M.B. o
210 APPLIED MECHANICS FOR BEGINNERS.

degrees of twist will be given to the test piece by turning the hand-
wheel through 235 revolutions ?
6. A lifting tackle is formed of two blocks, each weighing 15 lbs. ;

the lower block is a single movable pulley, and the upper or fixed
block has two sheaves. The cords are vertical and the fast end is
attached to the movable block. Sketch the arrangement and deter-
mine what pull on the cord will support 200 lbs. hung from the
movable block, and also what will then be the pressure on the point
of support of the upper block. (1896.)
7. Describe either a screw jack (pitch of screw h", handle 19" long)
or a simple winch for lifting weights up to 1 ton by one man. What
is the mechanical advantage neglecting friction? Describe what
sort of trial you would make to find its real mechanical advantage
under various loads, and what sort of result would you expect to
find? (1897.)
8. Describe any machine, workable by hand, for lifting weights.
Give the rule for its velocity ratio. When is its velocity ratio the
same as its mechanical advantage ? Describe carefully how you
would make tests to determine its real mechanical advantage under
various loads. (1898.)
9. A machineconcealed from sight except that there are two
is
vertical ropes when one of these is pulled downwards the other
;

rises. How would you find the efficiency of this lifting machine ?
What do we mean by the velocity ratio, and by the mechanical
advantage? (1900.)
CHAPTER XV.

INDICATED AND BRAKE HORSE-POWER. ABSORPTION


AND TRANSMISSION DYNAMOMETERS. FLY-
WHEELS. STEADINESS OF MACHINES. MOMENTUM.
IMPACT. FORCE OF BLOW. CENTIFRUGAL PUMP.

The indicator. The


student has now, from the preceding
chapters, a fair idea of how energy
supplied and energy delivered
are measured in the case of certain machines, and what sort of
results may be expected. The methods by which the same thing
is done in cases of machines running continuously, and where
considerable quantities of energy are being dealt with, should
now be considered.
In ordinary engines with pistons reciprocating in cylinders,
the energy delivered to the engineis measured from the diagram

of work done on the piston. These diagrams are drawn by


means of an instrument called an indicator. The essential parts
of an indicator consist of a small cylinder which can be con-
nected with the cylinder of the engine by means of a cock. A
piston moving in this small cylinder is controlled by a spring.

The elastic properties of this spring cause the piston to take up


a definite position in its cylinder depending on the pressure
exerted on it. The movement of the piston is communicated by
linkwork to a pencil, the linkwork being so arranged that a
straight line motion is given to the pencil. The pencil takes up
a position corresponding with the pressure in the cylinder. A
paper, stretched usually over a drum, is caused to move trans-
versely under the pencil, being driven to and fro by being
connected to some part of the engine so as to give a faithful
copy of the motion of the piston of the engine to a reduced
212 APPLIED MECHANICS FOR BEGINNERS.

scale. When the pencil is pressed on the paper, the indicator


being connected to one end of the engine cylinder, it will now
trace a curve showing the pressure on the piston at any part of

Gs|l|e
Pig. 267. Crosby Gas Engine Indicator.

the double stroke of the engine. From this curve the average
pressure on the piston may be found. A
datum line, showing
atmospheric pressure, is traced on the paper by putting the
indicator cylinder in communication with the atmosphere. A
small side hole in the communication cock enables this to be
done. Fig. 267 shows the Crosby
Gas Engine Indicator, and ex-
plains the construction of the
instrument clearly.
Calculation of I.H.P. from
"L the indicator diagram. This
Fig. 268. -Indicator diagram for end diagram (Fig. 268) represents
A of the cylinder in Fig. 270. ,
._ . ,. . ,,
what the indicator might draw if
connected to a steam engine cylinder. AL is the datum atmos-
pheric line, ah shows the admission of steam at high pressure
to the engine cylinder. At b the steam is cut off and expands
1

INDICATED AND BRAKE HORSE-POWER. 213

during the remainder of the stroke, falling in pressure as it does


so. df shows the back pressure on the piston during the return
stroke. The average breadth of the diagram is usually found
by dividing its length into ten equal parts and measuring its
breadths at the centres of these parts. The sum of these
breadths divided by 10 will give the average breadth. Suppose
this gives 1'25" as the average breadth. The strength of the
indicator spring being known, the height on the diagram
corresponding to a given pressure is known. Thus, supposing
in the given case a spring had been used of such strength that
30 lbs. per square inch pressure is represented by a height of
1" on the paper, then the average pressure on the piston will be

30 x 125 = 375 lbs. per square inch.


The average pressure on the other side of the engine piston
will be found, in the same way, from the diagram drawn by the
indicator when connected to the other end of the cylinder (Fig.
269).

A B

I _i
:<

Fig. 269. Indicator diagram for Fig. 270. Diagram of steam engine
endB of the cylinder in Fig. 270. cylinder.

Having these average pressures, the work done per stroke


may be easily found. In Fig. 270,
Let pA = average pressure for end A of cylinder, lbs. per sq. inch.
PB ~ B n n n
Total pressure on piston on side A =pA x area of piston

=pA x- = PA lbs.

Total pressure on piston on side B=p B x (area of piston - area


of piston rod)

-***{-rr)
=PBxi(D'i -cP)=PB \hs.
214 APPLIED MECHANICS FOR BEGINNERS.

Let N= revolutions of engine per minute.


L length of stroke in feet.
Work per stroke for side A = PA xL ft. -lbs.
minute = P A xLx A ft.-lbs. r

and Horse-power = ZTP^ = -4=r-s^


33,000
for side A.

In same way HP B PB xLxN for


and side B.
33,000
The total H.P. called the Indicated Horse-Power, will be the
sum of these,
A xLx N P b xLxN
LH.P.= 33,000""
33,000
LxN(P + /Js) '

33,000 ^
Brake horse-power. The quantity LxN(PA + PB) measures
the whole energy in foot-pounds given to the piston in one
minute. It is necessary to
know not only this, but also
the energy which the engine
can deliver per minute, as
from these two quantities, the
energy lost in overcoming
frictional resistances in the
engine, and the mechanical
efficiency of the engine can
be calculated. If the engine
is not too large, the energy

delivered can be conveniently


obtained by putting a brake
on the flywheel. The sim-
plest method is to pass a
Fig. 271. Arrangement of rope brake.
double rope round the wheel,
held in position by means of loosely fitting wood blocks secured
to the rope. One end of the rope is attached to a spring
balance, the other end carries a load W. The direction of
rotation of the wheel being as shown in Fig. 271, it will be
noticed that P is helping to turn the wheel and W is opposing
its rotation. The friction of the ropes on the rim communicates
these forces to the wheel.
INDICATED AND BRAKE HORSE-POWER. 215

Let R= radius, in feet, to the centre of the rope, then

WR-PR = R{W-P)
will be the net opposing moment in pound-feet, if and P are W
in pounds.
If now the wheel rotates once, the work done against this
moment will be
2ir.R(W-P)it.-\bs.,
and for N
revolutions per minute,
Work per minute = 2tt . R . N( W- P) ft. lbs.,

and HR= 33,000

This is called the Brake Horse-Power of the engine, written B.H.P.


Mechanical efficiency. If the B.H.P. and I.H.P. are known,
the mechanical efficiency of the engine may be calculated. For
the I.H.P. may be taken as a measure of the energy delivered
to the piston per minute and the B.H.P. as a measure of the
energy produced by the engine in the same time. So that
T> XT p
Mechanical efficiency = '
' '
x 100, per cent.

Prony brake. Brakes of the kind described above are called


Absorption Dynamometers. It will be noticed, in the one de-
scribed, that we depend on the pulls given to the ends of the rope

Fio. 272. Prony brake.

for the production of frictional resistance to motion, and that


these pulls bear a definite ratio to one another in any given
case. If a brake like this is used for absorbing large powers,
these pulls may have to be very great and make the use of a
large weight necessary on the lower end of the rope. To obviate
this a Prony brake may be used. The principle of this brake is

shown in Fig. 272. A is the wheel driven by the source of


216 APPLIED MECHANICS FOR BEGINNERS.

power, and has two brake blocks B and C fitted to it. These
blocks can be made to grip as tightly as may be
required on the
wheel A by tightening two thumbscrews on the bolts holding
the blocks together. A
lever B, balanced by a counterpoise at
J, carries a load P
lbs. in a scale pan at its end, and this load

gives a measure of the power. The lever has a limited B


freedom of movement between two fixed stops F, F. Let R be
the radius of the wheel A and B the horizontal distance between
}

the centre of A and P is applied, both expressed


the place where
in feet.Let i^=the total frictional forces generated all round
the rim of the wheel. Then, taking moments about J, and
remembering that the brake blocks and lever are balanced,
FxR=PxB,
PxB lbs.
I!

Work done against F in one revolution of A Fx2rfft.-lbs.,


so that if N= revolutions of A per minute,

B.H.P.=
Fx 2ttR x N
33,000
Px2ttBxN
33,000 '


Heating of the brake. In absorption dynamometers the
energy produced by the engine is absorbed by the frictional
resistances of the brake,
and is transformed into
heat. It is therefore neces-
sary to keep the flywheel
cool by lubrication with
soapy water, this being
assisted by the air draught
produced by the rotation
of the wheel. Sometimes
Froht Elevation Sectional Elevatioh flywheels have their rims
Fig. 273. Arrangement of water-cooled made of a channel section
flywheel.
so as to receive a stream
of water, which being whirled round by the wheel, retains its
position in the rim in the same way as a whirled stone at the
end of a string keeps its circular path. The water is kept
continually flowing into the rim and is drained away by a sharp-
'

INDICATED AND BRAKE HORSE-POWER. 217

edged scoop on the other side, and therefore keeps the rim cool.
The arrangement is shown in Fig. 273.
Transmission Dynamometers are used for measuring the
power delivered to a machine. They receive energy from a
moving belt or otherwise, measure it,
and deliver it to the machine with as
little loss as possible. They usually
work by measuring the torque trans-
mitted. Let two shafts, both in the rjy-
same straight line, have parallel discs
mounted as shown in Fig. 274. Let the
discs be connected by springs, one end
secured to a pin on one disc, the other
end to a pin on the other. If A is Fig. 274 -Spring transmission
driven in the direction shown, the
torque will be communicated to B by means of the springs,
which will therefore extend and cause A and B to take a new
relative position.This relative movement of A and B will
depend on the magnitude of the torque, which may therefore
be measured by observing the movement. Any convenient
arrangement of linkwork to cause, by utilising the relative
motion of A and Z?, a pointer to move over a scale will
answer.
Let T= observed torque, lb.- feet.

Work of one revolution = 2,irT ft. -lbs.,


<

ZttT.N
and H.P. transmitted =
33,000
Dynamometers measuring the difference in belt pulls.
In other forms of transmission dynamometers the difference in

Fig. 275. Alteneck or Siemens dynamometer.

the pulls of the belt driving the machine is measured. The


belt is led over guide pulleys at A and B (Fig. 275) mounted in
T

218 APPLIED MECHANICS FOR BEGINNERS.

a frame, the angles of deflection of the belt being made equal


by the pulleys C. If Tv 7\ are the greater pulls and T2 T2
,

the lesser, Tx
will give a resultant downward force, R lt
greater
than the resultant upward force, R2 due to T2 T2 A force
, , .

PR l
R 2 will therefore be required to hold the frame in
position, and if this force is measured, Tx and T2 may be found
from it and the angles of deflection of the belt.

Let R = radius of machine pulley in feet, then


Work of one revolution = ( T - T2 ) 2ttR
x
ft. -lbs

Fig. 276 shows the Froude, or Thorneycroft, transmission


dynamometer, in which also the difference between the tensions
of the belt driving the machine is measured for the re-
quired information. A is the driving pulley
and B the driven one. Two guide pulleys run
loose on spindles mounted at C and D on a
j_-shaped frame, this frame being pivoted at F.
The belt is led from A, round B, B, C, in the
manner shown. Neglecting frictional losses due
to the use of the guide pulleys, the tensions of
the belt will not be altered by passing round
them, so we have two forces Tx and Tx pulling
on the left-hand portion of the lever and other
two, T2 T2 acting on the right-hand portion.
, ,

A force P will be required to balance the


moments Taking the resultant
of these pulls.
of the on the tight side, equal to 2TX
pulls
acting through C, and also the resultant of the
Fig. 276. Froude
ulls on the slack side e(l ual to 2T2 acting
or ThoraeySroft P '

transmission through 2), then


dynamometer.
PxGF=(2T xFC)-(2T2 xFD), 1

and if the arms FC and FD are equal, as they usually are, this
will reduce to
Px GF=2 FC(T .
X
- 2)

T,-T =h.Px FC
GF
9

Knowing the dimensions GF and FC in a given brake of this


;

INDICATED AND BRAKE HORSE-POWER. 219

class, (T - x
T2) may
be calculated from the observed value of P,
and if the belt, or the revolutions and the
the speed of
diameter of the driven pulley be known, the H.P. transmitted
may be calculated in the same manner as in the preceding
case.

Measurement of large powers. The operation of measuring
the energy delivered by very large engines is a very difficult one.
It is not easy to imagine an absorption dynamometer suitable
for attachment to a marine engine of, say, 10,000 I.H.P. In
such cases, the Indicator Diagram is the sole means of estimating
the power of the engines. The case of large engines driving
electrical machinery is simpler. Here the electrical power
delivered by the machines can be measured with considerable
accuracy, and if the I.H.P. of the engine is obtained at the same
time, the mechanical efficiency of the combined engine and
dynamos can be calculated. Thus
Electrical power delivered by the dynamos

~ amperes x volts
_
746
horse-power = E.H.P.
pup
.*. Efficiency == -f-Wp
1 x 100 per cent, for the combined plant.

If the electrical and mechanical losses in the machine can be


obtained at the same time, we have the power actually delivered
by the engine to the machine from
Power delivered to machine = E.H.P. + H.P. absorbed by
electrical and mechanical losses in the machine.
Fluctuations in the speed of machines. In engines used
for the purpose of converting energy supplied from a natural
source into a form more suitable for practical purposes, such as
the driving of machinery, the energy taken in may be received
by the engine either at a uniform, or nearly uniform, rate, or it
may vary considerably.
Machines such as steam, gas, and oil engines, periodically
take in the energy to be converted and do not convert it at a
uniform rate. Again, in many cases, such as slotting, punching
and shearing machines, energy is supplied at a fairly uniform
rate, but work is done by the machine periodically only. In all

220 APPLIED MECHANICS FOR BEGINNERS.

cases, a want of uniformity in the rate of supply of energy to a


machine, or fluctuations in the rate at which energy is produced
from a machine, will produce unsteadiness in the motion.
Excess energy given to a machine above what is required to
satisfy the demand must remain in the machine as additional
kinetic energy in the parts, and these can only store additional
kinetic energy by moving faster. If the demand exceeds the
supply at any time, then this must be met from the stored
kinetic energy in the machine, which, accordingly, must diminish
its speed.
Methods of securing steady motion. It is customary, in
cases where the demand or supply fluctuates, and steadiness is
aimed at, to attach to the machine some body having a great
mass and capable of storing a large quantity of kinetic energy.
Comparatively small fluctuations in the demand or supply are
met from the store of energy in this body, with the effect that
the fluctuations in speed are greatly reduced in magnitude and
the machine moves more steadily.
In ordinary engines having a revolving crank shaft, the body
for storing energy consists usually of a flywheel attached to the
shaft and revolving with it. The great mass of a railway train
when in motion enables a large quantity of kinetic energy to be
stored and prevents small fluctuations of the energy produced
by the locomotive from being perceived, although one can detect
them for a few seconds as the train is starting when the speed is
very slow. Paddle steamers, especially those with one cylinder
only, depend on the mass of the vessel when in motion for steadi-
ness of speed. The " kick " of the vessel as she takes in more
kinetic energy can be plainly perceived. In the case of an
ordinary bicycle, the mass of the rider and the machine prevent
fluctuations in the energy supplied by the rider to the cranks
during a revolution being observed to any extent.
Energy stored in flywheels. We may find approximately
the kinetic energy of a flywheel by considering its mass to be
concentrated at the mean radius of the rim. Let v ft. per sec.

be the velocity of a point at this mean radius, and m the mass of


the wheel then ;

T _. ,. mv 2
Kinetic energy = -=
z9
INDICATED AND BRAKE HORSE-POWER. 221

Example. Suppose the mass of the rim of a flywheel is 10 tons


and its mean radius is 6 ft. Find its kinetic energy when rotating
90 times per minute.
90
Revolutions per sec. =z^-
60
Velocity of a point on the wheel at 6 ft. radius
90
=****
<6x
60
ft. per
7

Kinetic energy =-5

= 10x 396x396
64-4x7x7
= 496-9 ft. -tons.

M of a flywheel.
Let iV= revolutions per minute of a flywheel.
r=mean radius of rim, feet.
m=mass of wheel.
v = velocity of a point at r radius, ft. per sec.

Then
N
v=^ .2irr
Fi
oU
N.irr .
= -q- ft. per sec.

Kinetic energy of wheel =


~2g\ 30 )

V 1800 .a
9 )
The coefficient of N 2
in this result is constant for a given wheel,
so that we may state that the kinetic energy of a given wheel
depends simply on the square of the revolutions per minute. If,
therefore, we know the kinetic energy of the wheel at any given
speed, say one revolution per minute, we can calculate, by simple
proportion, its energy at any other speed.
Let M = kinetic energy of wheel at 1 revolution per min.
K= at N revolutions per min.
Then, M:A =l:^2,
r

or, A'=M^ 2
.
222 APPLIED MECHANICS FOR BEGINNERS.

Fluctuation of speed in flywheels. We may easily calculate


the fluctuation in the speed of a flywheel we know the if

fluctuation in the demand for energy. Thus, suppose a wheel of


mass m lbs., to have a mean radius r feet then, if the velocity ;

of a point at the mean radius is v ft. per sec. at a given instant,


x

Kinetic energy = K =-~


1
ft. -lbs.

Suppose now that there is a demand for W ft. -lbs. of energy


which has to be met by the store of energy in the wheel. The
wheel will slow down while supplying this. Call v 2 the velocity
of a point at the mean radius when the change is complete ;

then Kinetic energy = K = -~ 9 ft. -lbs.

The wheel loses kinetic energy = K K X 2,


and this has been
transformed into W ft.-lbs. of mechanical work,
.-. W=K -K, 1

_mv 2 mv22 1

=(V-V>,
or, p.8 -*J-?2. W.

Example. A wheel of mass 2000 lbs. at a mean radius of 3 feet


has a speed of 180 revolutions per minute. Suppose 4000 ft.-lbs. to
be abstracted from it and calculate its new speed.
180 _

= 3x2xyx3
= -=-=56-57 ft. per sec,

and ^2 = 3200,
and v 1 z_ v z2 = '* w
m t

3200 -V=^x 4000


= 128-8.
v22 = 3200 -128 8,

tf
2 =V307l
= 55 "4 ft. per sec.
INDICATED AND BRAKE HORSE-POWER. 223

r
Let new speed = A 2 revolutions per minute.
554x6
N.,=

55'4x (x7
2 x 22 x 3
= 176 '3 revolutions per min.

The wheel therefore loses 3 '7 revolutions per minute while giving
up 4000 ft. -lbs. of energy.
It forms a useful exercise for the student to perform some
experiments on the energy of a flywheel. For this purpose, a
small flywheel is mounted
on a horizontal shaft which
also has a drum on which
a cord is coiled (Fig. 277).
A pan is attached
scale
to the end of the cord.
Loads placed in this pan
and allowed to descend will
rotate the flywheel and give
energy to it. The cord must
be attached to the drum in
such a manner that it will
easily free itself from the
drum when the pan reaches
the floor.

Expt. (a). Place a suffi-

cient load in the pan, and let

its weight, together with that


of the pan, be lbs. W
Raise
the pan a measured height H Fio. 277. Experimental flywheel.
feetby rotating the wheel
by hand so as to coil up the cord. Now allow the pan to
descend, being careful not to aid it to start in any way, and
observe the time of its descent, using a stop-watch for this
purpose. Repeat this several times in order to obtain the
average time of descent let this be t seconds.
;

Potential energy lost by the descending load= JfZf ft.-lbs.


This energy has been transformed partly into kinetic energy
in the rotating wheel, partly into kinetic energy in the descend-
:

224 APPLIED MECHANICS FOR BEGINNERS.

ing load, and some has been utilised in overcoming the frictional
resistances of the bearings of the shaft.
To obtain the kinetic energy taken up by the load during its
descent
Let v = velocity acquired, feet per second.
Average velocity during descent =\v.
H= average velocity x time of descent
=%vxt ;

.*. v = ~ feet per second, and


I)
kinetic energy acquired by the load = -^ ft. -lbs.

Neglecting frictional losses at the bearings, the kinetic energy


acquired by the wheel would be

E^WH-^itAhs (1)


Expt. (b). Observe the number of revolutions made by the
wheel while the load is being wound through the measured height
If. The wheel will rotate the same number of times while the
load is descending. Call this number n. Since the wheel makes
n revolutions in a time t seconds, starting from rest, its average
7b
speed of rotation will be equal to - revolutions per second, and

its maximum speed of rotation will be


2?i
revolutions per

second. Let N be the maximum speed of rotation, stated in


revolutions per minute, then
2n .. 120n
JV=x60 =
t
t

The kinetic energy acquired by the wheel may now be stated


in terms of M, thus,

or MN*=WH-^-,
Wv 2
giving M= * ft.-lbs., (2)

this being the kinetic energy of the wheel when rotating with a
;

INDICATED AND BRAKE HORSE-POWER. 225

speed of one revolution per minute, frictional losses being still

neglected.
Expt. (c). To determine the energy utilised in overcoming the
wind up the load W again through the
friction of the bearings,
same height H, and allow it to descend as in Expt. (a). This
time, in addition to the other observations, note the total
number of revolutions made by the wheel from starting to
stopping. Call this number n.L .

Whole energy transformed = WH ft. -lbs.

Wv2
Kinetic energy acquired by descending load = --= ft.-lbs.
9
The difference between these, viz.,

(wH-^)ftAbs., (3)

reaches the drum, and is disposed of, while the load is descending,
partly in overcoming the frictional resistances and partly in
giving energy to the wheel. After the load has reached the
floor, the whole of the energy given to the wheel is absorbed in
overcoming the frictional resistances while the wheel is coming
to rest, consequently the whole of the energy reaching the drum
is ultimately expended in overcoming the frictional resistances
during n 2 revolutions. Assuming that the frictional resistances
are constant during the experiment, the energy required to
overcome them during one revolution will be
Wv2
=2- ft.-lbs.
n2
and during the time when the load is descending, in which
period the wheel rotates n times, the energy required will be

The equation showing the


(^-wt hAhs disposal of energy will now be
(4)

2g \ 2g Jn 2
from which the final value of M may be found, giving
M= 2^ SzA (6)

A.M.B. P
:

APPLIED MECHANICS FOR BEGINNERS.

Expt. (d). Nowperform a series of experiments, using different


loads and heights, and observe in each case all the particulars
required as mentioned above. Tabulate your observations thus

Experiment on Energy of Flywheel


',

No. of Load,
Height W Time of Revolutions,
Revs., total,
Experiment. jribs.
descends, descent, W descending n.2 .
H feet. t seconds. n.

The results must now be worked out, and are best stated in
tabular form in the following manner :

Results of Experiment.

Kinetic
Velocity Speed Energy Energy spent
acquired acquired by Energy
No. of acquired in Friction,
by load, wheel, transformed, by W,
Experi- 1H M.
ment. ^ = 120n
JV-2
WH V 2g f%t
foot pounds.
20 foot pounds.
feet per sec. revs, per min.
footpounds.

Find from the last column the average value of M, which will
now be very near to the true value.
Momentum. Momentum is possessed by a body when in
motion ; it is proportional to the mass of the body and to its
velocity jointly, and is measured by the product of these.
Momentum = mv.
Units of momentum will be stated by giving the units of
mass and velocity employed thus, if the pound is used for the
;

unit of mass, and one foot per second is the unit of velocity, then
Momentum = mv, pound-foot-seconds.
INDICATED AND BRAKE HORSE-POWER. 227

Example. Find the momentum of a body of mass 100 lbs. when


it has a velocity of 5 feet per second.
Momentum = mv
= 100 x 5 = 500 lb. -ft. -sees.

Forces generating momentum. Suppose


a body, of mass
m pounds, to be acted on by a force pounds during a time P
t seconds, and that the body is at first at rest.

An acceleration a feet per second per second p


will be produced, such that

P= lbs (1) Fia. 278.


9
Since P acts for a time t seconds, the velocity of the body at the
end of this time will be
v = at feet per second ;

.'. = - feet per second per second.

And from (1), by substitution,

P=
gt t w
(2)

Now mv is the momentum possessed by the body at the end

of the time t seconds, consequently will be the momentum it

acquires each second, and the force generating momentum will be


numerically equal to the momentum generated per second divided by
g. This relation is very useful when considering impulsive
forces, i.e. which act during a very small interval of time.
forces
A given momentum can be possessed either by a body of large
mass having a small velocity, or by a body of smaller mass
having a larger velocity. Suppose A is a body of mass and M
velocity v, and that B is another body of mass m and velocity
V, the velocities having been produced from rest by equal forces,
P, P, acting during the same time t.

P=
9*
, for the body A ;
[from (2) above]

/>? for the body B ;

9*
Mv __mV
~9t~~9t'
or, Mv=mV.
228 APPLIED MECHANICS FOR BEGINNERS.

may therefore be stated that equal forces, acting during


It
the same time on hodies of different masses, generate equal
momenta.
For example, consider the forces acting on a shell and gun
when the gun is discharged. Neglecting effects produced by
the inertia of the powder gases, it is evident that forces equal to
one another act for-
M
^^^Hu^^uhMmmmTrnmrmmnn^-^ ward on the shell and
TT^Ehfezfeaaawa **** v
backward on the gun
during the time that
the shell remains in the gun barrel, these being due to the
pressure of the powder gases. The forces being equal, and the
times of action being also equal, equal momenta will be
generated in the gun and in the shot, assuming of course
that the gun is so mounted, or suspended, that it is free to
move backwards.
Let M = mass of gun, lbs. (Fig. 279).

v= backward velocity of gun, ft. per sec.

m = mass of shell, lbs.

V= forward velocity of shell, ft. per sec.

Then Mv = m\\
which enables the velocity of either gun or shot to be calculated
if the other quantities are known.
Impulsive forces. Imagine
a body in motion to possess a
momentum abstracted by the body encoun-
equal to M, which is
tering a uniform resistance P. If this is accomplished in t
seconds, then

p M
9*
It will be noticed that becomes very small, P will become
if t

very large, and in fact, the force will be impulsive. Notice


also that, if the resistance encountered is not uniform, we may
still find its average value from this equation. This force we
may call, in the case of impulsive action, the average force of the
blow. It is quite impossible, in most cases of impulse, to state
exactly what the actual reactions are at any instant, and it is

very convenient to be able to calculate, at anyrate, their average


value.

INDICATED AND BRAKE HORSE-POWER. 229

Example. A hammer head, 2 lbs. mass, moving with a velocity


th
of 40 feet per second, arrested in ^s"o~
is second by meeting an
obstacle. Calculate the average force of the blow.
Momentum of hammer = 2 x 40
= 80 lb. -ft. -sees.

Momentum changed per second = 80 4- ^z

= 16,000 lb. -ft. -sees.,

and average force of blow = P = -&ip&-


= 500 lbs. nearly.

Change of momentum. Momentum depends on the velocity


of a body ; and, as velocity has direction, momentum will also be
a directed quantity. Change of momentum in any given case
must therefore be measured by taking the change in the body's
velocity. The method of ascertaining this has already been
described in Chap. XII. Having found the change in momentum
and its direction, the force required will act in the same line.
Example. Suppose a stream of 180 bullets per minute to impinge
at 90 to a plate with a velocity of 1000 feet per second, and then to
drop vertically downwards. If each bullet has a mass of 1 ounce,
what is the reaction of the plate ?
The change of velocity in this case will be at 90 to the plate and
will be equal and opposite in sense to the velocity of the bullets.
Change in velocity = 1000 feet per second.

Mass reaching plate per second = -^r- x ^=~ lb.

Reaction of plate = = 1
^oo-o

= 5-8 lbs.

If a jet of water be substituted for the bullets, the problem


will be similar.
Example. Suppose a jet to have a sectional area of y^ square
foot and a velocity of 200 feet per second.

Mass reaching plate per second = y^ x 200 x 62


-
5
= 125 lbs.

Reaction of plate =
= 125x200 __
nnn = 776C1U
lbs.
230 APPLIED MECHANICS FOR BEGINNERS.

Centrifugal pump. Suppose we have a hollow wheel with


openings at A, A (Fig. 280), for water to flow into it, and that
the wheel has blades arranged radially, so that if the wheel is at
B

Fig. 280. Diagram of a centrifugal pump with radial blades.

rest, the water will flow outwards and be discharged all round
the rim of the wheel in radial jets. Assuming that the radial
velocity of the water kept constant while passing along the
is

blades, there will be no change of momentum whatever, and


consequently no reaction required from the blades. If, how-

stillenters the blade portion of the wheel in radial lines, it will


be discharged at the outer circumference with not only radial
velocity, but also a tangential velocity of an amount equal to
that of the rim of the wheel. This must be the case, for the
water can only pass through the wheel by sliding along the
biades, and consequently at any part of the blade will have a
tangential velocity equal to that of the blade there.
Let F=the velocity of the rim of the wheel, feet per second,
then Change in tangential velocity of water = F,
being zero at the inner part of the blade and V at the outer.
Let M= mass of water, lbs., flowing per second, then
Change of tangential momentum = M F, and
Force which the wheel has to exert to produce this =
MV lbs.
g
This force may be imagined concentrated at the rim of the
wheel, so that

Work done per second =


MV x '

V
9
= foot-lbs.
9
,

INDICATED AND BRAKE HORSE-POWER. 231

Horse-power which must be applied' to the wheel


^ MV
2
60
X
g 33,000'

The arrangement is that of the ordinary centrifugal pump.


In practice the blades are generally curved to avoid shocks and
to allow the water to flow as quietly as possible through the
wheel. The only difference this will make to the above calcula-
tion is work done. For if the water still enters with zero
in the
tangential velocity and is discharged with V feet per second
tangential velocity, then the tangential force required at the

rim of the wheel to do this will be lbs.


9
Let Fp = tangential velocity of the wheel rim, then

Work per second =


MV V foot-lbs.
v .

9
_
Horse-power required =
.MV V
60 ,
T
.
.
.
p .

Example. 80 cubic feet of water per minute pass through a


pump, the velocity of the rim in a tangential direction
centrifugal
being 25 feet per second. Suppose also that the water enters
with no tangential velocity and is discharged with one of 20 feet per
second.

Work done
^
Then, tangential force required at wheel rim

per second = 51 7 x 25
= 1292
oU
25 '

ft. -lbs.
x 20 + 32-2=51 -7 lbs

~ 1292x60
H.P. required = - -^r =2oor
,
35.
33 -

EXERCISES ON CHAP. XV.


1. A friction dynamometer has a single load of 60 lbs. weight,
the arm of which is 30". Revolutions 100 per minute. Find the
Brake Horse-power.
2. In testing an engine for B. H. P. a brake similar to that shown
in Fig. 276 was used. The speed of the engine was found to be
200 revolutions per minute the pull of the spring balance 1 1 lbs.
;

and the dead load 70 lbs. The brake wheel was 5 feet diameter.
Calculate the B. H.P.
232 APPLIED MECHANICS FOR BEGINNERS.

3. In Question 2, calculate the heat developed by the brake.


State the result in British Thermal Units per minute.
4. In Question 2 the Indicated Horse-power was found to be 6*5.
Calculate the mechanical efficiency of the engine, and also the energy
lost per minute in overcoming frictional resistances in the engine.

5. The mean diameter of a flywheel is 12 feet. Its mass is 8


tons, and it is running at 90 revolutions per minute. Find its
Kinetic Energy.
6. Suppose the flywheel in Question 5 gives up 100,000 ft. -lbs. of
energy, what speed will it have ?
7. A flywheel has a mean radius of 3 feet and a normal speed of
120 revolutions per minute. It is required to supply 2000 ft. -lbs.
from its store of energy while slowing down to 118 revolutions per
minute. What mass of rim is required?
8. A truck, mass 10 tons, moving with a velocity of 4 feet per
second, comes into collision with fixed buffers and is stopped in
\ second. What is the average force of the blow ?
9. The mass of the moving parts of a steam hammer is 1000 lbs.,
and the hammer head reaches the work with a velocity of 20 feet
per second and is brought to rest in T J U second. Calculate the
average force of the blow.
10. A hammer head of 2\ lbs. moving with a velocity of 50 ft. per
second, is stopped in 0*001 second. What is the average force of
the blow? (1897.)
11. A
ship of 2000 tons, moving at 3 knots, is stopped in one
minute ; what is the average retarding force ? Neglect the motion
of the water. One knot is 6080 feet per hour. (1898.)

12. A car weighing 2^ tons and carrying 40 passengers, the


average weight of each of them being 145 pounds, is travelling on a
level rail at the rate of 6 miles an hour. What is the momentum in
engineer's units? If the propelling force be withdrawn, what
average force in pounds must be exerted to bring the car to rest in
two seconds? And supposing the force to be constant, what dis-
tance would the car travel before it came to rest? (1899.)

13. in any machine with which


Explain the use of the flywheel
you are acquainted. To what
class of machines is such a wheel
usually applied ? What is the kinetic energy in a wheel revolving
at 150 revolutions per minute, if the wheel loses 5000 ft. -lbs. of
energy when its speed is reduced to 147 revolutions per minute ?
(1899.)
14. If a bullets per minute, each weighing 0*5 oz.
gun delivers 400
with 2000 feet per second horizontal velocity ; neglecting the
momentum of the gases, what is the average force exerted upon the
gun? (1900.)
15. A flywheel is required to store 12,000 ft. -lbs. of energy as its
);

INDICATED AND BRAKE HORSE-POWER. 233

speed increases from 98 to 102 revolutions per minute ; what is its


kinetic energy at 100 revolutions per minute? (1900.)
16. A man and his bicycle weigh 170 lbs. ; he has a speed in-
dicator (not a mere counter). When going at 10 miles an hour on a
level road he suddenly ceases to pedal, and in 15 seconds finds that
his speed is 8 miles an hour. What is the force resisting motion ?
(1901.)
17. A machine is found to have 300,000 stored in it as
ft. -lbs.
kinetic energy when its main shaft makes 100 revolutions per
minute ; if the speed changes to 98 revolutions per minute, how
much kinetic energy has it lost ? (1901 .

18. The flywheel of a gas engine has a mass of 1000 lbs. at a mean
radius of 2 feet and runs at 200 revs, per min. The supply of gas to
the engine is stopped at a given instant and it is found that the fly-
wheel comes to rest in ^ minute. Calculate (a) the energy stored in
the wheel at first ; (b) the number of revs, while coming to rest
(c) the energy abstracted from the wheel per revolution, assuming
this to be the same for every revolution.
CHAPTER XVI.

CENTRIFUGAL FORCE. EXAMPLES OF WANT OF


BALANCE IN ROTATING BODIES. BURSTING
EFFECT IN FLYWHEELS. GOVERNORS.

Motion in a circle. If a body be attached to one end of a


string, the other end held in the hand, and the body whirled
round in a circle, it will be noticed that a pull along the string
has to be resisted. Suppose the body m
V ^ B t be at A (Fig. 281), the natural tendency
m WA "* s - * s ^ or ** to move m
tne * me ^^' tan gential
/' \ \ to the circle, and the pull P, applied to the
\P \ string A C by the hand at C, is required to
;'

\c I
overcome this tendency and to cause the
\ J
body to move in the circular path. The
inertia of the body* causes it to resist this
**
v "*""""
S pull with an opposite force F equal to P.
P is called the Central Force (sometimes
Centripetal Force), and F is called the
Centrifugal Force. Evidently, if the string is let go, these forces
would cease to exist, and the body would move off in a path
tangential to the circle.
Since a central force is required to overcome the inertia of
the body, it follows that there must be an acceleration caused
by it, towards the centre of the circle.

Let v = the velocity of the body in its circular path, feet per
second,
r=the radius of the circle, feet,
-

CENTRIFUGAL FORCE. 235

then it can be shown that the acceleration towards the centre


of the circle is given by
2
v
a= feet per second per second (1)

Again, D ma

;. P-Z* t (2)

the units being pounds or tons weight depending on the unit of


mass used for measuring m.
This equation (2) may be used to calculate the central force,
or the centrifugal force, acting on a rotating body.

Example. Calculate the central force required to cause a body of


mass 100 pounds to whirl in a circle 4 feet diameter 120 times per
minute.
120 ,
Here
= 2xttx4=I
p_mv i

gr
100 X (87T) 2
" 32-2x2
= 982 lbs.

Want of balance in machines.The effect of the centrifugal


force caused by a revolving mass in a machine running at a
high speed may be very serious, producing disturbances which
may cause the bearings to give out in a very short time and
possibly also to hammer other parts of the machine to pieces.

Example. In a machine running at 1800 revolutions per minute,


there an unbalanced mass of 1 pound at a radius of 1 foot. Find
is

the pull on the bearings due to the centrifugal force.


1800 _
v
=~w x2irr
= 30 x 2 x 7T x 1 =60tt ft. per sec.

Centrifugal force =
_1 X 60 X 60 X VX 7T

322x1
= 1120 lbs.
'

236 APPLIED MECHANICS FOR BEGINNERS.

This force, constantly directed from the centre, will revolve


with the shaft, causing pressures on every part of the bearings
during one revolution. The pressures
on the bearings may be got rid of by
fastening a mass of one pound in the
same plane of revolution as the given
one, and exactly opposite it at a radius
of one foot. The centrifugal forces of
the two masses will balance one another
(Fig. 282) without any aid from the
bearings, and the machine is now said
to be balanced. Usually the data in
Fig, 282.
practice are more complicated than
in the
simple case described here.
Fig. 282a shows an apparatus which may be used for detecting
any want of balance in rotating bodies. Four discs are mounted
on the same shaft, which
isdriven by a small elec-
tro motor, the whole being
mounted on a frame sus-
pended by means of three
chains. Masses may be
attached to the discs,
and, if unbalanced, will
produce vibrations in the
frame when rotation
occurs. If the masses
are in balance, then the
frame will remain steady
at all speeds of rotation.
This apparatus is due to
Professor Ewing.
Bursting effect in fly-
wheels.The effects of
Fig. 282a.
Apparatus for experiments on the centrifugal force have
balancing of rotating bodies.
also to be considered in
designing flywheels, as their rims are put under tensile
stressthereby, when the wheel rotates. Fig. 283 shows the
rim of a rotating flywheel every piece of material in the
;
CENTRIFUGAL FORCE. 237

rim will produce a centrifugal force directed from the centre


as shown. The case is analogous to a cylindrical shell under
internal pressure (p. 80), and we may treat it in the same
manner.
Let m=mass of rim, in lbs. per foot circumference,
v = velocity of rim, feet per second,
r=mean radius of rim, feet.

Then, centrifugal force per foot circumference = lbs.

Fig. 283. Rim of a rotating flywheel. Fig. 284.

The resultant centrifugal force for half the wheel, correspond-


ing to (p x d) in the boiler question, will be

mv2 x2r
R= o
gr

lbs.
9
Let = area of section of rim in square inches,
q = tensile stress on o, lbs. per square inch.
Then R = 2qa (Fig. 284),
2qa
*
9

q= lbs. per square inch.

Notice that m, the mass of rim per foot length is given by


a
m = mass of material per cubic foot x 1 x
144'

mlnerefore
, -
q = mass per
c
cubic foot xv 2
lbs. per square inch.
. ,

This equation shows that the tensile stress due to centrifugal


APPLIED MECHANICS FOR BEGINNERS.

force is independent of the sectional area of the rim and of the


radius of the wheel.
Example. A cast-iron wheel is run at a rim speed of 80 feet per

second. If the material has a mass of 450 pounds per cubic foot,
find the tensile stress due to centrifugal force.
2

q
_ 450xi;
~U4xg
nearly
=Yo
10
80x80
=640 lbs. per sq. inch.

Centrifugal governors. Engines supplying motive power


usually have to run at a fairly constant speed. This is managed

Fig. 285.Simple centrifugal Fio. 286. Diagram of forces


governor. acting on one ball.

by means of a governor, which controls the supply of steam, gas,


or oil, by opening or closing a valve. The governor usually
consists of two revolving balls mounted on two arms attached
to a shaft driven by the engine (Fig. 285). The balls move
out to a larger radius of rotation if the speed increases, and
move inwards if the speed falls, and this movement controls the
supply valve. Let us consider the forces acting on one of the
balls. There will be its weight, W, the pull of the arm, T, and
centrifugal force, P (Fig. 286). If the ball is rotating at a
CENTRIFUGAL FORCE.

steady speed it will keep at a constant radius r from the axis


AB f
and P, W and T will balance. Drawing the triangle of
forces for these forces, as at abc, it may be seen that this triangle
is similar to the triangle ABC. Therefore
P W=r
: : h,

or
r
W =r
: : h ;

"
.h~Wr,
r
9
or hv i =gri ;

*-* 0)
Let N= revolutions per minute of the ball, then
2ttt x N

and y= 60^T60 (2)

Substituting in (1) we get


,_ r2 x 60 x 60

_ 3600g JL^

=a constantx-^ (3)

This shows that h is independent of the mass of the hall and of


the length of the arm, and depends only on the reciprocal of the
square of the revolutions per minute in this type of governor.
Example 1. Find h for a governor of the above described type
when making 60 revolutions per minute. Take g = S2.
3600,7 J^
*~ X
\ir* ^V2
- 7x7
900 x 32 x
"22x22x60x60
= 0-809 feet, or 9708 inches.

Example 2. In this governor, if the speed be increased to 61


revolutions per minute, find the vertical movement of the balls.
X
4T2 61x61
= 0-784 feet, or 9-408 inches ;

.-. Vertical movement of the balls =9 708 -9 408 = '3 inch.


.

240 APPLIED MECHANICS FOR BEGINNERS.

This movement of 03" at the governor is transmitted by


leversand connecting links to the valve controlling the supply
of working stuff and partially closes this valve the speed of ;

the engine will in consequence be reduced to the required


value again.

Fig. 287. Loaded governor used for controlling the speed of an oil engine.
Governors are usually loaded by means of a spring or weight
as shown between the balls in Fig. 287. This is done in order
that they may be run at a higher speed than is possible in
an unloaded governor, and the
sensitiveness and power of con-
trolling the valve are increased
by so increasing the speed.
Simple harmonic motion.
Considering a point P
(Fig. 288)
rotating in a circle of radius r,
with a uniform velocity v, we
have seen that its acceleration
towards the centre of the circle,

Fig. 288. N, the projection of P on


C, is Taking components of
r
A B, has simple harmonic motion.
this acceleration parallel and at
right angles to AB when P is in any be seen
position, it may
that its horizontal acceleration, represented by bP, will be
CENTRIFUGAL FORCE. 241

bP 2
v

equal to multiplied by ; or, since the triangles aPb and
PNC are similar,

horizontal acceleration = -pr^. and PC=r,


PC r
;

2
v
.*. horizontal acceleration = NC . -5 (1)

=a constant x NC.
That is, the horizontal acceleration is proportional to the

distance, in feet, of N from C. When N is to the left of C, this


acceleration is directed towards the right ; and towards the
left when N is to the right of C ; that is, the acceleration is

always directed towards C. As the point P rotates in the cir-


cumference of the circle, N
will vibrate to and fro in AB, and
its motion is called simple harmonic. If a body of mass m lbs.
vibrates with N, a force will have to act on it to give it the
required acceleration, this force being found from
R = ma poundals,

a being the acceleration NC -y found above ; therefore

2
R=NC.m. v-^poundals, (2)

and, as can be seen by inspection, will be proportional to NC


(the rest of the expression being constant), and will be always
directed towards C. Conse-
quently, if a body vibrates A C B
>p , ,,

with simple harmonic motion, R R


a force must act on it, directed Fio. 289.

always towards the middle of


the swing, and proportional to the distance of the body from
the middle (Fig. 289). If we know the time that the body
takes to travel from A to B, we can easily find R in any given
case ; for in Fig. 288 the point P travels a distance equal to
half the circumference of the circle, i.e., irr, while N travels
from A to B. Let the time taken for this be t seconds. Then
_ distance
""velocity'

A.M.B.
242 APPLIED MECHANICS FOR BEGINNERS.

k r
v
and, JL
2 '7*
l
7T
2
7T .

and, from (2), R = NC.m.'-^ (3)

We may alter this equation so as to give the time of a swing,


thus,
NC* m~ **'

2
xmx NC
*
2 =
a
? xM7
and,
=^v i2 '
(4)

which gives the time of a swing in seconds, if we know the force


R acting on m when it is at a distance
JVC from the middle of its swing.
Simple pendulum. Referring to
Example 3 on p. 27 we saw that the
force, required to pull the weight out of
the vertical, was almost exactly propor-
tional to the distance from the vertical,
provided this is not too great. Conse-
quently, if the weight be released, it
will vibrate to and fro with simple
harmonic motion. The arrangement
constitutes a simple pendulum. Call I

the length of the suspending wire in


ft feet ; then from the diagram of forces
(Fig. 290) acting on ra,

W~mg ab AN'
Now if NC is small compared with I,

AN and AC will be practically equal to


one another, therefore

Fia. 290. Simple pendulum. A


mg
NC very
I
nearly, or = NC
m9 ^R'
CENTRIFUGAL FORCE. 243

Substituting this in the time equation (4) for one swing, from C
to C, we get

a/ l

Vg m
r * =7r (5 >
'
V^
as the time of swing of a simple pendulum.
is independent of the mass
It will be noticed that this result
of the body and of the distance from C to C
For this to be
true, the body should be of small dimensions and the distance
CC small also. The suspending wire or cord should be very
light, so that its weight may be, as above, neglected.
The compound pendulum consists of any body vibrating about
a horizontal axis under the influence of its own weight. An
equivalent simple pendulum can easily be found by suspending
a small bullet by means of a fine thread from the same axis, and
adjusting the length of the thread until both compound and
simple pendulums swing together in the same time.

Expt. The simple pendulum may be used for roughly
determining the value of g. Thus, arrange a small bullet to
swing through a small angle at the end of a fine thread 3 or
4 feet long take the time of, say, 100 swings
; call this T
;

seconds then time of one swing


;

T
t=TXf; seconds.

g can now be calculated, from

>=*&
6
2 2 -,
* =ir
9
TV*. I
and,

Expt. Hang a helical spring from a fixed support, and


attach a body of known mass to its lower end by means of a
fine cord 5 or 6 inches long. When the arrangement is at rest,
pull the body slightly downwards, and then let go. The body
will vibrate vertically, and as the force acting on it will always
244 APPLIED MECHANICS FOR BEGINNERS.

be proportional to the distance from the middle of the vibration,


the motion will be simple harmonic. Observe approximately
the time during which vibrations can be perceived while the
body is coming to rest. Now arrange a vessel containing water
so that the hanging body is immersed. Start vibrations as
before, and notice that each vibration is executed in a longer
time and that the body comes sooner to rest. Repeat the
experiment using a vessel containing oil. These experiments
show the effect of fluid friction in altering the time of vibration
and in stilling down or damping out, as it is called, the vibra-
tions.
Substitute a long piece of rubber for the helical spring. It
will now be found that the body will only make one or two
vibrations in coming to rest. The great molecular friction in
the rubber rapidly damps out the vibrations.

EH

'^W>

Fig. 291. Escapement mechanism and pendulum for controlling a clock.


Control of clock mechanisms. The pendulum, on account
of the uniform time in which it executes small vibrations, is
used to control the working of clocks. This it does by per-
mitting one tooth only of a wheel driven by the clock to pass
while it makes each swing. The escapement mechanism, as it is

called, consists of an anchor-shaped pallet CAB (Fig. 291)


CENTRIFUGAL FORCE. 245

mounted so as to vibrate with the pendulum. The pallet


engages with the teeth of the escape wheel E, which is driven
by the clock, and does so in such a way that one tooth escapes
past C while the pendulum swings from right to left, and again
another past D while the pendulum swings from left to right.
The rotation of E
is therefore controlled by the pendulum,

and the clock consequently moves at a uniform rate. Energy


is lost by the pendulum during each swing, due to the frictional

resistances of the atmosphere and of the pivot on which it is


mounted. This is made good by shaping the teeth of and E
the edges of the pallet at C and D in such a way that an
impulse is given to the pendulum each time a tooth is sliding
on the pallet edge while escaping.
A spiral spring and balance wheel are substituted in some
clocks and in all watches for the pendulum. The inertia of the
wheel rim takes the place of the inertia of the pendulum bob ;

the resistance of the spring


(which will be directly propor-
tional to the angle turned
through from its mean posi-
tion) takes the place of the
weight of the pendulum bob.
When the wheel is set vibrating
under the control of the spring,
it will experience a torque from

the spring directly proportional


from tne mpan Fig.
trip mean 292. Escapement mechanism and
to it*
its riom
ancrlp
angle ba ance whee i for controlling a clock,
i

position. Consequently it will


make simple harmonic vibrations. Impulses are given to it, and
control to the clock effected by means of a small lever pivoted
at A (Fig. 292) on the same axis as the pallet, which engages
with the escape wheel as before. Watch and clock control
movements take many different forms, two alone being
described here.

EXERCISES ON CHAP. XVI.


1. A
body, whose mass is 20 lbs., rotates round an axis at a
radius of 9", with a velocity of 40 feet per second. Calculate the
pull on the axis.
246 APPLIED MECHANICS FOR BEGINNERS.

2. A
disc rotates on a shaft 120 times per minute. A
wrought-
iron pin, mass 5 lbs., projects from the disc, its radius being 12
inches. Find the mass required to balance the pin at a radius of
4 inches.
3. A cylindrical drum, 12" long, has equal masses of 10 lbs. each
attached to its ends at radii of 9". Looking at the end elevation of
the drum, the masses appear on the same diameter, on opposite
sides of the centre. Calculate the rocking couple set up when the
drum rotates 300 times per minute.
4. A cast-iron wheel,
with solid rim in one piece, the material of
which has an ultimate tensile strength of 8 tons per square inch, is
run at a gradually increasing speed. What will be the speed of the
rim in feet per second when the wheel bursts ?
5. In a common unloaded governor, calculate the vertical height
of the cone of revolution when the balls are rotating 60 times per
minute. What will be the change in this height if the speed rises
to 62 revolutions per minute ?
6. A point rotates in the circumference of a circle of 6" radius
with a velocity of 10 feet per second. The plane of the circle is
vertical. Find the horizontal component of the acceleration of the
point when it is in positions differing by 30 round the complete
circumference, and plot these on a time base.
7. Find the length of a simple pendulum to beat seconds. Take
<7 = 32-2.
8. A
railway coach, mass 20 tons, runs round a curve of 1,600 feet
radius at a speed of 45 miles per hour. Calculate the centrifugal
force.
CHAPTER XVII.

HYDRAULICS. WATER PRESSURE AND PRESSURE


MACHINES.


Some properties of fluids. Fluids are substances which are
not able to offer permanent resistance to any forces, however
small, which tend to change their shape. Fluids are either
liquid or gaseous; gases possess the property of indefinite
expansion, liquids do not. Thus, a small quantity of gas in-
troduced into a perfectly empty vessel will at once expand and
occupy the whole of the vessel, while a small quantity of liquid
in the same circumstances will simply lie at the bottom of the
vessel. Gases exist either as vapours, or as so-called perfect
gases. Theperfect gas was supposed to exist under all con-
ditions as a gas, but it is now well known that all gases can be
liquefied by great pressure and cold. A vapour may be defined
as a gas near its liquefying point, and a perfect gas as the same
substance far removed from its liquefying point.
Some liquids are more easily able to change their shapes than
others. Liquids which change their shapes with difficulty are
said to be the more viscous, the property being called viscosity.
Mobile liquids change their shape very easily ; thus, chloroform
is used for delicate spirit levels on account of the extreme ease

with which the bubble can change its position, chloroform being
very mobile. Other liquids, such as cylinder oils, treacle, pitch,
shoemakers' wax, are very viscous, but all change their shape if
given sufficient time. As we have already seen in Chap. VII.,
change of shape is always produced by shearing forces. If
equal compressive stresses are applied to all the faces of a cube,
248 APPLIED MECHANICS FOR BEGINNERS.

the body will become smaller, but will remain cubical but ;

if shear stresses be applied, the shape changes. It follows,


therefore, that if shearing stresses be applied to a fluid, it will

not remain at rest, but will change its shape, and therefore,
if the fluid is at rest, there can he none hut normal stresses acting
anywhere on or in it.
Stress on horizontal immersed surfaces. Since there can
be no shearing stress in a fluid at rest, and since friction is always
brought about as a shearing stress, it follows that when a liquid
such as water is at rest, there can be
no frictional forces contributing to
preserve the equilibrium of any por-
tion of it. Suppose, in a tank of water
(Fig. 293), we think of the equilibrium
of a vertical column of it standing on
a horizontal base of one foot square.
The forces acting will be the weight of
Fig. 293. Equilibrium of a the column, which, if the depth is D
column of water.
feet, will be equal to the volume of
the column multiplied by 62^, the weight of a cubic foot of
water nearly, so that

W= D x 1 x 1 x 62| = 62-5 . D lbs.


There will also be stresses on each vertical side of the column,
everywhere directed perpendicular to the sides, these being due
to the pressures from the surrounding water, but as there can
be no friction between the surrounding water and the sides of
the column, these stresses merely serve to keep the column in
shape and do not help in any way to balance the vertical force W.
W is balanced by the upward stresses on the base of the column,

due to the pressure from the bottom of the tank. Consequently,


the total force on the base, which is one square foot in area,
will be equal to 62*5 D lbs. Any other horizontal square foct
.

at the same depth will have a similar and equal pressure on it.
If, therefore, we have a horizontal area, A square feet, at a

vertical depth D in water, the total pressure on it will be


found by multiplying the pressure per square foot by the area
A y
or
P=62o.D.A lbs.
;

HYDRAULICS. 249

If the liquid is not water, but some other which weighs w lbs.

per cubic foot, then


P=%o.D.A.
We notice from this that the pressure on a horizontal area
depends directly on its depth in the liquid and is proportional to it
at double the depth the pressure on a given horizontal area
will be doubled, and so on. It will be observed that the shape
of a tank does not influence the pressure on its bottom. So
long as the area of the bottom and the depth of liquid are kept
the same, the pressure will be unaltered. The student should
keep clear of the error of supposing that the weight of water in
the tank gives the pressure on the bottom. This is not the
case, as may be seen in the three examples shown in Figs. 294-6.

HiiHii^ rnrrn
Fig. 294. Fig. 295. Fig. 296.

As a consequence of the pressures of the sides on the liquid


being everywhere perpendicular to the sides, in the first case
shown, the vertical upward components of these pressures
sustain some of the weight of the contained water in the ;

second case, the vertical downward components of these pressures


give more pressure to the bottom and so compensate for the
diminished weight of water in the tank in the third case, the
;

pressure of the closed top has to be allowed for. In every case,


the total pressure on the bottom will be found as already stated,
by finding first the pressure on a square foot at the given depth
and multiplying this by the actual horizontal area.

Stress on inclined immersed surfaces. Suppose, now, we
think of the column as standing not on a horizontal base, but on
an inclined one, forming the sloping upper face of a wedge A BC,
(Fig. 297) making, for simplicity, an angle of 45 with the
horizontal. The pressures on this wedge will be, as before,
perpendicular to its faces. Let P be the resultant pressure on
the face AC, R the upward pressure on the bottom. For
equilibrium of the wedge a third force Q, is required, which
2.-0 APPLIED MECHANICS FOR BEGINNERS.

will act perpendicular to the face AB and, if we neglect the


weight of the wedge itself, will be equal to R by the triangle of
forces. That is to say, at a given
depth in a liquid, the stress on a
vertical area is the same as on a
horizontal one. It can be shown that
this is true for all areas sloping at
any angle. This is usually stated
as the principle that fluids transmit
stresses equally in all directions. It
should be noted that in the case of
liquids in tanks subjected to pressures
due to their own weight, that as the
stress varies directly as the depth, a
vertical square foot anywhere will not
have uniform stress, but one which
varies from the top to the bottom.
The stress at any point on the square
Fig. 297. Equilibrium of the
wedge ABC. foot will be that due to the vertical
column of water above it and is stated
as the pressure which would be exerted on a square foot
embracing that point if the stresses were uniform. Thus, the
stress at a point 4 ft. deep in water will be 4x62*5 = 250 lbs.

per square foot on any plane horizontal, vertical, or inclined.
Pressures on the sides
of a tank. This principle
enables us to calculate the
pressures on the sides of a
tank. Taking a rectangular
tank as shown (Fig. 298)
the stress on the bottom at
a depth AB feet will be, for
water,

AB x 62*5 lbs. per square foot.


Fl - 8 m r e "*** The stress at the surface
^ire ofrtaft?U o/w at*
be zero, and as it
level will
uniformly increases as we descend, we may represent the
stress at any depth by the breadth of a triangular diagram
HYDRAULICS. 251

ABK at that point, the base BK being made equal to AB x 625.


The average stress on any of the vertical sides will obviously be
one half of this maximum stress, and we may find the total
pressure on any of the sides by multiplying the area of that
side by this average stress.

Example. A rectangular tank, 10 feet long, 6 feet broad, 4 feet


deep, is full of fresh water. Calculate the resultant water pressure
(a) on the bottom, (6) on one end, (c) on one side.

(a) Resultant pressure on bottom = average stress x area of bottom


= (625 x 4) x (10x6)
= 15,000 lbs.
(b) Resultant pressure on one end = average stress x area of end
= (62-5x2)x(6x4)
= 3000 lbs.
(c) Resultant pressure on one side = average stress x area of side
= (62-5 x 2) x (10x4)
= 5000 lbs.
Average stress on an immersed area. The average stress
on any immersed area can be shown to be the stress at the depth
of its centre of area. Thus, for the rectangular sides of the
tank just considered, the average stress is that occurring half
way down, at the position of the centre of area of the rectangle.
In other cases where the
areas are not rectangular,
the centre of area must be
found first, or in some
cases, the area may be split
into others more conven-
iently dealt with.

Example. The vertical end


of a tank, 6 feet long, with
sloping sides, is shown in Fig. 299. End elevation of tank.
Fig. 299. If full of water,

Resultant pressure on bottom = average pressure x area of bottom


= (62-5 x 4) x (3x6)
= 4500 lbs.
252 APPLIED MECHANICS FOR BEGINNERS.

Resultant pressure on one sloping side = average pressure x area of side


= (62-5x2)x(5x6)
= 3750 lbs.
In dealing with the ends, draw two verticals and CF, throughBE
B and G, and consider the end as made up of two equal triangles,
A BE and DGF, and a rectangle BGFE.
Resultant pressure on end
= pressure on triangles + pressure on rectangle
= 2{(62-5 x |) x (3 x 2)} + {(62-5 x 2) x (4 x 3)}
= 1000+1500
= 2500 lbs.
Centre of pressure. The resultant pressure on an immersed
horizontal area acts at its centre of area, and in the case of a
vertical rectangular area,having one edge in the surface, at
rds
the depth of water from the surface. The case of other
more complicated areas cannot be dealt with here. The point
at which the resultant pressure on a surface acts is called the
Centre of Pressure.
Retaining wall for water. The overthrowing action of
water pressure on a retaining wall
can now be easily understood.
It is usual ato consider
portion of one foot
the wall
long, on the assumption that
whatever happens to it will
equally happen to every other
portion. This being so, the
resultant pressure P (Fig. 300)
will be given by
P= average pressure x wetted
area
= (62-5x|#)x(#xl)
Fig. 300. Section of retaining wall.
=4 x 62-5 xH2 lbs.
P will act at a distance from the bottom, and will tend to

make the wall turn about A, its moment about A being /}


TT
multiplied by lb. -feet. This moment will be resisted by the
o
weight, W lbs., of the portion of the wall under consideration,
HYDRAULICS. 253

acting at its centre of gravity G. The moment of W about A


will be opposite to that of P, and will be W multiplied by
AE lb. -feet. If the moment of P is less than the moment of
Jr, the wall will not be overthrown. For safety the overthrow-
ing moment will always be considerably less in practice than
the maximum resisting moment.
It is usual to test in this way Draw a section of the wall
to scale, and show P and W in
their proper positions. Find
the resultant R, of P and W,
by the parallelogram of forces.
Divide the base AD (Fig. 301)
into three equal parts at and F
K. If R passes within the
middle part FK, the wall is
safe, and if outside, it is not
strong enough.
Forces acting on a floating

body. When a body, such as a
ship, is floating in water, it is
subjected to two resultant
forces weight and the re-
its
Fig. 301 test for
sultant water pressure on its
sides and bottom. Consider
the ship as floating at rest in still water. Its weight will
be a downward vertical force W
(Fig. 302), acting through G,
the centre of gravity of the ship.
The resultant water pressure
balances W, and therefore must be
an upward vertical force R = W,
and in the same straight line ~ 1
jw _ _ :

with W. This force R, due to 1


the buoyant effect of the water, Jr-w
is called the buoyancy.
\ )
Imagine for a moment that
the surrounding water becomes
solid, and so can preserve its Fig 302. Equilibrium of a body
floating at rest in still water.
shape, and let the vessel be lifted
out, leaving a hole in the water which it exactly fits. Pour
254 APPLIED MECHANICS FOR BEGINNERS.

water into this hole until it is full to the surface level, and
then let the surrounding water become liquid again (Fig. 303).
The pressures on the water poured
in will be exactly the same as when
""""' '""_"
i the vessel occupied the hole, and

j - their
effect is similar the weight
TW of the water poured in is supported.
^ This being the case, we see that
jo w
the weight of the water poured in

PlQ 303
and the weight of the vessel must
be equal to one another, as each
is equal to R, the resultant pressure from the surrounding

water. Further, R must act through the centre of gravity of the


water poured in, in order to support its weight. This point
is called the centre of buoyancy the point through which the

resultant pressure of the water acts. Since R acts through G


when the vessel is in the hole and through B when water is
in, it follows that G and B must be in the same vertical line.

We may state, therefore, that when a vessel is floating at


rest in still water, the weight of the vessel is equal to the
weight of the water displaced, and that the centres of gravity
of the vessel and of the displaced water are both in the same
vertical line.
Specific gravity by experiment.
Since specific gravity is
defined as the weight of a substance compared with the weight
of an equal volume of water, it may be seen now how to deter-
mine experimentally the specific gravity of a body heavier than
water.
Expt. First
weigh the body in the pan of an ordinary
balance. Then suspend it from the balance beam by a fine
thread and weigh it again, this time the body being immersed
in water at a temperature of 60 F. The weight this time will
be diminished by the buoyancy of the water, which we have
seen is equal to the weight of the water displaced, i.e., the

weight of a volume of water equal to the volume of the body.


This loss of weight of the body while in water, therefore, gives
the weight of an equal volume of water.

Let W = weight of body in air,


,

W=
2 water,
;

HYDRAULICS. 255

W l
- W = weight of an equal volume of water,
2

and specific gravity =w _ 1


wr

Example. An iron rivet weighs 0*365 lbs. when weighed in the


balance pan and 320 lbs. when weighed in water at 60 F.
the loss of weight =0*365- 0'320
= 045 lbs.,
and this is the weight of an equal volume of water. The specific
gravity is therefore 0*365
O^Ol^^i
Pressure of the atmosphere. Air possesses weight, and
consequently the atmosphere exerts pressure on all bodies on
the earth. If we were to consider a vertical column of the
atmosphere, one foot square in section and reaching upwards to
the limit of the atmosphere, we could calculate the pressure
produced by its weight on but the problem is com-
its base,
plicated by the fact that the atmosphere is not equally dense at
all parts of its height, but diminishes in density to zero. The
pressure due to the weight of the atmosphere can easily be
measured in the fo^owing manner.

Expt. Take a glass tube closed at one end and open at the
other, about 36" long, and fill it with mercury. The open end
being inserted in a cup of mercury, and the tube being held
vertically, the level of the mercury inside
the tube will fall until it stands at a height
h inches above that in the cup (Fig. 304).
At A, the pressure inside the tube is that due
to a column of mercury h high, and is equal
to w multiplied by h lbs. per square inch, w
being the weight of a cubic inch of mercury.
The pressure on the surface of the mercury in
the cup will be equal to this and is produced p-ik-f
by the weight of the atmosphere. This FlG ^.-Barometer,
apparatus constitutes the common barometer.
The average height of the mercury column is 30 inches, and as
mercury weighs nearly 0*49 pound per cubic inch, this will give
a pressure of 0'49 multiplied by 30, or 14*7 pounds per square
inch ;and this may be taken as the average pressure of the
atmosphere on all bodies on the surface of the earth.
256 APPLIED MECHANICS FOR BEGINNERS.

Boyle's Law for perfect gases. The experiments of Boyle


and others on the connection between pressure and volume of
gases, show that the pressure varies inversely as the volume,
provided the temperature remains unaltered. For example, if we
have a volume v x cubic feet of air at a pressure p pounds per x

square inch and change the volume to v% and the pressure to p2


without change of temperature, then

p1 :p2 = v2 :v1 ,

Or PlVl=P22'
Notice that the pressure must be absolute, that is, measured
from zero, not from atmospheric, in applying Boyle's Law.
Suppose we have a cylinder fitted with a piston of area one
square inch and stroke I inches, and allow air at a pressure
p x pounds per square inch to enter it. The piston will be
moved forward and work may be done to the amount of
p l multiplied by I, the result being in inch-pounds. If the air
supply is stopped at some part of the stroke before the end is
reached, the pressure will fall as the piston moves on, but the
expanding air will continue to do work. Using Boyle's Law,
the pressures at various parts of the expansion may be found
and a diagram plotted, from which the average pressure may be
found. Thus, suppose the stroke is 12" and air is supplied at
rd
60 lbs. per square inch, absolute, and cut off at stroke.

vv = 4 cubic inches.
Pi = 60 lbs. per sq. inch.
^ = 60x4 = 240.
The product of any other pressure and corresponding volume
must be 240, so arranging a table of volumes differing by
1 cubic inch, we may calculate the corresponding pressures.

V V p V
lbs. per sq. inch. cubic inches. lbs. per sq. inch. cubic inches.

60 4 26 6 9
48 5 24 10
40 6 218 11
34 3 7 20 12
30 8
HYDRAULICS. 257

Using these numbers, the diagram in Fig. 305 has been


plotted. The average pressure and work done may now be
found as described in Chaps. X. and XV.

Lift pump. The common lift pump
depends for its action on the pressure
exerted by the atmosphere. A cylinder
0iSCHAC(

LB5.Pt'S(f/nc/t

iz Inches.

Fig. 305. Diagram showing expansion of air in


accordance with Boyle's law.

at .1 (Fig. 306) contains a piston, or


pump bucket as it is called, fitted with a
valve opening upwards. The cylinder
is connected by a pipe C, with a foot
valve at its bottom, to a cistern of
water E. On the up-stroke of the
bucket, the pressure of the air con-
tained in C falls and the atmo-
E "^ 11
I I

spheric pressure on the water in


causes some of the water to flow
into the pipe C. On the down- -E
stroke the valve D closes and the
valve B opens. No water can pass
D now, and air will be expelled
through B. On the next up- Fig. 306. Lift pump.
stroke B will close again and D
will open, and more water will flow into (7, and this process
repeated again and again will ultimately bring water into
the cylinder, when it will pass B and be discharged through
F. The process of starting in this manner is long and can
be hastened by first charging the pump cylinder and pipe
A.M. 11. R
258 APPLIED MECHANICS FOR BEGINNERS.

C with water through a plug placed near the top of the


suction pipe.

Head of water. Water under pressure is often spoken of as
being under a head. Head is the height from the point con-
sidered in the water to the surface level. The connection
between head and stress is easily
seen from the principles already
discussed ; thus, if H is the
head in feet from the surface
tank at A (Fig. 307) to
level of a
a pump B, then the fluid
at
stress on the pump piston will be
62*5ZT lbs. per square foot. In
general, if tp is the weight of the
liquid in pounds per cubic foot,
B
h * Hike head in feet, and p the fluid
stress in pounds per square foot,
Fig. 307.
then p=wH.
The total force on the pump piston will be found by multiplying
its area in square feet by p.

Fig. 308.Force pump.

Force pumps. Pumps are used for many purposes and have
HYDRAULICS. 259

different forms depending on the conditions. Fig. 308 shows a


pump suitable for forcing water against a considerable pressure,
such as would be the case in feeding a boiler. The plunger A is
solid, without valves on its up-stroke water enters through the
;

inlet, passing the suction valve B on the down-stroke B closes


;

and C opens, allowing the water to pass into the discharge.


Hydraulic pumps are used for supplying water under high
pressure for power purposes. They may be either belt-driven
from a line shaft, or direct-coupled to a steam engine or other
source of power. The object is to deliver a steady stream of
water to the pipes at high pressure, say from h ton to 2 tons per
square inch. To secure a fairly steady flow of water there are
usually three pumps driven from a crank shaft having three
cranks placed at angles of 120 to each other. The pumps are
usually single acting, that is, they deliver water during one
stroke and take in water during the next, the action being con-
fined to one side of the piston only. The arrangement mentioned
therefore insures that at least one pump shall always be deliver-
ing water to the mains.

Fig. 309. Diagram of the hydraulic press.

The hydraulic press. The hydraulic press depends for its


action on the fact that water transmits stress equally in every
direction and is also practically incompressible. In Fig. 309, in
260 APPLIED MECHANICS FOR BEGINNERS.

which all details have been omitted, A is a pump having a


plunger B, say dx inches diameter, and carrying a load P. C is
a large cylinder with a ram, or cylindrical piston D, say d2
inches diameter, carrying a load W. The pump cylinder and
the large cylinder are connected by a pipe at E. Due to the
load P on the plunger B, a stress will be produced on the con-
tained water which occupies the whole of the space in the
cylinders not taken up by the plunger and the ram, this stress
being
P P 4P =
area of plunger ird^ 2
Trdj ^ * y*

~T~
The stress be transmitted to all parts of the water, and
p will
will exert a pressure on the bottom of the ram 2), tending to
2
7rd
raise it, the resultant pressure being p multiplied by , W
will be equal to this, neglecting friction.

W=p x -,
4W
We see, therefore, that
4P 1 4 If
^^andalso^, .

P d*
The mechanical advantage (without friction) of the arrange-
ment is therefore equal to the ratio of the squares of the
diameters of the ram and the pump plunger. For example, if
dx is 1 inch, and d2 10 inches, then if P is 1 ton, W would be
100 tons.
It will be observed, also, that if P is allowed to descend, W will
be raised a much smaller Suppose, for example, that
distance.
the area of the pump plunger section is 1 square inch, and that
the ram sectional area is then, if P descends
100 square inches ;

1" it water to the other cylinder.


will deliver 1 cubic inch of
This cubic inch spread over the area of 100 square inches, will
give a movement of T ^j" to the ram. So we see that the velocity
ratio of the arrangement will be 100.
HYDRAULICS. 261

Leather packings. Leakage past the ram is prevented by

Fio. 310, -U leather packing.

means of leather packing, A leather ring, of U section, shown


separately in Fig. 310,
is inserted in a turned

recess, as illustrated in
Fig. 309. Water leaking
upwards from the cylin-
der enters the interior
of the ring and presses
it firmly against the
sides of the ram and the
recess ; leakage past this Fig. 311. Hat leather.

place is effectually prevented. Two other forms of leathers,

Fio. 312. Cup leather.

hat and shown in Figs. 311 and 312. There


cup, are is always
a considerable loss by friction at these packings.
262 APPLIED MECHANICS FOR BEGINNERS.

The hydraulic accumulator is used in connection with all


hydraulic power plants. Its functions are to absorb the work
done by the pumps when the presses,
cranes, lifts, or other machines are
at rest, and therefore taking no water,
and also to prevent the water pressure
exceeding a given maximum. It
consists of a hydraulic cylinder (Fig.
313) placed upright and connected
to both pumps and machines to be
driven. The ram is loaded with
heavy weights, and rises, when the
water pressure is applied, against
the resistance of these. When the
ram approaches the top of its stroke
it works a tappet arrangement con-
nected to the throttle valve of the
pump engine, or to the belt striking
gear in a belt-driven pump, and so
stops the pumps. The maximum
working pressure of water which
Fig. 313. Diagram of the
hydraulic accumulator. can be obtained is determined by
the weights placed on the ram. Let
IF be this weight in tons and d the diameter of the accumulator
ram in inches ; then the water pressure
W 4.W.
p=fi,= "jjt ^ ons P er s(l uare men,

neglecting the loss by friction of the ram leathers. The ram


will not begin to rise until the water pressure attains this value.
Suppose the accumulator ram is up at the top of its stroke, and
that the rise has been H
feet. The work done by the pumps in
raising it will be, neglecting friction, WH foot-tons. If now
one of the hydraulic machines, such as a crane, be started, it
draws its water supply at first from the accumulator, the
weights of which in consequence descend, giving up some of
their stored energy to the crane. Soon after starting, the
descending weights release the tappet arrangement, and the
pumps start off again, delivering water to the machine direct
HYDRAULICS. 263

until it is stopped, when the water from thepumps again goes


into the accumulator and raises the ram. The arrangement, as
will be seen from the above description,
prevents any damage being done through
stopping the hydraulic machines while the
pumps are still working. Without the
accumulator such stoppage, as water is

practically incompressible, would have to


be accompanied by a simultaneous stoppage
of the pumps, which could not easily be
accomplished.

Pm. 314. Simple hydraulic lift. Pio. 315. Hydraulic crane.

Hydraulic lift. Fig. 314 shows a simple form of hydraulic


lift. The cage is secured direct to the top of the ram of a
vertical hydraulic cylinder. Water entering the cylinder raises
the ram and so also the load. If d is the diameter of the ram in
inches, and p the water pressure in pounds per square inch,
then the total load which can be lifted, neglecting friction,

will be px lbs.

Hydraulic cranes are much used. Their action will be under-


stood from Fig. 315. The chain sustaining the load passes along
the tie, and down the interior of the hollow post to a hydraulic
264 APPLIED MECHANICS FOR BEGINNERS.

cylinder situated in a pit.


This cylinder has a piston
ram with a chain wheel
A at its outer end, and
another chain wheel B is
mounted on the base of
the cylinder.The chain
is secured to the side of
the cylinder, passes over
A, then back over B and
thence up the post. The
object is to obtain a larger
travel of the chain for a
small movement of the
ram ; in the present ex-
ample, the chain raising
the load will move twice
as fast as the ram. The
cylinder of a hydraulic
crane may be placed in
any convenient position
and the chain led to it.
Usually another cylinder
is fittedfor revolving the
post, so as to swing the
jib to any convenient
position for raising, a
load.
The hydraulic jack,
Fig. 316, consists of a hy-
draulic cylinder inverted
and working on a station-
ary ram. The cylinder
contains a small pump
operated by a lever out-
side. Water contained
in the upper portion of
the cylinder is forced, on
Fig. 316. Hydraulic jack.
working the outside lever,
HYDRAULICS. 265

into the lower part, and so raises the cylinder and any load
which may be placed on its top. These machines are very
convenient for raising heavy loads through a short distance a ;

large velocity ratio is possible. Fig 317 shows a small hydraulic


jack arranged with levers for experimental purposes. Loads

Fig. 317. Experimental hydraulic jack. Fig. 318.Hydraulic punching bear.

up to 300 pounds can be conveniently applied to it. The


mechanical advantage, effect of friction and efficiency of this
machine are determined bv the methods described in Chapter
XIV.
The hydraulic punching bear (Fig. 318) is a convenient tool
operating on the same principle as the jack. The machine is
entirely self-contained, and is consequently portable and may
be set up in the most suitable position for performing any work.
266 APPLIED MECHANICS FOR BEGINNERS.

EXERCISES ON CHAP. XVII.


1. A
tank, 12 feet long, 8 feet broad, and 5 feet deep, is full of
sea water (64 lbs. per cubic foot). Calculate the pressures on the
bottom, on one side and on one end of the tank.
2. Fresh water stands to a depth of 6 feet on one vertical side of
a wall 20 feet long. Calculate the resultant water pressure on the
wall and its overthrowing moment.
3. If the section of the wall in Question 2 is rectangular, material
120 lbs. per cubic foot, and the height of the wall is 7 feet, what
should be its thickness in order to have a righting moment of twice
the overthrowing moment ?
4. 2 cubic feet of air at an absolute pressure of 15 lbs. per square
inch are compressed till the pressure is 95 lbs. per square inch
absolute. Assuming no change of temperature, find its volume.
5. What is the maximum height that a common lift pump may
be placed above the level of the supply water ?
6. A
pump, the diameter of the plunger being 2", forces water
against a pressure of 700 lbs. per square inch. If its stroke is
6 inches, and it makes 40 effective strokes per minute, how much
work is done per minute ? Suppose the efficiency to be 60 per cent.,
what horse-power is absorbed in driving the pump ?
7. of a water tank measures 7' in length and 3' 4"
The bottom
in width. When
the tank contains 900 gallons of water, what will
be the depth of the water, and what would be the pressure on the
bottom, on each side and end of the tank respectively ? One gallon
of water weighs 10 lbs. One cubic foot weighs 62*3 lbs. (1897.)
8. An accumulator is to work at 700 lbs. per sq. inch, the ram
is 10" diam., what must be the total load? If the lift is 8 ft., what
is the total store of energy when the weight is up? What is the
total store of pressure water, that is, the extra amount due to the
ram being up ? Sketch the leather packing of an hydraulic press.
(1897.)
Sketch in section and describe the action of the ordinary
9.
lifting pump. In such a pump the pump rod is f inch in diameter,
and the pump barrel is 5 inches in diameter, while the spout at
which the water is delivered is 20 ft. above the surface of the pump
bucket when the latter is at its lowest point what would be the
;

maximum tension on the pump rod in the upstroke of the pump,


neglecting the weight of the pump rod and the pump bucket (the
weight of a cubic foot of water is 62*5 lbs. ) ? (1896.)

10. Name
the chief physical properties of a liquid, and show in
what respect a liquid differs from a gas and from a solid. How is
the pressure of water on the vertical sides of a tank calculated ?
(1898.)
;

HYDRAULICS. 267

A water tank is 10' long, 10' wide, and 10' deep. When it is
11.
with water, what will be the force with which the water acts
tilled
on one side of the tank ? (1898.)
12. Water at 750 lbs. per square inch pressure acts on a piston
1 square foot in area, through a stroke of 1 foot ; what is the work
that such water does per cubic foot and per gallon ? If a hydraulic
company charges 18 pence for a thousand gallons of such water,
how much work is given for each penny ? (1898.)
13. Distinguish between the velocity ratio and the mechanical
advantage of a machine.
In a hydraulic lifting-jack the ram is 6" in diameter, the pump
plunger is " diameter; the leverage for working the pump is 10 to 1.
What is the velocity ratio of the machine? Experimentally we
find that a force of 20 lbs. applied at the end of the lever lifts a
weight of 8500 lbs. on the end of the ram. What is the mechanical
advantage of the machine? What is the efficiency of the machine?
(1899.)
14.Describe the construction and action of an ordinary suction
pump for raising water from a well. If 200 gallons of water are
raised per hour from a depth of 20 feet, and if the efficiency of the
pump is 60 per cent., what horse-power is being given to the pump?
(1899.)
15. A
hydraulic crane is supplied with water at a pressure of
700 lbs. per sq. inch, and uses 2 cubic feet of water in order to lift
4 tons through a height of 12 feet. How much energy has been
supplied to the crane, and how much has been converted into useful
work? (1899.)

Sketch and describe the construction and working of any


16.
hydraulic accumulator with which you are acquainted. If an ac-
cumulator has a ram 20" diam. with a lift of 15', and the gross
weight of the load lifted is 130 tons, what is the pressure of water
per square inch and the maximum energy in ft. -lbs. stored in the
accumulator, neglecting friction ? (1900.)

17. A rectangular areahas sides 18" (horizontal) and 22" (vertical)


the uppermost edge is 5 ft. vertically below the surface of still
water what is the total pressure on the area ? If the water is at
:

rest only at the surface, is the pressure greater or less than before ?
(1900.)
18. A
single-acting hydraulic engine has three rams, each of
3 inches diam. ; common crank, 3 inches long ; pressure of water
above that of exhaust, 100 lbs. per sq. inch 100 revolutions per
;

minute; no slip of water. What is the horse-power? If this engine


does 2*15 horse-power usefully by means of a rope, what is the
efficiency? (1901.)
CHAPTEK XVIII.

HYDRAULICS. FLOW OF WATER. WATER MOTORS.


The laws of fluid friction have already been explained, and
the student should refer to the statement of them in Chap. XL
before studying the following.
Discharge from an orifice. Oneof the simplest examples
of the flow of water is to be found when a jet discharges
through a sharp edged circular
hole in a thin plate. Let ab
(Fig. 319) be such a hole formed
in the vertical side of a tank, WL
being the surface level, giving
a steady head II over the orifice
ab. Outside some boundary such
as cde, the water will be moving
comparatively very slowly, and
a particle will only begin to
acquire important velocity when
it crosses to the orifice side of
the boundary. A particle situated
move straight for the
at d, will
and go out one at c will
orifice ;

move downwards along the tank


it arrives at a, having
side until
Fig. 319. Flow of water through
then a considerable velocity. Its
an orifice.
inertia prevents it from turning
the corner at a sharply, it can only do so in a curve, and the
same thing will occur to a particle situated at e, moving out at b.
9

HYDRAULICS. FLOW OF WATER.


Particles crossing the boundary at other places will move in
curved paths, somewhat like those shown, towards the orifice.
The general effect of all these movements is to cause the jet to
contract in cross sectional area after the orifice is passed and it

is only at some little distance from the orifice that parallel flow
is attained. The section at which contraction
is complete is

called the contracted obvious that the water has


vein ; it is

attained its maximum velocity at this place. In obtaining the


quantity discharged there are two things which must be

allowed for contraction and viscosity. It has been found
experimentally that, for orifices like the one under consideration,
the area of the jet at the contracted vein is 0*64 of the area of
the orifice. This is called the coefficient of contraction. It has
also been found that the actual velocity is 0*97 of that calculated
from a purely theoretical basis this number is called the ;

coefficient of velocity. The product of these two is called the


coefficient of discharge ; thus

c=(V64x 0-97 = 0-62.


Velocity of Discharge. Theoretically, the velocity of the
jet may be calculated from a consideration of the energy of a
pound of water at various places. Thus, one pound of water at
the surface level WL will have potential energy due to its
elevation Hover the orifice this potential energy will be equal
;

to 1 x 27, or H
foot-lbs. At d, on the same level as the orifice,
this potential energy will have been changed to energy of
another kind, as the pound of water is no longer elevated over
the orifice. This other kind of energy may be called pressure
energy and the expression for it may be obtained from a
consideration of the work done by the
pressure on a pound of water at d in
displacing it. Thus, let the volume of
one pound of water be v cubic feet,
and the pressure acting on it p pounds
rV *'

per square foot. Imagine the pound


Fig 320
of water to be contained in a cylinder
one square foot in sectional area (Fig. 320) it will then ;

occupy a length of the cylinder equal to v feet. The work


done by p in displacing the pound of water a distance v will be
%
:
270 APPLIED MECHANICS FOR BEGINNERS.

p multiplied by v foot-pounds, and this is taken as the measure


of the pressure energy. Therefore
Pressure energy of one pound of water at d in the tank
=p x v ft. -lbs.,
and if w = weight of one cubic foot of water, -

v= cubic foot
w
:. pressure energy =pv= foot-lbs.

The potential energy H foot-lbs. at WL will have been


changed into - foot-lbs. of pressure energy at d, so that

w
This statement will be very nearly true if there is only a very
small velocity of descent from WL to d, as then the losses due
to fluid friction having to be overcome will be very small. The
pressure energy possessed by one pound of the water at d will
be gradually converted into kinetic energy as the particle moves
towards the orifice, and will be completely converted when the
particle has attained its maximum velocity, that is, at the
contracted vein. Let F=the velocity at the contracted vein,
then
1 x V2
Kinetic energy of one pound of water there = -
V
= 2
foot-lbs.
2g
Assuming for a moment no losses by fluid friction between d
and the contracted vein, the pressure energy lost must be equal
to the kinetic energy acquired, so that

w 2g
'

Y2
or, since ^ = Sp H=^r:\

or

and
HYDRAULICS. FLOW OF WATER. 271

The actual velocity Va will be 0*97 of this, allowing for fluid


friction, so that
Va = 0-91\f 2gH.
Quantity discharged from orifice.
Let Q be the quantity
of water discharged per second, in cubic feet, then Q will be
the volume of a stream Ya feet long, this being the length
discharged in one second, and of cross sectional area equal to
64. J, A being the area of the orifice in square feet.
.-. Qmfrte.A. Va
= 0-64 x 0*97 x A xJtyH
= 0*62 A \f%gH cubic
. . feet per second,

or writing c, the coefficient of discharge for 0*62,


Q=c A \f2gH cubic feet per second.
. .

Experimental hydraulic apparatus. Fig. 321 shows the


whole arrangement of an experimental hydraulic apparatus,

Fig. 321. Arrangement of apparatus for hydraulic experiments.

means being provided for measuring the quantity of water


discharged from various shaped orifices in thin plates. An
upright cylindrical vessel made of wrought iron can be supplied
with water at a pressure of from 15 up to 100 pounds per
square inch. The water enters the vessel at the top, centrally,
being controlled by a valve outside, and the pipe is carried
down inside the vessel a little more than half way. This
272 APPLIED MECHANICS FOR BEGINNERS.

pipe perforated near its end with a large number of small


is

holes and is fitted with gauze baffle sheets to still down


whirling and eddies as far as possible. For small heads, a glass
water gauge with a scale of feet divided into tenths is provided.
This shows the head of water above the orifice level. For
larger heads, a pressure gauge of the Bourdon type is attached
to the vessel at the level of the orifice this gauge is divided;

both in pounds per square inch and in feet head. When using
heads of 2 feet or more, a cock which puts the top of the tank
into communication with the atmosphere is closed. A quantity
of air is thus entrapped above the water in the tank and is
compressed as the pressure rises. When the desired pressure is
attained, by adjusting the water regulating valve, this air cushion
enables it to be maintained with great constancy. In fact the
vessel plays the part, when used thus, of an ordinary air vessel.
This arrangement had to be adopted in the apparatus shown,
as the pressure in the ordinary water mains was very variable,
sometimes zero, and it was found necessary to put down a small
Worthington pump for supplying experimental water. The
tank air vessel effectually stops pulsations from the pump.
The jet is discharged from the left side of the vessel and is
caught by the sloping baffle plate shown and directed into a
rectangular tank below, whence it falls into the measuring tank
shown on the extreme left.

An actual experiment. Some results are given, obtained
with this apparatus, using a circular orifice in a tMn brass plate
0-25" diameter.

Experiment on the Discharge from an Orifice.


Quantity of
water Calculated
Pressure Duration quantity per
lbs. per
Head discharged
of test second
quantity per
square inch.
H feet. in time
t seconds.
second
t seconds =~ cubic feet. = 0-62 A V2gH
. .

Q cubic feet.

20 46 7-2 626 00115 00115


30 6912 7-2 510 0*0141 00141
40 92-16 7'2 445 0-0162 0-0163
50 115-2 7-2 400 0-0180 0-0182
60 138-4 7 2 365 0197 0-0200
HYDRAULICS. FLOW OF WATER. 273

It will be seen that the calculated results agree fairly well with
the experimental ones. The principal difficulty in carrying out
such experiments lies in the measurement of the dimensions of
the orifice. In the above experiments, the hole was fitted to a
standard " diameter cylindrical gauge, so that its diameter was
known with considerable accuracy. Other orifices which are
very interesting to experiment with, chiefly on account of the
beautiful forms the jet takes, are triangular and square shaped.
There is great difficulty, however, in obtaining their areas to
any degree of accuracy.

Flow over gauge notches. Water flowing along a stream
may be measured in two different ways depending on the
magnitude of the stream. If small, the most convenient method

Fio. 322. Arrangement of a weir for measuring the quantity of water flowing
along a stream.

is to place a weir across the stream with a gauge notch formed in

a thin plate at one part of the weir for the water to tumble over
(Fig. 322). From the head of water and the dimensions of the
notch, the quantity flowing may be calculated. If the stream is
large, then soundings must be taken across one section of it so
that the shape of the bed and the area of the cross section of the
stream may be found. The velocity is then measured at various
places by means of an instrument with blades like a propeller,
the revolutions of which depend on the velocity of the stream
and are registered by means of an attached counter. The
average velocity being found, this multiplied by the cross
sectional area will give the volume flowing per second. In the
case of a straight stream with an ordinary river bed, the
maximum velocity would be found near its centre, a little
below the surface.
A.M.B. s
274 APPLIED MECHANICS FOR BEGINNERS.

An experimental weir is easily arranged. In Fig. 321 the long


box has a brass plate bolted to its end with a notch cut in it.
Either of two plates can be fitted, one with a rectangular notch,
the other with a V notch, the angle of the V being 90 (Fig. 323).

Fig. 323.Rectangular and V Fig. 324. Baffle plates in the experimental


gauge notches. apparatus for stilling down eddies.

Water is supplied at the other end of the long box and has to
pass several battle plates arranged as shown in the section (Fig.
324), the last one being a sheet of wire gauze. These still down
all eddies, so that the water reaches the weir with steady motion.
After passing the weir, the water falls into
the measuring tank at the left-hand end of
the apparatus. The measurement of the
head of water above the lower edge of the
notch is effected by means of a hook gauge.
This consists of a round brass rod AB
(Fig. 325), having a hook of brass wire fixed
to it and brought to a sharp point at C.

Fig.
U
325. Hook gauge,
The rod A B can slide vertically in a tube at
D which is clamped securely to a fixed
support. The tube D is split at the top
along one side and has a vernier cut on
it,a scale of inches divided into tenths being cut on AB.
In rod AB is drawn up until the point at C just breaks
use, the
the water surface. From the scale reading the head of water
above the weir may be obtained. The hook gauge, both in the
experimental tank and in actual practice must be placed a
considerable distance from the weir, as the water surface always
slopes slightly downwards as the weir is approached.
=

HYDRAULICS. FLOW OF WATER. 275

Calculation of discharge. There is considerable difference


in the formulae used to calculate the flow over V and rect-
angular notches. In the case of the V notch, the stream
lines and the general shape of the jet remain similar to one
another with all heads, while in the rectangular notch a
change of head produces dissimilar
jets. Fig. 326 is from a photograph
of water falling over a triangular
gauge notch and Fig. 327 shows a
plan of the same. The curved paths
taken by the various particles remain
similar with all heads, and when this
is the case, Prof. James Thomson
has shown that the quantity flow-
ing depends on the | th power of
the head. A coefficient has to be
introduced from experimental data,
thus
Q = c(h)h
where
Q = cubic feet flowing per second,
h = head of water over notch in feet,
Fio. 326. Water flowing
over a V notch.
c coefficient of discharge = 2 635.

In the case of the rectangular notch, a portion near its centre


will have parallel stream lines as seen in the plan and elevation

Fig. 327. Fio. 328. Water flowing over a rectangular


notch. Two end contractions.

(Fig. 328) and near its edges A and B, curved stream lines.
Alteration of head produces alterations in the shape of these
stream lines, so that a formula has to be used containing a term
276 APPLIED MECHANICS FOR BEGINNERS.

applying to the straight portion in the middle and another term


applying to the contracting portion. Thus,

# = 3'33(Z- TV^)Ai,
where Q = cubic feet flowing per second,
L = length of notch in feet,
h = head in feet,
n = 2 for a notch as in Fig. 328,
n=\ Fig. 329,
n=0 Fig. 330.

n simply means the number of end contractions of the weir.


Evidently there are two in Fig. 328, one in Fig. 320, and none in
Fig. 330.

Fig. 329. Rectangular notch ; one Fig. 330. Rectangular notch ; no


end contraction. end contractions.

Flow through pipes. Let A and B in Fig. 331 be two tanks


at different levels, connected by a pipe the length of which is
L Let the water in
feet.
the tanks be preserved at
a constant difference of
level H feet. Water will
flow from A into B, losing
as it does so H foot-pounds
of energy per pound of
Fig. 331. Virtual slope of a pipe. water. This loss is made
up of three quantities :

(a) the kinetic energy possessed by the water entering B is


lost in producing surging of the water in B ;

(b) the water entering the pipe from A loses energy by the
production of eddies in the pipe ;

(c) energy is lost in overcoming frictional resistances to


motion in the pipe.
HYDRAULICS. FLOW OP WATER. 277

In practice (c) alone is considered, as the loss under this


heading in a long pipe is much greater than the others. The
loss of head per foot length of the pipe will be
.
%m H
T
This quantity is called the virtual slope of the pipe.
The flowwater in pipes has been shown by Prof. Osborne
of
Reynolds to be of two different kinds, steady flow and eddy
How. Up to a certain critical velocity, which depends on the
temperature, the flow is steady, and the resistance is pro-
portional to the velocity of the water at velocities above the ;

critical one, thewater breaks up into eddies and the resistance


is proportional to some power of the velocity, this power being

1*7 for very smooth pipes, 1'722 for lead pipes, and 2 for rough

pipes.
Many experimenters have observed the flow of water in
pipes and there are several formulae representing the results
obtained. That due to Prof. Unwin is

l
H cvi

~L~d^ ;

in which d is the diameter of the pipe in feet c = 0*0004, , = 1 '87 ;

and n 2 = l'4 for riveted wrought-iron pipes which are fairly


smooth and c 00007, /i x = 2 and %=1*1 for very rough pipes.
;

This formula may be used for pipes of from one foot up to four
feet in diameter.

Example. How much water will flow per second from a reservoir
through a pipe 1 foot diameter, 5000 feet long, the fall of surface
level being 25 feet ? The inside of the pipe is fairly smooth.
Unwin's equation gives
H_ 0-0004 xt-J
Z~ d^ '

OR
.= 0-005 =0-0004 xv 1 *,
500O

te
0-005
(o 0004/
W 7
'

logv=p^ log 12-5,


v = 3*4 feet per second.
;

278 APPLIED MECHANICS FOR BEGINNERS.

Let Q quantity flowing per second, cubic feet


A = cross sectional area of pipe, square feet.
Q-v x A
=3*4x--r-
4
= 3-4x0-7854
= 267 cubic feet per second.

In addition to the losses due to fluid friction in the straight


parts of a pipe, any sudden enlargement or contraction, or
bends in the pipe, will cause the flowing water to break up into
eddies, and will therefore produce further losses. Any change
of section should be gradual, and all bends should be made
very easy, merging gradually from the straight to the curved
portion of the pipe.
Expt. Arrange a horizontal glass pipe AC, Fig. 332, con-
tracted at B and having a short branch D
attached about the

Fig. 332. Apparatus showing change of pressure in water flowing along a pipe
of varying section.

middle of the contracted part. Let D dip into a vessel E


containing water coloured with red ink. Connect the end A,
of the horizontal pipe, by means of a rubber tube to a water
tap. The other end, C, should be over a sink, into which the
water passing along the pipe AC will be discharged. Now
turn on the water. In a few seconds steady flow will occur in
A C and it will be found that the coloured water in will pass E
up D and join the flowing water in AC. The explanation of
this is as follows. Since the water is flowing steadily in AC,
equal quantities must pass every section in the same time,
consequently the velocity of the water must be greater at B
than at A or C By the principle of the conservation of
energy, and neglecting frictional losses, the sum of the pressure
energy and the kinetic energy of a given quantity of the flowing
HYDRAULICS. FLOW OF WATER. 279

water must remain constant. But the kinetic energy is greater


at B than at A or C, for the velocity is maximum there, there-
fore the pressure energy must be less at B than it is at A or C:
Now the pressure at C is that due to the atmosphere, and
therefore, the pressure at B must be
less than atmospheric.
It follows that the pressure of the atmosphere on the surface
of the water in E will cause a flow from E up D.

Thomson's jet pump. This experiment illustrates the action
of Thomson's jet pump, a section of which is shown in Fig. 333.
Water enters at A, and the
reduced pressure in the con-
tracted part at B enables a
flow of water to take place
from a tank situated on a
Fio. 333. Diagram of Thomson's jet
lower level, to which the pump.
pump connected at D.
is

The mixed water is discharged into the atmosphere at C.

Conversion of the energy of water. Let us now consider


what happens when water falls
from a height into a pond of
water at rest. Suppose one pound
of water to overflow from a cistern
A (Fig. 334), of surface level H
1
feet above the surface level of a
pond B, into which the water falls.
In A, the pound of water possesses
potential energy equal to // ft.-lbs.
This energy is gradually changed
into kinetic energy during the
fall, until at the surface level of
B whole of the potential
the
energy has been transformed into
kinetic energy, which, if the
Fio. 334.
velocity of the water is v feet
2
v
per second there, will be equal to 5- foot-pounds.
2
v
Potential energy lost = kinetic energy gained, or H= ^-.
The water in B will be disturbed by the water entering it, but.
280 APPLIED MECHANICS FOR BEGINNERS.

presently quietens down again, that is, the whole of the kinetic
energy of the pound of water has been' dissipated in creating
disturbances in B, and none has been utilised in producing
useful work. Useful work may be derived from the H
foot-
pounds of potential energy available by permitting the water to
descend through a pipe, thereby producing pressure energy at
the level of B, which may be converted into mechanical work by
driving the pistons of a water engine. Or, the energy available
may be utilised by means of a water-wheel of which there are
three varieties over-shot, breast-shot, and under-shot.
In the over-shot wheel (Fig. 335) water is brought to the top
of the wheel, which has buckets fastened all round its rim the
;

water enters these buckets


and remains in them until
the wheel, turned by the
extra weight of water on -

one side, has brought the


buckets into such a position
that the water is spilled
out. In large wheels such
as this it is usual to have
teeth secured to the inner
rim of the wheel and the
power is taken from the
wheel by a shaft carrying
a pinion gearing with these
Fig. 335. Over-shot water wheel. teeth. This is to prevent
the large stresses which
would be introduced into the arms of the wheel if the power
were taken from the shaft on which the wheel turns.
In breast-shot wheels the water enters the buckets half-way up
and remains in them until the bottom of the wheel is nearly
reached, when it is spilled out.
In under-shot wheels the water is allowed to acquire as much
velocity as possible before reaching the wheel and is then
allowed to impinge on the blades. In this last case the change
is from kinetic energy to mechanical work, in the others the

mechanical work is done directly by the gravitational effort on


the water in the buckets.
HYDRAULICS. FLOW OF WATER. 281

Turbines are machines used for converting the energy of water


coming from a height into mechanical work. These are of two

kinds one in which the energy of the water is partly pressure
and partly kinetic in passing through the machine, these being
called reaction turbines and another kind in which the energy
;

of the water is wholly kinetic on reaching the machine, these


being called impulse turbines.
The turbine consists of a wheel having blades running in a
casing furnished with guide blades. The entering water is
guided by these blades so as to have tangential velocity and
consequently tangential momentum. This momentum is
abstracted during the passage through the wheel by the action
of the curved blades on the wheel. Consequently, pressure is
exerted on the rotating wheel, and work is done thereby. Re-
action wheels, in which the water has its energy partly in the
pressure form, must run full of water in impulse wheels, on
;

the other hand, the pressure of the water is atmospheric or


nearly so, and the water slides along the blades in compara-
tively thin streams.

Fio. 336. Diagram of Thomson's turbine.

In Thomson's turbine (Fig. 336) the water enters the wheel at


its outer circumference, being guided by four blades which may
282 APPLIED MECHANICS FOR BEGINNERS.

be adjusted to suit varying quantities of water passing it then ;

passes through the wheel, moving inwards, and is discharged at


the inner circumference. The shape of the guide blades and of
the wheel blades at the outer circumference is such that the
velocity of the entering water relative to the wheel is along the
wheel blade consequently the water enters without shock. At
;

the inner circumference the shape of


the blade is such that the water leaves
with radial velocity only.
Wheel having radial blades.
Suppose we have a wheel with radial
blades, as in Fig. 337, and that the
velocity of the blade at A is V. Let
AB be the direction of the guide blades,
then the direction of the water moving
towards the wheel will be along BA.
For the water to enter the wheel
without shock, its velocity relative to
337. Turbine wheel hav- the wheel must be along AO. Give
ing radial blades.
the wheel at A and a particle of water
just leaving the guide blade each a velocity equal to F, to the
right. This will stop the wheel. Let P (Fig. 338) be the
particle of water. It will now have
a velocity V represented by PC, to
the right, and a velocity Vw along
the direction of the guide blade.
The resultant of these is Yn repre-
sented by PD, and this will be the
relative velocity of water and wheel blade. PD will evidently
be along the blade at A if we give the proper value PE to
Vw the velocity of the water along the guide blade.
Wheel having curved blades. A similar construction will
give the proper velocities if the wheel blade is curved at A
instead of being radial. This is shown in Figs. 339and 340.
PD equal to Vr is here the relative velocity of water and wheel,
and is directed along the blade at A. PE equal to Vw is the
velocity of the water along the guide blade. At 0, the outlet,
the relative velocity of the water and the blade must be along the
blade, that is, along FO, and for radial discharge, the direction
HYDRAULICS. FLOW OF WATER. 283

of this velocitymust be chosen to suit the speed of the wheel


blade there. Let V be the tangential velocity of a point on
the blade at 0, and let this
be represented by PG in /
Fig. 341. Stop the wheel as
before, by giving P, and a
particle of water at P,
velocities equal to Vq, but
in the opposite sense ; this
is shown by PH equal to
VQ . A particle of water at
P will now have a radial
velocity v represented by
PK at 90 to and also PH
a velocity V represented
Fiu. 339. Turbine wheel having curved
by PH. The resultant of blades.
these, Vr equal to PL,
will be the relative velocity of water and blade at P, and
consequently the direction of the blade at the inner circum-
ference must be along
p y
LP. =~C
Horse-power of
wheel. The entering
velocity Vw being now
known, and the exit velocity being radial, we may easily find the
momentum changed by passage through the wheel. Thus, let
D

Pig. 341. Fio. 342.

BA, equal to V^ be the velocity of the entering water in Fig. 342 ;

resolving this into tangential and radial components, we find


284 APPLIED MECHANICS FOR BEGINNERS.

the tangential velocity Vt represented by CA. This disappears


on passage through the wheel, consequently the tangential
momentum changed per pound of water is equal to 1 x Vt .

Let m lbs. of water pass per second, then


Change of momentum per second = m V and t,

Force given to wheel at A in consequence of this = -lbs.

Therefore, Work done per second = -. Fft.-lbs., where Fis,

wheel at A, and
as before, the velocity of the

Horse-power = t V
J mV

60
. .

33,000
Example. Suppose 500 lbs. of water per second to be delivered
to a wheel with a tangential velocity of 40 feet per second. The
velocity of the wheel rim is 35 feet per second. The water leaves
the wheel radially. What horse-power can be developed ?
500 x 40
Pressure on wheel
32 2
500x40
Work per second

H.P. = 39-5.

Fig. 343. Diagram of the Pelton wheel.

The Pelton wheel is an example of an impulse wheel. It con-


sists of a wheel running in an outer casing (Fig. 343), and
HYDRAULICS. FLOW OF WATER. 285

having blades or buckets arranged round its rim. A jet of


water impinges on the buckets, gives up its momentum to the
wheel, and escapes by a pipe from
the lower part of the casing.
The shape of the buckets is shown
in plan in Fig. 344 the jet *
; r - .=^==^7=
impinges centrally, divides and
circles round the curved bucket,
fc

and is then discharged. The fio. 344. -Plan of bucket,


shape is semi-circular in plan, and
is such that the maximum amount of momentum is abstracted
from the water. If the bucket were at rest, the water would
be directed backwards with a velocity equal to its original one,
V|. The whole change of momentum per pound of water would
be 2 x v Xi and the pressure on the bucket due to this would be

2#
- lbs. If the wheel had such a speed of rotation that the
9
was equal to that of the jet, no momentum
velocity of the bucket
would be changed, and the pressure would be zero. In either
case, no work would be done. At a speed of rotation such that
the buckets have a velocity half that of the jet, these con-
ditions giving the theoretical maximum efficiency, the water
would leave the bucket with little or no velocity relative to the
earth, and consequently would have a maximum quantity of
energy abstracted from it. The whole momentum 1 x \\ of a
pound of water in the jet would be changed. The efficiency of
such a wheel would be 100 per cent., only the imperfect action
of the water reaching the buckets, due to their different inclina-
tions caused by the rotation of the wheel, and the interference
of one bucket just entering the jet with the supply going to
another, prevent this. An efficiency of about 80 per cent, can
be attained.
Experimental Pelton wheel. In Fig. 321 a small Pel ton
wheel may be observed at the extreme right of the apparatus.
This is connected to the upright tank so that water can be
supplied to it at any pressure up to 100 lbs. per square inch.
A speed counter driven by a small worm and worm wheel
counts the revolutions of the wheel. A pulley on the wheel
shaft has a brake fitted to it by which the horse-power of the
286 APPLIED MECHANICS FOR BEGINNERS.

wheel can be measured. The water is discharged from the


wheel casing, when done with, into the trough below, whence
it finds its way over the gauge notch into the measuring tank

on the left of the apparatus. As both the head of water and


the speed of the wheel can be altered independently of one
another, this forms a very useful experiment for the student.
The records of a test on this wheel with constant head are
given.

Test on a Pelton Wheel.


Diameter of bucket wheel 5|" to centres of buckets.
Diameter of brake wheel 6" to centre of cord.
Pressure of constant water supply 40 lbs. per sq. inch, giving
head = H=
92'16 feet.
The theoretical velocity corresponding to this head would be

v= s/2gH=76'S feet per second.

The actual velocity of the jet was obtained by measuring


the water used by the wheel per minute this amounted to
;

91-5 lbs. = W.
Water per second = 0*0244 cubic feet.
Diam. of tapered nozzle =0'25".
Area of jet = 0'000341 sq. feet.

]\ = Velocity of jet=
n ^^^ = 71_5 feet per second.

This latter value has been used in working out the results.

V2 = velocity of bucket

= x revolutions of wheel per second,


J.3S

= 1 '5 x JV feet per second.


W77 = energy supplied per minute = 91 5x92"16 ,
ft. -lbs.

H.P. supplied in water = rr^ =0-278.

HP - (P-W)2*R.N _ 1'57
***
B y
33,000 "33,000^ V
= 0-000048 (P- W)A7 .
HYDRAULICS. FLOW OF WATER, 287

Results of Test.

Brake loads. Efficiency


Speed of wheel Velocity of Brake Ratio
B.H.P
revolutions bucket, horse-
per min., N. V., ft. per sec. power. H.P. supplied
V\
Plbs. Wlba. X100.

650 162 201 575 0-117 0-227 421


800 20 1-51 5-00 0134 0-28 48 3
1000 25 I -01 4-25 0-155 0-35 55-9
1150 2874 0-71 3 50 0-154 402 55-5
1200 30 0-51 3 00 143 0-42 51-5
1400 35 0-21 2 00 0120 0-49 43-2
1480 37 on 1-75 I
0-116 0-518 41-8
1

The speed of the wheel was adjusted by varying the brake


load, and as the conditions, when altered, settled down at once,

BMP.

H / ~\\
<H*
/
1
ft

)
\\
M
/
/
\

\
M

(HO

M
200 400 600 800 1000 1200 ItOO 1600*0?

Fig. 345.Test on a Pelton wheel ; curve of B.H.P. and revolutions.

each test lasted only about 3 minutes. Fig. 345 shows a plotted
curve of B.H.P. at the different speeds of revolution, and in
Fig. 346 the efficiency of the motor for the varying values of

has been plotted. It will be noticed that maximum


vx
288 APPLIED MECHANICS FOR BEGINNERS.

efficiency is obtained under the given conditions when the


speed of the bucket is about 0*38 that of the jet.

fin

.V) r\
u / \\
M
/

\v
\
70
/ S
N
i

HI
/

87 M 03 04 OS Of 07
\0-9 I0 V

Fig. 346. Test on a Pelton wheel ; curve of efficiency for ratios

Centrifugal pumps. In centrifugal pumps, water enters the


centre of a wheel furnished with blades. The water enters as
nearly as possible with radial velocity only, and the blades are
so shaped as to gradually give rotational velocity to the water as
it passes outwards. This, of course, simply means that a further
store of kinetic energy is added to the water while passing

through the wheel. The water escapes at the outer circum-


ference of the wheel into a large circular chamber, called the
whirlpool chamber. Here its velocity is allowed gradually to
diminish, with the effect that its pressure energy increases. It
will nowconsequently be able to overcome the resistance of a
considerable head of water and will be able to flow up a pipe
into a tank above. Centrifugal pumps deriving their supply of
water from a reservoir on a lower level than the pump require
a foot valve on the pipe in the reservoir and the supply pipe
and pump must be charged with water before starting. When
the pump the increase of kinetic energy of the water
starts,
near the wheel centre causes the pressure energy to diminish
there, the pressure consequently falls below atmospheric, and
HYDRAULICS. FLOW OF WATER. 2S9

the pressure of the atmosphere on the surface of the water in


the reservoir causes it to flow up the pipe into the wheel. At
A energy of a pound of water
(Fig. 347) therefore, the is wholly
pressure energy and is that due to
atmospheric pressure. At B, the wheel
centre, the energy is partly potential,
due to the elevation AB partly ;

pressure, but less than that due to


atmospheric pressure and partly ;

kinetic. The wheel adds largely to


the kinetic energy, and in the whirl-
pool chamber this partly is changed
again into pressure energy. At C, the
discharge pipe entrance, the energy is
largely pressure energy, partly poten-
tial due to the elevation A C and partly
kinetic, this last being due to the
velocity in the discharge pipe necessary
to produce flow. It should be observed
here, that if the discharge pipe is of
too great height, the whole of the
energy of the water at C may be made
up of potential and pressure forms
only, caused by the head being too
great. The absence of kinetic energy
means that there will be no velocity
of flow and consequently no water will Fig. 347. Arrangement of a
i j- t. t x "
ax.
" i i centrifugal pump.
be discharged. Lowering the level of
D even a small amount may correct this and give a very good
efficiency where previously there was none. The potential
energy of the water gradually becomes greater as it ascends the
discharge pipe and if the pipe is of uniform section, its velocity
will remain constant and therefore its kinetic energy also will be
constant. The gain of potential energy, is therefore at the
expense of pressure energy, which accoidingly becomes less.
Finally, the water is discharged with potential energy, pressure
energy due to atmospheric pressure, and kinetic energy, which
last is soon dissipated in the upper tank in surging of the water.
The total gain of energy due to the whole arrangement will be
A.M.B. T
;

290 APPLIED MECHANICS FOR BEGINNERS.

simply the difference between the potential energies of a pound


of water in 2? and in A, that is, IT foot-lbs.

EXERCISES ON CHAP. XVIII.


1. Water
discharged through a circular orifice in a thin plate
is
under a head of 60 feet. If the diameter of the orifice is 1", what
quantity will be discharged ; give your answer in gallons per
minute.
Water flows over a V-notch, the angle of which is 90 if the
2. ;

head what quantity passes per minute ?


is 15",

3. A rectangular gauge notch is 2 feet wide. The head of water


measured by hook gauge is 5" calculate the quantity of water
;

flowing per minute. There are two end contractions.


4. A pipe 24" diam. is 2500 feet long and has a fall of level of
20 feet. What probable quantity of water will flow per hour?
State the answer in gallons.
5. 4 cubic feet of water per second enter an overshot wheel
whose diameter is 40 feet. Taking an efficiency of 65 per cent.,
what horse-power can be obtained from the wheel ?
6. A wheel has radial blades, and water flows through it from
outside to centre. The speed of the wheel at the outer rim is
40 feet per second and the radial velocity of the entering water is
8 feet per second. Find, and show in a diagram, the actual velocity
of the entering water, if there is to be no shock.
7. In Question 6, what will be the tangential velocity of the
water leaving the wheel blades if the radius there is one half the
outer radius ? Supposing the radial velocity of the water there is
8 feet per second, show in a diagram what alteration must be made
on the blade in order to discharge the water radially ?
8. The radial velocity of water in a centrifugal pump wheel is
2 feet per second ; the vane makes an angle 35 with the outer
circumference what is the velocity of the water relatively to the
;

wheel? (A graphical method of solution may be used.) (1900.)


9. A horizontal pipe of 12 inches diameter gradually becomes
of 3 inches diameter and then becomes of 12 inches diameter again.
There is a flow of water of 5 cubic feet per second. Neglecting
friction, calculate and state how the pressure alters along the axis
of the pipe. (1901.)
10. Ten cubic water per second enters a turbine wheel
feet of
with a tangential velocity of 50 feet per second it enters without
;

shock, the velocity of the rim of the wheel being 50 feet per second;
the water leaves the centre of the wheel with only a radial velocity
what energy does the water give to the wheel per second? (1901.)
CHAPTER XIX.

MATERIALS.

Iron in its many forms the metal chiefly used by the


is

engineer and builder. It reduced from its ores in the blast


is

furnace, the resulting product being called pig iron. Pig iron
contains from 2 to 5 per cent, of carbon, which may be simply
mechanically mixed with the iron, such iron being called grey
iron, or the carbon may be in combination with or in solution in
the iron, the iron being then known as white iron. Grey iron is
used for foundry purposes, as it gives good castings. White
iron is brittle and very hard. It fuses at a lower temperature
than grey iron, but assumes a pasty condition before becoming
quite liquid. White iron is used for wrought iron and steel
production.

Cast iron. Casting into the required forms is performed
from a furnace called a cupola. The pigs are melted in this
furnace, different grades being combined in certain proportions,
and the resulting mixture is cast into sand or loam moulds
having the desired form. Rapid cooling after the iron is in the
mould tends to produce white iron hence, when castings are
;

required to be very hard they are cast into moulds chilled by


circulating water and are known as chilled castings. Cast iron
is a crystalline metal, weak under tension and strong under com-

pression. It lacks ductility and cannot be welded.


Wrought iron is simply iron, as nearly pure as possible,
which has been manipulated so as to produce & fibrous structure
instead of a crystalline. It is produced from pig iron by a pre-
liminary refining process in which many of the impurities are
removed. Fusion in a puddling furnace follows, the iron being
292 APPLIED MECHANICS FOR BEGINNERS.

removed from the furnace in a pasty condition to be hammered


or squeezed. This process gets rid of slag and consolidates the
mass. The iron is then rolled into puddled bars. To improve
the quality, these bars are cut into short lengths, bundled, heated,
and hammered together, the quality of the iron being improved
with each repetition of these processes. The iron is then rolled
into the market forms of plates and bars. Wrought iron is a
ductile, fibrous metal, easily malleable, and can be welded.
Steel is iron as nearly pure as possible with the addition of a
small quantity of carbon, the proportion varying according to
the purpose for which the steel is intended. Mild steel, which
closely resembles wrought iron, may contain from 0*15 to 0*25
per cent., and hard steel up to 1*5 per cent, of carbon.
The finest qualities of hard steel are produced by the cementa-
tion process, which consists in baking high quality wrought-iron
bars in contact with charcoal for about a fortnight. Carbon
from the charcoal enters into the iron during the process, con-
verting it into blister steel. The process may be stopped at
different stages, giving steel suitable for the manufacture of
springs, tools, cutlery, etc., by subsequent reheating, rolling,
and hammering. Cast steel is produced by remelting blister
steel in crucibles. A more uniform metal results than can be
obtained by hammering.
In the Bessemer process steel suitable for rails, etc., is produced
direct from pig iron containing silicon, by burning off the
carbon and impurities in a converter, a powerful blast being
supplied. Spiegeleisen (pig iron containing manganese and a
large proportion of carbon), or ferro-manganese, is added to
bring the carbon to the right proportion, and the metal is then
cast into ingots, being afterwards rolled into plates or bars of
the desired form. A " blow " in the Bessemer process occupies

about 30 minutes.
In the Martin process wrought iron or steel scrap and pig iron
are melted together to produce steel ; in the Siemens process
pig iron and ore are used in the Siemens-Martin process, which
;

is a combination of the two, scrap iron, pig iron, and ore are

used together. These processes are conducted in a Siemens


regenerative furnace, and are known as open-hearth processes.
The steel is cast from the furnace into ingots, which are then
MATERIALS. 293

cogged, or hammered, to consolidate the metal, arid sheared and


rolled into plates and bars.
The process by open-hearth may occupy
of steel production
from 9 to 12 hours, and the whole being under perfect control,
steel can be produced to specification.
(
The mild steel or ingot
iron produced differs from wrought iron only in being more
homogeneous, having been produced from a cast ingot instead of
from a puddled ball containing slag. It may contain the same
proportion of carbon as wrought iron, and has a, greater strength
and is more ductile. It also forges well.

Hardening and Tempering. Steel containing 0-5 per cent,
and upwards of carbon can be hardened by dipping it when hot
it suddenly.
into water, or other liquid, so as to cool It is then
very hard and brittle and not suitable for many purposes, but
can be tempered by carefully reheating it to certain tempera-
tures, when it loses its brittleness and some of its hardness.
The temperature at which steel acquires a temper suitable for
various purposes is practically ascertained by the colour of the
skin of oxide which comes over its originally clean surface
during reheating. These colours and approximate temperatures
are :

Colours asd Temperatures during Tempering.

Temperature. Colour. Suitable for

220 C. Faint yellow. (Very hard. ) Surgical instruments.


230 Straw. Razors.
245 Dark straw. Wood-working tools, taps, and dies.
255 Brownish yellow. Chisels.
265 Purplish brown. Axes, planes, chipping chisels.
275 Purple. Table knives.
285 Light blue. Swords, springs.
295 Dark blue. Fine saws, augers.
315 Blackish blue. Large saws.

The ordinary workshop method of tempering a turning tool,


or chisel, is to heat it to a bright red, then plunge about one
inch of the point into water till withdraw
this portion is cold,
and clean the point with a piece emery the
of sandstone or ;

heat travels down into the point from the hot body of the tool
294 APPLIED MECHANICS FOR BEGINNERS.

and the colours of the oxide formed on the point are watched.
When the proper colour has been attained, the whole tool is
dipped again into water and held there till cold.
Malleable castings are produced from the ordinary cast-iron
castings by heating them for several days in contact with some
substance, such as red haematite, which will remove the carbon.
Case hardening consists in giving a surface of steel to wrought
This process is effected by heating the articles in
iron articles.
contact with some substances, such as charcoal, leather, and
ferrocyanide of potassium, which will give carbon to the iron.
The operation resembles the cementation process, but is not
allowed to go so far, as only a thin layer, J" to J* thick, of steel
is required.
Copper is largely found native, and is also produced by
reduction from its ores. It is much used in sheets, bars, and
as wire, for fire boxes, sheathing, tubes, boiler stays, nails,
electrical conductors, etc. Copper castings are not much used,
being poor and expensive. Copper is strong and malleable, is
easily wiredrawn, and can be made up into many forms by
hammering or brazing. The metal is best forged at a moderate
red heat and has its strength improved by the process. It is
hardened by working, but may have its ductility restored, and
at the same time its strength reduced, by heating and quenching
in water. When pure, copper has a high electrical conductivity.
It is a good conductor of heat and has a high power of resisting
corrosion by air and water.
Till not found native, being generally extracted from its ore
is

tinstone. Its strength and ductility are low, but it is very


malleable and is easily made into foil tin-foil.
It is very
brittle when at temperatures near its melting point. When
pure tin is bent, it makes a crackling noise. The metal does
not tarnish much at ordinary temperatures, and its melting
point is low. The alloys of tin with other metals are valuable,
and the metal is much used for coating other metals to protect
them.
Thin iron plates are coated with tin by first undergoing a
process of scouring and pickling in order to thoroughly clean
their surfaces and then dipping in a bath of molten tin. The
resulting tin plate preserves well so long as the tin coat remains

MATERIALS. 295

unbroken, but rusts very rapidly directly this is damaged, on


account of the galvanic action set up. Copper and brass are
often tinned.
Zinc is obtained from its ores by a process of distillation. It
is brittle malleable and can be
at ordinary temperatures, but is

rolled into sheets at temperatures from 200 to 300F. At


temperatures about 400F. it becomes brittle again. Zinc gives
good castings suitable for art work. Its principal use in
engineering is for coating iron plates in order to protect them,
the resulting sheets being known as galvanised iron. The
protection afforded by zinc is more effective than that by tin.
Lead is reduced from its ores by treatment in a reverberatory
furnace connected to a long flue in which the lead particles are
condensed. Lead is very soft and heavy. Its tensile strength
is low, and it does not give good castings. The metal is easily
rolled into sheets and made into pipes. Lead pipes are some-
times lined with tin in order to prevent the formation of
poisonous salts under the action of water. Lead is used for a
variety of minor purposes in engineering.
Aluminium is a very widely distributed metal, and forms the
base of clay. Aluminium resembles zinc in colour and hard-
ness ; it is very light, having a specific gravity about one-third
that of wrought iron. As the tensile strength of aluminium is

also about one -third that of wrought iron, it follows that


wrought iron and aluminium, weight for weight, have equal
tensile strengths. The metal is easily rolled into sheets, and
can be wiredrawn, forged, and cast. A
skin of oxide forms
very rapidly on the surface of the metal, preventing further
corrosion, but at the same time making it very difficult to
secure satisfactory soldered or brazed joints. Aluminium alloys
readily with other metals, and the alloys are very valuable.
The strength of the metal is increased by cold hammering.
Copper alloys with zinc are called brasses, with tin bronzes.

Brass. Ordinary brass is made of two parts of copper to one
part of zinc by weight. A higher proportion of copper gives a
better metal. Lead, present in small quantities, gives an alloy
easier to machine, but too much lead produces brittleness.
Sterro metal consists of copper and zinc in about the same pro-
portions as for brass, with the addition of small quantities of tin
296 APPLIED MECHANICS FOR BEGINNERS.

and iron. The metal is and non-porous.


strong, non-corrosive,
Delta metal consists of brass with small quantities of iron and
phosphorus. The metal can be wrought both hot and cold, and
can be wire drawn. Muntz metal contains 66 per cent, copper,
33 per cent, zinc, and 1 per cent. lead. It can easily be rolled
into sheets.
Bronzes. Gun -metal consists of 90 per cent, copper and
10 per cent. tin. This metal is much used by engineers for
bearings, cocks, valves, etc. good castings. Aluminium
It gives
bronze contains 90 per cent, copper and 10 per cent, aluminium.
The metal can be forged, and is very durable. Phosphor bronze
consists of any brass or bronze of copper, tin, and zinc, with the
addition of phosphorus. The metal is very strong. Manganese
bronze contains 88 per cent, copper, 10 per cent, tin, and 2 per
cent, manganese. Its strength and ductility are about the same
as those of mild steel. The metal can be forged well when hot,
and may be rolled into sheets.

Building stones. The commonest stones used for building
purposes are granites, sandstones, and limestones. These stones
are derived from rocks which have either been first in a state
of fusion under the action of heat and have then consolidated,
or have been deposited under water and stratified, that is, built
up of layers. Granite is unstratified, and is composed of quartz,
felspar, and mica. The best quality is very strong, hard, and
durable, and is much used in engineering work. The stone
takes a high polish. Sandstone consists of small grains of
quartz cemented together. As the quartz is practically
indestructible, the strength of the stone depends on the nature
of the cementing material, which in the best stone is silica, and
in the worst
alumina. Limestones vary in compactness from
chalk to marble. Portland stone and Bath stone are prized for
their durability.
Bricks are made from clay which is dug in the autumn and

cleared of gravel, by hand picking, or, if much gravel is


etc.,

present, crushing between rollers is necessary. The clay is left


over one or more winters, so that frost may disintegrate it.
Tempering follows, consisting of working the clay with the
spade or by machine. The prepared clay is then moulded into
bricks, air dried, and burned either in stacks or kilns.
MATERIALS. 297

Lime is made from limestone, or other mineral containing


carbonate of lime, by calcination in kilns to drive off water and
carbonic acid, the resulting material being called quicklime.
Quicklime, when sprinkled with water, swells and breaks up
into powder, and heat is evolved. This is called slaking. This
powder which results, when made into a paste with water and
left, soon hardens, or sets, the operation consisting in the absorp-

tion of carbonic acid from the air, thereby reconverting the


material into carbonate of lime.
Cements artificial. The most important
are either natural or
natural cement Roman cement, which is made by calcination
is

from nodules found in London clay. It sets quickly but is not


strong. Portland cement is artificial, and is made generally from
chalk and clay by mixing in suitable proportions, burning in
kilns, and then grinding to a fine powder. Portland cement is
much used by the engineer and builder. Sand is often mixed
with cement for building purposes. The strength of Portland
cement diminishes as the proportion of sand is increased. The
action of the cement is to form a binding material uniting the
grains of sand together, consequently the cement and sand
should be thoroughly mixed both dry and wet. The strength
of Portland cement alone, and also mixed with sand, gradually
months after setting.
increases for several
Concrete is made from ballast, broken bricks, etc., by
mixing with cement or lime, and water. It is much used for
foundations.
Timber is obtained from exogenous trees, that is, trees in
which the growth takes place by successive additions on the
outside of the woody matter already formed. The oak, fir,
beech, etc., are examples of these. Endogenous trees, such as
palms, are unsuitable for timber, as their structure consists of
independent fibres cemented together, thus making the wood
too flexible for structural purposes.
If the section of the trunk of a tree suitable for timber be
examined, the following facts may be noticed. The pith
occupies the centre or nearly the centre ; outside of the pith is

seen a broad ring of fully-formed and matured wood called the


heart wood ; outside of this, again, sap wood occurs, and then a
thin layer of slimy matter called cambium between the wood

298 APPLIED MECHANICS FOR BEGINNERS.

and the bark, and outside of all the bark. The woody matter is
made up of more or less distinct rings, one of these being added
each year in our climate, hence the name annual rings.
Radiating from the centre may be noticed narrow strips called
medullar!/ rays.
The growth of the tree takes place by the ascent from the
roots of water and mineral salts, this ascent being promoted by
the medullary rays ; these substances on reaching the leaves,
and being acted on by the constituents of the atmosphere in
the presence of sunlight, take in carbon from the carbonic acid
of the atmosphere and then descend to the cambium, which
and transforms the food supplied from
consists of active cells,
the leaves into woody matter to be added to that already
existing.
Timber is best felled in winter, as the resulting wood is
better than that felled during active growth in the warmer
months. It is converted after felling into forms suitable for
the market, and seasoned to remove the sap and water which
would, if left, destroy it.
Seasoning may be done by natural means, viz., stacking the
timber in a dry sheltered place and providing for ample ventila-
tion of dry air, or artificially by hot air, or otherwise. Timber
is liable to crack or develop shakes during seasoning, such as

heart shakes extending radially from the outer parts, star shakes
radiating from the centre, and cup shakes in which separation
occurs along the circumference of the annual rings.
Timber also shrinks in seasoning, chiefly circumferentially
along the annual rings, not so much
radially and very little
in the direction of the length. Knowledge of these facts is
taken advantage of in the conversion, by cutting up the stem of
the tree in such a manner that shrinkage affects the pieces in
the least harmful way.
Timber is liable to dry rot, in which case the woody structure
becomes powdery. This fault arises from want of sufficient
ventilation. The growing tree is often injured by attacks of
insects and certain plants, and the timber from them also, is
liable to insect attacks after the structure is erected.
Timber is preserved by painting or tarring, but the wood
must be thoroughly dry first, or the moisture is simply con-
MATERIALS. 299

fined by the coating and the wood will rot. Creosoting is the
most effective way of preserving timber. It is conducted in a
closed vessel, the timber being first subjected to a partial
vacuum, to withdraw air and moisture from the pores, and then
tarry oil mixed with creosote is forced in hot, under a pressure
of about 170 lbs. per square inch. The creosote fills all the
pores and the oily matter remains also as a coating to the
woody fibres, protecting them from damp. Properly carried
out creosoting is very effective.
For our purpose, timber may be divided into hard woods and
soft woods.
Hard woods. Oak one of our strongest and most durable
is

timbers. It does not warp much


after it is thoroughly dry
and if cut properly along the medullary rays shows a beautiful
silver grain much prized for ornamental purposes. The acid
present in oak corrodes metal fastenings, and oak should there-
fore be put together with wooden pins called trenails. The
timber may be used in wet positions.
Beech also corrodes metal fastenings it stands well if kept
;

constantly dry or constantly wet, but will not preserve its


shape with alternate wetting and drying. It can be used for
positions under water.
Ash is very tough and flexible, and consequently is useful
for structures, etc., subject to shocks.
Elm also will not stand alternate wetting and drying, but
if constantly in one or the other condition it can be preserved.
It warps greatly, is strong against crushing, and is also strong
across the grain.
Mahogany can be preserved well dry and does not warp or
shrink much. It takes metal fastenings and glue well.
Teak is a very strong and durable timber for engineer's work.
It takes metal fastenings well, is very stiff, and resists insect

attacks.
Greenheart is very strong, heavy and hard. It burns freely.

Soft woods. Northern pine, also called red or yellow fir,
comes from Scotland, Russia and the Baltic. When of good
quality the timber is strong and durable and is much used for
carpenter's work.
American yellow pine works very easily and is much used for

300 APPLIED MECHANICS FOR BEGINNERS.

Table of Ultimate Strength of Materials.

Tensile Compressive Shearing


strength, strength, strength,
M.tt 6 rial.
tons per tons per tons per
sq. inch. sq. inch. sq. inch.

Cast iron, .... 5 to 15,


average 8
25 to 65 6 to 13

Wrought iron
Tested in direction of rolling, 20 to 29 )
Tested across direction of Il6to20 22

Mild
Cast steel,
rolling,
steel, ....
....
19 to 24
27 to 32
35 to 70
I
21 to 25

Copper, cast, . 8 to 12
,, rolled, 15
,, wire (hard drawn), 28
Tin, 2
Zinc, cast,
,,

Lead
rolled, ....
. 1 to 3
8 to 10
1

Aluminium, cast, 5
,, rolled, 6 to 10
Brass, ordinary,
,, wire,
Sterro metal,
....
.
11
20 to 25.
35
Delta metal, cast, . 22
,, forged, 34

Muntz metal,
Gun metal,
,,
....
wiredrawn,

....
55
22
15
Aluminium bronze, 40
Phosphor bronze, annealed, 25
',, unannealed, up to 70
Manganese bronze,
Granite,
Sandstone,
.....
... . 28
6 to 10
2 to 5
Portland stone, 2
Brick, London stock, 1

Pine,
,,
.....
Staffordshire blue),
5
2 to
2}
6
"i
Oak
Leather, ..... 7
2
H
)

MATERIALS. 301

internal work. It takes glue but not nails well, and is durable
in dry climates.
Pitch pine contains much resinous matter, is durable, but
difficult to work. It rots soon if not kept dry.
Spruce or white deal is tough, but is much subject to warping
and shrinkage, and is inferior to red deal in strength ; it also
breaks easily under shock.

EXERCISES ON CHAP. XIX.


1. Describe very briefly the differences in composition, properties
and uses, of cast iron, wrought iron, mild steel, tool steel, and any
three alloys of copper. (1896.)
2. What is the difference between hardening and tempering a
piece of steel ? Describe the process of hardening and tempering a
chisel for cutting wrought iron. In what order do the colours
successively appear during the process of tempering? (1897.)
3. Choose some cast iron object, and explain why, and where, it
may have initial strains and weaknesses. (1898.)
4. What is a chilled casting? A malleable casting? How is
each produced? Describe how a wrought, iron body is case-
hardened. (1898.)
5. Describe the manufacture of any kind of steel describe its ;

chemical composition and physical properties. (1901.)


6. How is cement made ? What is your notion of what occurs
when it sets and gradually hardens? What is the effect of the
addition of sand ? ( 1898.
COURSE OF LABORATORY WORK.

General Instructions.
Two Laboratory Note-books are
required ; rough notes of the experiments should be
in one
made, and in the other a fair copy of them in ink should be
entered.
Before commencing any experiment, make sure that you
understand what its object is also the construction of the
;

apparatus and instruments employed.


Eeasonable care should be exercised in order to avoid damage
to apparatus, and to secure fairly accurate results.
In writing up the results, enter the notes in the following
order :

(1) The
title of the experiment and the date on which it was
performed.
(2) Sketches and descriptions of any special apparatus or
instruments used.
(3) The object of the experiment.
(4) Dimensions and weights required for working out the
results'; from these values calculate any constants required.
(5) Log of the experiment, entered in tabular form, together
with any remarks necessary.
(6) Work out the results of the experiment and enter them
in tabular form.
(7) Plot any curves required.
(8) Work out the equations for the curves where possible.
Notes should not be left in the rough form for several days ;

it is much better to work out the results and enter them directly
after the experiments have been performed.
COURSE OF LABORATORY WORK. 303

MEASUREMENTS.
1. Using an ordinary caliper and steel rule, find the dimensions of
the pieces of bar metal provided.
2. Find the breadths and thicknesses, or the diameters, of the
same pieces of material as in Expt. 1, using a micrometer caliper.
First find the zero error, if any, of the instrument and correct your
readings accordingly. pp. 3-5.
Repeat Expt. 2, using a vernier caliper.
3. pp. 5, 6.
4. Weigh the same pieces of materials calculate their volumes
;

from the dimensions obtained in Expts. 2 and 3 ; then calculate the


specific gravities of the materials. p. 16.

FORCES. SIMPLE STRUCTURES.


5. Parallelogram of forces. Apparatus as in Figs. 33, 34. Per-
form the Expt. as directed. p. 24.
6. Forces acting on a pendulum. Apparatus as in Fig. 39.
Perform the Expt. as directed. Confirm each reading of P and T
graphically by the parallelogram of forces. p. 28.
Forces in a simple roof truss. Apparatus as in Fig. 42. Find
7.
graphically the forces in all the parts for a given load and confirm
the results by experiment. p. 29.
Triangle of forces.
8.
Apparatus as in Figs. 33, 34. Find and E
R for two given forces by applying the triangle of forces. Confirm
the result by trial. p. 32.
Resultant of several forces. Apparatus as in Fig. 51a. Find
9.
E andR for several given forces by repeated applications of the
parallelogram of forces. Confirm the result by trial. p. 33.
10. Polygon of forces. Apparatus as in Fig. 51a. Arrange a
number of forces in the apparatus and let the ring come to rest.
Draw the polygon of forces for them and ascertain if it closes.
pp. 34-36.
11. Forces in a derrick crane.
Apparatus as in Fig. 52. Perform
the Expt. as directed (a) when the cord is attached to the top of the
jib, {b) when the cord passes over a pulley at the top of the jib.
pp. 36, 37.
Forces in a wall crane.
12. Apparatus as in Fig. 57. Same
instructions as for Expt. 11. p. 37.
13. Forces in sheer legs. Apparatus as in Fig. 58. Perform the
Expt. as directed. p. 39.
14.Forces on a carriage on an inclined plane. Apparatus as in
Fig. 60. Perform the Expt. as directed. p. 40.
304 APPLIED MECHANICS FOR BEGINNERS.

MOMENTS. PARALLEL FORCES.


Balance of two forces of equal moment. Apparatus as in
15.
Figs. 63, 64, 65. Prove experimentally that a given force may be
balanced by one of equal moment. pp. 42-44.
16. Principle of moments.
Apparatus as in Fig. 66. Perforin the
Expt. as directed. p. 44.
17. Reactions of the supports of a beam. Apparatus as in Fig. 69.
Place given loads on the beam. Calculate the reactions of the
supports and confirm by reading the spring balances. pp. 45-47.
18.Resultant of parallel forces. Apparatus as in Fig. 70. Per-
form the Expts. as directed (a) when P
and Q have the same sense,
(b) when P
and Q are of opposite sense. pp. 47, 48.
19. Couples. Apparatus as in Fig. 72. Perform the Expts. as
directed. pp. 49, 50.
20. Forces in the parts of
an engine. Apparatus as in Fig. 77.
Take P=10 1bs. Find by experiment Q, T, S, and V for crank
angles 45, 90, 135 ; confirm the results by applications of the
parallelogram of forces. Draw a turning moment diagram for the
crank moving from 0 to 180. pp. 50-52.
21. Centre of gravity. Apparatus as in Figs. 79, 80. Find
experimentally the centres of gravity of the thin plates provided.
pp. 55, 56.
22. Tensions in a hanging cord.
Apparatus as in Fig. 92. Per-
form the Expt. as directed. p. 60.
23. Tensions in a stretched chain. Apparatus as in Fig. 96.
Measure the dip and span of the chain ; calculate from these and
the weight of the chain H, Tvy T h , and T. Confirm the results by
experiment. pp. 62, 63.

STRENGTH AND ELASTICITY OF MATERIALS.


24. Extensions of pulled rubber.
Apparatus as in Fig. 110.
Gradually increase the load, noting the extension of the portion CD
for each increment then gradually diminish the load, again noting
;

the changes of length of CD. Plot loads and extensions for both
sets of readings. p. 70.
25. Extensions of pulled wires. Apparatus as in Fig. 111. Use
wires of steel, iron, copper, brass, etc. Same instructions as for
Expt. 24. p. 70.
26. Wires loaded to breaking under tension. Apparatus as in
Fig. 111. Perform the Expt. as directed. pp. 77-79.
27. Stiffness of beams.
Apparatus as in Fig. 150. Use pieces of
tool steel, each 3' 3" long, sections 1" x ", f" x ", " x ", " x ",
l"xl", respectively, arranging them (a) as beams supported at
COURSE OF LABORATORY WORK. 305

two places, (6) as cantilevers. Note the deflections produced by


gradually increasing and diminishing the loads. From the results,
verify the laws stated for proportional stiffness (p. 99). Plot
deflections and loads for each case. From the results work out the
value of Young's modulus for each sample. pp. 103-105.
28. Strength of timber test bars. Apparatus as in Fig. 150.
Perform the Expt. as directed. Calculate the value of the modulus
of Transverse Rupture (p. 102) for each sample. p. 106.
29. Stiffness of wires under torsion. Apparatus as in Fig. 162.
Use wires of the same material and having different diameters.
Vary the length under test in each case. Plot angles of twist and
torque. From the results verify the laws of proportional stiffness.
pp. 116-118.
30. Elastic extensions of a spring.
Apparatus as in Fig. 170.
Perform the Expt. as directed. Plot extensions and loads. From
the results verify the proportional laws. p. 120.

LAWS OF FRICTION.
31. Friction of a Apparatus as in Figs. 183, 184. Perform
slider.
the Expt. as directed. pp. 138-140.
32. Slider on an inclined plane. Apparatus as in Fig. 186. Per-
form the Expt. as directed. pp. 140, 141.
33. Effect of extent of surfaces in contact. Apparatus as in Figs.
184, 188. Perform the Expt. in the same manner as for Expt. 32.
p. 142.
34. Friction of a cord coiled on a drum. Apparatus as in Fig. 191.
Determine the ratio of the pulls for angles of lap differing by 90.
pp. 144-146.
35.
Rolling friction Apparatus as in Fig. 184 with the addition
of the small carriage shown in Fig. 60. Determine, for different
loads, the resistances offered to rolling on roads of cast iron, teak,
and rubber. Plot loads and resistances. pp. 146-148.

VELOCITY. ACCELERATION. MECHANISM.


36. The law, P= Apparatus
. as in Fig. 202. Perforin the

Expt. as directed. p. 160.

37. Crank and connecting-rod. Apparatus as in Fig. 235. De-


termine, from the model, piston positions corresponding to crank
angles differing by 30?. Plot these in a diagram. p. 183.
38. Infinite connecting-rod.
Apparatus as in Fig. 238. Same
instructions as for Expt. 38. p. 185.
39. Oscillating engine. Apparatus as in Fig. 239. Same instruc-
tions as for Expt. 38. p. 186.
U
306 APPLIED MECHANICS FOR BEGINNERS.

MECHANICAL ADVANTAGE. EFFICIENCY OF


MACHINES.
In all the following Expts., follow the procedure given on
pp. 196-202.
40. Simple pulley block. Apparatus as in Fig. 252. p. 195,
41. Pulley blocks.Apparatus as in Fig. 253. p. 197.
42. Rope blocks. Apparatus as in Fig. 257. p. 202.
43. Weston's blocks. Apparatus as in Fig. 258. p. 202.
44. Wheel and differential axle. Apparatus as in Fig. 261. p. 205.
45. Helical blocks. Apparatus as in Fig. 263. p. 206.
46. The crab. Apparatus as in Fig. 264. pp. 207-208.
47.
Screw jack. Apparatus as in Fig. 266. p. 208.
48.
Hydraulic jack. Apparatus as in Fig. 317. p. 264.

MISCELLANEOUS.
49. Energy of a flywheel. Apparatus as in Fig. 277. Perform
the Expt. as directed. pp. 223-226.
50. Simple pendulum. Perforin the Expt. as directed. p. 243.
51. Vibrations of springs. Perform the Expt. as directed, p. 243.

52. Specific gravity. Find the specific gravities of the pieces of


metal supplied. p. 254.
53. Pressure of the atmosphere. Apparatus as in Fig. 304. Per-
form the Expt. as directed. Compare the result with the reading
of a standard barometer and account for any difference noted, p. 255.

HYDRAULICS.
54.
Discharge from an orifice. Apparatus as in Fig. 321. Com-
pare the* actual flow with the result obtained by calculation.
pp. 268-273.
55.
Flow-over gauge notches. Apparatus as in Figs. 321, 323.
Using (a) the V-notch, (b) the rectangular notch, compare the actual
flow with the result obtained by calculation. pp. 274-276.
56. Pipe of varying section. Apparatus as in Fig. 332. Perform
the Expt. as directed. p. 278.
MATHEMATICAL TABLES,
ANSWERS, AND
INDEX.
308 APPLIED MECHANICS FOR BEGINNERS.

LOGARITHMS.

1 2 3 4 5 6 7 8 9 12 3 4 5 6 7 8 9

10 0000 0043 0086 0128 0170 0212 0253 0294 0334 0374 4 8 12 17 21 25 29 33 37

11 0414 0453 0492 0531 0569 0607 0645 0682 0719 0755 4 8 11 15 19 23 26 30 34
12 0792 0828 0864 0899 0934 0969 1004 1038 1072 1106 3 7 10 14 17 21 24 28 31
13 1139 1173 1206 1239 1271 1303 1335 1367 1399 1430 3 6 10 13 16 19 23 26 29
14 1461 1492 1523 1553 1584 1614 1644 1673 1703 1732 3 6 9 12 15 18 21 24 27
15 1761 1790 1818 1847 1875 1903 1931 1959 1987 2014 3 6 8 11 14 17 20 22 25

16 2041 2068 2095 2122 2148 2175 2201 2227 2253 2279 3 5 8 11 13 16 18 21 24
17 2304 2330 2355 2380 2405 2430 2455 2480 2504 2529 2 5 7 10 12 15 17 20 22
18 2553 2577 2601 2625 2648 2672 2695 2718 2742 2765 2 5 7 9 12 14 16 19 21
19 2788 2810 2833 2856 2878 2900 2923 2945 2967 2989 2 4 7 9 11 13 16 18 20
20 3010 3032 3054 3075 3096 3118 3139 3160 3181 3201 2 4 6 8 11 13 15 17 19
|

21 3222 3243 3263 3284 3304 3324 3345 3365 3385 3404 2 4 6 8 10 12 14 16 IS
22 3424 3444 3464 3483 3502 3522 3541 3560 3579 3598 2 4 (i 8 10 12 14 15 17
23 3617 8636 3655 3674 3692 3711 3729 3747 3766 3784 2 4 6 7 9 11 13 15 17
24 3802 3820 3838 3856 3874 3892 3909 3927 3945 3962 2 4 5 7 9 11 12 14 16
25 3979 3997 4014 4031 4048 4065 4082 4099 4116 4133 2 3 5 7 9 10 12 14 15

26 4150 4166 4183 4200 4216 4232 4249 4265 4281 4298 2 3 5 7 8 10 11 13 15
27 4314 4330 4346 4362 4378 4393 4409 4425 4440 4456 2 3 5 6 8 9 11 13 14
28 4472 4487 4502 4518 4533 4548 4564 4579 4594 4609 2 3 5 6 8 9 11 12 14
29 4624 4639 4654 4669 4683 4698 4713 4728 4742 4757 1 3 4 6 7 9 10 12 13
30 4771 4786 4800 4814 4829 4843 4857 4871 4886 4900 1 3 4 6 7 9 10 11 13

31 4914 4928 4942 4955 4969 4983 4997 5011 5024 5038 1 3 4 6 7 8 10 11 12
32 5051 5065 5079 5092 5105 5119 5132 5145 5159 5172 1 3 4 5 7 8 9 11 12
33 5185 5198 5211 5224 5237 5250 5263 5276 5289 5302 1 3 4 5 6 8 9 10 12
34 5315 5328 5340 5353 5366 5378 5391 5403 5416 5428 1 3 4 5 6 8 9 10 11
35 5441 5453 5465 5478 5490 5502 5514 5527 5539 5551 1 2 4 5 6 7 9 10 11

36 5563 5575 5587 5599 5611 5623 5635 5647 5658 5670 1 2 4 5 6 7 8 10 11
37 5682 5694 5705 5717 5729 5740 5752 5763 5775 5786 1 2 3 5 6 7 8 9 10
38 5798 5809 5821 5832 5843 5855 5866 5877 5888 5899 1 2 3 5 6 7 8 9 10
39 5911 5922 5933 5944 5955 5966 5977 5988 5999 6010 1 2 3 4 5 7 8 9 10
40 6021 6031 6042 6053 6064 6075 6085 6096 6107 6117 1 2 3 4 5 6 8 9 10

41 6128 6138 6149 6160 6170 6180 6191 6201 6212 6222 1 2 3 4 5 6 7 8 9
42 6232 6243 6253 6263 6274 6284 6294 6304 6314 6325 1 2 3 4 5 6 7 8 9
43 6335 6345 6355 6365 6375 6385 6395 6405 6415 6425 1 2 3 4 5 6 7 8 9
44 6435 6444 6454 6464 6474 6484 6493 6503 6513 6522 1 2 3 4 5 6 7 8 9
45 6532 6542 6551 6561 6571 6580 6590 6599 6609 6618 1 2 3 4 5 6 7 8 9

46 6628 6637 6646 6656 6665 6675 6684 6693 6702 6712 1 2 3 4 5 6 7 7 8
47 0721 6730 6739 6749 6758 6767 6776 6785 6794 6803 1 2 3 4 5 5 6 7 8
48 6812 6821 6830 6839 6848 6857 6866 6875 6884 6893 1 2 3 4 4 5 6 7 8
49 6902 6911 6920 6928 6937 6946 6955 6964 6972 6981 1 2 3 4 4 5 6 7 8
50 6990 6998 7007 7016 7024 7033 7042 7050 7059 7067 1 2 3 3 4 5 6 7 8

51 7076 7084 7093 7101 7110 7118 7126 7135 7143 7152 1 2 3 3 4 5 6 7 8
52 7160 7168 7177 71S5 7193 7202 7210 7218 7226 7235 1 2 2 3 4 5 6 7 7
53 7243 7251 7259 7267 7275 7284 7292 7300 7308 7316 1 2 2 3 4 5 6 6 7
54 7324 7332 7340 7348 7356 7364 7372 7380 7388 7396 1 2 J. 3 4 5 6 ft 7
MATHEMATICAL TABLES. 309

LOGARITHMS.

1 2 3 4 5 6 7 8 9 1 2 3 4 5 6 7 8 9

55 7404 7412 7419 7427 7435 7443 7451 7459 7466 7474 1 2 2 3 4 5 5 6 7

56 7482 7490 7497 7505 7513 7520 752S 7536 7543 7551 1 2 2 3 4 5 5 6 7
57 7559 7566 7574 7582 7589 7597 7604 7612 7619 7627 1 2 2 3 4 5 5 6 7
58 7634 7642 7649 7657 7664 7672 7679 7686 7694 7701 1 1 2 3 4 4 5 6 7
59 7709 7716 7723 7731 7738 7745 7752 7760 7767 7774 1 1 2 3 4 4 5 6 7
60 7782 7789 7796 7803 7810 7818 7825 7832 7839 7846 1 1 2 3 4 4 5 6 6

.u 7853 7860 7868 7875 7882 7889 7896 7903 7910 7917 1 1 2 3 4 4 5 6 6
62 7924 7931 7938 7945 7952 7959 7966 7973 79S0 79S7 1 1 2 3 3 4 5 6 6
63 7993 8000 8007 8014 8021 8028 8035 8041 S048 S055 1 1 a 3 8 4 5 5 6
64 8062 8069 8075 8082 8089 8096 8102 8109 8116 8122 1 1 2 3 3 4 5 5 6
65 8129 8136 8142 8149 8156 8162 8169 8176 8182 8189 1 1 2 3 3 4 5 5 6

06 8195 8202 8209 8215 8222 8228 8235 8241 8248 8254 1 1 2 3 3 4 5 5 6
67 8261 8267 8274 8280 8287 8293 8299 8306 8312 8319 1 1 2 3 3 4 5 5
68 8325 8331 8338 8344 8351 8357 8363 8370 8376 8382 1 1 2 3 3 4 4 5 6
69 8388 8395 8401 8407 8414 8420 8426 8432 8439 8445 1 1 2 2 3 4 4 5 6
70 8451 8457 8463 8470 8476 8482 8488 8494 8500 8506 1 1 2
2 3 4 4 5 6

8513 8519 8531 8537 8543 8459 8555 8561 1 o 4 4 5 5


71 8525 8567 1 2 3
72 8573 8579 8585 8591 8597 8603 8609 8615 8621 8627 1 1 2 2 3 4 4 5 5
73 8633 8639 8645 8651 8657 8663 8669 8675 8681 8686 1 1 2 2 3 4 4 5 5
74 8692 8698 8704 8710 8716 8722 8727 8733 8739 8745 1 1 2 2 3 4 4 5 5
75 8751 8756 8762 8768 8774 8779 8785 8791 8797 8802 1 1 2 2 3 3 4 5 5

7.; 8808 8814 8820 8825 8831 8837 8842 8848 8854 8859 1 1 2 2 3 3 4 5 5
77 8865 8871 8876 8S82 8887 8893 8899 8904 8910 8915 1 1 2 2 3 3 4 4 5
78 8921 8927 8932 8938 8943 8949 8954 8960 8965 8971 1 1 2 2 3 3 4 4 5
79 8976 8982 8987 8993 8998 9004 9009 9015 9020 9025 1 1 2 2 3 3 4 4 5
80 9031 9036 9042 9047 9053 9058 9063 9069 9074 9079 1 1 2 2 3 3 4 4 5

81 9085 9090 9096 9101 9106 9112 9117 9122 9128 9133 1 1 2 2 3 3 4 4 5
82 9138 9143 9149 9154 9159 9165 9170 9175 9180 9186 1 1 2 2 3 3 4 4 5
83 9191 9196 9201 9206 9212 9217 9222 9227 9232 9238 1 1 2 2 3 3 4 4 5
84 9243 9248 9253 9258 9263 9269 9274 7279 9284 9289 1 1 2 2 3 3 4 4 5
85 9294 9299 9304 9309 9315 9320 9325 9330 9335 9340 1 1 2 2 3 3 4 4 5

86 9345 9350 9355 9360 9365 9370 9375 9380 9385 9390 1 1 2 2 3 3 4 4 5
87 9395 9400 9405 9410 9415 9420 9425 9430 9435 9440 1 1 2 2 3 3 4 4
88 9445 9450 9455 9460 9465 9469 9474 9479 9484 9489 1 1 2 2 3 3 4 4
89 9494 9499 9504 9509 9513 9518 9523 9528 9533 9538 1 1 2 2 8 3 4 4
90 9542 9547 9552 9557 9562 9566 9571 9576 9581 9586 1 1 2 2 8 3 4 4

91 9590 9595 9600 9605 9609 9619 9624 9628 9633 2 o 3 3 4 4


9614 1 1
92 9638 9643 9647 9652 9657 9661 9666 9671 9675 9680 1 1 2 2 3 3 4 4
93 9685 9689 9694 9699 9703 9708 9713 9717 9722 9727 1 1 2 2 3 3 4 4
94 9731 9736 9741 9745 9750 9754 9759 9763 9768 9773 1 1 2 2 3 3 4 4
95 9777 9782 9786 9791 9795 9S00 9805 9809 9814 9818 1 1 2 2 8 3 4 4

96 9823 9827 9832 9836 9841 9845 9850 9854 9859 9863 Q 1 1 o 2 3 3 4 4
97 9S68 9872 9877 9881 9886 9S90 9894 9899 9903 9908 1 1 2 2 3 3 4 4
98 9912 9917 9921 9926 9930 9934 9939 9943 9948 9952 1 ] 2 2 3 3 4 4
i
99 9956 9961 9965 9969 9974 9978 9983 9987 9991 9996 Q 1 1 2 3 3 3 4
310 APPLIED MECHANICS FOR BEGINNERS.

ANTILOGARITHMS.

1 2 3 4 5 6 7 8 9 1 2 3 4 5 6

oo 1000 1002 1005 1007 1009 1012 1014 1016 1019 1021 1 1 1 1

01 1023 1026 1028 1030 1033 1035 1038 1040 1042 1045 1 1 1 1
02 1047 1050 1052 1054 1057 1059 1062 1064 1067 1069 1 1 1 1
03 1072 1074 1076 1079 10S1 1084 1086 1089 1091 1094 1 1 1 1
04 1096 1099 1102 1104 1107 1109 1112 1114 1117 1119 1 1 1 1 2
05 1122 1125 1127 1130 1132 1135 1138 1140 1143 1146 1 1 1 1 2

06 1148 1151 1153 1156 1159 1161 1164 1167 1169 1172 1 1 1 1 2
07 1175 1178 1180 1183 1186 1189 1191 1194 1197 1199 1 1 1 1 2
08 1202 1205 120S 1211 1213 1216 1219 1222 1225 1227 1 1 1 1 2
09 1230 1233 1236 1239 1242 1245 1247 1250 1253 1256 1 1 1 1 2
10 1259 1262 1265 1268 1271 1274 1276 1279 1282 1285 1 1 1 1 2

11 1288 1291 1294 1297 1300 1303 1306 1309 1312 1315 1 1 1 2 2
12 1318 1321 1324 1327 1330 1334 1337 1340 1343 1346 1 1 1 2 2
13 1349 1352 1355 1358 1361 1365 1368 1371 1374 1377 1 1 1 2 2
u 1380 1384 1387 1390 1393 1396 1400 1403 1406 1409 1 1 1 2 2
15 1413 1416 1419 1422 1426 1429 1432 1435 1439 1442 1 1 1 2 2

16 1445 1449 1452 1455 1459 1462 1466 1469 1472 1476 1 1 1 2 2
17 1479 1483 1486 1489 1493 1496 1500 1503 1507 1510 1 1 1 2 2
18 1514 1517 1521 1524 1528 1531 1535 1538 1542 1545 D 1 1 1 2 2
19 1549 1552 1556 1560 1563 1567 1570 1574 1578 1581 1 1 l 2 2
20 1585 1589 1592 1596 1600 1603 1607 1611 1614 1618 1 1 1 2 2

21 1622 1626 1629 1633 1637 1641 1644 1648 1652 1656 1 1 2 2 2
22 1660 1663 1667 1671 1675 1679 1683 1687 1690 1694 1 2 2 2
23 1698 1702 1706 1710 1714 1718 1722 1726 1730 1734 1 1 2 2 2
24 1738 1742 1746 1750 1754 1758 1762 1766 1770 1774 1 1 2 2 2
25 1778 1782 1786 1791 1795 1799 1803 1807 1811 1816 1 1 2 2 2

26 1820 1824 1828 1832 1837 1841 1845 1849 1854 1858 1 1 2 2 3
27 1862 1866 1871 1875 1879 1884 1888 1892 1897 1901 1 1 2 2 3
28 1905 1910 1914 1919 1923 1928 1932 1936 1941 1945 1 1 2 2 3
29 1950 1954 1959 1963 196S 1972 1977 1982 1986 1991 1 1 2 2 3
30 1995 2000 2004 2009 2014 2018 2023 2028 2032 2037 1 1 2 2 3

31 2042 2046 2051 2056 2061 2065 2070 2075 2080 2084 1 1 2 2 3
32 2089 2094 2099 2104 2109 2113 2118 2123 2128 2133 1 1 2 2 3
33 2138 2143 2148 2153 2158 2163 2168 2173 2178 2183 1 1 2 2 3
34 2188 2193 2198 2203 2208 2213 2218 2223 222S 2234 1 1 '2
2 3 3
35 2239 2244 2249 2254 2259 2265 2270 2275 2280 2286 1 1 2 2 3 3

36 2291 2296 2301 2307 2312 2317 2323 2328 2333 2339 1 1 2 2 3 3
37 2344 2350 2355 2360 2366 2371 2377 2382 2388 2393 1 1 2 2 3 3
38 2399 2404 2410 2415 2421 2427 2432 2438 2443 2449 1 1 2 1 3 3
39 2455 2460 2466 2472 2477 24S3 2489 2495 2500 2506 1 1 a 2 3 3
40 2512 2518 2523 2529 2535 2541 2547 2553 2559 2564 1 1 2
2 3 4

41 2570 2576 2582 2588 2594 2600 2606 2612 2618 2624 1 1 2 2 3 4
42 2630 2636 2642 2649 2655 2661 2667 2673 2679 2685 1 1 >_)
2 3 4
43 2692 2698 2704 2710 2716 2723 2729 2735 2742 2748 1 1 I 3 3 4
44 2754 2761 2767 2773 2780 2786 2793 2799 2805 2812 1 1 2 3 3 4
45 2818 2825 2831 2838 2844 2851 2858 2864 2871 2877 1 1 2 3 3 4

46 2884 2891 2897 2904 2911 2917 2924 2931 2938 2944 1 1 2 8 3 4
o
47 2951 2958 2965 2972 2979 2985 2092 2999 3006 3013 1
~2
1 3 4
48 3020 3027 3034 3041 3048 3055 3062 3069 3076 3083 1 1 8 4 4
49 13090 3097 3105 3112 3119 3126 3133 3141 3148 3155 1 1 i 3 4 4
MATHEMATICAL TABLES. 311

ANTILOGAEITHMS.

1 2 3 5 6 7 8 9 1 2 3 4 5 6 7 8 9

50 3162 3170 3177 3184 3192 3199 3206 3214 3221 3228 1 1 2 3 4 4 5 6 7

51 3236 3243 3251 3258 3266 3273 3281 3289 3296 3304 1 2 2 8 4 5 5 6 7
52 3311 3319 3327 3334 3342 3350 3357 3365 3373 3381 1 I 2 3 4 5 5 6 7
53 3388 3396 3404 3412 3420 3428 3436 3443 3451 3459 1
2
2 8 4 5 6 6 7
54 3467 3475 3483 3491 3499 3508 3516 3524 3532 3540 1 2 2 3 4 5 6 6 7
55 3548 3556 3565 3573 3581 3589 3597 3606 3614 3622 1 2 2 3 4 5 6 7 7

56 3631 3639 3648 3656 3664 3673 3681 3690 3698 3707 1 2 1 8 4 5 6 7 8
57 3715 3724 3733 3741 3750 3758 3767 3776 3784 3793 1 2 3 8 4 5 6 7 8
58 3802 3311 3819 3828 3837 3846 3855 3864 3873 3882 1 2 3 4 4 5 6 7 8
59 3890 3899 3908 3917 3926 3936 3945 3954 3963 3972 1 2 3 4 5 5 6 7 8
60 3981 3990 3999 4009 4018 4027 4036 4046 4055 4064 1 2 3 4 5 6 6 7 8

61 4074 4083 4093 4102 4111 4121 4130 4140 4150 4159 1 2 3 4 5 6 7 8 9
62 4169 4178 4188 4198 4207 4217 4227 4236 4246 4256 1 2 3 4 5 6 7 8 9
63 4266 4276 4285 4295 4305 4315 4325 4335 4345 4355 1 2 8 4 5 6 7 8 9
64 4365 4375 4385 4395 4406 4416 4426 4436 4446 4457 1 2 8 4 5 6 7 8 9
*65 4467 4477 4487 4498 4508 4519 4529 4539 4550 4560 1 2 3 4 5 6 7 8 9

66 4571 4581 4592 4603 4613 4624 4634 4645 4656 4667 1 2 3 4 5 6 7 9 10
67 4677 4688 4699 4710 4721 4732 4742 4753 4764 4775 1 2 3 4 5 7 8 9 10
68 4786 4797 4808 4819 4831 4842 4853 4864 4875 4887 1 2 3 4 6 7 8 9 10
69 4898 4909 4920 4932 4943 4955 4966 4977 4989 5000 1 2 3 5 6 7 8 9 10
70 5012 5023 5035 5047 5058 5070 5082 5093 5105 5117 1 2 4 5 6 7 8 9 11

71 5129 5140 5152 5164 5176 5188 5200 5212 5224 5236 1 2 4 5 6 7 8 10 11
72 5248 5260 5272 5284 5297 5309 5321 5333 5346 5358 1 2 4 5 6 7 9 10 11
73 5370 5383 5395 5408 5420 5433 5445 5458 5470 5483 1 3 4 5 6 8 9 10 11
74 5495 5508 5521 5534 5546 5559 5572 5585 5598 5610 1 3 4 5 6 8 9 10 12
75 5623 5636 5649 5662 5675 5689 5702 5715 5728 5741 1 3 4 5 7 8 9 10 12

76 5754 5768 5781 5794 5808 5821 5834 5848 5861 5875 1 3 4 5 7 8 9 11 12
77 5888 5902 5916 5929 5943 5957 5970 5984 5998 6012 1 3 4 5 7 8 10 11 12
78 6026 6039 6053 6067 6081 6095 6109 6124 6138 6152 1 3 4 8 7 8 10 11 13
79 6166 6180 6194 6209 6223 6237 6252 6266 6281 6295 1 3 4 6 7 9 10 11 13
80 6310 6324 6339 6353 6368 6383 6397 6412 6427 6442 1 3 4 6 7 9 10 12 13

81 6457 6471 6486 6501 6516 6531 6546 6561 6577 6592 2 3 B 8 8 9 11 12 14
82 6607 6622 6637 6653 6668 6683 6699 6714 6730 6745 2 3 5 8 8 9 11 12 14
83 6761 6776 6792 6808 6823 6839 6855 6871 6887 6902 2
8 5 8 8 9 11 13 14
84 6918 6934 6950 6966 6982 6998 7015 7031 7047 7063 2
3 E 8 8 10 11 13 15
85 7079 7096 7112 7129 7145 7161 7178 7194 7211 7228 2 3 5 7 8 10 12 13 15

86 7244 7261 7278 7295 7311 7328 7345 7362 7379 7396 2 3 5 7 8 10 12 13 15
87 7413 7430 7447 7464 7482 7499 7516 7534 7551 7568 2 3 5 7 9 10 12 14 16
88 7586 7603 7621 7638 7656 7674 7691 7709 7727 7745 1 4 6 7 9 11 12 14 16
89 7762 7780 7798 7816 7834 7852 7870 7889 7907 7925 2 4 5 7 9 11 13 14 16
90 7943 7962 7980 7998 8017 8035 8054 8072 8091 8110 2 4 7 9 11 13 15 17

91 8128 8147 8166 8185 8204 8222 8241 8260 8279 8299 I 4 6 8 9 11 13 15 17
92 8318 8337 8356 8375 8395 8414 8433 8453 8472 8492 2 4 8 8 10 12 14 15 17
93 8511 8531 8551 8570 8590 8610 8630 8650 8670 8690 2 4 8 8 10 12 14 16 18
94 8710 8730 8750 8770 8790 8810 8831 8851 8872 8892 2 4 8 8 10 12 14 16 18
95 8913 8933 8954 8974 8995 9016 9036 9057 9078 9099 2 4 6 8 10 12 15 17 19

96 9120 9141 9162 9183 9204 9226 9247 926S 9290 9311 2 4 6 8 11 13 15 17 19
97 9333 9354 9376 9397 9419 9441 9462 9484 9506 9528 2 4 7 9 11 13 15 17 20
98 9550 9572 9594 9616 9638 9661 9683 9705 9727 9750 2 4 9 11 13 16 18 20
99 9772 9795 9817 9840 9863 9886 9908 9931 9954 9977 8 5 7 9 11 14 16 18 20
312 APPLIED MECHANICS FOR BEGINNERS.
I .

TRIGONOMETRICAL TABLE.

Angle. Radians. Sine. Tangent. Cotangent. Cosine.

0 00 1 1-5708 90
1 0175 0175 0175 57-2900 9998 1-5533 89
2 0349 0349 0349 28-6363 9994 1-5359 88
3 0524 0523 0524 19-0811 9986 1-5184 87
4 0698 069S 0699 14-3006 9976 1-5010 86
8 0873 0872 0875 11-4301 9962 1-4835 85
6 1047 1045 M051 9-5144 9945 1-4661 84
7 1222 1219 1228 8-1443 9925 1-4486 83
8 1396 1392 1405 7-1154 9903 1-4312 82
9 1571 1564 1584 6-313S 9877 1-4137 81
10 1745 1736 1763 5-6713 9848 1-3963 80
11 1920 1908 1944 5-1446 9816 1-3788 79
12 2094 2079 2126 4-7046 9781 1-3614 78
13 2269 2250 2309 4-3315 9744 1-3439 . 77
14 2443 2419 2493 4-0108 9703 1-3265 76
15 2618 2588 2679 3-7321 9659 1-3090 75
16 2793 2756 2867 3-4874 9613 1-2915 74
17 2967 2924 3057 3-2709 9563 1-2741 73
18 3142 3090 3249 3-0777 9511 1-2566 72
19 3316 3256 3443 2-9042 9455 1-2392 71
20 3491 3420 3640 2-7475 9397 1-2217 70
21 3665 3584 3839 2-6051 9336 1-2043 69
22 3840 3746 4040 2-4751 9272 1-1868 68
23 4014 3907 4245 2-3559 9205 1-1694 67
24 4189 4067 4452 2-2460 9135 1-1519 66
25 4363 4226 4663 2-1445 9063 1-1345 65
26 4538 4384 4877 2 0503 8988 1-1170 64
27 4712 4540 5095 1-9626 8910 1-0996 63
28 4887 4695 5317 1-8807 8830 1-0821 62
25 5061 4848 5543 1-8040 8746 1-0647 61
30 5236 5000 5774 1-7321 8660 1-0472 60
31 5411 5150 6009 1-6643 8572 1-0297 59
32 5585 5299 6249 1-6003 8480 1-0123 58
33 5760 5446 6494 1-5399 8387 9948 57
34 5934 5592 6745 1-4826 8290 9774 56
35 6109 5736 7002 1-4281 8192 9599 55
36 6283 5878 7265 1-3764 8090 9425 54
37 ;6458 6018 7536 1-3270 7986 9250 53
38 6632 6157 7813 1-2799 7880 9076 52
39 6807 6293 8098 1-2349 7771 8901 51
40 6981 6428 8391 1-1918 7660 8727 50
41 7156 6561 8693 1-1504 7547 8552 49
42 7330 6691 9004 1-1106 7431 8378 48
43 7505 6820 9325 10724 7314 8203 47
44 7679 6947 9657 1-0355 7193 8029 46
45 7854 7071 1-0000 1-0000 7071 7854 45

Cosine. Cotangent. Tangent. Sine. Radians. Angle.


ANSWERS.

Chapter I., p. 11.

1. 2-908 metres. 2. 9 feet 7 '74 inches. 3. 5*129 kilometres.


4. 2*114 inches. 5. 12 -58 square inches. 6. 53 square cms.
7. 7 "3 square inches. 8. (a) 44 cms. ;
(b) 154 square cms.
9. 381*9 cubic inches. 10. 5*44 square inches. 11. 14*5 square inches.

Chapter II., p. 19.

1. 6-72 lbs. 2. 2-21 lbs. 3. 277 lbs. 4. 17 26. lbs.


5. 82-9. lbs. 6. 17-49 lbs. 7. 3 27 lbs. 8. 3 05 lbs.
9. 14-4 lbs. 10. 12-4 tons. 11. 496 lbs. 12. 87".

Chapter III., p. 30.

2. 5-7 lbs. 3. 5 lbs. 4. 29 lbs.

5. 21 lbs., acting towards the left.

6. (a) 11-7 lbs.; (b) 8'7 lbs. ; (c) 14 lbs.

7. 19 '-5 lbs. 8. 14 5 lbs. 10. 8 96 lbs. ; 7 32 lbs.

11. 9-65 cwts. 13. 1086 lbs.; 51 lbs. 14. 160 lbs.; 81 lbs.

15. 10 lbs.; 17-32 lbs.

Chapter IV., p. 40.

1. Push in AB =0*16 ton ;


push in AC =089 ton.
2. Push in jib =7*12 lbs.; pull in tie =575 lbs.
3. Push in jib =5*28 tons ;
pull in tie =4 36 tons.
4. Push in jib =7*78 tons ;
pull in tie =4 '86 tons.
5. Pull in AC =0-16 ton ;
push in BC =0 83 ton.
6. Push in AC =1*16 tons ;
pull in BC =083 ton.
314 ANSWERS.

7. Pull in AB = 30 tons pull in BC = 2 *425 tons ; ;

push in CD =0*97 ton push in BD =2*55 tons. ;

8. Push in each leg = 21 tons pull in back leg = 23 tons. ;

9. Pull in AP =2000 lbs push in BP =3464 lbs. ;

11. (a) 4-66 lbs.; {b) 4-23 lbs.; (c) 4*88 lbs.

Chapter V., p. 53.

1. At 25" from C, on the other side of the pivot from W.


2. 69*2 lbs. 3. P = 64*l lbs. 4. At 2-25 feet from the 3 lb. load.
6. 0-833 ton ; 0*417 ton. 7. 355 tons ; 2*45 tons.
8. 1 -33 tons at A ;
4 '66 tons at C.
10. (a) 2400 lbs.; (&) 5376 lbs. 11. 41-8".

Chapter VI., p. 65.

1. 345 lbs.; 245 lbs. 2. 5 '417 tons ; 4-583 tons.


3. 200 lbs.; 100 lbs. 4. 663 lbs. 5. 67 lbs.

6. 1600 lb. -feet ; 707 lb. -feet ; wall will fall.

8. 536 lbs. 9. 13-25 tons ; 9 "75 tons.


10. On the shorter portion of the beam, 2 24 ft. from the pivot.
11. 1 '77 ft. from the heavier end of the ladder.
12. Pull at middle = 25 lbs.; pull at each end = 26 "9 lbs.

13. 177-5 lbs.; 216*9 lbs. 14. 25*3 lbs.; tension increases.

Chapter VII., p. 86.

1. 2*8 tons per sq. inch. 2. 12-5 tons.


3. Strain = 0-00083; #=30,000,000 lbs. per sq. inch.
4. 0-5Q4". 5. 0-0556". 6. 076 1b.

7. 9062 lbs. 8. 29-46 tons ;


66 "6 tons per sq. inch.
9. 9-82 tons ; 199*8 tons per sq. inch.
10. #=15,700,000 lbs. per sq. inch. 11. 0*747 sq. inch.
13. 3600 lbs. per sq. inch ; '000125 ; 28,800,000 lbs. per sq. inch.

Chapter VIII., p. 107.

1. (a) M=-0; =300 lbs. (b) M =1500 lb. -ft,; 5=300 lbs.

2. (a) M= 3000 lb. -ft. ; S= 500 lbs. (6) M = 1500 lb. -ft, ; S= 500 lbs.
3. Bending moments, at middle, 15 ton-feet; at each 1 ton load,
JO ton-feet. Shearing force = 1 ton.
APPLIED MECHANICS FOR BEGINNERS. 315

Place. Bending Moment. Shearing Force.

At wall 1600 lb. -ft. 400 lbs.


2 ft. from wall 900 300
4 ft. 400 200
6 ft. 100 100
8 ft.

5. M
At middle = 3600 lb. -feet S = 0. ;

At 3 feet from each end, ^7=2500 lb. -feet ; =600 lbs.

1-375 tons. 7. 2-875 tons. 8. 18-16 tons ; 4 83 tons.


122*4 ton-feet; 5'1 sq. inches.
10. Tensile stress =1-66 tons per sq. inch ; compressive stress = 8 "33
tons per sq. inch.
5 21 cwts. 12. 18 tons ; 3 6 tons. 13. 4*77 tons per sq. in.

30,190,000 lbs. per sq. inch.

Chapter IX., p. 121.

1. 24-75 tons. if"; 3|"; 74 per cent.


2. 3. (a) 0'5"; (6) tf"-

4. 1". 5. 1-42:1. 6. 7636 lbs. per sq. inch. 7. 3-126".

8. 785*7 ton-inches ;
29'1 tons. 9. M: T=l :2.

11. (a) 15710 lb.-inches ; (6) 426". 12. 2'28".

Chapter X., p. 133.

1. 600 ft. -tons. 2. 135,000 ft. -lbs. 3. 697,000 ft. -lbs.

4. 375 ft. -tons. 5. 3400 ft. -lbs. 6. (a) 140 ft. -lbs.; (&) 220 ft. -lbs.

7. 2,250,000 ft. -lbs. 8. 324 ft. -lbs. 9. 24,550,000 ft. -lbs.

10. 7500 ft. -lbs. 11. 463,200 ft. -lbs. 12. 15,440,000 ft. -lbs.

13. 0-64 H.P. 14. 318 H.P. 15. 101 H.P.


16. 19*8 per cent. 17. 160 H.P. 18. 0-215 inch-ton.
19. 3,780,000 ft. -lbs. 20. 0091 H.P.; 0-68 H.P. 21. 288,000 ft.-lbs.

22. 247,000 ft. -lbs.; 1 12, 700 ft. -lbs. ; 1 34, 300 ft. -lbs.

Chapter XI., p. 151.

1. 0266. 2. 0-433. 3. 0'288. 4. 147 lbs.

5. 1530 lbs.; 1 85 H.P. 6. 3 28 feet. 7. 416 lbs.; 166 lbs.

8. 1800 lbs.; 144 H.P. 9. 08 H.P. 342B.T.U. 10.; 0'179.


11. 423-4 ft. -lbs. 13. 192 H.P 480 H.P. ;
;

316 ANSWERS.

Chapter XII., p. 164.


1. 440 feet. 2. 32*7 miles per hour. 3. 17 "4 feet per sec.
4. 47 miles per hour. 5. 0*084 feet per sec. per sec. 6. 12 feet.
7. 1 *703 tons. 8. 400 feet per min. 9. 1863 lbs.

10. 1500 ft. -lbs. 11. 49-7 ft. -lbs. 12. 265,600 ft. -tons.
14. 10 feet per sec. 15. 43 feet per sec.

16. Heights fallen -9 '82 feet ; 9676 feet ; 0*0966 feet.

Average velocities 98*21 ft. per sec; 96*76 ft. per sec; 96*616
ft. per sec.
17. 1770 ft. -lbs.

Chapter XIII., p. 192.

(a) 212; 318. (6) 207*8 ; 305*5 revs, per min.


14*5 H.P. 5. 25; 50. 7. 14*66 feet per min.
B, 3 clockwise ; C, ^ anticlockwise ; D, 3 clockwise.
80 on leading screw ; 45 on pin, gearing with the 80 wheel
60 on same pin, gearing with the 20 wheel,
10. (a) 628*6 feet per min.; (b) 400 feet per min.;

Crank angle 0 30 60 90 120 150 180


(<')

Piston speed,
392 624 628*6 464 236
feet per min.

12. 2*25". 13. 19*1"; 22 revs.


16. 2514 feet per min.; 262 lbs. 17. 10". 18. i"

Chapter XIV., p. 209.

1. 46*6 lbs. 2. 15. 3. 238 lbs.

4. 48. 5. 940 degrees. 6. 71*6 lbs.: 301*6 lbs.

Chapter XV ,
p. 231.

2*86. 2. 5*62. 3. 240.


86*4 per cent.; 29010 ft. -lbs. 5. 896,100 ft. -lbs.

84*7 revs, per min. 7. 2,740 lbs. 8. 5 tons weight.


62,100 lbs. weight. 10. 3,882 lbs. weight. 11. 5*24 tons weight.
12. 100, 3201b. ft. -sec ; 1,557 lbs weight ; 8*8 feet.
13. 126,200 ft. -lbs. 14. 12*9 lbs. weight. 15. 150,000 ft. -lbs.

16. 1 *03 lbs. weight. 17. 11,880 ft. -lbs.

18. (a) 42,520 ft. -lbs. (6) 75 revs. ; (c) 567 ft. -lbs.
APPLIED MECHANICS FOR BEGINNERS. 317

Chapter XVI., p. 245.

1. 1,325 lbs. 2. 15 lbs. 3. 234 4 lb. -feet.

4. 428 ft. per sec. 5. 0'8 foot ; 6 inch.

0 330 60 1
90 120 150
Angle.
1

180
360 30 300 270 240 210

Acceleration,
+ 200 + 173 2 + 100 -100 - 173 2 -200
ft. sec. sec.

7. 3912 inches. 8. 1 7 tons.

Chapter XVII., p. 266.

1. 30,720 lbs.; 9,600 lbs.; 6,400 lbs. 2. 22,500 lbs.; 45,000 lb. -feet.

3. 3 25 feet. 4. 316 cubic foot. 5. 33 9 feet.


6. 44,000 ft. -lbs.; 2 -2 H. P. 7. 6*19 ft.; 9,000 lbs.; 8,400 lbs.; 4,000 lbs.

8. 55,000 lbs.; 440,000 ft. lbs.; 4'36 cubic feet. 9. 166 6 lbs.

11. 31,250 lbs. 12. 108,000 ft, -ibs.; 17,280 ft. -lbs.; 960,000 ft. -lbs.

13. 470 ; 425 ; 904 per cent. 14. 0033 H.P.


15. 201,600 ft. -lbs.; 107,520 ft. -lbs.
16. 41 ton per sq. inch ; 1,950 ft. -tons. 17. 1,017 lbs.
18. 3-21 H.P.; 67 percent.

Chapter XVIII., p. 290.

1. 78*8 gallons per min. 2. 276 cubic feet per min.


3. 102 cubic feet per min. 4. 349,000 gallons per hour.
5. 11-8 H.P. 6. 40-7 feet per sec. 7. 20 feet per sec.
8. 3-48 feet per sec. 9. The pressure in the 12" part exceeds
that in the 3" part by 70 lbs. per sq. inch. 10. 48,830 ft. -lbs.
INDEX.

Absolute pressure of a gas, 256. Beams, bending moments of, 90.


,, unit of force, 159. ,, commercial tests on, 102.
Accelerating force, 158. ,, curves of bent, 107.
Acceleration, 155. ,, distributed load on, 93.
,, due to gravity, 157. ,, effect of notching, 106.
,, expt. on, 160. ,, model showing forces in,
,, in a mechanism, 183. 90.
of a rotating body, Beams of I section, 95.
234. ,, of similar sections, 99.
Accumulator, hydraulic, 262. ,, reactions of supports of,
Activity, 128. 45.
Air, expansion of, 256. Beams, resisting moment of, 96.
Aluminium, 295, 300. ,, section of cast-iron, 96.
bronze, 74, 296, 300. ,, shearing force in, 91.
American yellow pine, 299. ,, six standard cases of, 99,
Angle of resistance, limiting, 141. 100.
Angle of twist, 116. Beams, stresses in, 89.
Angles, unit, 10. ,, strength and stiffness of,
Annealing, 76. 99.
Arch, experimental, 65. Bearings, ball, 146.
,, line of resistance in an, ,, heating of, 150.
64. ,, power absorbed in, 151.
Areas, determination of, 6. roller, 146.
Ash, 299. Beech, 299.
Atmosphere, pressure of the, Belt driving, 166.
255. ,, pulley arrangements, 169.
Autographic records, 79, 80, Belts, guide pulleys for, 168.
105. , ,
power transmitted by, 1 67.
Average stress of a liquid, 251. slip of, 168.
Bending moment, 90, 91.
Balance, ordinary, 14. ,, ,, diagram, 93,
spring, 15. 94.
Barometer, 255. Bessemer process, 292.
Beams, 45, 88. Bevel wheels, 177.
,, apparatus for expts. on, Blades of a centrifugal pump,
46, 103. 230.
APPLIED MECHANICS FOR BEGINNEHS. 319

Blades of a turbine, 282. Compressive stress, 68.


Body, 13. Concrete, 297.
,, falling freely, 157. Cones, friction, 171.
Boiler, stresses in a, 83. ,, speed, 170.
Booms of a girder, 98. Conservation of energy, 125.
Boyle's law, 256. Contracted vein, 269-
Brake, heating of a, 216. Contraction during tensile test,
,, horse-power, 214. 74.
,, prony, 215. Copper, 74, 294, 300.
,, rope, 214. Couples, 49, 95.
Brass, 74, 295, 300. Course of laboratory work, 302.
Bricks, 296, 300. Crab, 207.
Bronzes, 296, 300. Crane, experimental derrick, 36.
Buckling, 96. ,, experimental wall, 37.
Bulging of flat plates, 83. Crank and connecting-rod, 50.
Buoyancy, 253. Crank, maximum torque on,
133.
Calipers, 2. Creeping, 74.
vernier, 5. Creosoting, 299.
Cams, 192. Crushing, 69.
Cantilever, model of a, 89, 97. Cylindrical shell, stresses in, 80.
Case hardening, 294.
Cast iron, 291. Deflection of beams, 99, 104, 107.
,, beams, 96. Delta metal, 296, 300.
,, factor of safety for, 76. Density, 13.
strength of, 102, 300. ,, specific, 14.
,, test bars, 105. Dip and span, 62.
Castings, chilled, 291. Discharge from an orifice, 268,
,, malleable, 294. 272.
,, weight of, 17. Discharge through notches, 273,
Cementation process, 292. 275.
Cements, 297. , Ductility, 74.
Centre, instantaneous, 183. Dynamometers, absorption, 215.
,, of buoyancy, 254. ,, transmission, 217.
,, of gravity, 55.
,, of parallel forces, 54. Eccentric, 191.
,, of pressure, 252. Eddy motion, 149.
Centrifugal force, 234. Efficiency of an engine, mechani-
,, governors, 238. cal, 215.
,, pump, 230. Efficiency of machines, 125, 196,
Chains, stretched, 61, 63. 201.
Clock escapements, 244. Elastic curve, 72, 79, 130.
Coefficient of contraction, 269. limit, 73.
,, of discharge, 269. Elasticity, 73.
,, of friction, 136. ,, Young's modulus of,
of velocity, 269. 73, 104.
Column, 22. Elm, 299.
Columns, failure of, 69. End measuring rods, 6.
Components of a force, 23, 26. Energy, 124.
320 INDEX.

Energy, conservation of, 125. Force, producing acceleration,


,, electrical, 127. 158.
,,heat, 126. Force, representation, of a, 21.
kinetic, 126, 161, 220. Forces, apparatus, for expts. on,
,, loss of useful, 126. 25.
,, of a fluid, pressure, 269. Forces, centre of parallel, 54.
of a flywheel, 220. ,, generating momentum,
potential, 126, 161, 269. 227.
,, transformation of, 124, Forces, impulsive, 228.
161, 279, 288. ,, in the parts of a structure,
Energy, useful forms of, 126. 29, 36, 38, 64.
Epicyclic trains, 175. Forces in the parts of an engine,
Equations of motion, 156. 50, 52.
,, for a machine, 125, Forces in the same plane, 59.
150, 199. ,, in the same straight line,
Equilibrant, 24. 21.
Equilibrium, stable, unstable Forces, parallelogram of, 23.
and neutral, 57. polygon of, 34.
Equilibrium of a floating body, ,, triangle of, 32.
253. ,, two intersecting, 23.
Extension of a pulled bar, 70. Fracture, 74.
Extensometers, 71. Friction, angle of, 141.

,, coefficient of , 136.
Factor of safety, 75, 76. ,, conditions influencing,
Falling body, 126, 157, 161. 136.
Fatigue, 76. Friction, effect of extent of
Floating body, 253. rubbing surfaces on, 142.
Flow in streams, 273. Friction, expt. on rolling, 147.
,, over gauge notches, 275. ,, force of, 136.
,, steady and eddy, 277. ,, in machines, effects of,
,, through orifices, 268. 150.
through pipes, 276. Friction gearing, 170.
Fluctuation in speed of machines, ,, influence of speed of
219. rubbing on, 142.
Fluid friction, 148, 247. Friction, laws of fluid, 148.
,, pressure, 248. of a belt, 146, 166.
Fluids, some properties of, 148, ,, of a rolling wheel, 146.
247. ,, of a rope on a drum, 144.
Flywheels, bursting effect in, 236. of a slider, 138.
,, energy stored in, 220. ,, of bearings, 149.
,, expt. on, 223. ,, ofdry surfaces, 137.
,, fluctuation in speed ,, on an inclined plane,
of, 222. 140.
Flywheels, of, 221.
Foot-pound, 122.
M Gas, perfect, 247.
Force, 13. ,, pressure of a, 256.
,, absolute unit of, 159. Gauge notches, 273.
,, central or centripetal, 234. Gauges, limit,. 2.
centrifugal, 234. ,, standard cylindrical, 2.
APPLIED MECHANICS FOR BEGINNERS. 321

Gauges, standard screw, 3. Iron, ingot, 293.


Girders, action of diagonal parts pig, 291.
in, 97. wrought, 74, 105, 291, 300.
Girders, bridge, 98.
,, distribution of material Jet, 268.
in, 97. pump, 279.
Girders, plate, 98.
Governors, 238. Kinetic coefficientof friction, 136.
Gram mass, 13. energy, 126, 161, 220,
Granite, 296, 300. 270, 279, 288.
Gravitational effort, 14. Knuckle joint, 84.
Gravity, 14, 157.
,, specific, 15, 254. Laboratory work, course of, 302.
,, table of specific, 17. Lead, 295, 300.
Guide pulleys, 168. Leather packings, 261.
Gun-metal, 296, 300. Lime, 297.
Limestone, 296.
Hardening, 293, 294. Limiting angle of resistance,
Harmonic motion, simple, 240. 141.
Head of water, 258. Link mechanisms, 179.
Heat transformed into work, 126. Link polygon, 59.
,, unit of, 126. Liquid, flow of, 268.
Heating and cooling, effect of, friction, of, 149, 276.
76, 294. Load, alternating, 76.
Heating of bearings, 150. ,, suddenly applied, 76, 131.
of brake, 216.
Helical blocks, 206. Machines, 123.
Hook gauge, 274. ,, equations for, 199.
Hooke's law, 72. ,, expts. on, 195.
Horse-power, 128, 212, 231, 283. ,, steadiness in, 220.
Hydraulic accumulator, 262. ,, want of balance in,
,, apparatus, experi- 235.
mental, 271. Mahogany, 299.
Hydraulic crane, 263. Manganese bronze, 296, 300.
jack, 264. Mass, 13.
lift, 263. ,, of a body, 16.
press, 259. Matter, 13.
pumps, 259. Mechanical advantage, 124, 127,
punching bear, 265. 195.
Mechanism, acceleration in, 183.
Idle wheels, 173. ,, crank and connect-
Impulsive action, 228. ing-rod, 180.
Inclined plane, 40, 140. Mechanism, infinite connecting-
Indicated horse-power, 212. rod, 185.
Indicator, 211. link, 179.
Inertia, 158, 234. ,, models, 183.
Instantaneous centre. 183. oscillating engine,
Iron, cast, 74, 102, 105, 291, 300. 186.
galvanised, 295. Mechanism, parallel motion, 187.
322 INDEX.

Mensuration of areas, 6. Potential energy, 126, 161, 269,


,, of volumes, 9. 279, 289.
Micrometer, 3. Pound mass, 13.
Mobile liquids, 247. ,, weight, 15.
Modulus of a beam section, 97. Power, 128.
,, of rigidity, 86. Powers, measurement of large,
,, of transverse rupture, 219.
102. Press, hydraulic, 259.
Modulus, Young's, 73, 104. Pressure in a liquid, 249.
Moment, bending, 90. ,, of the atmosphere, 255.
,, of a couple, 49. ,, on the walls of a tank,
,, of a force, 42. 250.
,, resisting, 96, 114. Procedure for expts. on machines,
,,
turning, 51. 196.
Moments, principle of, 44. Puddling, 291.
Momentum, 226. Pulley blocks, 195, 197, 202.
,, and impulse, 228. Pulleys, belt, 166.
change of, 229, 230, ,, rope, 169.
284, 285. Pump, centrifugal, 230.
Momentum, forces generating, ,,force, 258.
227. ,, hydraulic, 259.
Motion in a circle, 234 lift, 257.

,, in a straight line, 153. ,, Thomson's jet, 279.


Muntz metal, 296, 300. Punching, 84, 109.

Northern pine, 299. Radian, 9,


Notches, gauge, 273, 275. Reaction, 22, 29, 45.
Records, autographic, 79, 105.
Oak, 299, 300. Resilience, 130.
Orifice, flow through, 268. Rest, 161.
,, friction of, 136.
Parallel forces, 46. Resultant, 23, 26, 33, 47, 48.
,, motions, 187. Retaining wall for water, 252.
Pawl and ratchet wheel, 204. Reversing motion, 170.
Pelton wheel, 284. Rigidity, modulus of, 86.
Pendulum, 27, 243. Rivet holes, 109.
used to determine g, Riveted joints, 109.
243. Rolling, resistance to, 146.
Phosphor bronze, 74, 296, 300. Roman cement, 297.
Pipes, change of pressure in, Roof truss, simple, 29.
278. Rope pulleys, 169.
Pipes, flow in, 276. Rotating body, 180, 220, 235.
Pitch circle, 172. Running down of a load, 204.
,, of teeth, 172.
of thread, 178. Safety, factor of, 75, 76.
,, pine, 301. Sandstone, 296, 300.
Polygon of forces, 34. Screw, 177.
link, 59. jack, 208.
Portland cement, 297. Seasoning of timber, 298.
APPLIED MECHANICS FOR BEGINNERS. 323

Shaft coupling, flanged, 115. Table of antilogarithms, 310.


hollow, 115. logarithms, 308.
horse power transmitted ,, strength, etc., of beams,
by a, 132. 100.
Shaft section, moment of resis- Table of strength of materials,
tance of a, 114. 300.
Shaft, torque on a, 113. Table of trigonometrical ratios,
Shearing action, 84. 312.
,, force in a beam, 91. Table of useful constants, 10.
Shear diagram, 93. ,, weights and specific
strain, 85. gravities, 17.
,, stress produced by torque, Table of Young's modulus, 74.
114. Teak, 299.
Siemens-Martin process, 292. Teeth of wheels. 176.
Simpson's rule, 7. Tempering, 293.
Specific density, 14. Tensile tests on wires, 77.
gravity, 15, 16, 254. Tensions at ends of a chain, 61.
,, gravity, table of, 17. ,, in a stretched cord, 60,
Speed cones, 170. Test bars, 102, 105, 106.
Springs, 118. Thomson's jet pump, 279.
,, elastic extension of , 120. ,, turbine, 281.
,, vibrations of, 243. Tie bar, 22.
Spruce, 301. Ties, 68.
Steady motion of a liquid, 149, Timber, 297.
277. Tin, 294, 300.
Steady motion in machines, 220. Tin-plate, 294.
Steel, 74, 105, 292, 300. Toothed wheels, 172, 177.
,, tape, 6. Torsion, 113.
Sterro metal, 295, 300. Trains of wheels, 173.
Stiffness of beams, 99. Trapezoidal rule, 7.
,, of wires under torsion, Triangle of forces, 32.
116. Trigonometrical ratios, 10, 312.
Stones, building, 296. Turbines, 281.
Straight edge, 1. Turning moment diagram, 51.
Straight line motion, Peaucellier,
191. Ultimate strength, 75, 300.
Strain, tensile and compressive, Useful constants, 10.
72.
Strain, shear, 85. Vapour, 247.
Strength of materials, table of, Velocities, parallelogram of, 154.
300. Velocity, 153.
Stress, 67. ,, change in, 156, 163.
in a liquid, 149, 248. ,, of discharge, 269.
,, shear, 84. ratio, 124, 195.
,, tensile and compressive, ,, relative, 161.
68. ,, time diagrams, 154.
Structures, models of, 36. ,, uniform and variable,
Struts, 22, 68. 153.
Surface plate, 1. Vernier, 5.
, .

324 INDEX.

Vernier caliper, 5. Weston's blocks, 202.


Vibrations of a pendulum, 242, Wheel and differential axle, 205.
244. Winch, simple, 123.
Vibrations of a spring, 243, 245, Wires under tension, 62, 70, 77.
,
simple harmonic, 241 ,, under torsion, 118.
Virtual slope, 277. Work 122.
Viscosity, 149, 247. of anexpanding gas, 256.
Volumes, determination of, 9. ,, done in stretching a bar,
130.
Wall retaining, 252. Work done on a piston, 213.
Water, flow of, 268. ,, representation of, 129.
, ,
pressure of, 248. Worm-wheel gearing, 178.
wheels, 280.
Weight, 14, 15. Yield point, 74.
Weights, calculation of, 16. Young's modulus, 73, 104.
table of, 17.
Weir, 273. Zinc, 295, 300.

GLASGOW : PRINTED AT THE UNIVERSITY PRESS BY ROBERT MACLEHOSE AND CO.


MACMILLAN & CO.'S

SCIENCE CLASS BOOKS


Adapted to the South Kensington Syllabuses

I. PRACTICAL PLANE AND SOLID GEOMETRY.


Practical Plane and Solid Geometry. By J. Harrison, M. Inst. M.E., etc.,
Instructor in Mechanics and Mathematics, and G. A. Baxandall, Assistant
Instructor Royal College of Science, London. Part I. Elementary. 2s. 6d.
Adapted to the Elementary Stage of the South Kensington Syllabus. Part U.
Advanced. 4s. 6d.

Test Papers in Practical Plane and Solid Geometry. Elementary


Stage. By George Grace, B.Sc. (Lond.). 24 Tests printed on Cartridge Paper,
(kf.

II. MACHINE CONSTRUCTION AND DRAWING.


Machine Construction for Beginners. By F. Castle, M.I.M.E.
[In preparation.
III. BUILDING CONSTRUCTION.
Building Construction for Beginners. Adapted to the Elementary Stage of
the South Kensington Syllabus. By J. W. Riley, Rochdale Technical School.
2s. 6d,

Building Construction for Advanced Students. By J. W. Riley.


[In preparation.
V. MATHEMATICS.
An Elementary Course of Mathematics. Comprising Arithmetic, Algebra,
and Euclid. Adapted to the Elementary Stage of the South Kensington Syllabus.
By H. S. Hall, M.A., and F. H. Stevens, M.A., Masters of the Military Side,
Clifton College. 2s. M.
Graduated Test Papers in Elementary Mathematics. Adapted to the
Elementary Stage of the South Kensington Syllabus. By Walter J. Wood, B.A.
Is.

Elementary Practical Mathematics. Adapted to the South Kensington


Syllabus. By F. Castle, M.I.M.E. 3s. M.
Practical Mathematics for Beginners. Adapted to the new Elementary
South Kensington Syllabus. By F. Castle, M.I.M.E. 2.s. 6rf.

VI. THEORETICAL MECHANICS.


Elementary Mechanics of Solids. By W. T. A. Emtage, M.A., Director of
Public Instruction in Mauritius. 2s. CkI.

Mechanics for Beginners. By W. Gallatly, B.A. 2s. 6d. Adapted to the


Elementary Stage of the South Kensington Syllabus.
Mechanics for Beginners. By Rev. J. B. Lock, M.A. Part I. Mechanics and
Solids. 2s. 6d. Adapted to the Elementary Stage of the South Kensington
Syllabus.
Hydrostatics for Beginners. ByF. W. Sanderson, M.A. 2*. tW.

VII. APPLIED MECHANICS.


Lessons in Applied Mechanics. By Professor J. H. Cotterill, F.R.S., and
J. H. Slade. 5s. 6d.

Applied Mechanics for Beginners. By J. Di scan, Wh.Sc. [/ prtpamtion,

MACMILLAN AND CO., LTD., LONDON


VIII. SOUND, LIGHT, AND HEAT.
Elementary Lessons in Heat, Light, and Sound. By D. E. Jones,
B.Sc, Inspector of Science Schools under the Science and Art Department.
Adapted to the Elementary Stage of the South Kensington Syllabus. 2s. dd.
Lessons in Heat and Light. By D. E. Jones, B.Sc. 3s. 6d.
Practical Exercises in Light. By R. S. Clay, D.Sc. [In preparation.
Heat for Advanced Students. Adapted to Advanced Stage of South
Kensington Syllabus. By E. Edser, A.R.C.Sc. 4s. 6d.

Light for Advanced Students. By E. Edser, A.R.C.Sc. [In preparation.


Elementary Physics. By Balfour Stewart, F.R.S. New Edition, thoroughly
Revised. 4s. 6d. Questions, 2s.

IX. MAGNETISM AND ELECTRICITY.


Electricity and Magnetism for Beginners. Adapted to the Elementary
By P. W. Sanderson, M.A. 2s. 6d.
Stage of the South Kensington Syllabus.
Magnetism and Electricity for Beginners. Adapted to the Elementary
Stage of the South Kensington Syllabus. By H. E. Hadley, B.Sc. (Lond.).
2s. 6d.

Practical Exercises in Magnetism and Electricity. Being a Labora-


tory Course for Schools of Science. By H. E. Hadley, B Sc. Globe Svo. 2s. 6d.
Elementary Lessons in Electricity and Magnetism. By Prof. Silvanus
P. Thompson, F.R.S. New and Enlarged Edition. 4s. 6d.

X. and XI. CHEMISTRY.


INORGANIC CHEMISTRYTHEORETICAL.
Chemistry for Beginners. Adapted to the Elementary Stage of the South
Kensington Syllabus. By Sir Henry Roscoe, F.R.S., Assisted by J. Lunt, B.Sc.
New Edition, revised. 2s. 6d.
The Elements of Chemistry. Adapted to the South Kensington Syllabus.
By Prof. Ira Remsen. 2s. 6d.
Inorganic Chemistry for Advanced Students. By Sir H. E. Roscoe,
F.R.S., and Dr. A. Harden. 4s. 6d.
Chemical Problems.. By Prof. T. E. Thorpe, F.R.S. With Key, 2s.
Chemical Arithmetic. By S. Lupton, M.A. With 1200 Problems. 4s. 6d.

Inorganic Chemistry. By Prof. Ira Remsen. 6s. 6d.

INORGANIC CHEMISTRY PRACTICAL.


Chemistry for Organised Schools of Science. By S. Parrish, B.Sc,
A.R.C.S. (Lond.), with Introduction by Dr. Forsyth. 2s. 6d.

Practical Inorganic Chemistry. By G. S. Turpin, M.A., D.Sc. Adapted to


the Elementary Stage of the South Kensington Syllabus, and to the Syllabus for
Organised Science Schools. 2s. 6d.
Practical Inorganic Chemistry for Advanced Students. By Chapman
Jones, F.I.C., F.C.S. 2s. 6d.

The Junior Course of Practical Chemistry. By F. Jones, F.C.S. 2s. 6d.


The New Edition of this book covers the Syllabus of the South Kensington
Examination.
ORGANIC CHEMISTRY.
Organic Chemistry for Beginners. By G. S. Turpin, M.A., D.Sc. Adapted
to the South Kensington Syllabus. 2s. 6d.

Organic Chemistry. By Ira Remsen. 6s. 6d.


Prof.
Practical Organic Chemistry for Advanced Students. By J. B.
Cohen, Ph.D. 3s. 64.

Organic Chemistry for Students. J. B. Cohen, Ph.D. [In preparation.

MaCMTLLAN AND CO., LTD., LONDON


XII. GEOLOGY.
Geology for Beginners. By W. W. Watts, M.A., F.G.S. Adapted to the
Elementary Stage of the South Kensington Syllabus. 2s. 6d.

XIV. HUMAN PHYSIOLOGY.


Physiology for Beginners. By
Michael Foster and Dr. L. E. Shokb.
Sir
Adapted to the Elementary Stage of the South Kensington Syllabus. 2s. 6d.
Lessons in Elementary Physiology. By the Right Hon. T. H. Huxlky,
F.R.S. 4s. 6d. Questions, Is. 6d.

XVI. ZOOLOGY.
A Manual of Zoology. By the lateT. Jeffery Parker, D.Sc, F.R.S., and
W. A. Habwell, M.A. D.Sc, F.R.S., Illustrated. 10s. 6d.

XVII. BOTANY.
Botany for Beginners. Adapted to the Elementary Stage of the South
Kensington Syllabus. By Ernest Evans, Burnley Technical School. 2s. 6d.

XVIII. PRINCIPLES OF MINING.


Coal Mining for Beginners. By Morgan W. Davies, M.I.M.E. [In the press.

XIX. METALLURGY.
A ementary Metallurgy. By
Text-Book of Elementary "
A H. Hiorns, Principal of
A.
the School of Metallurgy, Birmingham and Midland
"'llau' Institute. 3s. Questions, Is.

XXII. STEAM.
Steam, Gas, and Oil Engines. By John Perry, F.R.S. 7s. 6d. net

XXIII. PHYSIOGRAPHY.
Experimental Science (Section I. Physiography). By Prof. R. A.
Gregory and A. T. Simmons, B.Sc. 2s. 6d.

Physiography for Beginners. By A. T. Simmons, B.Sc. 2s. 6d. Adapted to


the Elementary Stage of the South Kensington Syllabus.
Physiography for Advanced
Students. By A. T. Simmons, B.Sc. 4s. 6d.
Elementary Lessons in Astronomy. By Sir Norman Lockyer. New
Edition. 5s. 6d. This book contains all the Astronomy required for the Advanced
and Honours.

XXIV. THE PRINCIPLES OF AGRICULTURE.


Elementary Lessons in the Science of Agricultural Practice. By
H. Tanner, F.C.S. 3s. 6d.

XXV. HYGIENE.
Hygiene for Beginners. By E. S. Reynolds, M.D. Adapted to the Elementary
Stage of the Soutn Kensington Syllabus. 2s. 6d.
Experimental Hygiene (Section I.). By A. T. Simmons, B.Sc., and E. Stkn-
house, B.Sc. 2s. 6d.

Hygiene for Students. By E. F. Willouohby, M.B. Adapted to the Advanced


Stage of the 8outh Kensington Syllabus. Being a New and Revised Edition. 4s. 6d.

MACMILLAN AND CO., LTD., LONDON


BOOKS FOR

SCHOOLS OF SCIENCE
ELEMENTARY PHYSICS.
Exercises in Practical Physics for Schools of Science. By Prof. R. A.
Gregory and A. T. Simmons, B.Sc. Part I. First Year's Course. 2s. Part II.
Second Year's Course. 2s.

An Introduction to Practical Physics. By D. Rintoul, M.A. 2s. 6d.

An Exercise Book of Elementary Practical Physics. By Prof. R. A.


Gregory. Fcap. 4to. 2s. 6d.

Elementary Course of Practical Science. Part I. By Hugh Gordon,


M.A , Inspector of Science Schools, Science and Art Department. Is.

Practical Lessons in Physical Measurement. By A. Earl. 5s.


A Primer of Physics. By Prof. Balfour Stewart. Is.
Elementary Physics. By Prof. Balfour Stewart. 4s. 6d. Questions. 2s.
Elements of Physics. By C. E. Fessenden. I. Matter and its Properties.
II. Kinematics. III. Dynamics. IV. Heat. 3s.

A Graduated Course of Natural Science. By B. Loewy. Part I., 2s.


Part II., 2s. 6d.

Practical Exercises in Magnetism and Electricity. Being a Labora-


tory Course for Schools of Science. By H. E. Hadley, B.Sc. Globe 8yo. 2s. 6d.

ELEMENTARY CHEMISTRYTHEORETICAL.
Chemistry for Beginners. By Sir Henry Roscoe, F.R.S., and J. Lunt, B.Sc.
2s. 6d.

The Elements of Chemistry. By Prof. Ira Remsen. Xew Edition. 2*. 6d.

ELEMENTARY CHEMISTRYPRACTICAL.
Elementary Chemistry for Schools of Science and Higher Grade
Schools. By S. Parrish, B.Sc, Central Higher Grade School, Leeds. With
Introduction by Dr. Forsyth. 2s. C>d.

Practical Inorganic Chemistry. By G. S. Turpin, M.A., D.Sc. 2s. 6d.


An Introduction to the Study of Chemistry. By W. H. Perkin, Jun.,
Ph.D., F.R.S., and Bevan Lean, D.Sc. 2s. 6d.
Introduction to Chemistry and Physics. By W. H. Perkin, Junr., Ph.D.
and B. Lean, D.Sc. 2 vols. 28. each.

The Junior Course of Practical Inorganic Chemistry. By F. Jones,


F.C.S. U. 2s.

A Primer of Chemistry. By Sir Henry Roscoe, F.R.S. Is.

MACMILLAN AND CO., LTD., LONDON


5.0.02
Ktf A>uMC*^ ,

p App)'eA wv OACiutfij

4-or v^-e Q v v\ n v* <^

PLEASE DO NOT REMOVE


CARDS OR SLIPS FROM THIS POCKET

UNIVERSITY OF TORONTO LIBRARY

You might also like